Advanced Neuroradiology Cases. Challenge Your Knowledge 9781107088719, 2016007903

408 41 34MB

English Pages [689] Year 2017

Report DMCA / Copyright

DOWNLOAD FILE

Polecaj historie

Advanced Neuroradiology Cases. Challenge Your Knowledge
 9781107088719, 2016007903

  • Commentary
  • eBook

Table of contents :
Advanced Neuroradiology Cases
Half Title
Title Page
Copyright
Dedication
Contents
List of Chapters with Case Titles
Contributors
Foreword 1
Foreword 2
Preface
Acknowledgements
How to Use This Book
CASE 1
CASE 2
CASE 3
CASE 4
CASE 5
CASE 6
CASE 7
CASE 8
CASE 9
CASE 10
CASE 11
CASE 12
CASE 13
CASE 14
CASE 15
CASE 16
CASE 17
CASE 18
CASE 19
CASE 20
CASE 21
CASE 22
CASE 23
CASE 24
CASE 25
CASE 26
CASE 27
CASE 28
CASE 29
CASE 30
CASE 31
CASE 32
CASE 33
CASE 34
CASE 35
CASE 36
CASE 37
CASE 38
CASE 39
CASE 40
CASE 41
CASE 42
CASE 43
CASE 44
CASE 45
CASE 46
CASE 47
CASE 48
CASE 49
CASE 50
CASE 51
CASE 52
CASE 53
CASE 54
CASE 55
CASE 56
CASE 57
CASE 58
CASE 59
CASE 60
CASE 61
CASE 62
CASE 63
CASE 64
CASE 65
CASE 66
CASE 67
CASE 68
CASE 69
CASE 70
CASE 71
CASE 72
CASE 73
CASE 74
CASE 75
CASE 76
CASE 77
CASE 78
CASE 79
CASE 80
CASE 81
CASE 82
CASE 83
CASE 84
CASE 85
CASE 86
CASE 87
CASE 88
CASE 89
CASE 90
CASE 91
CASE 92
CASE 93
CASE 94
CASE 95
CASE 96
CASE 97
CASE 98
CASE 99
CASE 100
CASE 101
CASE 102
CASE 103
CASE 104
CASE 105
CASE 106
CASE 107
CASE 108
CASE 109
CASE 110
CASE 111
CASE 112
CASE 113
CASE 114
CASE 115
CASE 116
CASE 117
CASE 118
CASE 119
CASE 120
CASE 121
CASE 122
CASE 123
CASE 124
CASE 125
CASE 126
CASE 127
CASE 128
CASE 129
CASE 130
CASE 131
CASE 132
CASE 133
CASE 134
CASE 135
CASE 136
CASE 137
Index

Citation preview

Advanced Neuroradiology Cases

Advanced Neuroradiology Cases Challenge Your Knowledge Edited by

Lázaro Luís Faria do Amaral Neuroradiologist and Chief of the Neuroradiology Department, Medimagem – Hospital Beneficência Portuguesa and Hospital São Jose, São Paulo, SP, Brazil Neuroradiologist and Head of the Neuroradiology Department, Telemedimagem – Hospital Santa Catarina, São Paulo, SP, Brazil Neuroradiologist of the Neuroradiology Department, Santa Casa de Misericórdia de São Paulo, São Paulo, SP, Brazil

Asim K. Bag Assistant Professor, Section of Neuroradiology, The Department of Radiology, School of Medicine, The University of Alabama at Birmingham, AL, USA

Fabrício Guimarães Gonçalves President of the Radiology Society of Brasília, and Neuroradiologist and Chief of the Radiology Department, Department of Radiology, Children’s Hospital of Brasilia, Brazil

Prasad B. Hanagandi Pediatric Neuroradiology Fellow at Hospital for Sick Children, University of Toronto, Ontario, Canada

Associate Editor

Suzanne Byan-Parker Birmingham, AL, USA

University Printing House, Cambridge CB2 8BS, United Kingdom One Liberty Plaza, 20th Floor, New York, NY 10006, USA 477 Williamstown Road, Port Melbourne, VIC 3207, Australia 4843/24, 2nd Floor, Ansari Road, Daryaganj, Delhi - 110002, India 79 Anson Road, #06-04/06, Singapore 079906 Cambridge University Press is part of the University of Cambridge. It furthers the University’s mission by disseminating knowledge in the pursuit of education, learning, and research at the highest international levels of excellence. www.cambridge.org Information on this title: www.cambridge.org/9781107088719 © Cambridge University Press 2017 This publication is in copyright. Subject to statutory exception and to the provisions of relevant collective licensing agreements, no reproduction of any part may take place without the written permission of Cambridge University Press. First published 2017 Printed in the United Kingdom by Clays, St Ives plc A catalog record for this publication is available from the British Library Library of Congress Cataloging in Publication data Names: Amaral, Lázaro Luís Faria do, editor. | Bag, Asim K., editor. | Gonçalves, Fabrício Guimarães, editor. | Hanagandi, Prasad B., editor. | Byan-Parker, Suzanne, associate editor. Title: Advanced neuroradiology cases : challenge your knowledge / edited by Lázaro Luís Faria do Amaral, Asim K. Bag, Fabricio Guimarães Gonçalves, Prasad B. Hanagandi, associate editor, Suzanne Byan-Parker. Description: Cambridge, United Kingdom; New York: Cambridge University Press, 2016. | Includes bibliographical references. Identifiers: LCCN 2016007903 | ISBN 9781107088719 (Hardback) Subjects: | MESH: Central Nervous System Diseases–radiography | Central Nervous System Diseases–diagnosis | Neuroradiography | Diagnostic Techniques, Neurological | Case Reports Classification: LCC RC386.6.M34 | NLM WL 141.5.N47 | DDC 616.8/047548–dc23 LC record available at http://lccn.loc.gov/2016007903 ISBN 978-1-107-08871-9 Hardback Cambridge University Press has no responsibility for the persistence or accuracy of URLs for external or third-party internet websites referred to in this publication, and does not guarantee that any content on such websites is, or will remain, accurate or appropriate.

............................................................................. Every effort has been made in preparing this book to provide accurate and up-to-date information which is in accord with accepted standards and practice at the time of publication. Although case histories are drawn from actual cases, every effort has been made to disguise the identities of the individuals involved. Nevertheless, the authors, editors, and publishers can make no warranties that the information contained herein is totally free from error, not least because clinical standards are constantly changing through research and regulation. The authors, editors, and publishers therefore disclaim all liability for direct or consequential damages resulting from the use of material contained in this book. Readers are strongly advised to pay careful attention to information provided by the manufacturer of any drugs or equipment that they plan to use.

With love to my wife, Denise, inseparable companion of my life, and to my son Danilo, and my daughter, Beatriz, for giving my life immeasurable meaning. To my parents Iolanda and Pedro: thank you very much for your love and support. “Wisdom is supreme; therefore get wisdom. Though it cost all you have, get understanding.” Proverbs 4:7 Lázaro Luís Faria do Amaral, MD To all the residents, fellows, and trainees who are striving to be the best: I hope this book makes you stronger and sharper. To all the physicians who want to familiarize themselves to the imaging appearances of some of the rare diseases of the brain for confident diagnosis: I hope you find this book helpful. To Ras, my soul mate now and forever: To Rishi, my loving son: To my parents: Thank you for your EXCEPTIONAL patience, Love, and STRONG support. Asim K. Bag, MD To all my teachers, fellow colleagues, and residents. A special thank you to my parents for their support. “Neel” my adorable son. Very special thanks to my lovely woman; for standing by me and extending emotional support and encouragement during my tough times. Prasad B. Hanagandi, MD To God, who gives me strength, to my wife Flávia for her love and patience, who provided support and encouragement and who took care of me and especially of two princesses Rafaela and Marcela, during those months dedicated to the completion of this dream. A special thank you to Asim Bag (aka “the president”), our editor, for his support and great patience organizing the material, to Dr. Lázaro (aka “the boss”) for his mentorship and being always available to teach and share his vast knowledge and for Prasad my special friend (my Indian brother) who always shared the dream of writing together this book and for his always present words of support. Fabrício Guimarães Gonçalves, MD

Contents List of chapters with case titles page xi List of contributors xvi Foreword 1 by Àlex Rovira xviii Foreword 2 by Mauricio Castillo xix Preface xxi Acknowledgements xxii How to Use This Book xxiii

Part I: Neurodegenerative Diseases 1 Asim K. Bag, Lázaro Luís Faria do Amaral

18 Asim K. Bag, Victor Hugo Rocha Marussi, Lázaro Luís Faria do Amaral 59

1

19 Prasad B. Hanagandi, Rahul J. Vakharia, Lázaro Luís Faria do Amaral 63

2 Asim K. Bag, Aparna Singhal, Lázaro Luís Faria do Amaral 3 3 Asim K. Bag, Victor Hugo Rocha Marussi, Lázaro Luís Faria do Amaral 7 4 Asim K. Bag, Aparna Singhal, Lázaro Luís Faria do Amaral 9

20 Prasad B. Hanagandi, Satya Patro, Lázaro Luís Faria do Amaral, Antônio José da Rocha 69

Part II: Neurovascular Diseases

5 Asim K. Bag, Lázaro Luís Faria do Amaral

13

21 Asim K. Bag, Lázaro Luís Faria do Amaral

6 Asim K. Bag, Lázaro Luís Faria do Amaral

15

22 Prasad B. Hanagandi, Satya Patro, Salo Haratz, Lázaro Luís Faria do Amaral 81

7 Asim K. Bag, Ricardo Heusi

17

23 Fabrício Guimarães Gonçalves

75

87

8 Asim K. Bag, Lázaro Luís Faria do Amaral

19

9 Asim K. Bag, Lázaro Luís Faria do Amaral

21

10 Asim K. Bag, Lázaro Luís Faria do Amaral

25

25 Asim K. Bag, Aparna Singhal, Fabrício Guimarães Gonçalves 97

11 Asim K. Bag, Lázaro Luís Faria do Amaral

27

26 Fabrício Guimarães Gonçalves, Prasad B. Hanagandi

103

12 Asim K. Bag, Lázaro Luís Faria do Amaral

31

27 Prasad B. Hanagandi, Leslie Lamb, Jennifer Kamps

107

13 Lázaro Luís Faria do Amaral, Afonso C. P. Liberato

24 Asim K. Bag, Lázaro Luís Faria do Amaral

33

14 Prasad B. Hanagandi, Santanu Chakraborty, Lázaro Luís Faria do Amaral 37

93

28 Prasad B. Hanagandi, Santanu Chakraborty, Lázaro Luís Faria do Amaral 113 29 Asim K. Bag, Glenn H. Roberson

117

15 Prasad B. Hanagandi, Taleb Al Mansoori, Lázaro Luís Faria do Amaral 43

30 Prasad B. Hanagandi, Santanu Chakraborty, Lázaro Luís Faria do Amaral 119

16 Lázaro Luís Faria do Amaral, Bruno Shigueo Yonekura Inada, Leonardo Furtado Freitas, Prasad B. Hanagandi 49

31 Fabrício Guimarães Gonçalves, Lázaro Luís Faria do Amaral 125

17 Fabrício Guimarães Gonçalves

55

32 Lázaro Luís Faria do Amaral, Lucídio Portella Nunes Neto, Christiane Monteiro Siqueira Campos 131

vii

Contents

33 Lázaro Luís Faria do Amaral, Anderson B. Belezia

135

34 Prasad B. Hanagandi, Jeffrey Chankowsky, Raquel del Carpio-O’Donovan 141

57 Fabrício Guimarães Gonçalves, Lázaro Luís Faria do Amaral 245 58 Fabrício Guimarães Gonçalves, Márcio Olavo, Gomes Magalhães, Lázaro Luís Faria do Amaral

35 Lázaro Luís Faria do Amaral, Lucídio Portella Nunes Neto, Christiane Monteiro Siqueira Campos 145

59 Fabrício Guimarães Gonçalves, Asim K. Bag

36 Lázaro Luís Faria do Amaral, Leonardo Furtado Freitas 149

60 Fabrício Guimarães Gonçalves, Asim K. Bag

Part III: Neuroinfectious Diseases 38 Lázaro Luís Faria do Amaral, Heitor Castelo Branco R. Alves 157

261

61 Fabrício Guimarães Gonçalves, Márcio Olavo Gomes Magalhães 267 62 Asim K. Bag, Rasmoni Roy

271

39 Lázaro Luís Faria do Amaral, Ingrid Aguiar Littig, Asim K. Bag 163

63 Fabrício Guimarães Gonçalves, Lázaro Luís Faria do Amaral 275

40 Lázaro Luís Faria do Amaral, Ricardo Tavares Daher 169

64 Asim K. Bag, Aparna Singhal

41 Lázaro Luís Faria do Amaral

65 Prasad B. Hanagandi, Rahul J. Vakharia

175

66 Asim K. Bag, Harry S. Hardin

181

43 Fabrício Guimarães Gonçalves, Guilherme Cássia

183

44 Lázaro Luís Faria do Amaral, Bruno Shigueo Yonekura Inada, Bruno Siqueira Campos Lopes 189 45 Fabrício Guimarães Gonçalves, Karenn Barros Bezerra 193 46 Fabrício Guimarães Gonçalves, Lázaro Luís Faria do Amaral 199

48 Prasad B. Hanagandi, Sonali H. Shah, Bernardo Jose Alves Ferreira Martins, Lázaro Luís Faria do Amaral 209 49 Lázaro Luís Faria do Amaral, Ricardo Tavares Daher 213 50 Prasad B. Hanagandi, Sunila Jaggi

215

53 Asim K. Bag

223

227

54 Lázaro Luís Faria do Amaral, Ricardo Tavares Daher 231 55 Lázaro Luís Faria do Amaral, Ivanildo Castro Pereira Jr. 237 56 Asim K. Bag, Benson Tran, Lázaro Luís Faria do Amaral 241

291

67 Prasad B. Hanagandi, Rahul J. Vakharia, Lázaro Luís Faria do Amaral 295 68 Prasad B. Hanagandi, Taleb Al Mansoori, Jeffrey Chankowsky 301 69 Fabrício Guimarães Gonçalves

307

71 Asim K. Bag, Rasmoni Roy, Lázaro Luís Faria do Amaral 317 72 Prasad B. Hanagandi, Jeffrey Chankowsky, Raquel del Carpio-O’Donovan 321 73 Prasad B. Hanagandi, Jeffrey Chankowsky, Raquel del Carpio-O’Donovan 327

75 Lázaro Luís Faria do Amaral, Anderson B. Belezia

217

52 Lázaro Luís Faria do Amaral, Ricardo Tavares Daher

285

74 Lázaro Luís Faria do Amaral, Ricardo Tavares Daher 333

Part IV: Neuroinflammatory Diseases 51 Asim K. Bag, Glenn H. Roberson

281

70 Prasad B. Hanagandi, Rahul J. Vakharia, Lázaro Luís Faria do Amaral 311

47 Prasad B. Hanagandi, Sunila Jaggi, Lázaro Luís Faria do Amaral 205

viii

255

Part V: Metabolic Diseases Involving Central Nervous System

37 Lázaro Luís Faria do Amaral, César Augusto P. F. Alves 153

42 Asim K. Bag, Lázaro Luís Faria do Amaral

249

339

76 Lázaro Luís Faria do Amaral, Renato Hoffmann Nunes 345 77 Prasad B. Hanagandi, Rahul J. Vakharia, Inder Talwar 351 78 Lázaro Luís Faria do Amaral, Anderson B. Belezia

357

79 Lázaro Luís Faria do Amaral, Ricardo Tavares Daher, Asim K. Bag 363 80 Asim K. Bag

369

Contents

81 Lázaro Luís Faria do Amaral, Kelly Fiorini, Asim K. Bag, Leonardo Furtado Freitas 373

106 Lázaro Luís Faria do Amaral, Victor Hugo Rocha Marussi 495

82 Lázaro Luís Faria do Amaral, Bruno Siqueira Campos Lopes, Asim K. Bag 377

107 Lázaro Luís Faria do Amaral, Afonso C. P. Liberato

501

108 Lázaro Luís Faria do Amaral, Afonso C. P. Liberato

507

Part VI: Central Nervous System Tumors

109 Prasad B. Hanagandi, Leslie Lamb, Lázaro Luís Faria do Amaral 511

83 Fabrício Guimarães Gonçalves, Lázaro Luís Faria do Amaral 381

110 Lázaro Luís Faria do Amaral, Bruno Augusto Telles

517

111 Lázaro Luís Faria do Amaral, Afonso C. P. Liberato

521

84 Fabrício Guimarães Gonçalves, Asim K. Bag, Lázaro Luís Faria do Amaral 383 85 Fabrício Guimarães Gonçalves, Asim K. Bag, Lázaro Luís Faria do Amaral 389

112 Lázaro Luís Faria do Amaral, Lucído Portella Nunes Neto, Christiane Monterio Siqueira Campos 527 113 Lázaro Luís Faria do Amaral, Anderson B. Belezia

86 Fabrício Guimarães Gonçalves, Lázaro Luís Faria do Amaral 395

114 Lázaro Luís Faria do Amaral, Bruno Augusto Telles, Asim K. Bag 539

87 Asim K. Bag, Lázaro Luís Faria do Amaral

115 Lázaro Luís Faria do Amaral, Bruno Augusto Telles, Asim K. Bag 545

401

88 Lázaro Luís Faria do Amaral, Anderson B. Belezia

407

89 Lázaro Luís Faria do Amaral, Taciana Mara Filomeno Orsini, Leonardo Furtado Freitas 411 90 Prasad B. Hanagandi, Sonali H. Shah, Lázaro Luís Faria do Amaral 415 91 Prasad B. Hanagandi, Leslie Lamb, Jeffrey Chankowsky, Raquel del Carpio-O’Donovan 421 92 Prasad B. Hanagandi 93 Asim K. Bag

433

94 Asim K. Bag

439

95 Asim K. Bag

443

427

98 Prasad B. Hanagandi, Leslie Lamb

101 Prasad B. Hanagandi, Jeffrey Chankowsky, Raquel del Carpio-O’Donovan 473

569

120 Prasad B. Hanagandi, Lázaro Luís Faria do Amaral

573

122 Prasad B. Hanagandi, Satya Patro, Santanu Chakraborty 581

123 Lázaro Luís Faria do Amaral, Bruno Siqueira Campos Lopes 587 124 Asim K. Bag, Aparna Singhal

589

125 Asim K. Bag, Aparna Singhal

593

126 Asim K. Bag

477

103 Asim K. Bag, Lázaro Luís Faria do Amaral

491

119 Prasad B. Hanagandi, Lázaro Luís Faria do Amaral

Part VIII: Miscellaneous

100 Prasad B. Hanagandi, Stephanie Lam, Lázaro Luís Faria do Amaral 467

105 Asim K. Bag

Part VII: Congenital Diseases Manifesting in Adults

455

99 Prasad B. Hanagandi, Stephanie Lam, Jeffrey Chankowsky 461

485

117 Lázaro Luís Faria do Amaral, Bruno Augusto Telles, Asim K. Bag 557

121 Lázaro Luís Faria do Amaral, Bruno Siqueira Campos Lopes 577

445

97 Fabrício Guimarães Gonçalves, Asim K. Bag, Lázaro Luís Faria do Amaral 449

104 Asim K. Bag

551

118 Lázaro Luís Faria do Amaral, Auro Augusto Junqueira Côrtes, Leonardo Furtado Freitas, Asim K. Bag 563

96 Asim K. Bag, Philip R. Chapman

102 Asim K. Bag, Harry S. Hardin

116 Prasad B. Hanagandi, Satya Patro, Rahul J. Vakharia

533

481

599

127 Lázaro Luís Faria do Amaral, Victor Hugo Rocha Marussi 603 128 Prasad B. Hanagandi, Lázaro Luís Faria do Amaral

607

ix

Contents

129 Asim K. Bag

613

130 Lázaro Luís Faria do Amaral, Afonso C. P. Liberato

615

131 Prasad B. Hanagandi, Satya Patro, Lázaro Luís Faria do Amaral 619

135 Prasad B. Hanagandi, Taleb Al Mansoori, Lázaro Luís Faria do Amaral 643 136 Prasad B. Hanagandi, Marc Dilauro

137 Prasad B. Hanagandi, Santanu Chakraborty

132 Fabrício Guimarães Gonçalves, Asim K. Bag, Lázaro Luís Faria do Amaral 625 133 Fabrício Guimarães Gonçalves, Lázaro Luís Faria do Amaral 631 134 Prasad B. Hanagandi, Stephanie Lam, Lázaro Luís Faria do Amaral 637

x

647

Index

658

653

List of Chapters with Case Titles Part I: Neurodegenerative Diseases

13 Hippocampal Sclerosis Dementia 36 Lázaro Luís Faria do Amaral, Afonso C. P. Liberato

1 Beta-Propeller Protein-Associated Neurodegeneration (BPAN) 2 Asim K. Bag, Lázaro Luís Faria do Amaral 2 Mitochondrial Membrane Protein-Associated Neurodegeneration (MPAN) 4 Asim K. Bag, Aparna Singhal, Lázaro Luís Faria do Amaral 3 Neuroferritinopathy 8 Asim K. Bag, Victor Hugo Rocha Marussi, Lázaro Luís Faria do Amaral 4 Pantothenate Kinase-Associated Neurodegeneration (PKAN) 10 Asim K. Bag, Aparna Singhal, Lázaro Luís Faria do Amaral 5 Phospholipase-Associated Neurodegeneration (PLAN) 14 Asim K. Bag, Lázaro Luís Faria do Amaral

7 Whipple Disease with Central Nervous System Involvement 18 Asim K. Bag, Ricardo Heusi

16 Cerebrotendinous Xanthomatosis 52 Lázaro Luís Faria do Amaral, Bruno Shigueo Yonekura Inada, Leonardo Furtado Freitas, Prasad B. Hanagandi 17 Huntington Disease 56 Fabrício Guimarães Gonçalves 18 Frontotemporal Lobar Degeneration Associated with Fused in Sarcoma Protein 60 Asim K. Bag, Victor Hugo Rocha Marussi, Lázaro Luís Faria do Amaral

20 Machado-Joseph Disease 72 Prasad B. Hanagandi, Satya Patro, Lázaro Luís Faria do Amaral, Antônio José da Rocha

20

9 Multiple System Atrophy-Parkinsonian Type Asim K. Bag, Lázaro Luís Faria do Amaral

15 Idiopathic Progressive Ataxia and Palatal Tremor 46 Prasad B. Hanagandi, Taleb Al Mansoori, Lázaro Luís Faria do Amaral

19 Leukoencephalopathy with Brainstem and Spinal Cord Involvement and Lactate Elevation 66 Prasad B. Hanagandi, Rahul J. Vakharia, Lázaro Luís Faria do Amaral

6 Aceruloplasminemia 16 Asim K. Bag, Lázaro Luís Faria do Amaral

8 Multiple System Atrophy-Cerebellar Type Asim K. Bag, Lázaro Luís Faria do Amaral

14 Primary Lateral Sclerosis 40 Prasad B. Hanagandi, Santanu Chakraborty, Lázaro Luís Faria do Amaral

22

Part II: Neurovascular Diseases

10 Corticobasal Degeneration 26 Asim K. Bag, Lázaro Luís Faria do Amaral

21 Cerebral Proliferative Angiopathy 78 Asim K. Bag, Lázaro Luís Faria do Amaral

11 Progressive Supranuclear Palsy 28 Asim K. Bag, Lázaro Luís Faria do Amaral

22 Spectacular Shrinking Deficit 84 Prasad B. Hanagandi, Satya Patro, Salo Haratz, Lázaro Luís Faria do Amaral

12 Right Temporal Variant of Frontotemporal Lobar Degeneration 32 Asim K. Bag, Lázaro Luís Faria do Amaral

23 Cerebral Amyloidoma 90 Fabrício Guimarães Gonçalves

xi

List of Chapters with Case Titles

24 Cerebral Amyloid Angiopathy with Inflammation Asim K. Bag, Lázaro Luís Faria do Amaral 25 Reversible Cerebral Vasoconstriction Syndrome Asim K. Bag, Aparna Singhal, Fabrício Guimarães Gonçalves

94 100

26 Susac Syndrome 104 Fabrício Guimarães Gonçalves, Prasad B. Hanagandi 27 Cerebral Autosomal Dominant Arteriopathy with Subcortical Infarcts and Leukoencephalopathy (CADASIL) 110 Prasad B. Hanagandi, Leslie Lamb, Jennifer Kamps 28 Transient Global Amnesia 116 Prasad B. Hanagandi, Santanu Chakraborty, Lázaro Luís Faria do Amaral 29 Perimesencephalic Subarachnoid Hemorrhage Asim K. Bag, Glenn H. Roberson

118

30 Cerebral Fat Embolism 122 Prasad B. Hanagandi, Santanu Chakraborty, Lázaro Luís Faria do Amaral 31 Developmental Venous Anomaly with Complications 128 Fabrício Guimarães Gonçalves, Lázaro Luís Faria do Amaral 32 Radiation-Induced Tumefactive Cyst 134 Lázaro Luís Faria do Amaral, Lucídio Portella Nunes Neto, Christiane Monteiro Siqueira Campos 33 Thrombosis of Bilateral Internal Cerebral Veins 138 Lázaro Luís Faria do Amaral, Anderson B. Belezia 34 Hemolytic Uremic Syndrome 144 Prasad B. Hanagandi, Jeffrey Chankowsky, Raquel del Carpio-O’Donovan 35 Sinus Pericranii 148 Lázaro Luís Faria do Amaral, Lucídio Portella Nunes Neto, Christiane Monteiro Siqueira Campos 36 Ramsay Hunt Syndrome with Involvement of Spinal Nucleus and Tract of Trigeminal Nerve 152 Lázaro Luís Faria do Amaral, Leonardo Furtado Freitas 37 Herpes Simplex Cerebellitis 156 Lázaro Luís Faria do Amaral, César Augusto P. F. Alves

Part III: Neuroinfectious Diseases 38 Listeria Rhombencephalitis 160 Lázaro Luís Faria do Amaral, Heitor Castelo Branco R. Alves

xii

39 Progressive Multifocal Leukoencephalopathy – Immune Reconstitution Inflammatory Syndrome Lázaro Luís Faria do Amaral, Ingrid Aguiar Littig, Asim K. Bag

166

40 Neurosyphilis Involving Temporal Lobe 172 Lázaro Luís Faria do Amaral, Ricardo Tavares Daher 41 Syphilitic Gumma 178 Lázaro Luís Faria do Amaral 42 Central Nervous System Blastomycosis 182 Asim K. Bag, Lázaro Luís Faria do Amaral 43 Central Nervous System Aspergillosis 186 Fabrício Guimarães Gonçalves, Guilherme Cássia 44 Cerebral Schistosomiasis 192 Lázaro Luís Faria do Amaral, Bruno Shigueo Yonekura Inada, Bruno Siqueira Campos Lopes 45 Tubercular Vasculitis 196 Fabrício Guimarães Gonçalves, Karenn Barros Bezerra 46 Chagas Disease 202 Fabrício Guimarães Gonçalves, Lázaro Luís Faria do Amaral 47 Cerebral Malaria 208 Prasad B. Hanagandi, Sunila Jaggi, Lázaro Luís Faria do Amaral 48 HTLV-1-Associated Myelopathy/Tropical Spastic Paraparesis 212 Prasad B. Hanagandi, Sonali H. Shah, Bernardo Jose Alves Ferreira Martins, Lázaro Luís Faria do Amaral 49 Racemose Neurocysticercosis Complicated with Infarction 214 Lázaro Luís Faria do Amaral, Ricardo Tavares Daher 50 Neurocysticercosis and Japanese B Encephalitis Prasad B. Hanagandi, Sunila Jaggi

Part IV: Neuroinflammatory Diseases 51 Primary Angiitis of CNS (PACNS) Asim K. Bag, Glenn H. Roberson

220

52 Anti-N-Methyl-D-Aspartate Receptor (NMDAR) Encephalitis 226 Lázaro Luís Faria do Amaral, Ricardo Tavares Daher 53 Neuro-Behçet Disease Asim K. Bag

228

54 Parenchymal Neurosarcoidosis 234 Lázaro Luís Faria do Amaral, Ricardo Tavares Daher

216

List of Chapters with Case Titles

55 Bickerstaff Encephalitis 240 Lázaro Luís Faria do Amaral, Ivanildo Castro Pereira Jr.

71 Methotrexate-Induced Subacute Encephalopathy 318 Asim K. Bag, Rasmoni Roy, Lázaro Luís Faria do Amaral

56 CLIPPERS 242 Asim K. Bag, Benson Tran, Lázaro Luís Faria do Amaral

72 Metronidazole-Induced Encephalopathy 324 Prasad B. Hanagandi, Jeffrey Chankowsky, Raquel del Carpio-O’Donovan

57 Balo Concentric Sclerosis 246 Fabrício Guimarães Gonçalves, Lázaro Luís Faria do Amaral 58 Acute Necrotizing Encephalopathy 252 Fabrício Guimarães Gonçalves, Márcio Olavo, Gomes Magalhães, Lázaro Luís Faria do Amaral 59 Neuromyelitis Optica (NMO) Spectrum Disorder Fabrício Guimarães Gonçalves, Asim K. Bag

258

Part V: Metabolic Diseases Involving Central Nervous System 60 Vanishing White Matter Syndrome 264 Fabrício Guimarães Gonçalves, Asim K. Bag 61 Lipoid Proteinosis 268 Fabrício Guimarães Gonçalves, Márcio Olavo Gomes Magalhães 62 Central Variant of Posterior Reversible Encephalopathy Syndrome (CV-PRES) 272 Asim K. Bag, Rasmoni Roy 63 Posterior Reversible Encephalopathy Syndrome with Hemorrhage 278 Fabrício Guimarães Gonçalves, Lázaro Luís Faria do Amaral 64 Hyperglycemic Hemiballismus and Hemichorea Asim K. Bag, Aparna Singhal

282

65 Delayed Encephalopathy due to Carbon Monoxide Poisoning 288 Prasad B. Hanagandi, Rahul J. Vakharia 66 Chronic Hepatic Encephalopathy Asim K. Bag, Harry S. Hardin

292

67 Menkes Disease 298 Prasad B. Hanagandi, Rahul J. Vakharia, Lázaro Luís Faria do Amaral 68 Acute Hyperammonemic Encephalopathy 304 Prasad B. Hanagandi, Taleb Al Mansoori, Jeffrey Chankowsky 69 Marchiafava-Bignami Disease Fabrício Guimarães Gonçalves

308

70 Hypomelanosis of Ito 314 Prasad B. Hanagandi, Rahul J. Vakharia, Lázaro Luís Faria do Amaral

73 Transient Splenial Lesion 330 Prasad B. Hanagandi, Jeffrey Chankowsky, Raquel del Carpio-O’Donovan 74 Mitochondrial Encephalomyopathy, Lactic Acidosis, and Stroke-like Episodes Syndrome: MELAS Syndrome 336 Lázaro Luís Faria do Amaral, Ricardo Tavares Daher 75 Wilson Disease 342 Lázaro Luís Faria do Amaral, Anderson B. Belezia 76 Hereditary Hemochromatosis 348 Lázaro Luís Faria do Amaral, Renato Hoffmann Nunes 77 Wolfram Syndrome 354 Prasad B. Hanagandi, Rahul J. Vakharia, Inder Talwar 78 Wernicke Encephalopathy 360 Lázaro Luís Faria do Amaral, Anderson B. Belezia 79 Subacute Combined Degeneration with Optic Tract Involvement 366 Lázaro Luís Faria do Amaral, Ricardo Tavares Daher, Asim K. Bag 80 Osmotic Demyelination Syndrome Asim K. Bag

372

81 Adult-Onset Alexander Disease 376 Lázaro Luís Faria do Amaral, Kelly Fiorini, Asim K. Bag, Leonardo Furtado Freitas 82 Mucopolysaccharidosis – MPS Type II (Hunter Syndrome) 380 Lázaro Luís Faria do Amaral, Bruno Siqueira Campos Lopes, Asim K. Bag

Part VI: Central Nervous System Tumors 83 Rosette-Forming Glioneuronal Tumor 382 Fabrício Guimarães Gonçalves, Lázaro Luís Faria do Amaral 84 Meningioangiomatosis 386 Fabrício Guimarães Gonçalves, Asim K. Bag, Lázaro Luís Faria do Amaral 85 Astroblastoma 392 Fabrício Guimarães Gonçalves, Asim K. Bag, Lázaro Luís Faria do Amaral

xiii

List of Chapters with Case Titles

104 Diffuse Astrocytoma/Gliomatosis Cerebri Asim K. Bag

86 Protoplasmic Astrocytoma 398 Fabrício Guimarães Gonçalves, Lázaro Luís Faria do Amaral 87 Medulloblastoma with Extensive Nodularity Asim K. Bag, Lázaro Luís Faria do Amaral

105 Persistent Diffusion Restriction after Bevacizumab Therapy 494 Asim K. Bag

404

88 Multifocal Desmoplastic Medulloblastoma 410 Lázaro Luís Faria do Amaral, Anderson B. Belezia

106 Pineoblastoma with Drop Metastasis 498 Lázaro Luís Faria do Amaral, Victor Hugo Rocha Marussi

89 Malignant Transformation of Dermoid Cyst 414 Lázaro Luís Faria do Amaral, Taciana Mara Filomeno Orsini, Leonardo Furtado Freitas

107 Renal Cell Carcinoma Metastasis to Posterior Fossa 504 Lázaro Luís Faria do Amaral, Afonso C. P. Liberato

90 Extraventricular Neurocytoma 418 Prasad B. Hanagandi, Sonali H. Shah, Lázaro Luís Faria do Amaral

108 Intraventricular Hemangiopericytoma/Solitary Fibrous Tumor 510 Lázaro Luís Faria do Amaral, Afonso C. P. Liberato

91 Cerebellar Liponeurocytoma 424 Prasad B. Hanagandi, Leslie Lamb, Jeffrey Chankowsky, Raquel del Carpio-O’Donovan

109 Interhemispheric and Pericallosal Lipoma with Cortical Dysplasia 514 Prasad B. Hanagandi, Leslie Lamb, Lázaro Luís Faria do Amaral

92 Delayed Postradiation Temporal Lobe Necrosis Prasad B. Hanagandi 93 Pseudoprogression due to Chemoradiation Asim K. Bag

430

110 Supratentorial Ependymoma 520 Lázaro Luís Faria do Amaral, Bruno Augusto Telles

436

111 Multifocal Dysembryoplastic Neuroepithelial Tumor 524 Lázaro Luís Faria do Amaral, Afonso C. P. Liberato

94 Pseudoresponse Associated with Antiangiogenic Therapy 442 Asim K. Bag

112 Cavernous Sinus Hemangioma 530 Lázaro Luís Faria do Amaral, Lucído Portella Nunes Neto, Christiane Monterio Siqueira Campos

95 Distal Glioblastoma Recurrence after Antiangiogenic Therapy 444 Asim K. Bag 96 Anti-Ma2-Associated Paraneoplastic Encephalitis Asim K. Bag Philip R. Chapman

446

97 Microcystic Meningioma 452 Fabrício Guimarães Gonçalves, Asim K. Bag, Lázaro Luís Faria do Amaral 98 Dural Convexity Chondroma 458 Prasad B. Hanagandi, Leslie Lamb 99 Third Ventricular Craniopharyngioma 464 Prasad B. Hanagandi, Stephanie Lam, Jeffrey Chankowsky 100 Intraparenchymal Epidermoid Cyst 470 Prasad B. Hanagandi, Stephanie Lam, Lázaro Luís Faria do Amaral 101 White Epidermoid Cyst 476 Prasad B. Hanagandi, Jeffrey Chankowsky, Raquel del Carpio-O’Donovan 102 Brain Metastases from Mucin-Producing Primary Asim K. Bag, Harry Hardin 103 Intra-axial CNS Dermoid Cyst 484 Asim K. Bag, Lázaro Luís Faria do Amaral

xiv

488

480

113 Ruptured Dermoid Cyst with Cerebral Ischemia Lázaro Luís Faria do Amaral, Anderson B. Belezia

536

114 Cystic Glioblastoma 542 Lázaro Luís Faria do Amaral, Bruno Augusto Telles, Asim K. Bag 115 Pilomyxoid Astrocytoma 548 Lázaro Luís Faria do Amaral, Bruno Augusto Telles, Asim K. Bag 116 Central Nervous System Tuberculoma 554 Prasad B. Hanagandi, Satya Patro, Rahul J. Vakharia 117 Temporal Horn Choroid Plexus Papilloma 560 Lázaro Luís Faria do Amaral, Bruno Augusto Telles, Asim K. Bag

Part VII: Congenital Diseases Manifesting in Adults 118 Hereditary Spastic Paraplegia with Hypoplastic Corpus Callosum 566 Lázaro Luís Faria do Amaral, Auro Augusto Junqueira Côrtes, Leonardo Furtado Freitas, Asim K. Bag

List of Chapters with Case Titles

119 Pallister-Hall Syndrome 572 Prasad B. Hanagandi, Lázaro Luís Faria do Amaral

128 Giant Arachnoid Granulation 610 Prasad B. Hanagandi, Lázaro Luís Faria do Amaral

120 Incontinentia Pigmenti 576 Prasad B. Hanagandi, Lázaro Luís Faria do Amaral

129 Mineralizing Microangiopathy Asim K. Bag

121 Hybrid Phakomatosis 580 Lázaro Luís Faria do Amaral, Bruno Siqueira Campos Lopes

130 Ring-Shaped Lateral Ventricular Nodules 618 Lázaro Luís Faria do Amaral, Afonso C. P. Liberato 131 Erdheim-Chester Disease 622 Prasad B. Hanagandi, Satya Patro, Lázaro Luís Faria do Amaral

122 Fragile X Tremor Ataxia Syndrome 584 Prasad B. Hanagandi, Satya Patro, Santanu Chakraborty

132 Hypertrophic Olivary Degeneration 628 Fabrício Guimarães Gonçalves, Asim K. Bag, Lázaro Luís Faria do Amaral

Part VIII: Miscellaneous

133 Neurocutaneous Melanosis 634 Fabrício Guimarães Gonçalves, Lázaro Luís Faria do Amaral

123 Anterior Thalamic Infarct with Transneuronal Degeneration of Mammillothalamic Tract (MTT) 588 Lázaro Luís Faria do Amaral, Bruno Siqueira Campos Lopes 124 Intracranial Hypotension with Midbrain Swelling Asim K. Bag, Aparna Singhal 125 Idiopathic Intracranial Hypertension Asim K. Bag, Aparna Singhal

590

596

126 Calcineurin Inhibitor-Mediated Limbic Injury Asim K. Bag

614

602

127 Sinking Skin Flap Syndrome (Trephine Syndrome) 606 Lázaro Luís Faria do Amaral, Victor Hugo Rocha Marussi

134 Progressive Hemifacial Atrophy: Parry-Romberg Syndrome with Localized Scleroderma 640 Prasad B. Hanagandi, Stephanie Lam, Lázaro Luís Faria do Amaral 135 Cutis Verticis Gyrata 646 Prasad B. Hanagandi, Taleb Al Mansoori, Lázaro Luís Faria do Amaral 136 Trigeminal Trophic Syndrome 650 Prasad B. Hanagandi, Marc Dilauro 137 Bilateral Middle Cerebellar Peduncle Infarcts Prasad B. Hanagandi, Santanu Chakraborty

656

xv

Contributors

Taleb Al Mansoori Department of Medical Imaging, the Ottawa Hospital, University of Ottawa

Raquel del Carpio-O’Donovan Department of Diagnostic Radiology, the Montreal General Hospital

César Augusto P. F. Alves Santa Casa de Misericórdia de São Paulo

Ricardo Tavares Daher Hospital Beneficência Portuguesa de São Paulo

Heitor Castelo Branco R. Alves Santa Casa de Misericórdia de São Paulo

Marc Dilauro Department of Medical Imaging, the Ottawa Hospital, University of Ottawa

Anderson Benine Belezia Hospital Beneficência Portuguesa de São Paulo Karenn Barros Bezerra Diagnostic Imaging, Radiology Department, Hospital Universitário de Brasilia, Universidade de Brasilia

Leonardo Furtado Freitas Hospital Beneficência Portuguesa de São Paulo

Christiane Monteiro Siqueira Campos Hospital Beneficência Portuguesa de São Paulo

Salo Haratz Tel Hashomer Medical Center

Guilherme Cassia Department of Radiology, Hospital Santa Helena

Harry S. Hardin The Department of Radiology, School of Medicine, the University of Alabama at Birmingham

Santanu Chakraborty Department of Medical Imaging, the Ottawa Hospital, University of Ottawa Jeffrey Chankowsky Department of Diagnostic Radiology, the Montreal General Hospital

xvi

Kelly Fiorini Hospital Beneficência Portuguesa de São Paulo

Ricardo Heusi Clinica São Lucas, Universidade do Vale do Itajai – Univale and Unimed de Balneário Camboriú Bruno Shigueo Yonekura lnada Hospital Beneficência Portuguesa de São Paulo

Philip R. Chapman The Department of Radiology, School of Medicine, the University of Alabama at Birmingham

Sunila Jaggi Department of CT Scan and MRI, Bombay Hospital and Medical Research Centre

Auro Augusto Junqueira Côrtes Hospital Beneficência Portuguesa de São Paulo

Jennifer Kamps Stavanger University Hospital

Antônio José da Rocha Neuroradiology Department at Fleury Medicina Diagnóstica and Santa Casa de Misericórdia de São Paulo.

Stephanie Lam Department of Radiology, McGiIl University Health Centre – Montreal General Hospital

List of contributors

Leslie Lamb Department of Medical Imaging, the Ottawa Hospital, University of Ottawa Afonso Celso Pedrotti Liberato Hospital Beneficência Portuguesa de São Paulo Ingrid Aguiar Littig Santa Casa de Misericórdia de São Paulo Bruno Siqueira Campos Lopes Hospital Beneficência Portuguesa de São Paulo and Hospital Santa Catarina Márcio Olavo Gomes Magalhães Radiology Department, Hospital Universitário de Brasilia, Universidade de Brasilia Victor Hugo Rocha Marussi Hospital Beneficência Portuguesa de São Paulo Lucídio Portella Nunes Neto Hospital Beneficência Portuguesa de São Paulo Renato Hoffmann Nunes Santa Casa de Misericórdia de São Paulo

Ivanildo Castro Pereira Junior Hospital Beneficência Portuguesa de São Paulo Glenn H. Roberson The Department of Radiology, School of Medicine, the University of Alabama at Birmingham Rasmoni Roy The Department of Radiology, School of Medicine, the University of Alabama at Birmingham Sonali H. Shah Department of CT Scan and MRI, Bombay Hospital and Medical Research Centre Aparna Singhal The Department of Radiology, School of Medicine, the University of Alabama at Birmingham Inder Talwar Department of CT Scan and MRI, Bombay Hospital and Medical Research Centre Bruno Augusto Telles CETAC/INC CURITIBA – PR

Taciana Mara Filomeno Orsini Hospital Beneficência Portuguesa de São Paulo

Benson Tran The Department of Radiology, School of Medicine, the University of Alabama at Birmingham

Satya Patro Department of Medical Imaging, the Ottawa Hospital, University of Ottawa

Rahul J. Vakharia Department of CT Scan and MRI, Wockhardt Hospital

xvii

Foreword 1

I am delighted to write a foreword for this book, prepared by a group of enthusiastic and committed physicians who have dedicated their careers to the practice of diagnostic neuroradiology. All four authors participated in and successfully passed the two-year Pierre Lasjaunias European Course of Neuroradiology, in which I had the pleasure and honor to be a faculty member and past-Director. Energized by a mutual passion for honing their diagnostic skills, they set about building a collective repository of definitive diagnostic neuroradiology case studies. The fruit of their thirst for knowledge, this book harmoniously unites a unique collection of atypical neuropathologic case studies in a methodical, readily accessible manner. Framed as “Diagnosis Please” format of the Radiology journal, each case study presents a brief clinical history, relevant laboratory results, and supporting diagnostic images. The reader is then invited to diagnose the case based on the clinical and imaging information provided. This interactive format stimulates the reader to actively ponder over the data and work out the appropriate diagnosis, while simultaneously and systematically considering potential differential diagnoses.

xviii

The cases conclude with a thorough discussion of the definitive primary diagnosis in light of the analytical, clinical, and imaging findings, effectively eliminating the less likely differential possibilities. Thus, in a concise, inviting manner, the book provides readers with an essential basis for day-to-day use to develop and strengthen their diagnostic neuroradiology skills. This volume will serve as a valuable resource for neurology and radiology trainees preparing for advanced certification, a reference for established neuroradiologists, and a guide for neurologists in search of readily available examples of atypical neuropathologies. The authors should be congratulated on their earnest wish to share their desire for excellence in practicing neuroradiology. Àlex Rovira, MD Head of the Neuroradiology Section, and Director of the MR Unit. Vall d’Hebron University Hospital, Barcelona, Spain Director of the X and XI Cycles of the European Course of Neuroradiology Vice President of the European Society of Neuroradiology

Foreword 2

As editor-in-chief of American Journal of Neuroradiology and someone who has being practicing neuroradiology for over 25 years, I am well aware of the paucity of relevant and easily accessible information on rare neurologic cases amongst today’s academic publications. I also understand the need for practicing neuroradiologists to remain abreast of current trends and practices in order to accurately diagnose complicated patient pathologies. The electronic age and information highway have certainly greatly affected academic and clinical medicine by creating vast amounts of information. Paradoxically this sheer volume of information can be an impediment for a busy clinician in search of a rare neurologic finding, an unusual presentation, or confounding patient history. Recognizing this difficulty in searching for information on atypical neurologic cases, the authors of this book have created a compendium of cases featuring rare, adult neurologic brain diseases. This task was a result of a desire to extricate illusive nuggets of pertinent information from the ever-increasing amount found elsewhere. Targeting neuroradiologists and other specialists alike, this text incorporates relevant patient information and laboratory values accompanied by typical imaging findings which enable one to offer a diagnosis. Although I agree that multiple pathologies found herein may rarely be encountered, this does not detract from having this book close by as a handy reference.

This book seeks to incorporate recent advances in neuroradiology but also serves as a resource on seldom-encountered neurologic pathologies. Merging these two aspects of neuroradiology, innovations and advances in imaging modalities/ techniques with representative samples of atypical patient cases, into one text should be welcomed by those who practice neuroradiology. The authors sought to include representative cases from all major categories of rare neurologic pathologies ranging from degenerative and infectious diseases, to metabolic diseases and tumors. This text will be an added value to those seeking to diagnose an atypical case or preparing for examinations. The authors have collated a large number of atypical cases that will prove helpful when offering a differential diagnosis based on unusual diagnostic and imaging findings. I would like to congratulate the authors for creating a resource that unifies rare cases in one reference that will readily enable us to increase our knowledge of atypical neurologic pathologies seen in the brain. Mauricio Castillo, MD, FACR Professor of Radiology, Section Chief Neuroradiology University of North Carolina School of Medicine Editor-in-Chief, American Journal of Neuroradiology

xix

Preface

Neuroradiology is a fast-paced subspecialty. The expanse of worldwide knowledge and global electronic accessibility to journals, databases, and search engines constitutes a daunting wealth of information. However, this vast oasis of information does not readily yield easily accessible information on atypical neuroradiologic case presentations. If anything, the magnitude of available data may impede acquisition of information with any discernibly diagnostic value when searching for unusual cases and atypical presentations. Traditional data mining or exhaustive literature searches are out of bounds for most practicing neuroradiologists who often find themselves overwhelmed with voluminous caseloads, unable to spare extended periods of time to look up a particular case. This is further complicated by the fact that rare cases find no place in current mainstream radiology journals that are more focused on achieving and maintaining a high impact factor rather than providing practical information to clinical neuroradiologists. Thus, finding consultative or reference materials becomes extremely arduous for a busy radiologist. The introduction of numerous speciality journals and influx of open access journals have added to the cryptic informational overload for clinicians seeking material on unusual case presentations. The significance of our text lies within the historic origins of neuroradiology as a subspecialty which began, unquestionably, before visual images of the brain were conceivable – or obtainable. A series of benchmarks built one upon another laid the foundation and therefore generated the imperative need to obtain visual access to the human brain, or as William Macewen aptly described it, “. . .the dark continent” [1]. Subsequent fine-tuning and advancement of radiographic technologies enable and sustain exploration of neurologic disease with such

a rapid pace that practicing neuroradiologists stagger to keep abreast of these tidal waves. The purpose of our book is to create a readily accessible and reliable repository of rare cases that can then be confidently diagnosed by informed assessment of characteristic imaging and clinical findings. The intent is that this text will challenge a reader’s knowledge base and enlighten him or her about the characteristic imaging findings of central nervous system (CNS) pathology. Thus, the reader will be able to then confidently make the same diagnosis in real time, when seen in a clinical setting. In hope to retain the information, the book is formatted as unknown quiz cases with clinical and imaging information in the front facing page (the recto) and the case discussion in the back facing page (the verso). Although this book is targeted towards the full-time neuroradiology professionals, it will benefit any physician who is involved with the care of a patient with a rare CNS disorder. Whether the physician is a general radiologist practicing in a community hospital, an academic neuroradiologist, or a practicing neurologist – they will find value in this text that serves as a readily available guide. Lázaro Luís Faria do Amaral, MD Asim K. Bag, MD Fabrício Guimarães Gonçalves, MD Prasad B. Hanagandi, MD

Reference 1.

Macewen W. Intra-cranial lesions: illustrating some points in connexion with the localisation of cerebral affections and the advantages of antiseptic trephining. Lancet 1881; 118(3031): 581–3.

xxi

Acknowledgements

This book is a celebration of exceptional friendship, sharing, patience, and commitment. The preparation of this casebook was a project of several years and could not have been concluded successfully without the support and collaboration of many people. In addition to the people who grace the pages of this book, there are many others needing noteworthy mention. First, we want to thank Suzanne Byan-Parker, whose orchestrated efforts made this book possible. She has been heavily involved in the project from early on. She spent hours in coordinating, editing, and compiling the manuscript. We find no appropriate words to thank her. We simply want to say, “Thank you Suzanne for everything you have done with this project.” The editors also want to thank their family members for being so supportive and for encouraging them to finish this project that needed countless hours away from their families. LA, PH, and FG also want to appreciate AB for his leadership role in this project without which it would not have been possible to finalize this process. We also want to thank all the contributors for their gracious help.

xxii

LA wants to thank to his colleagues: Sérgio Santos Lima, Christiane Monteiro Siqueira Campos, Victor Hugo Rocha Marussi, Bruno Siqueira Campos Lopes, Lucas Ávila Lessa, Leonardo Furtado Freitas, and Anderson A. Belezia, Neuroradiologists staff at MEDIMAGEM, Hospital Beneficência Portuguesa de São Paulo, São Paulo, Brazil, for their continuous support during this effort. FG wishes to thank personally the following people for their contributions to his inspiration and knowledge and other help in creating this book: Raquel del Carpio, MD, Professor and Vice-Chair of Radiology at McGill University and Jeffrey Chankowsky, MD, CM, FRCP(C), Associate Professor; Program Director (Residency Program) McGill University, Montreal. All the editors also acknowledge the continuous friendly and encouraging support of the editorial staff of Cambridge University Press. Lázaro Luís Faria do Amaral, MD, Asim K. Bag, MD, Prasad B. Hanagandi, MD, Fabrício Guimarães Gonçalves, MD

How to Use This Book

The patient cases that follow feature pathologies that may present unique challenges to a practicing neuroradiologist, or radiologist. Drawn from both academic and private practice, the samples encompass a range of pathologies categorized as neurodegenerative diseases, neuroinfectious, metabolic disorders and more that specifically target the central nervous system, central nervous system tumors, and a myriad of miscellaneous cases. The chapters are organized according to overarching extraneurologic pathologic involvement. Each case correlates with the specific underlying etiology, so to speak, that best represents the pathology or pathophysiology related to the thematic category of one of eight chapters. This framework allows the reader to associate each case discussed with a correlative pathologic or physiologic category of the underlying etiology. Spanning two continents, this collection of cases represents relatively rare pathologic entities not seen in everyday radiology practice. The text was designed to be used as a guide, reference, and study companion that houses atypical patient cases with representative imaging and correlative imaging findings. Each case section represents an aspect of a physician-patient encounter beginning with initial presentation, diagnostic procedures, consolidation, reporting, diagnosis, and clinical decision-making. The cases are divided into sections designed to introduce the reader to the patient, observe pertinent clinical and radiologic findings, and engage in the diagnostic thought process in order to arrive at the most likely diagnosis based on the data presented. Optimally, the reader should be able to identify the most likely primary diagnosis after close reading of the clinical presentation and examining the accompanying images. Clinical Presentation. Includes the essential patient findings at presentation in a clinical context in order to prompt the

reader to begin patient assessment and determine what optimal diagnostic imaging modalities would best ascertain the underlying diagnosis. Imaging. Images are presented with figure legends in order to complete the clinical presentation. Primary Diagnosis. Primary diagnosis based on findings, evidence, clinical decision-making, and outcome after careful consideration of the clinical presentation and imaging studies; the most likely diagnosis should be evident. Differential Diagnoses. A list of the most pertinent differential diagnoses based on the clinical findings and the images featured with each case. The reader should be able to explain why the featured list of differential diagnoses is excluded based on the clinical and imaging studies presented in each case. Imaging Findings. Images are presented with figure legends describing the important findings, which include functional and anatomic characteristics, striking features, and notation of usual and/or pathologic landmark findings of each image. After examining the images, in conjunction with the clinical presentation, the reader should be able to develop and articulate a basis for discussing the primary diagnosis and eliminating differential diagnoses. Discussion. Explains the basis for the primary diagnosis based on the clinical, laboratory, and imaging findings. The discussion offers the reader an opportunity to engage in clinical consultation as though reporting on patient findings during a consult in order to arrive at the most likely diagnosis based on imaging findings and characteristic presentations of each case. Key Points. A summary of combined clinical and imaging features from each case. Suggested Reading. A list of source and reference material that will provide the reader with additional information pertinent to each pathologic entity.

xxiii

Neurodegenerative Diseases

CASE

Part I

1

Asim K. Bag, Lázaro Luís Faria do Amaral

Clinical Presentation A 31-year-old woman presented with a three-year history of gradually worsening bradykinesia, rigidity, tremor, dystonia, dysphagia, and severe neurocognitive decline. Her symptoms were not levodopa-responsive. Her family states that she had severe developmental delay in early childhood, along with mental retardation. Although the mental retardation was static for an extended period, the family noted history of recent neurocognitive decline. Magnetic resonance imaging was performed (shown below) for further follow-up.

Imaging (A)

(A)

(B)

(B)

Fig. 1.1 (A) Axial T2WI and (B) T1WI through the level of the basal ganglia. Taken with permission from: Amaral LLF, Gaddikeri S, Chapman PR, Roy R, Gaddikeri RS, Marussi VH, Bag AK. Neurodegeneration with brain iron accumulation: clinicoradiological approach to diagnosis. J Neuroimaging 2015; 25(4): 539–51.

Fig. 1.2 (A) Axial T2WI and (B) T1WI through the level of the substantia nigra. Taken with permission from: Amaral LLF, Gaddikeri S, Chapman PR, Roy R, Gaddikeri RS, Marussi VH, Bag AK. Neurodegeneration with brain iron accumulation: clinicoradiological approach to diagnosis. J Neuroimaging 2015; 25(4): 539–51.

Advanced Neuroradiology Cases, ed. Amaral et al. Published by Cambridge University Press. © Cambridge University Press 2016.

1

Part I. Neurodegenerative Diseases: Case 1

Beta-Propeller Protein-Associated Neurodegeneration (BPAN) Primary Diagnosis Beta-propeller protein-associated neurodegeneration (BPAN)

Differential Diagnoses Pantothenate kinase-associated neurodegeneration (PKAN) Phospholipase-associated neurodegeneration (PLAN) Mitochondrial membrane protein-associated neurodegeneration (MPAN) Early-onset Parkinson disease

Imaging Findings Fig. 1.1: (A) Axial T2-weighted MR image through the level of basal ganglia demonstrated T2 hypointensity secondary to iron deposition involving bilateral globus pallidi (arrows) and mild diffuse brain atrophy as suggested by prominence of ventricles and sulci. (B) Axial T1-weighted MR image through the same level demonstrated subtle, increased T1 signal in the globus pallidi (arrows). Fig. 1.2: (A) Axial T2-weighted MR image demonstrated evidence of increased iron deposition involving bilateral substantia nigra (arrows). (B) Axial T1-weighted MR image through the level of substantia nigra demonstrated bilateral symmetric, high T1 signal involving the substantia nigra (black arrows), with a band of central T1 hypointensity (white arrowhead), virtually pathognomonic of BPAN.

Discussion Global developmental delay, early childhood neurocognitive decline that remains static for years, juvenile or young-adult onset of worsening of neurocognitive changes, and the development of progressive, levodopa-resistant Parkinsonian features suggest the diagnosis of beta-propeller protein-associated neurodegeneration (BPAN). Virtually pathognomonic MRI findings in the substantia nigra (bilateral symmetric, high T1 signal involving substantia nigra with a band of central T1 hypointensity) confirm the diagnosis of BPAN. Late-onset PKAN (see Part I: Case 4) may be considered in the differential diagnosis; however, typical eye-of-the-tiger sign in the globus pallidus is absent. Moreover, the T1 hyperintensity of the subthalamic nucleus (STN) is not characteristic of PKAN. PLAN (see Part I: Case 5) is a disease of early childhood, and is characterized by more severe symptoms, most notably cerebellar atrophy. Most patients with PLAN succumb to the disease by 10 years of age. The characteristic T2 hyperintensity of the medial medullary lamina, between the globus pallidus interna and externa, the characteristic imaging finding in MPAN (see Part I: Case 2) is not present in this patient. Although symptoms complex is suggestive of Parkinson disease, levodopa-nonresponsiveness and evidence of excessive deep nuclei iron accumulation are not features of early-onset Parkinson disease.

2

Recently defined, BPAN is the only X-linked subtype of neurodegeneration with brain iron accumulation (NBIA) due to a mutation in the WDR45 gene at the Xp11.23 locus. In the past, this entity was named static encephalopathy (of childhood) with neurodegeneration in adulthood (SENDA). After the underlying genetic defect was identified, this NBIA subtype was renamed or reclassified as BPAN, similar to the other subtypes of NBIA. In BPAN, patients present with global developmental delay, including delayed language and motor skills. Unlike other subtypes of early manifested NBIA, the clinical presentation of delayed development remains relatively static until adolescence/young adulthood. Abnormal movement or neurocognitive decline is absent in childhood. As the patient approaches adolescence and early adulthood, progressive neurodegeneration symptoms including dystonia, Parkinsonian syndromes, and neurocognitive decline start to appear. Although the benefit is only short lasting, Parkinsonian symptoms may improve with initiation of levodopa therapy. This characteristic pattern of disease progression, from childhood to young adulthood, was the basis for the previous name, SENDA. As in other subtypes of NBIA, iron deposition is the key imaging abnormality. The earliest and most concentrated iron accumulation occurs in the substantia nigra. Although iron deposition does occur in the globus pallidus, it usually follows iron deposition in the substantia nigra and is less severe, unlike the other NBIA subtypes. Evidence of iron deposition can only be seen in the STN on iron-sensitive sequences, as disease progression occurs when there may not be any apparent iron deposition in the globus pallidus. Paired bands of linear T1 hyperintensity separated by a relatively T1 hypointense area is a distinguishing and virtually pathognomonic finding in patients with BPAN. The bright T1 signal may be due to formation of iron-melanin complexes from the release of neuromelanin in the dying pigmented neurons of the STN pars compacta.

Key Points  BPAN should be clinically suspected when the onset of global developmental abnormalities is early in childhood and remains relatively stable until adolescence before it deteriorates and with the development of seizures and stereotypes.  Bilateral symmetric, high T1 signal involving substantia nigra with a band of central T1 hypointensity is a virtually pathognomonic imaging sign of BPAN.  If characteristic clinical and radiologic findings are present, the diagnosis can be confirmed by molecular genetic analysis.

Suggested Reading Amaral LLF, Gaddikeri S, Chapman PR, et al. Neurodegeneration with brain iron accumulation: clinicoradiological approach to diagnosis. J Neuroimaging 2015; 25(4): 539–51. Hayflick SJ, Kruer MC, Gregory A, et al. Beta-propeller proteinassociated neurodegeneration: a new X-linked dominant disorder with brain iron accumulation. Brain 2013; 136(Pt 6): 1708–17.

Neurodegenerative Diseases

CASE

Part I

2

Aparna Singhal, Lázaro Luís Faria do Amaral, Asim K. Bag

Clinical Presentation An 11-year-old boy presented with a history of relatively rapidly progressive cognitive decline and motor neuropathy. This was accompanied by gradual development of extrapyramidal signs, dystonia, visual abnormalities, and Parkinsonian features. On further questioning, his parents revealed that their other child also had similar symptoms. Extensive hematologic studies were negative. Ophthalmologic evaluation revealed optic atrophy but no features of pigmentary retinal degeneration.

Imaging (A)

(B)

(C)

Fig. 2.1 (A) Axial T2WI and (B) Coronal T2WI through the level of basal ganglia. (C) Axial T2WI through the posterior fossa. Taken with permission from: Amaral LLF, Gaddikeri S, Chapman PR, Roy R, Gaddikeri RS, Marussi VH, Bag AK. Neurodegeneration with brain iron accumulation: clinicoradiological approach to diagnosis. J Neuroimaging 2015; 25(4): 539–51.

Advanced Neuroradiology Cases, ed. Amaral et al. Published by Cambridge University Press. © Cambridge University Press 2016.

3

Part I. Neurodegenerative Diseases: Case 2

Mitochondrial Membrane Protein-Associated Neurodegeneration (MPAN) Primary Diagnosis Mitochondrial membrane protein-associated neurodegeneration (MPAN)

Differential Diagnoses Pantothenate kinase-associated neurodegeneration (PKAN) Phospholipase-associated neurodegeneration (PLAN) Beta-propeller protein-associated neurodegeneration (BPAN) Early-onset Parkinson disease

Imaging Findings Fig. 2.1: (A) Axial T2WI through the level of basal ganglia demonstrated diffuse T2 hypointensity involving the bilateral globus pallidi, and diffuse brain atrophy. Subtle linear T2 hyperintensity (arrowhead) was seen in the medial medullary lamina between the globus pallidus interna and globus pallidus externa. (B) Coronal T2WI through the basal ganglia demonstrated T2 hypointensity involving the bilateral basal ganglia. T2 hyperintensity involving the medial medullary lamina (arrowheads) is more clearly visualized on both sides compared to the axial images. (C) Axial T2WI through the posterior fossa demonstrated significant atrophy of the cerebellar hemispheres as evinced by prominence of sulci.

Discussion Genetic analysis demonstrated a mutation of the C19orf12 gene, confirming the diagnosis of mitochondrial membrane protein-associated neurodegeneration (MPAN). The early onset of gradually worsening neurologic symptoms is suggestive of a familial neurodegenerative disorder; this is confirmed by the presence of brain atrophy on imaging. Increased T2 hypointensity on the globus pallidus is suggestive of inappropriate iron accumulation and raises the concern for neurodegeneration associated with brain iron accumulation (NBIA). Linear increased T2 signal in the medial medullary lamina is suggestive of MPAN, a subtype of NBIA. BPAN (see Part I: Case 1) is excluded because of the absence of typical T1 hyperintensity in the basal ganglia and in the substantia nigra. Patients with PLAN (see Part I: Case 5) typically present very early in life, usually before 10 years of age. Inappropriate brain iron deposition is a non-specific finding and can be seen as a manifestation of a neurodegenerative disease or normal aging. Presentation in early childhood excludes many neurodegenerative diseases such as Parkinson disease, Parkinson-plus syndromes, and Alzheimer disease. Neuroferritinopathy (see Part I: Case 3) and aceruloplasminemia (see Part I: Case 6), NBIA subtypes that typically manifest in older age groups, were also excluded, as there were no typical imaging findings and the other blood work did not demonstrate low ferritin or ceruloplasmin, respectively. The clinical presentation of this patient overlaps with

4

symptoms of PKAN (see Part I: Case 4). The eye-of-the-tiger sign, the characteristic imaging manifestation of PKAN, was not present on imaging. No features suggestive of pigmentary retinal degeneration, a frequent manifestation of PKAN, were noted. A recently described phenotype, MPAN may account for approximately 5% of all NBIA subtypes. In Online Mendelian Inheritance in Man® (OMIM®, http://www.omim.org), MPAN is classified as NBIA type 4 (OMIM #614298). It is an autosomal recessive disorder caused by mutation in the C19orf12 gene, the product of which is a mitochondrial membrane protein. This mutation causes mitochondrial dysfunction and altered lipid metabolism. As of 2013, 67 cases have been reported. The mean age of presentation is 11 years, with a range of 4–30 years. Progression of cognitive decline to dementia is almost a universal finding of MPAN. Motor neuropathy is another frequent manifestation that typically starts with upper motor signs followed by lower motor signs. Other clinical presentations include slowly progressive gait disorder from generalized dystonia and spastic paraparesis, cognitive decline, neuropsychiatric abnormalities, optic atrophy, and motor axonal neuropathy. In contrast to PKAN, MPAN is associated with optic atrophy, not pigmentary degeneration of retina. The onset of symptoms in MPAN is later in childhood with more gradual psychomotor regression, as compared to PKAN. However, cognitive decline progressing to severe dementia is more commonly associated with MPAN, as compared to PKAN. Magnetic resonance imaging shows T2 hypointensity from iron accumulation in the globus pallidus and in the substantia nigra, in the vast majority of patients. T2 hyperintense streaking of the medial medullary lamina between the globus pallidus interna and externa is a characteristic finding, though not present in all cases. If present, this may discriminate MPAN from other NBIA subtypes. The eye-of-the-tiger sign is absent, distinguishing this NBIA subtype from the more common PKAN. The presence of nigral T2 hypointensity is similar to features seen in patients with another NBIA subtype, PLA2G6associated neurodegeneration (PLAN), who may also show optic atrophy. However, PLAN patients show cerebellar atrophy, which is not always seen in MPAN (although it is seen on the given case). Magnetic resonance imaging of MPAN patients demonstrates additional T1 hyperintensity of the caudate nucleus and putamen, not seen in PLAN patients. On pathology, neuronal loss, iron deposits, axonal spheroids, Lewy bodies, and hyperphosphorylated tau-containing inclusions are seen.

Key Points  MPAN should be suspected if presentation includes relatively rapidly progressing cognitive decline in older children with evidence of excessive iron deposition in the basal ganglia.  Increased T2 signal in the medial medullary lamina between the globus pallidus interna and globus pallidus

Part I. Neurodegenerative Diseases: Case 2

externa is a more specific, but less sensitive imaging finding of MPAN.  If characteristic clinical and radiologic findings are present, the diagnosis should be confirmed by molecular, genetic analysis.

Suggested Reading Hartig M, Prokisch H, Meitinger T, Klopstock T. Mitochondrial membrane protein-associated neurodegeneration (MPAN). Int Rev Neurobiol 2013; 110: 73–84.

Hogarth P, Gregory A, Kruer MC, et al. New NBIA subtype: genetic, clinical, pathologic, and radiographic features of MPAN. Neurology 2013; 80(3): 268–75. Kniffin CL. Neurodegeneration with Brain Iron Accumulation 4; NBIA 4. Baltimore, MD: Johns Hopkins University; 2011 [updated 11/12/2013; cited 2014]. Available from: http://omim.org/entry/ 614298. Schulte EC, Claussen MC, Jochim A, et al. Mitochondrial membrane protein associated neurodegeneration: a novel variant of neurodegeneration with brain iron accumulation. Mov Disord 2013; 28(2): 224–7.

5

Neurodegenerative Diseases

CASE

Part I

3

Asim K. Bag, Victor Hugo Rocha Marussi, Lázaro Luís Faria do Amaral

Clinical Presentation A 35-year-old man presented with dysarthria and gradually worsening gait and ataxia. His medical history included long history of hand tremor. Neurologic examination revealed cerebellar symptoms including dysmetria, hypotonia, micrographia, and

mild cognitive impairment. Routine hematologic examinations including serum chemistries and hematologic profile were normal. Further testing demonstrated low serum ferritin levels. Cerebrospinal fluid examinations were within normal range. Magnetic resonance of the brain was performed and is shown below.

Imaging (A)

(A)

(B)

(B)

Fig. 3.1 (A) Axial T2WI through the basal ganglia. (B) Axial GRE image through the same level. Taken with permission from: Amaral LLF, Gaddikeri S, Chapman PR, Roy R, Gaddikeri RS, Marussi VH, Bag AK. Neurodegeneration with brain iron accumulation: clinicoradiological approach to diagnosis. J Neuroimaging 2015; 25(4): 539–51.

Fig. 3.2 (A) Axial T2WI. (B) GRE sequence through the substantia nigra. Taken with permission from: Amaral LLF, Gaddikeri S, Chapman PR, Roy R, Gaddikeri RS, Marussi VH, Bag AK. Neurodegeneration with brain iron accumulation: clinicoradiological approach to diagnosis. J Neuroimaging 2015; 25(4): 539–51.

Advanced Neuroradiology Cases, ed. Amaral et al. Published by Cambridge University Press. © Cambridge University Press 2016.

7

Part I. Neurodegenerative Diseases: Case 3

Neuroferritinopathy Primary Diagnosis Neuroferritinopathy

Differential Diagnoses Aceruloplasminemia Pantothenate kinase-associated neurodegeneration (PKAN) Wilson disease Leigh syndrome Enlarged perivascular space

Imaging Findings Fig. 3.1: (A) Axial T2WI through the basal ganglia demonstrated bilateral, almost symmetric, cystic degeneration of the globus pallidus (arrows) and putamen. A subtle T2 hypointense rim was noted at the anterior aspect of the globus pallidus (arrowheads). Also significant brain atrophy was noted as evidence by bilateral enlargement of sylvian fissures. (B) Axial gradient echo image through the same level demonstrated more obvious peripheral hypointensity on the lateral aspect (arrowheads) of the globus pallidus (arrows) due to abnormal iron deposition. Fig. 3.2: (A) Axial T2WI and (B) GRE sequence through the substantia nigra demonstrated no evidence of abnormal iron deposition in the substantia nigra (arrows).

Discussion In an adult patient, the presence of cystic degeneration of the bilateral basal ganglia with evidence of abnormal iron deposition accompanied by progressively worsening of abnormal movement, cognitive disorders, and low serum ferritin are diagnostic of neuroferritinopathy. Aceruloplasminemia (ACP) (see Part I: Case 6) is a subtype of neurodegeneration with brain iron accumulation (NBIA) presenting in the adult population. In ACP, there is more prominent signal abnormality involving the deep gray matter nuclei without any evidence of cystic degeneration. Patients with ACP develop diabetes mellitus and retinal degeneration. Additionally, ACP is characterized by low serum ceruloplasmin and high serum ferritin level. PKAN (see Part I: Case 4), usually a disease of childhood, can also present in early adulthood. Presentation of PKAN in the fourth decade, as in this patient, is extremely rare. In addition, intense, abnormal pallidal iron deposition, with a central area of rarefaction evidenced by a T2 hyperintensity (eye-of-the-tiger sign), is the typical imaging presentation of PKAN, not complete cystic pallidal degeneration as seen in neuroferritinopathy (NFT).

8

The presence of abnormal T2 hyperintensity in the basal ganglia can also be seen in Leigh syndrome, but Leigh syndrome typically manifests in the first two years of life with ophthalmoplegia, cranial nerve palsy, and respiratory failure. In addition, there is high brain and CSF lactate levels on MR spectroscopy. Although bilateral basal ganglia T2 hyperintensity can be present in Wilson disease, Kayser-Fleischer ring in bilateral iris and hepatic failure are additional clinical findings that were absent in our patient. Though enlarged perivascular space can have a large cyst-like appearance, on occasion, similar to NFT, it is not associated with peripheral T2/GRE hypointensity due to excessive iron accumulation. Neuroferritinopathy is one of the many subtypes of NBIA that is transmitted as an autosomal dominant trait and is due to mutation of the ferritin light chain gene on chromosome 19q. Mutated ferritin light chains lose their capacity for iron storage and metabolism, thus free iron is released within affected cells. This free iron can induce oxidative stress and subsequent neuronal degeneration, particularly in basal ganglia. This also explains characteristic low serum ferritin level. Unlike many other NBIA subtypes, NFT presents later in life, usually after 30 years of age. Extrapyramidal symptoms such as choreoathetosis, dystonia, and Parkinsonian symptoms usually predominate. Palatal myoclonus and orolingual dyskinesias ataxia can also be presenting symptoms. Neurocognitive decline is usually a late finding. Neuroferritinopathy has distinctive imaging findings including bilateral, almost symmetric, cystic degeneration involving the bilateral globus pallidus, substantia nigra, and deep cerebellar nuclei. Areas of cystic degeneration are lined by hypointense rim due to excess iron deposition. Mild to moderate brain atrophy is frequently associated with cystic degeneration.

Key Points  Neuroferritinopathy should be suspected in adult patients with early-onset Parkinsonian syndrome and cystic pallidal degeneration surrounded by hypointense rim on GRE and/or T2WI.  Low serum ferritin level is characteristic.  If characteristic clinical and radiologic findings are present, the diagnosis is almost certain and should be confirmed by genetic analysis.

Suggested Reading Amaral LLF, Gaddikeri S, Chapman PR, et al. Neurodegeneration with brain iron accumulation: clinicoradiological approach to diagnosis. J Neuroimaging 2015; 25(4): 539–51. Fatima Z, Ishigame K, Araki T. Case 193: Neuroferritinopathy–a brain iron accumulation and neurodegenerative disorder. Radiology 2013; 267(2): 650–5.

.

Neurodegenerative Diseases

CASE

Part I

4

Aparna Singhal, Lázaro Luís Faria do Amaral, Asim K. Bag

Clinical Presentation A five-year-old boy presented with history of gradual development of postural and gait abnormalities, pyramidal and extrapyramidal symptoms, oromandibular dystonia, and neurocognitive decline. On neurologic evaluation, profound oromandibular dystonia was noted. Ophthalmologic evaluation revealed night blindness and visual field constriction, but no feature of optic atrophy. Hematologic studies did not reveal any significant abnormality. An MRI was performed with a 1.5T magnet.

Imaging (A)

(B)

(C)

Fig. 4.1 (A) Axial FLAIR and (B) T2WI and (C) GRE images through the globus pallidi. Taken with permission from: Amaral LLF, Gaddikeri S, Chapman PR, Roy R, Gaddikeri RS, Marussi VH, Bag AK. Neurodegeneration with brain iron accumulation: clinicoradiological approach to diagnosis. J Neuroimaging 2015; 25(4): 539–51.

Advanced Neuroradiology Cases, ed. Amaral et al. Published by Cambridge University Press. © Cambridge University Press 2016.

9

Part I. Neurodegenerative Diseases: Case 4

Pantothenate Kinase-Associated Neurodegeneration (PKAN) Primary Diagnosis Pantothenate kinase-associated neurodegeneration (PKAN)

Differential Diagnoses Other neurodegeneration with brain iron accumulation (NBIA) subtypes Carbon monoxide poisoning Metabolic abnormality

Imaging Findings Fig. 4.1: (A) Axial FLAIR MR image demonstrated increased T2 signal in the center of the globus pallidi (arrows). (B) Axial T2WI through the same level demonstrated increased T2 signal in the center of the globus pallidi (arrows). (C) Axial GRE image through the same level demonstrated hypointensity, secondary to iron deposition, in the posterior and lateral aspect of globus pallidi (arrows). Please note there is also no associated brain atrophy or abnormal T2 signal in the white matter.

Discussion The clinical presentation is classic for typical PKAN. The disease manifests in children between three and five years of age, with gradually progressing gait abnormalities and pyramidal-extrapyramidal symptoms. Presence of profound oromandibular dystonia, night blindness, and visual field constriction further support the diagnosis of PKAN. Evidence of excessive iron accumulation in the globus pallidi with central area of rarefaction, absence of white matter disease, and presence of brain or cerebellar atrophy confirms the diagnosis of PKAN. The eye-of-the-tiger sign, demonstrated in this case, is characteristic, but not pathognomonic for PKAN. This sign can be seen in neuroferritinopathy (see Part I: Case 3), corticobasal degeneration (see Part I: Case 10), and multiple system atrophy (see Part I: Case 8). It can also be an imaging manifestation of normal aging. All of these diseases typically affect older patients, not children. Although there may be overlap of clinical symptoms between other NBIA subtypes such as mitochondrial membrane protein-associated neurodegeneration (MPAN) and phospholipase-associated neurodegeneration (PLAN), characteristic eye-of the-tiger sign is typically absent in these subtypes. Carbon monoxide poisoning can present as T2 hyperintensities in the globus pallidus, without surrounding T2 hypointensity related to excessive iron accumulation. In addition, other brain structures are often involved in carbon monoxide poisoning, such as the putamen, thalamus, and peripheral cortex. PKAN is an autosomal recessive disorder due to mutations in the PANK2 (pantothenate kinase 2) gene located on chromosome 20p13, with an estimated prevalence of 1:1,000,000, accounting for approximately half of all NBIA cases. In Online Mendelian Inheritance in Man® (OMIM®, http://www.omim .org) PKAN is described as NBIA type 1 (OMIM #234200).

10

There are two types: typical and atypical. In the typical variant, onset occurs before six years of age in almost 90% of patients, typically with gait difficulty as the presenting symptom. There are pyramidal features (spasticity, hyperreflexia, and extensor plantar toe response) and extrapyramidal features (prominent dystonia) often with typical predominant orolingual-mandibular involvement. Other extrapyramidal features such as Parkinsonism, chorea, and neuropsychiatric features including attention deficit hyperactivity disorder, cognitive decline, and behavioral changes may be seen. Oculomotor abnormalities are common, seen partly because of midbrain degeneration. Around 70% of patients have electroretinographic evidence of retinopathy. Unlike PLAN and MPAN, optic atrophy is typically absent in PKAN. Typical PKAN has a rapidly progressive course with affected children usually becoming wheelchair-bound within a few years of disease onset. Unlike typical PKAN, atypical PKAN is classically an adultonset disease with age of onset between 20 and 30 years of age. Patients with atypical PKAN demonstrate less severe motor involvement as compared to the typical form and have frequently been reported to present with speech difficulties and atypical phenotypical features such as focal dystonia, less prominent extrapyramidal features, retinopathy, predominant cognitive decline, and psychiatric features. On MRI, there is typical iron accumulation in the anteromedial part of the globus pallidus with extension into the genu of the internal capsule that is manifested as prominent T2 hypointensity. The presence of a central hyperintensity on the T2WI sequence within the surrounding area of hypointensity is called the eye-of-the-tiger sign and is characteristic of PKAN. It has been suggested that the hyperintense central pallidum indicates a primary tissue insult leading to neuronal loss, gliosis, and cavitation of the neuropil. The surrounding hypointense region indicates high iron deposition. It is unclear if the iron deposition plays a primary or secondary effect in PKAN. Iron deposition in the subthalamic nucleus and substantia nigra, in addition to globus pallidi has also been described. If present very early in life, bilateral hyperechogenicity in the substantia nigra and lenticular nucleus can be seen on transcranial sonography. Pathologically, iron accumulation is seen as rustbrown pigmentation in the globus pallidus grossly and microscopically; iron is mostly in the ferric form, in a perivascular distribution in the microglia and macrophages. The exact pathophysiology of PKAN is poorly understood. The PANK2-encoded enzyme is key in coenzyme A synthesis, which is essential for fatty acid synthesis. Dysfunction of PANK2, thus, likely causes alteration in lipid metabolism. PANK2 is mainly targeted to mitochondria; its mutation may therefore also cause dysfunction of cellular energy metabolism. Null mutations of the PANK2 gene result in complete absence of the enzyme and are more commonly seen in early-onset typical PKAN with rapidly progressive disease; whereas missense mutations result in partial loss of enzymatic function and are more commonly seen in late-onset, atypical PKAN with more slowly progressive disease. No drugs are currently available that can halt neurodegenerative progression in

Part I. Neurodegenerative Diseases: Case 4

patients with PKAN. Patients are treated symptomatically with baclofen, levodopa, anticholinergics, and tetrabenazine. Deep brain stimulation is also helpful in controlling symptoms.

Key Points  PKAN should strongly be considered when the typical presentations including gradually worsening gait and motor control, visual field constriction, night-blindness, neurocognitive decline, pyramidal-extrapyramidal symptoms (including oromandibular dystonia and tongue protrusion) are seen in a patient of < 6 years of age.  Presence of eye-of-the-tiger sign in a patient with the abovementioned clinical findings is consistent with the diagnosis of PKAN that can be further confirmed by genetic analysis.

Suggested Reading Amaral LLF, Gaddikeri S, Chapman PR, et al. Neurodegeneration with brain iron accumulation: clinicoradiological approach to diagnosis. J Neuroimaging 2015; 25(4): 539–51. Gregory A, Polster BJ, Hayflick SJ. Clinical and genetic delineation of neurodegeneration with brain iron accumulation. J Med Genet 2009; 46(2): 73–80. Schneider SA, Dusek P, Hardy J, et al. Genetics and pathophysiology of neurodegeneration with brain iron accumulation (NBIA). Curr Neuropharmacol 2013; 11(1): 59–79. Schneider SA, Hardy J, Bhatia KP. Syndromes of neurodegeneration with brain iron accumulation (NBIA): an update on clinical presentations, histological and genetic underpinnings, and treatment considerations. Mov Disord 2012; 27(1): 42–53.

11

Neurodegenerative Diseases

CASE

Part I

5

Asim K. Bag, Lázaro Luís Faria do Amaral

Clinical Presentation An 18-month-old girl presented with a 1-month history of spastic quadriplegia, and recent history of language and motor skill regression. Her parents noted that although she was born healthy, over the past months, she had developed

severe muscle weakness and it had become increasingly difficult to feed her. They also noted that she had developed visual problems accompanied by jerky eye movements. Magnetic resonance imaging was performed and results are noted below.

Imaging (A)

(B)

(C)

(E) (D)

Fig. 5.1 (A) Axial T2WI through the level of the basal ganglia. (B) Axial GRE through the same level. (C) Midline sagittal T1WI. (D) Coronal T2WI through the cerebellum. (E) Coronal MR image through the level of the sella. Taken with permission from Amaral LLF, Gaddikeri S, Chapman PR, Roy R, Gaddikeri RS, Marussi VH, Bag AK. Neurodegeneration with brain iron accumulation: clinicoradiological approach to diagnosis. J Neuroimaging 2015; 25(4): 539–51.

Advanced Neuroradiology Cases, ed. Amaral et al. Published by Cambridge University Press. © Cambridge University Press 2016.

13

Part I. Neurodegenerative Diseases: Case 5

Phospholipase-Associated Neurodegeneration (PLAN) Primary Diagnosis Phospholipase-associated neurodegeneration (PLAN)

Differential Diagnoses Lysosomal storage disorders Mitochondrial disorders Defects of carbohydrate glycosylation

Imaging Findings Fig. 5.1: (A) Axial T2WI through the level of basal ganglia demonstrated T2 hypointensity (arrows) secondary to iron deposition involving bilateral putamen, globus pallidi, and severe cerebral atrophy for an 18-month-old as evidenced by the enlargement of the ventricles and cerebral sulci. (B) Axial GRE image through the same level demonstrated more pronounced hypointensity involving bilateral basal ganglia (arrows), confirming iron deposition. (C) Midline sagittal T1WI demonstrated severe atrophy of the vermis (large arrow), thinning of the corpus callosum (small arrows), and optic chiasm (arrowhead). (D) Coronal T2WI through the cerebellum also demonstrated severe atrophy of bilateral cerebellar hemispheres (arrows). (E) Coronal MR image through the level of the sella demonstrates atrophy of the optic chiasm (arrowheads).

Discussion This patient has the typical clinical and radiologic presentation of phospholipase-associated neurodegeneration (PLAN). Relatively rapid-onset motor signs, severe muscular weakness, regression of language and motor skills, and visual disturbances that developed after birth are typical clinical presentations of PLAN. A diagnosis of PLAN is more certain with the characteristic radiologic findings seen in this case including diffuse cerebellar atrophy, optic atrophy, thin corpus callosum, and evidence of iron deposition in the basal ganglia. Cerebellar atrophy and retinal degeneration can be a manifestation of a wide variety of congenital errors of metabolism including lysosomal storage disorders, mitochondrial disorders, and defective carbohydrate glycosylation. Although optic atrophy can be part of these inborn metabolic errors, abnormal iron deposition is not a feature of these disorders. In patients without abnormal iron deposition, the diagnosis can be difficult: metabolic and genetic screening should be performed to establish a definitive, correct diagnosis. PLAN is due to mutation of the calcium-independent phospholipase A2 gene (PLA2G6). Phenotypic presentation of PLAN can be infantile, before two years of age, and has been variably referred to in the literature as infantile neuroaxonal dystrophy (INAD), Seitelberger disease, and neurodegeneration

14

with brain iron accumulation (NBIA) 2A in Online Mendelian Inheritance in Man® (OMIM®, http://www.omim.org) (OMIM #256600). PLAN can have atypical presentation, referred to as atypical PLAN, with delayed presentation, usually manifesting after three years of age. This atypical PLAN has also been described in literature as atypical neuroaxonal dystrophy (ANAD). Both typical and atypical PLAN patients are asymptomatic at birth. Over time, they develop developmental delay. In typical PLAN, patients develop language and motor regression, diffuse muscular weakness and hypotonia, followed by spastic quadriplegia. Muscular weakness can be so severe that feeding and breathing may be difficult and patients frequently develop pneumonia. Optic atrophy results in visual disturbances and nystagmus is commonly present. Like other NBIA subtypes, the atypical form is slowly progressive as compared to the typical presentation. Atypical PLAN is characterized by movement disorders and dystonia with or without pyramidal symptoms. The dominant imaging finding, present in all typical PLAN cases, is cerebellar atrophy that involves both the vermis as well as the cerebellar hemispheres and usually precedes the inconsistent abnormal iron deposition that can be seen in up to 50% of patients. Iron deposition typically occurs in the globus pallidus. Optic atrophy is another typical imaging manifestation and can best be evaluated in coronal T2 STIR sequences. The corpus callosum appears thin, particularly the splenium, which appears smooth and may be vertically oriented (slitlike). Unlike other NBIA subtypes, diffuse FLAIR hyperintensity in the deep white matter is not a feature of PLAN.

Key Points  PLAN should be considered in younger infants with regression of motor and language skills, severe diffuse muscular weakness, hypotonia, and spastic quadriplegia.  Typical imaging findings in PLAN include severe cerebellar atrophy, optic atrophy, thinning of the corpus callosum, and abnormal iron deposition in the basal ganglia (usually after the cerebellar atrophy).

Suggested Reading Amaral LLF, Gaddikeri S, Chapman PR, et al. Neurodegeneration with brain iron accumulation: clinicoradiological approach to diagnosis. J Neuroimaging 2015; 25(4): 539–51. McKusick, VA. Neurodegeneration with Brain Iron Accumulation 2A; NBIA2A. Baltimore, MD: Johns Hopkins University; 2011 [updated 9/28/2011; cited 2014]. Available from: http://omim.org/ entry/256600. Spiegel R, Pines O, Ta-Shma A, et al. Infantile cerebellar-retinal degeneration associated with a mutation in mitochondrial aconitase, ACO2. Am J Hum Genet 2012; 90(3): 518–23.

Neurodegenerative Diseases

CASE

Part I

6

Asim K. Bag, Lázaro Luís Faria do Amaral

Clinical Presentation A 38-year-old woman, previously diagnosed with diabetes mellitus and anemia, presented with several-year history of gradually progressing blepharospasm, abnormal extremity movement, and ataxia. Her family member complained about her unusual forgetfulness and frequent difficulty in recognizing known people and everyday objects. Her vision had also

begun recently deteriorating. She requires insulin to maintain the normal glycemic status. Routine hematologic tests demonstrated microcytic anemia and iron deficiency that was non-responsive to iron supplementation. Further serum chemistries demonstrated high ferritin level, low transferrin level, and low ceruloplasmin level. An MRI of the brain was performed and results are shown below.

Imaging (A)

(A)

(B)

(B)

Fig. 6.1 (A) Axial GRE through the level of the cerebral peduncles. (B) Axial GRE through the level of the thalami.

Fig. 6.2 (A) Axial T2WI through the level of cerebral peduncles. (B) Axial T2WI through the level of the thalami.

Advanced Neuroradiology Cases, ed. Amaral et al. Published by Cambridge University Press. © Cambridge University Press 2016.

15

Part I. Neurodegenerative Diseases: Case 6

Aceruloplasminemia Primary Diagnosis Aceruloplasminemia

Differential Diagnoses Neuroferritinopathy Familial idiopathic basal ganglia calcification (FIBGC) Physiologic calcification Altered calcium metabolism Hemochromatosis

Imaging Findings Fig. 6.1: (A) Axial GRE image through the level of cerebral peduncles demonstrated hypointensity in the bilateral substantia nigra, suggesting excessive iron deposition, and (B) Axial GRE image through the level of the thalami demonstrated extensive iron deposition involving bilateral basal ganglia and thalami. Fig. 6.2: (A) Axial T2WI through the level of cerebral peduncles demonstrated hypointensity in the bilateral substantia nigra and red nuclei, suggesting excessive iron deposition, and (B) Axial T2WI through the level of the thalami demonstrating hypointensity, secondary to extensive iron deposition involving bilateral basal ganglia and thalami.

Discussion Early-onset movement disorders and neurocognitive decline are indicative of metabolic diseases with CNS involvement. In this patient, MRI of the brain demonstrated abnormal iron accumulation in the deep gray nuclei. In addition to the CNS manifestations, she also has diabetes and microcytic anemia. Similar presentations are seen in both neuroferritinopathy (NFT) and aceruloplasminemia (ACP). However, NFT has microcytic anemia that is responsive to iron supplementation. In addition, in NFT, serum ferritin level is low, not high. High serum ferritin and low ceruloplasmin levels are diagnostic of ACP. Typically, patients with ACP develop visual problems due to retinal degeneration. This patient also developed retinal degeneration that further supports the diagnosis of ACP. Patients with extensive calcification of deep gray nuclei, such as FIBGC, and patients with altered calcium metabolism, such as hypo- and hyperparathyroidism, pseudohypoparathyroidism, or renal failure, may demonstrate extensive signal loss on GRE and SWI sequences. Of all these, movement disorders are prevalent only in patients with FIBGC. In physiologic calcification, the extent of calcification is not as severe as with these conditions and abnormal serum chemistry is not present. Aceruloplasminemia is one of the two known, adult-onset neurodegeneration with brain iron accumulation (NBIA) subtypes (the other one is NFT) and is caused by mutation in the ceruloplasmin gene located in chromosome 3q. The mutation results in the loss of ceruloplasmin protein function, the key transporter of copper, which indirectly results in altered homeostasis of systemic and CNS iron trafficking and tissue

16

iron mobilization. This causes abnormal iron deposition in different areas of the brain such as deep gray nuclei of the basal ganglia, red nuclei, substantia nigra, dentate nuclei, thalami, cerebral cortex, and cerebellum. Deep gray nuclei and dentate nuclei are affected more severely than the cerebral cortex. Iron typically deposits in the perivascular space with degeneration of astrocytes. Enlarged and/or deformed astrocytes and spheroid-like globular structures are characteristic neuropathologic findings in ACP. In addition to the brain, abnormal iron accumulation also occurs in the retina, myocardium, pancreas, and liver. Dementia, abnormal movements, and diabetes in a middleaged patient are typical clinical findings of ACP. Patients classically have blepharospasm, chorea, torticollis, and ataxia. In addition to the movement disorders, patients frequently have retinal degeneration. There is also microcytic anemia that is responsive to ceruloplasmin rather than iron supplementation. Serum ceruloplasmin levels are characteristically low, in contrast to high ferritin levels. Similar to the other NBIA subtypes, the classic imaging finding seen is abnormal excessive iron accumulation in the brain. In addition to the globus pallidus and substantia nigra, excessive iron accumulation is seen in the caudate nuclei, putamen, thalami, red nuclei, and dentate nuclei. The degree of iron deposition in patients with ACP is more extensive compared to the other NBIA subtypes. There is no cystic degeneration of the basal ganglia, a feature that can be used to differentiate ACP from NFT, as both of these entities share common clinical presentations. There may be associated cerebellar atrophy.

Key Points  Typical clinical presentation of ACP is demonstrated by 1) the presence of gradually progressing neurocognitive decline and movement disorder, 2) difficult-to-control diabetes mellitus, 3) microcytic anemia non-responsive to iron supplementation, and 4) retinal degeneration.  Typical imaging findings include evidence of excessive abnormal iron deposition in the caudate nuclei, putamen, thalami, red nuclei, and dentate nuclei, in addition to globus pallidus and substantia nigra. There is also cortical involvement (NBIA in the cortex) in patients with ACP.

Suggested Reading Amaral LLF, Gaddikeri S, Chapman PR, et al. Neurodegeneration with brain iron accumulation: clinicoradiological approach to diagnosis. J Neuroimaging 2015; 25(4): 539–51. Kruer MC, Boddaert N, Schneider SA, et al. Neuroimaging features of neurodegeneration with brain iron accumulation. AJNR Am J Neuroradiol 2012; 33(3): 407–14. Schipper HM. Neurodegeneration with brain iron accumulation – clinical syndromes and neuroimaging. Biochim Biophys Acta 2012; 1822(3): 350–60.

Neurodegenerative Diseases

CASE

Part I

7

Asim K. Bag, Ricardo Heusi

Clinical Presentation A 59-year-old man presented to a neurology clinic with a history of fatigue, hypersomnia, weight loss, bifrontal headache, supranuclear ophthalmoplegia, oculofacial-skeletal myorhythmia, and slowly progressing neurocognitive decline. He also had a long history of abdominal pain and treatmentunresponsive diarrhea. In addition, he had a history of

fleeting-type pain in multiple large joints that lasted for several days. He does not have any known cancer. Hematologic studies were normal. He was evaluated with brain MRI (shown below) and CT scan of the chest, abdomen, and pelvis (not shown). The CT images demonstrated multiple, fat-containing mesenteric and retroperitoneal adenopathies. Routine CSF analysis demonstrated mild lymphocytic pleocytosis.

Imaging (A)

(B)

Fig. 7.1 (A) Axial FLAIR image through the level of the cerebral peduncles. (B) Axial FLAIR image at slightly higher level.

(A)

(B)

Fig. 7.2 (A) Postcontrast axial T1WI sequence through same level as Fig. 7.1A. (B) Postcontrast axial T1WI through the same level as Fig. 7.1B.

Fig. 7.3 Coronal postcontrast T1WI sequence through the hypothalamus.

Advanced Neuroradiology Cases, ed. Amaral et al. Published by Cambridge University Press. © Cambridge University Press 2016.

17

Part I. Neurodegenerative Diseases: Case 7

Whipple Disease with Central Nervous System Involvement Primary Diagnosis Whipple disease with central nervous system involvement

Differential Diagnoses Paraneoplastic limbic encephalitis Pilomyxoid astrocytoma Metastasis

Imaging Findings Fig. 7.1: (A) Axial FLAIR image through the level of the cerebral peduncles demonstrated increased FLAIR signal involving the left inferior frontal lobe, bilateral medial temporal lobes, and inferior hypothalamus. Mass effect is absent. (B) Axial FLAIR image at slightly higher level demonstrated increased FLAIR signal involving the left inferior frontal lobe, left hypothalamus, and left inferior basal ganglia. Subtle signal abnormality in the right inferior basal ganglia was also noted. Fig. 7.2: (A) Postcontrast axial T1WI sequence through same level as Fig. 7.1A demonstrated nodular enhancement of the left inferior frontal lobe and the hypothalamus. (B) Postcontrast axial T1WI sequence at the same level as Fig. 7.1B demonstrated nodular enhancement involving the left inferior basal ganglia and the hypothalamus. Fig. 7.3: Coronal postcontrast T1WI sequence through the hypothalamus demonstrated left-predominant nodular enhancement of the hypothalamus. In addition, subtle nodular enhancement was noted in the right medial temporal lobe, right basal ganglia, and in the left cingulum.

Discussion The triad of dementia, ophthalmoplegia, and oculofacioskeletal myorhythmia is almost exclusively diagnostic of CNS Whipple disease (WD). Hypersomnia, diarrhea, migratory arthritis, and lipid-containing abdominal lymphadenopathy are consistent with the diagnosis of WD. Characteristic imaging findings of CNS WD include abnormal FLAIR signal involving the left inferior frontal lobe, left medial temporal lobes, hypothalamus, and left midbrain with nodular enhancement. The clinicoradiologic profile of this patient is typical of WD and was subsequently confirmed by duodenal biopsy. Paraneoplastic limbic encephalitis, particularly associated with anti-Ma2 (see Part VI: Case 96), may have similar imaging manifestations; however, the clinical presentation is different. In addition, this patient did not have history of cancer. Metastasis is also excluded based on the same arguments. Pilomyxoid astrocytoma may also have similar imaging manifestations, but usually presents earlier in life and lacks systemic manifestations. Whipple disease is a chronic, multisystem disease caused by an infection of Tropheryma whipplei that usually affects middle-aged male patients. Typical clinical presentation includes weight loss, low-grade fever, joint pain, and diarrhea and abdominal cramping. The joint pain is due to symmetric, migratory large-joint, non-deforming arthritis that may present long before the gastrointestinal manifestations, making

18

the diagnosis difficult. Although uncommon, WD may manifest as endocarditis and may be the presenting symptom without histologic or clinical history of gastrointestinal disease or arthralgias. Diagnosis of WD was previously confirmed via jejunal biopsy and demonstrated classic PAS-positive macrophages at the lamina propria containing gram-positive and acid-fast staining negative bacteria. However, PCR-based analyses of saliva and stool specimens are now considered firstline, non-invasive diagnostic tests for WD. In patients with suspected CNS involvement, PCR analysis of CSF also confirms a diagnosis of WD. Whipple disease may present with CNS manifestations in up to 20% of cases. Central nervous system involvement in WD can be manifested in three different settings: 1) as a manifestation of WD along with other systemic features (most common), 2) as CNS recurrence, and 3) as isolated CNS manifestation. Dementia is the most common clinical presentation of CNS WD. Oculofacial myorhythmia or oculofacioskeletal myorhythmia is a pathognomonic clinical manifestation of CNS WD and can be seen in up to 20% of patients. This characteristic myorhythmia is usually accompanied by ophthalmoplegia. The most common imaging abnormality in patients with WD is bilateral (often asymmetric) abnormal T2 signal involving the inferior frontal lobes, medial temporal lobes, basal ganglia, periaqueductal gray matter, thalamus and hypothalamus. Nodular enhancement may also be present, predominantly involving the hypothalamus, although the enhancement may be transient. Areas of T2 abnormality typically show diffusion facilitation on DWI. Other less common imaging findings include abnormal T2 signal involving the gray matter-white matter interface, which, on occasion, may become confluent, and abnormal T2 signal along the corticospinal tracts, similar to amyotrophic lateral sclerosis. Enhancement may be either absent or subtle in these two conditions. Magnetic resonance imaging may be completely normal in some patients.

Key Points  The presenting hallmark of CNS WD is the clinical triad of dementia, ophthalmoplegia, and oculofacioskeletal myorhythmia.  Diarrhea, steatorrhea, abdominal pain, and migratory arthritis involving the larger joints are the most common systemic manifestations of late-staged WD.  Abnormal FLAIR signal involving bilateral medial temporal lobes, hypothalamus, and the midbrain thalamus with nodular enhancement are the most common imaging findings of CNS WD.

Suggested Reading Black DF, Aksamit AJ, Morris JM. MR imaging of central nervous system Whipple disease: a 15-year review. AJNR Am J Neuroradiol 2010; 31(8): 1493–7. Louis ED, Lynch T, Kaufmann P, Fahn S, Odel J. Diagnostic guidelines in central nervous system Whipple’s disease. Ann Neurol 1996; 40(4): 561–8.

Neurodegenerative Diseases

CASE

Part I

8

Asim K. Bag, Lázaro Luís Faria do Amaral

Clinical Presentation A previously healthy 65-year-old male patient presented with progressively worsening muscle rigidity, tremor, akinesia, and stridor that were poorly controlled by levodopa. In addition, he reported several months’ history of constipation, postural hypotension, dysphagia, and bladder dysfunction. Prior to admission, he reported development of progressively worsening cerebellar symptoms including gait ataxia, dysarthria, and oculomotor dysfunction. Prior to onset of current complaints, he had never experienced neurologic problems. He has no known primary malignancies, no history of alcohol abuse, and hematologic abnormalities, or any vitamin deficiencies. Neurologic examination revealed Babinski sign.

Imaging

Fig. 8.2 Sagittal T1WI through the midline.

Fig. 8.1 Axial T2WI through the level of the pons.

Fig. 8.3 Directionally encoded color-coded FA map at the level of the pons.

Advanced Neuroradiology Cases, ed. Amaral et al. Published by Cambridge University Press. © Cambridge University Press 2016.

19

Part I. Neurodegenerative Diseases: Case 8

Multiple System Atrophy-Cerebellar Type Primary Diagnosis Multiple system atrophy-cerebellar type

Differential Diagnoses Idiopathic late-onset cerebellar ataxia Toxin-mediated cerebellar ataxias Paraneoplastic cerebellar ataxias Hereditary cerebellar ataxias

Imaging Findings Fig. 8.1: Axial T2WI through the level of the pons demonstrated a cruciform-shaped, T2 hyperintensity through the middle of the pons (hot-cross-bun sign) due to atrophy of the cerebellopontine and transverse pontine fibers. Atrophy of the vermis is demonstrated by the enlargement of the sulci. Fig. 8.2: Sagittal T1WI through the midline demonstrated atrophy of the pons. Fig. 8.3: Colored FA map demonstrated absence of red transverse pontine fibers.

Discussion Gradually progressing Parkinsonian syndromes (tremor, rigidity) in association with autonomic dysfunction and cerebellar signs are suggestive of possible multiple system atrophy (MSA), according to the second consensus statement on diagnosis of multiple system atrophy. Presence of cerebellar atrophy, hotcross-bun sign, and pontine atrophy is suggestive of multiple system atrophy: cerebellar type, MSA-C, by the same definition. Patients with MSA-P have similar clinical presentation, excluding the cerebellar symptoms, cerebellar and pontine atrophy. Idiopathic, late-onset cerebellar ataxia is a descriptive term defined by the presence of a primary progressive form of ataxia in the absence of any known exposure to common cerebellar toxins such as alcohol and antiseizure medications, and the absence of any known primary malignancies or hypothyroidism, with careful exclusion of MSA. The gradual unfolding of underlying genetic causes has led to the identification of many specific entities under the umbrella of idiopathic late-onset cerebellar ataxia (ILOCA), such as Friedreich ataxia, ataxia-telangiectasia, and fragile X-associated tremor. Although ILOCA and MSA-C share many clinical features, MSA-C is more commonly associated with Parkinsonian and premotor symptoms, such as urinary dysfunction, stridor, constipation, dysphagia, and sleep disturbances. Cerebellar atrophy can be seen in either of these syndromes, but pontine atrophy, middle cerebellar peduncle atrophy, and hot-cross-bun signs are not seen in any other ILOCA variety. Other causes of acquired cerebellar ataxias can easily be excluded by the patient’s history. Multiple system atrophy is a descriptive term for a group of neurodegenerative disorders that were previously known as olivopontocerebellar degeneration, Shy Dragger syndrome, and striatonigral degeneration. It is one of the known three Parkinson-plus syndromes, namely progressive supranuclear

20

palsy, corticobasal degeneration, and MSA. An adult-onset, sporadic neurodegenerative disease, MSA is characterized by autonomic failure in addition to Parkinsonian or cerebellar symptoms. In MSA-C, the cerebellar symptoms dominate the clinical findings. Histopathologically, MSA-C is characterized by alpha-synuclein-positive glial cytoplasmic inclusions and olivopontocerebellar degeneration. Diagnosis of MSA subtypes is difficult, particularly during early stages. Like many other neurodegenerative diseases, diagnosis of MSA and its subtypes can be described as 1) definite (confirmed at autopsy); 2) probable or possible; or 3) with differing degrees of diagnostic certainty based on clinical and imaging findings. Indicative findings include the presence of autonomic failure (urinary incontinence, erectile dysfunction, or orthostatic hypotension), Parkinsonian symptoms that are refractory to treatment with levodopa, or cerebellar symptoms that consist of gait ataxia with cerebellar dysarthria, limb ataxia, or cerebellar oculomotor dysfunction. Common abnormalities associated with MSA-C include atrophy of the pons, middle cerebellar peduncle, and cerebellum. Hot-cross-bun sign is characteristic of MSA-C; however, it is not pathognomonic, and is secondary to degeneration and gliosis of pontocerebellar fibers – irrespective of underlying pathologic processes – and usually develops late in the disease process. All of these imaging findings are usually late in the disease process, limiting their role in early diagnosis of MSAC. On 18F-fluorodeoxyglucose positron emission tomography CT images, there is decreased glucose uptake in the brainstem and cerebellum in patients with MSA-C, as compared to control patients. There is also evidence of denervation of presynaptic nigrostriatal dopaminergic innervation on dopamine transporter scan images (DaT scan). Treatment for MSA-C mainly targets Parkinsonism and dysautonomia symptoms; however, currently there is no treatment for the cerebellar symptoms.

Key Points  MSA-C should be suspected in adult patients presenting with progressive Parkinsonian syndrome that is nonresponsive to levodopa, in added presence of autonomic and cerebellar signs.  Atrophy of the pons with hot-cross-bun sign and atrophy of middle cerebellar peduncle, and cerebellum are characteristic imaging manifestations of a MSA-C diagnosis.

Suggested Reading Gilman S, Wenning GK, Low PA, et al. Second consensus statement on the diagnosis of multiple system atrophy. Neurology 2008; 71(9): 670–6. Lin DJ, Hermann KL, Schmahmann JD. Multiple system atrophy of the cerebellar type: clinical state of the art. Mov Disord 2014; 29(3): 294–304.

Neurodegenerative Diseases

CASE

Part I

9

Asim K. Bag, Lázaro Luís Faria do Amaral

Clinical Presentation A 43-year-old man presented with a history of inability to control the release of urine from his bladder. He stated that this problem was gradually worsening. In addition, he also found it difficult to maintain his erection. Ultrasonography of the pelvis demonstrated a normal prostate gland and a CT angiogram of the abdomen demonstrated no aortoiliac atherosclerotic disease. Medical history included more than five years’

history of Parkinson disease that was initially levodopa responsive; however, his symptoms are not controlled by levodopa anymore. On questioning, his wife mentioned that she had noted an increase of inappropriate laughing and crying in her husband. In addition, his sleep at night was not as good as it used to be, and he appeared more forgetful. He also complained of unsteadiness, but there was no clinical ataxia. Magnetic resonance imaging was performed (see below).

Imaging Fig. 9.1 Axial T2WI through the basal ganglia.

Fig. 9.2 Axial FLAIR image through the basal ganglia.

Fig. 9.3 Axial GRE image through the basal ganglia.

Fig. 9.4 Axial spin echo T1WI sequence through the centrum semiovale with added magnetization transfer pulse.

Advanced Neuroradiology Cases, ed. Amaral et al. Published by Cambridge University Press. © Cambridge University Press 2016.

21

Part I. Neurodegenerative Diseases: Case 9

Multiple System Atrophy-Parkinsonian Type Primary Diagnosis Multiple system atrophy-Parkinsonian type

Differential Diagnoses Parkinson disease Multiple system atrophy-cerebellar type Parkinson-plus syndrome Corticobasal degeneration

Imaging Findings Fig. 9.1: Axial T2WI through the basal ganglia demonstrated subtle curvilinear T2 hyperintensity along the dorsolateral margins of the putamen (arrows) – the hyperintense putaminal slit sign. Putaminal hypointensity, which sometimes can be seen because of abnormal iron accumulation, is not present in this particular case. Fig. 9.2: Axial FLAIR image through the basal ganglia better demonstrated the hyperintense putaminal slit sign. Fig. 9.3: Axial GRE image through the basal ganglia demonstrated subtle hypointensity in the posterior aspect of the right putamen, due to abnormal iron deposition (arrow) that was not evident on the T2WI sequences. Fig. 9.4: Axial spin echo T1WI sequence with added magnetization transfer pulse demonstrated hyperintensity involving the corticospinal tract at the level of the centrum semiovale (arrows).

Discussion Typically, Parkinson disease responds well to levodopa. Levodopa-unresponsive Parkinsonism suggests one of the atypical or Parkinson-plus syndromes, namely multiple system atrophy (MSA), corticobasal degeneration (CBD), or progressive supranuclear palsy (PSP). The presence of autonomic failure (urinary problems and erectile dysfunction) is suggestive of MSA. The absence of prominent ataxic signs, cerebellar and pontine atrophy, and hot-cross-bun sign disfavors diagnosis of MSA-C. This patient did not have eye movement problems, and his MRI did not show foreshortening of the midbrain, thus PSP can be eliminated as a potential diagnosis. The absence of significant brain atrophy, predominant behavioral symptoms, or cognitive decline eliminates CBD as an optional diagnosis. Multiple system atrophy is a gradually worsening, neurodegenerative disorder characterized by a combination of levodopa-nonresponsive abnormal movements and symptomatic autonomic nervous system failure. As movement disorder is the predominant presenting symptom in all patients with MSA, it is considered one of the Parkinson-plus syndromes, in addition to PSP and CBD. Depending upon the predominant brain regions involved and subsequent clinical symptoms, MSA can be categorized as Parkinsonian type (MSA-P) or cerebellar type (MSA-C, see Part I: Case 8). There is an ethnic difference in the incidence of MSA-C verses MSA-P. MSA-P is more prevalent in North American and European populations,

22

as compared to MSA-C, which is more prevalent in the Japanese population. Multiple system atrophy is now classified as one of the alpha-synucleinopathies, due to the presence of cytoplasmic inclusion bodies that contain alpha-synuclein. However, no specific mutation of the synuclein (SNCA) gene has been implicated as a cause of MSA although several genetic polymorphisms at the SNCA locus have been linked to MSA. Recently, homozygous or compound heterozygous mutations in the COQ2 gene (which encode enzymes involved in the biosynthetic pathway of coenzyme Q10) have been linked to familial MSA. Common presentation of MSA-P includes decreased spontaneous movement, tremor, or rigid muscles, clumsiness, loss of balance, and frequent falls. In addition, patients also demonstrate features of autonomic nervous system dysfunction manifested as fainting or lightheadedness due to orthostatic hypotension, and bladder control problems, such as a sudden urge to urinate or difficulty emptying the bladder completely. Although patients may initially demonstrate MSA-P symptoms, over time, they may develop MSA-C symptoms. Moreover, common presentations of MSA also include development of contractures in the hands and feet, disproportionate antecollis, emotional lability such as inappropriate laughing or crying, as well as sleep and sexual disturbances. Unlike MSA-C, there is no characteristic imaging appearance associated with MSA-P. On conventional imaging, there may be posterior putaminal hypointensity from abnormal iron accumulation, which can be better seen on 3T magnets, or subtle hyperintensity at the dorsolateral margin of the putamen on T2-weighted sequences, which has been described as hyperintense putaminal rim or slit sign. Volumetric segmentation of putaminal volume has been shown to help differentiate MSA-P from controls and Parkinson disease. Although these are helpful indicators, these changes are not specific to MSA-P. Advanced MRI techniques may add complementary information to the conventional MRI. Diffusion MRI imaging has shown low anisotropy of the motor cortex and increased diffusivity of putamen in MSA-P. T1 spin echo MRI with addition of magnetization transfer contrast pulse has shown that there is an increase in signal in the motor cortex as well as in the pyramidal tracts at the centrum semiovale, particularly if MSA-P is associated with C9orf32 mutation associated amyotrophic lateral sclerosis. Striatal and cerebellar hypometabolism on 18F-fluorodeoxyglucose PET scan in a patient with clinically absent ataxia is suggestive of MSA-P.

Key Points  MSA-P should be suspected in adult patients presenting with progressive Parkinsonian syndrome that is nonresponsive to levodopa, in addition to autonomic signs.  Presence of hypointensity in the posterior putamen and hyperintense putaminal rim sign on T2 is suggestive of MSA-P.

Part I. Neurodegenerative Diseases: Case 9

Suggested Reading Baudrexel S, Seifried C, Penndorf B, et al. The value of putaminal diffusion imaging versus 18-fluorodeoxyglucose positron emission tomography for the differential diagnosis of the Parkinson variant of multiple system atrophy. Mov Disord 2014; 29(3): 380–7. da Rocha AJ, Maia AC, Jr., da Silva CJ, et al. Pyramidal tract degeneration in multiple system atrophy: the relevance of magnetization transfer imaging. Mov Disord 2007; 22(2): 238–44.

Gilman S, Wenning GK, Low PA, et al. Second consensus statement on the diagnosis of multiple system atrophy. Neurology 2008; 71(9): 670–6. Mitsui J, Tsuji S. Genomic aspects of sporadic neurodegenerative diseases. Biochem Biophys Res Commun 2014; 452(2): 221–5. Tir M, Delmaire C, Besson P, Defebvre L. The value of novel MRI techniques in Parkinson-plus syndromes: diffusion tensor imaging and anatomical connectivity studies. Rev Neurol (Paris) 2014; 170(4): 266–76.

23

CASE

Part I

10

Neurodegenerative Diseases Asim K. Bag, Lázaro Luís Faria do Amaral

Clinical Findings A 67-year-old man presented with recent history of slowly progressive clumsiness and a more than two-year history of loss of function of his left hand. He also noted that he does not have full control of his left hand as it is frequently involved in unwanted movements. He reports no previous history of neurologic disease or any known familial neurologic disease. In addition, he also complained that he is more forgetful about his day-to-day activities. Particularly, he found it is very difficult to manage his financial matters. On neurologic

examination, he was found to have bradykinesia, ideomotor and limb-kinetic apraxia, left limb dystonia, and stimulussensitive myoclonus. His symptoms did not improve with levodopa. A follow-up MRI was performed (shown below). Computed tomography angiogram of his head was obtained (not shown) but did not demonstrate any flow-limiting vascular stenosis. FDG-PET demonstrated hypometabolism of the posterior cingulate, and bilateral parietal regions, with relative sparing of the sensorimotor areas and frontal lobes (not shown).

Imaging

Fig. 10.1 Axial FLAIR image through the centrum semiovale.

Fig. 10.2 Axial FLAIR image through the centrum semiovale inferior to level of Fig. 10.1.

Advanced Neuroradiology Cases, ed. Amaral et al. Published by Cambridge University Press. © Cambridge University Press 2016.

25

Part I. Neurodegenerative Diseases: Case 10

Corticobasal Degeneration Primary Diagnosis Corticobasal degeneration

Differential Diagnoses Progressive supranuclear palsy Alzheimer disease Posterior cortical atrophy Stroke Creutzfeldt-Jakob disease Frontotemporal lobar degeneration

Imaging Findings Fig. 10.1: Axial FLAIR image through the centrum semiovale demonstrated bilateral posterior frontal and parietal atrophy as evidenced by secondary enlargement of the sulci and predominantly right-sided thinning of the gyri. Fig. 10.2: Axial FLAIR image through the centrum semiovale (obtained inferior to Fig. 10.1) demonstrated mild, posterior-predominant atrophy, in addition to abnormal FLAIR signal in both posterior parietal lobes, as well as in the right posterior frontal lobe.

Discussion The clinical description of this case is typical of corticobasal syndrome (CBS). Imaging demonstrates posteriorpredominant atrophy involving both the parietal lobes and the right posterior frontal lobe. Combined clinical and radiologic findings are suggestive of CBS, likely associated with Alzheimer disease (AD). No focal encephalomalacia was detected, ruling out largeartery distribution stroke. The clinical course of CreutzfeldtJakob disease features a rapidly progressive dementia with or without myoclonus typically followed by death within the same year. Our patient was symptomatic for more than two years. Posterior cortical atrophy is a radiologic description of posterior-predominant brain atrophy and can be seen in many different neurodegenerative diseases including CBS. Although progressive supranuclear palsy (PSP) and AD are two distinct diseases, both of these conditions can present clinically with CBS. Therefore, it is possible that even though the patient has CBS, he/she will have pathologic diagnosis of corticobasal degeneration (CBD), PSP, or AD (see the discussion below). Corticobasal syndrome is a rare, progressive neurodegenerative disease with a heterogeneous clinical presentation consisting of motor, sensory, behavioral, and cognitive symptoms

26

that frequently overlap with other neurodegenerative disorders such as frontotemporal lobar degeneration (FTLD), AD, and PSP. The classical clinical manifestation of CBD is known as corticobasal syndrome (CBS) and is characterized by levodopaunresponsive Parkinsonism, progressive asymmetric rigidity, apraxia, aphasia, and dystonia, which can be found in up to 50% of pathologically proven cases of CBD (CBS-CBD). However, CBS can be a presenting symptom of Alzheimer disease (CBS-AD), progressive supranuclear palsy (CBS-PSP), and frontotemporal lobar degeneration (CBS-FTLD) making its diagnosis difficult. Typically, CBD demonstrates asymmetric focal atrophy of the frontoparietotemporal lobes, sparing the brainstem and cerebellum; however, the substantia nigra may demonstrate loss of pigmented neurons. Manifestation of brain atrophy in CBS is heterogeneous and condition-dependent such as frontotemporal atrophy in CBS-FTLD, parietotemporal atrophied CBS-AD, and superior posterior frontal in CBS-CBD/CBSPSP. Predominant dominant frontal lobe atrophy is seen in CBS-FTLD, whereas predominant dominant parietal lobe atrophy is seen in CBS-FTLD and CBS-AD. Predominant cortical atrophy is not a feature of CBS-CBD and CBS-PSP. Corticobasal degeneration is a 4R-tauopathy similar to PSP (Part I: Case 11). On histopathology, there are neuronal spherical corticobasal bodies that are tau-4R immunoreactive, and are found mostly in the frontal and parietal cortex. However, the histopathologic hallmark of CBD is the presence of numerous Gallyas – and tau-immunoreactive neuronal threads/astrocytic plaques – in the neuropil of affected gray matter and adjacent white matter.

Key Points  Corticobasal syndrome is a clinical descriptor of a syndrome characterized by motor, sensory, cognitive, and behavioral problems associated with CBD, PSP, FTLD, and AD.  Brain atrophy associated with CBS depends upon phenotypic variant. CBS-FTLD has predominant frontotemporal atrophy, CBS-AD has predominant parietotemporal atrophy, and CBS-CBD has predominant posterior superior frontal lobe.

Suggested Reading Rohan Z, Matej R. Current concepts in the classification and diagnosis of frontotemporal lobar degenerations: a practical approach. Arch Pathol Lab Med 2014; 138(1): 132–8.

CASE

Part I

11

Neurodegenerative Diseases Asim K. Bag, Lázaro Luís Faria do Amaral

Clinical Presentation A primary care physician referred a 45-year-old previously healthy man with a self-described myriad of neurologic complaints to a subspecialty neurology clinic for assessment. This man explained that lately, he found it very difficult to walk unassisted, was experiencing a sensation of losing his balance, and had a tendency to fall backwards. He also found that smooth reading and eating without spoiling his clothes had become troublesome. He was receiving complaints that his behavior had become rude because he no longer maintained socially appropriate eye contact. He also described frustrating memory loss and inability to remember routine

things such as computer passwords. He shared that his girlfriend laughed at him because of his new illegible handwriting, peculiar, slow (octogenarian-type) gait, and rather inappropriate impulsive behavior (uncontrollable laughter for no good reason). He also reported poor sleep, and anxiety regarding ability to continue management of his financial affairs. He stressed the lack of previous neurologic disease. On examination, rigidity, stiff broad-based gait, downward gaze palsy, and neuropsychologic features suggestive of frontotemporal dementia were all noted. An MRI was performed and results are shown. Routine hematologic studies were negative.

Imaging Fig. 11.1 Sagittal T1WI through the midline.

Fig. 11.2 Sagittal FLAIR through the midline.

Fig. 11.3 Axial T2WI sequence through the midbrain.

Fig. 11.4 Axial T2WI sequence through the pons.

Advanced Neuroradiology Cases, ed. Amaral et al. Published by Cambridge University Press. © Cambridge University Press 2016.

27

Part I. Neurodegenerative Diseases: Case 11

Progressive Supranuclear Palsy Primary Diagnosis Progressive supranuclear palsy

Differential Diagnoses Frontotemporal dementia (FTD) Corticobasal syndrome (CBS) Parkinson disease (PD) Parkinson-plus syndrome (PPS)

Imaging Findings Fig. 11.1: Sagittal T1WI. Fig. 11.2: Sagittal FLAIR image through the midline demonstrates foreshortening and significant atrophy of the midbrain, with relative sparing of the pons (known as humming bird sign or penguin-silhouette sign). Also, note that the superior border of the midbrain has slight concavity instead of regular convexity (due to significant atrophy). Fig. 11.3: Axial T2WI sequence through the midbrain demonstrates decreased anteroposterior diameter at the level of the superior colliculi (< 12 mm), giving the midbrain a Mickey Mouse appearance. Also, the loss of lateral convexity of the cerebral peduncles (morning glory sign) is noted. Fig. 11.4: Axial T2WI sequence through the pons demonstrates bilateral superior cerebellar peduncle atrophy that is more severe on the right side (arrow) compared to the left side (arrowhead).

Discussion The gradually progressive supranuclear palsy (downward gaze palsy), Parkinsonism (rigidity, slow movements, and illegible handwriting), and frontotemporal-type dementia in a patient who is aged 40 years or more epitomizes the typical presentation of progressive supranuclear palsy (PSP). Imaging studies demonstrating characteristic midbrain atrophy further confirm a diagnosis of PSP. The complex and extremely heterogeneous group of diseases including FTD, CBS, PD, and PPS overlap and share multiple clinical features. Thus, on a clinical basis, it is extremely difficult to differentiate between these disease types. It is also difficult to differentiate between these diseases immunopathologically, as they all can be tau-immunoreactive. Thus, all of these diseases are considered variants on a spectrum of a common histopathologic process. Individual disease types can be diagnosed by combination of clinical and radiologic findings. Progressive supranuclear palsy is a tauopathy accompanied by a heterogeneous clinical syndrome and variable histopathologic findings. As expected in any tauopathy, PSP shares clinical findings with other tau-related neurodegenerative diseases and demonstrates symptoms related to degeneration of the cerebral cortex, deep gray matter nuclei of the basal ganglia, brainstem, and cerebellum. The clinical and pathologic spectrum of PSP depends upon the degree and region of brain involved. In one extreme, PSP can be associated with predominant cortical involvement and presents as PSP-progressive non-fluent aphasia (PSA-PNFA). However, classic PSP as described by Richardson (PSP-R), is at the other end of

28

the spectrum that predominantly features degeneration of the brainstem nuclei. Other PSP variants, such as PSP-associated Parkinsonism (PSP-P), PSP-associated pure akinesia with gait freezing (PSP-PAGF), and PSP-corticobasal syndrome (PSP-CBS) are in the middle of the spectrum. Clinical presentations of typical PSP-R include Parkinsonism features (rigidity and bradykinesia), supranuclear gaze palsy, and postural instability with falls. Characteristic impairment of downward gaze results in problems reading, eating, and walking. Nystagmus is also common. Varying degrees of associated neuropsychiatric disturbances such as cognitive decline, psychomotor slowing, impaired decision-making, and apathy are also common. Obsessive-compulsive and disinhibitory features have also been described in patients with PSP-R. In patients with PSP, there is severe involvement of the subthalamic nucleus, pallidum, striatum, red nucleus, substantia nigra, pontine tegmentum, third nerve nuclei in the pons, cerebellar white matter, and dentate nuclei. In PSP-R, the substantia nigra pars compacta and ventro-tegmental areas are most severely affected, with selective destruction of the dopaminergic nigrostriatal system. This discriminate tissue damage explains why PSP-R is non-responsive to levodopa. Although the substantia nigra can be affected in other PSP variants, it is less severe compared to PSP-R. Generalized brain atrophy is commonly seen in PSP, with predominant involvement of the frontal lobes. The distinguishing imaging finding in PSP is the striking atrophy of the midbrain in comparison to the rest of the brainstem. This distinctive feature has been described as a humming bird sign or penguin-silhouette sign on midline sagittal images and the Mickey Mouse and morning glory sign on axial images. Several quantitative measurements have been proposed for accurate diagnosis of PSP. The most specific and most sensitive sign in discriminating PSP from PD or other multiple system atrophy subtypes is Quattrone’s MR Parkinsonism index. It is calculated as the ratio of pons area to midbrain area measured in the midsagittal plane, multiplied by the ratio of the middle cerebellar peduncle and superior cerebellar peduncle width. On FDG-PET, there is hypometabolism in the frontosubcortical regions (anterior cingulate gyrus and lingual gyrus). The histopathologic hallmarks of PSP are the presence of star-shaped argyrophilic astrocytic tufts and globose neurofibrillary tangles in neuronal cytoplasm that are tau-immunoreactive and have four repeats of the phosphate-binding domain (i.e., 4R tauopathy) similar to CBD.

Key Points  Progressive supranuclear palsy is a tauopathy that has heterogeneous clinical and pathologic manifestations.  Depending upon the areas of brain involvement, PSP can present as an FTD variant or a variant of Parkinsonism (Parkinson-plus syndrome).  Characteristic imaging findings include striking midbrain and superior cerebellar peduncle atrophy

Part I. Neurodegenerative Diseases: Case 11

with hypersignal on FLAIR and hypometabolism of the frontosubcortical regions.

Suggested Reading Hattori N. Autosomal dominant parkinsonism: its etiologies and differential diagnoses. Parkinsonism Relat Disord 2012; 18(Suppl 1): S1–3.

Quattrone A, Nicoletti G, Messina D, et al. MR imaging index for differentiation of progressive supranuclear palsy from Parkinson disease and the Parkinson variant of multiple system atrophy. Radiology 2008; 246(1): 214–21. Williams DR, Lees AJ. Progressive supranuclear palsy: clinicopathological concepts and diagnostic challenges. Lancet Neurol 2009; 8(3): 270–9.

29

CASE

Part I

12

Neurodegenerative Diseases Asim K. Bag, Lázaro Luís Faria do Amaral

Clinical Presentation A right-handed, 60-year-old man, with no history of neurologic disease, presented with a four-year history of progressive, worsening forgetfulness of day-to-day-life activities. His wife noticed that he had been having trouble recognizing his relatives and friends that seems to be worsening with time. She also noted that he had been spending money inappropriately and

making inappropriate public comments about other women. She also stated that he got lost several times in the market they regularly visited. He had also developed the strange behavior of checking the garage door several times to make sure that it was locked properly. However, on neurologic exam he did not demonstrate any difficulties naming common objects. Magnetic resonance imaging was performed (images shown below).

Imaging (A)

(B)

Fig. 12.1 (A) Coronal T2WI through the anterior temporal lobe. (B) Coronal T2WI obtained through the temporal lobe but more posteriorly.

(A)

(B)

Fig. 12.2 (A–B) Axial T2WI through the temporal lobes demonstrates striking atrophy of the right anterior and medial temporal lobe.

Fig. 12.3 Axial FLAIR image through the temporal lobes.

Advanced Neuroradiology Cases, ed. Amaral et al. Published by Cambridge University Press. © Cambridge University Press 2016.

31

Part I. Neurodegenerative Diseases: Case 12

Right Temporal Variant of Frontotemporal Lobar Degeneration Primary Diagnosis Right temporal variant of frontotemporal lobar degeneration

Differential Diagnosis Frontotemporal lobar degeneration variants (refer to Part I: Case 18)

Imaging Findings Fig. 12.1: (A) Coronal T2WI through the anterior temporal lobe demonstrated striking atrophy of the right anterior and inferior temporal lobe. Atrophy of the frontal and left temporal lobes is present; however, it is mild, in comparison to the right temporal lobe. (B) Coronal T2WI obtained more posteriorly, demonstrated striking atrophy of the right medial temporal lobe, in comparison to the remainder of the visualized areas of brain. Fig. 12.2: (A–B) Axial T2WI through the temporal lobes demonstrates striking atrophy of the right anterior and medial temporal lobe. Fig. 12.3: Axial FLAIR image through the temporal lobes demonstrated increased FLAIR signal in the remainder of the anterior and medial temporal lobe, in addition to the severe atrophy of the right anterior and medial temporal lobe.

Discussion The presence of constellating clinical findings including neurocognitive decline, progressive prosopagnosia (PP; inability to recognize familiar faces), behavioral changes, topographic disorientation, and striking, characteristic right temporal lobe atrophy (RTLA) is diagnostic of right temporal variant of frontotemporal lobar degeneration (FTLD). In other variants of FTLD, left temporal atrophy dominates not right temporal lobe. Progressive prosopagnosia, a clinical syndrome characterized by a progressive and selective inability to recognize known faces, is due to preferential atrophy of the anterior and inferior right hemisphere (non-dominant). It is classified as a right

32

hemispheric variant of semantic dementia, a variant of frontotemporal dementia (FTD). This is different from apperceptive prosopagnosia in which there is functional damage of visuoperceptual processing in the visual association areas. In PP, there is general cognitive and intellectual decline, with striking, progressive inability to recognize previously known faces of relatives, friends, and famous historic or media persons. Although the majority of RTLA case reports have focused on PP, other common clinical symptoms of RTLA include impairment of episodic memory, topographic disorientation, and disinhibited social behaviors. Unlike semantic dementia, anomia and problems of naming are less common presentations. An asymmetric, striking atrophy of the anterior and inferior right temporal lobe on both visual interpretation as well as voxel-based morphometric analysis is the characteristic imaging finding in patients with PP. In addition to the right frontal lobe, there is also frontal-predominant generalized brain atrophy. Perfusion SPECT studies also demonstrate profound decreased perfusion of the right temporal lobe that progresses over time. Although heterogeneous, the constellation of clinical presentations and characteristic atrophy of the anterior and inferior right temporal lobe is considered a rare variant of FTD.

Key Points  Progressive prosopagnosia manifests as RTLA, a distinct variant of FTD differentiated by characteristic atrophy of the anterior and inferior right temporal lobe.  Other symptoms are impairment of episodic memory and symptomatic topographic disorientation.

Suggested Reading Chan D, Anderson V, Pijnenburg Y, et al. The clinical profile of right temporal lobe atrophy. Brain 2009; 132(Pt 5): 1287–98. Joubert S, Felician O, Barbeau E, et al. Progressive prosopagnosia: clinical and neuroimaging results. Neurology 2004; 63(10): 1962–5.

CASE

Part I

13

Neurodegenerative Diseases Afonso C. P. Liberato, Lázaro Luís Faria do Amaral

Clinical Presentation An otherwise healthy 59-year-old woman, with no previous history of trauma or epilepsy, presented to our facility with progressive memory loss. Neuropsychologic testing confirmed presence of short-term memory loss.

Imaging (B) (A)

Fig. 13.1 (A–B) Axial FLAIR through the level of the hippocampus.

Advanced Neuroradiology Cases, ed. Amaral et al. Published by Cambridge University Press. © Cambridge University Press 2016.

33

Part I. Neurodegenerative Diseases: Case 13

(A) (B)

(C)

Fig. 13.2 (A–C) Coronal FLAIR through the same level.

34

Part I. Neurodegenerative Diseases: Case 13

(B) (A)

(C)

Fig. 13.3 (A–C) Coronal T2 STIR through the same level.

35

Part I. Neurodegenerative Diseases: Case 13

Hippocampal Sclerosis Dementia Primary Diagnosis Hippocampal sclerosis dementia

Differential Diagnoses Alzheimer disease Vascular dementia Frontotemporal lobar degeneration

Imaging Findings Fig. 13.1: (A–B) Axial FLAIR MR images showed hyperintense signal in the right hippocampus. Fig. 13.2: (A–C) Coronal FLAIR MR images showed isolated hypersignal, as well as signs of marked atrophy on the right hippocampus (head, body, and tail). Fig. 13.3: (A–C) Coronal T2 STIR also showed hypersignal of the right hippocampus, more clearly demonstrating hippocampal atrophy.

Discussion Pure hippocampal sclerosis (HS) is a rare cause of dementia whose name is derived from a constellation of associated neuropathologic findings. Hippocampal sclerotic neuropathology is characterized by severe neuronal loss and gliosis in the cornu ammonis area 1 (CA1) region of the hippocampus and subiculum. However, the associated findings of other dementing illnesses such as Alzheimer disease (AD), vascular dementia (VD), or frontotemporal lobar denegation (FTD), or clinical evidence of syncope, hypotension, or hypoxia, in association with dementia onset are noticeably absent. Thus, correlating clinical and imaging findings in cases of suspected HS is imperative in order to narrow the differential diagnosis. In this patient, the presence of marked hippocampal atrophy in conjunction with confirmed memory loss strongly suggests a diagnosis of HS. In addition to demential disorders, other neuropathologies are associated with HS. Mesial temporal sclerosis, or HS, is most commonly associated with prolonged seizures in early infancy and cerebral infarcts are associated with HS. However, in these conditions, the clinical symptomatology and age of affected individuals differs, in comparison to the dementia syndromes.

36

The estimated prevalence of HS among patients with dementia is variable, ranging from 0.4% to 8%, being more frequent in patients 80 years of age or older. The etiology of HS dementia is unknown. Generally, patients with HS present with cognitive disturbances, mimicking AD and/or FTD, making its clinical diagnosis extremely difficult. It has been reported that these patients have a tendency to become progressively demented and that the duration of their illness is shorter, in comparison to patients with AD without HS. At imaging, unilateral or bilateral hippocampal atrophy in conjunction with increased hippocampal signal intensity on FLAIR and T2-weighted MR images constitutes the prominent findings.

Key Points  Pure HS is a rare neuropathologic finding in daily neuroradiology practice.  Neuroradiologists should have a clear understanding of the distinguishing clinical and radiologic findings of HS, especially in older patients with dementia.  Abnormal signal in the hippocampus on T2 and FLAIR are classic findings of HS.  Neurologically confirmed dementia in the presence of hippocampal atrophy are suggestive of pure HS.

Suggested Reading Blass DM, Hatanpaa KJ, Brandt J, et al. Dementia in hippocampal sclerosis resembles frontotemporal dementia more than Alzheimer disease. Neurology 2004; 63(3): 492–7. Jack CR, Jr., Dickson DW, Parisi JE, et al. Antemortem MRI findings correlate with hippocampal neuropathology in typical aging and dementia. Neurology 2002; 58(5): 750–7. Onyike CU, Pletnikova O, Sloane KL, et al. Hippocampal sclerosis dementia: an amnesic variant of frontotemporal degeneration. Dement Neuropsychol 2013; 7(1): 83–7. Probst A, Taylor KI, Tolnay M. Hippocampal sclerosis dementia: a reappraisal. Acta Neuropathol 2007; 114(4): 335–45. Zarow C, Weiner MW, Ellis WG, Chui HC. Prevalence, laterality, and comorbidity of hippocampal sclerosis in an autopsy sample. Brain Behav 2012; 2(4): 435–42.

CASE

Part I

14

Neurodegenerative Diseases Santanu Chakraborty, Prasad B. Hanagandi, Lázaro Luís Faria do Amaral

Clinical Presentation A 48-year-old woman presented with a history of gradually progressive left-sided weakness that initially involved her left leg, which began when she was 36 years old. She reported a two-year history of symptomatic progression including difficulty in speaking and progressive involvement of her left upper limb that was associated with inability to perform fine motor functions. She denied memory, sensory, visual, and swallowing impairments. No history of trauma or drug abuse was noted. Family history was non-contributory. On examination, mini mental testing was normal. She had spastic dysarthria with spastic tongue that showed some atrophy,

but no fasciculation. Bilateral asymmetric spasticity involving all four limbs was noted, with brisk deep tendon reflexes, bilateral ankle clonus, and Babinski sign. No sensory or cerebellar dysfunction was noted in the bedside testing. Routine hematologic and biochemical investigations including chest X-ray were normal. Hematologic investigations for HIV, HTLV-1, VDRL, antinuclear antibody, and abnormal vitamin B12 levels were negative. Visual evoked potentials were normal. Electrophysiologic studies including nerve conduction study and electromyography were normal and did not show any evidence of lower motor neuron involvement.

Imaging

Fig. 14.1 Axial FLAIR image of the brain at the level of motor cortex.

Fig. 14.2 Axial T1 magnetization transfer contrast sequence at the level of motor cortex.

Advanced Neuroradiology Cases, ed. Amaral et al. Published by Cambridge University Press. © Cambridge University Press 2016.

37

Part I. Neurodegenerative Diseases: Case 14

Fig. 14.4 Axial FLAIR image of the brain at the level of cerebral peduncles. Fig. 14.3 Axial GRE MR sequence of the brain at the level of motor cortex.

Fig. 14.5 Coronal FLAIR volumetric 3D reconstruction sequence of the brain at the level of body of lateral ventricles and frontoparietal convexities.

38

Part I. Neurodegenerative Diseases: Case 14 Fig. 14.6 Oblique reconstruction of 3D FLAIR image of the brain showing the corticospinal tracts.

Fig. 14.7 Superior view of the surface rendering image of the brain obtained from 3D volumetric FLAIR image showing the atrophied right motor cortex (arrow).

39

Part I. Neurodegenerative Diseases: Case 14

Primary Lateral Sclerosis Primary Diagnosis Primary lateral sclerosis Mills variety

Differential Diagnoses Amyotrophic lateral sclerosis Uncomplicated forms of hereditary spastic paraplegia Multiple sclerosis Adrenoleukodystrophy and metachromatic leukodystrophy Vitamin B12 deficiency HTLV-1 and HIV infection Compressive cervical myelopathy

Imaging Findings Fig. 14.1: Axial FLAIR image demonstrated abnormal hyperintense signal with focal cortical thinning and volume loss in the right motor cortex (arrow). Fig. 14.2: T-MTC (magnetization transfer contrast) sequence image is more sensitive than conventional FLAIR sequence in depicting the signal changes. Fig. 14.3: Axial GRE image showed T2 shortening in the motor cortex (arrow) representing iron deposition. Fig. 14.4: Axial FLAIR image showed asymmetric hyperintense signal in the right cerebral peduncle (arrow). Fig. 14.5: Coronal FLAIR volumetric 3D reconstruction sequence and Fig. 14.6: Oblique 3D reconstruction image also denoted abnormal signal changes (arrows) along the corticospinal tract. Fig. 14.7: Superior view of the surface rendering image of the brain obtained from 3D volumetric FLAIR image showed the atrophied right motor cortex (arrow).

Discussion On imaging, amyotrophic lateral sclerosis (ALS) has similar features to primary lateral sclerosis (PLS).The most distinctive PLS feature, however, is the lack of lower motor neuron involvement. On the contrary, ALS presents with upper and lower motor neuron involvement. Hereditary spastic paraplegia can have overlapping features but presents in a younger age group. It has various forms on inheritance with multiple pheno- and genotypes. Few multiple sclerosis (MS) cases present with diffuse thinning of corpus callosum, white matter changes, and volume loss. Clinically, the primary progressive and pure motor variant of MS can have a similar clinical presentation. However, the typical MRI demyelinating pattern involving the periventricular-pericallosal regions, callososeptal interface, brainstem, and spinal cord features helps in differentiating MS and its variants from PLS. Metachromatic leukodystrophy and adrenoleukodystrophy have multiple types and their adult forms tend to present earlier in life. The distribution pattern is symmetric with periventricular and central deep white matter changes and is associated with cognitive disturbances as the disease progresses. HTLV-1 and HIV infections can have similar clinical presentation and HTLV-1 can have imaging

40

appearances similar to PLS and ALS; however, serologic findings can exclude these infections as possible diagnoses. Vitamin B12 deficiency can often mimic the presentation of PLS but has predominant cord signal changes with posterior column involvement and symptoms. Apart from vitamin B12 deficiency, cervical compressive myelopathy is another treatable condition, which can manifest with a similar clinical presentation to PLS. Affected individuals present with intractable pain; cord compression is often evident on cervical spine imaging and lacks the brain imaging features of PLS. Primary lateral sclerosis is a rare form of motor neuron disease, which is typically differentiated from the more common amyotrophic lateral sclerosis by the absence of lower motor neuron involvement. It is characterized by spinobulbar spasticity due to upper motor neuron degeneration. In 1992, Pringle et al. described eight cases and proposed the current clinical diagnostic criteria. Primary lateral sclerosis is a disease of middle age, occurring most frequently during the fourth to sixth decades of life. It is slowly progressive and is common in both sexes. The most common PLS manifestation is leg weakness and spasticity, followed by spastic dysarthria. Although upper extremity symptoms develop over time, they are uncommon at presentation. Symptoms may unilaterally affect one side of the body, before progressing to involve the other side. Bulbar symptoms initially manifest as dysarthria, followed by dysphagia, and may evolve into emotional lability and inappropriate laughing or crying, the so-called pseudobulbar affect. The diagnostic criteria involves adult onset, negative family history, gradual progression, spasticity for more than three years, and dysfunction clinically limited to the corticospinal tracts. Hematologic and biochemical investigations are normal but help in excluding pertinent conditions such as vitamin B12 deficiency, HIV, HTLV-1, syphilis, Lyme disease, and some leukodystrophies that can closely resemble PLS. Pathologically, primary lateral sclerosis is limited to the primary motor cortex, corticobulbar, and corticospinal tracts. Although PLS lacks lower motor neuron involvement clinically, evidence of mild denervation with occasional fibrillation may be seen on electromyography. In addition, cortically evoked motor potentials are often absent in patients with PLS. When present, the central motor conduction time (CMCT) may be considerably prolonged, two to three times normal. There is no positive marker for PLS. Autopsy may help diagnosis by showing selective Betz cell loss in the central motor cortex. Amyotrophic lateral sclerosis and PLS share MR imaging features including T2 hyperintensity within the corticospinal tracts and atrophy of the motor and premotor cortex. The atrophy may be noticed in the CT images. The typical pattern of hyperintensity in bilateral corticospinal tracts in the coronal images is also described as a wine glass appearance. A band of hypointensity involving the cortex of precentral gyrus in gradient T2* or susceptibility-weighted images are also

Part I. Neurodegenerative Diseases: Case 14

described in the literature. Decreased fractional anisotropy is noted in the pyramidal tracts in diffusion tensor imaging. Criteria require disease duration of at least three years before a PLS diagnosis can be made: the lack of clinical involvement of lower motor neurons will help differentiate PLS from ALS.

Key Points  Primary lateral sclerosis is a rare form of motor neuron disease characterized by the absence of lower motor neuron involvement.  The disease is slowly progressive with normal life expectancy.  Characteristic imaging features include T2-FLAIR hyperintensity within the corticospinal tracts and atrophy of the motor and premotor cortex.

 Vitamin B12 deficiency and compressive cervical myelopathy are the two commonly treatable conditions that mimic PLS.

Suggested Reading Butman JA, Floeter MK. Decreased thickness of primary motor cortex in primary lateral sclerosis. AJNR Am J Neuroradiol 2007; 28(1): 87–91. Kuipers-Upmeijer J, de Jager AE, Hew JM, Snoek JW, van Weerden TW. Primary lateral sclerosis: clinical, neurophysiological, and magnetic resonance findings. J Neurol Neurosurg Psychiatry 2001; 71(5): 615–20. Kuruvilla A, Joseph S. ‘Wine Glass’ appearance: a unique MRI observation in a case of primary lateral sclerosis. Neurol India 2002; 50(3): 306–9. Pringle CE, Hudson AJ, Munoz DG, et al. Primary lateral sclerosis. Clinical features, neuropathology and diagnostic criteria. Brain 1992; 115(Pt 2): 495–520.

41

CASE

Part I

15

Neurodegenerative Diseases Taleb Al Mansoori, Prasad B. Hanagandi, Lázaro Luís Faria do Amaral

Clinical Presentation A 62-year-old, right-handed man presented with a 10-year history of frequent falls, gait disturbances, progressive abnormal eye movements, and dysarthria. Past medical history and family history was non-contributory. Clinical examination revealed mild contractions of the soft palate consistent with

palatal tremor. Ophthalmologic examination revealed vertical nystagmus with normal visual acuity. The examination of the upper and lower limbs demonstrated normal power, but also demonstrated a wide gait and inability to perform the heel-totoe walk. Hematologic profile, serum vitamin B12, and TSH evaluation was normal. Laboratory CSF analysis was negative.

Imaging (B)

(A)

Fig. 15.1 (A) Axial T2WI at the level of inferior olivary nuclei. (B) Coronal T2WI through the brainstem and lower medulla.

Advanced Neuroradiology Cases, ed. Amaral et al. Published by Cambridge University Press. © Cambridge University Press 2016.

43

Part I. Neurodegenerative Diseases: Case 15 Fig. 15.2 Axial FLAIR image at the level of inferior olivary nuclei.

Fig. 15.3 Axial DWI at the level of inferior olivary nuclei.

Fig. 15.4 Axial GRE image at the level of pons and brachium pontis.

44

Part I. Neurodegenerative Diseases: Case 15

(A)

(B)

Fig. 15.5 (A–B) Axial T1-MTC (magnetization transfer contrast) images at the level of inferior olivary nuclei.

45

Part I. Neurodegenerative Diseases: Case 15

Idiopathic Progressive Ataxia and Palatal Tremor Primary Diagnosis Idiopathic progressive ataxia and palatal tremor

Differential Diagnoses Causes of bilateral hypertrophic olivary degeneration (brainstem stroke, tumor, hemorrhage, or demyelinating diseases) Adult-onset Alexander disease with progressive ataxia and palatal tremor Metronidazole toxicity

Imaging Findings Fig. 15.1: (A) Axial T2WI and (B) Coronal images of the brainstem showed hyperintense signal changes in the bilateral inferior olivary nuclei (arrows). Fig. 15.2: Axial FLAIR image through the same level also demonstrated signal changes (arrows). Fig. 15.3: DWI showed no evidence of diffusion restriction. Fig. 15.4: GRE sequence of the proximal brainstem did not show any evidence of hemorrhage or mass lesion, similar to findings on T2WI (Fig. 15.1). Fig. 15.5: (A–B) Axial T1-MTC images show more pronounced signal changes (arrows).

Discussion Insidious and progressive onset of ataxia and associated palatal tremors, with no identifiable structural lesion in the GuillainMollaret triangle to explain the presence of abnormal T2FLAIR hyperintense signal changes in the inferior olivary nuclei that are more pronounced on T1-MTC images, as seen in this patient, are the key clinicoradiologic features suggestive of idiopathic progressive ataxia and palatal tremor (PAPT). Several etiologies such as stroke, hemorrhage, demyelination, and neoplasms resulting in bilateral hypertrophic olivary degeneration have been noted to cause symmetric T2-weighted and FLAIR signal abnormality changes that closely mimic PAPT; however, the lack of any predisposing factors excludes these pathologies from the diagnosis. Neurology and neuroradiology literature describes a specific phenotype of adult-onset Alexander disease that can have imaging features similar to PAPT. Other imaging findings that are commonly associated with Alexander disease include signal changes in the basal ganglia and periventricular white matter. It has been suggested that involvement of the white matter along the hypothalamo-pituitary axis is responsible for the hypothermia, primary ovarian failure, and hypothyroidism often seen in these patients. Inferior olivary nuclei changes have also been described in toxic-metabolic conditions such as metronidazole toxicity. The acute presentation, involvement of dentate nuclei and splenium of corpus callosum with diffusion restriction and T2-FLAIR signal changes are the classical findings associated with metronidazole toxicity. However, there is complete resolution of imaging findings and symptomatic

46

improvement after drug withdrawal, differentiating it from PAPT. Palatal tremor is a rare neurodegenerative movement disorder of uncertain etiology. It is divided into essential palatal tremor (EPT), which comprises approximately 25% of cases, and symptomatic palatal tremor (SPT). Symptomatic palatal tremor is subdivided into sporadic and familial forms and is associated with rhythmic movements of the anterior soft palate and ear clicks that are attributable to levator palatini activity. Several etiologies such as stroke, hemorrhage, and tumors cause sporadic SPT. A small subgroup of the sporadic SPT form has no predisposing factors and is categorized as an idiopathic form. Symptomatic palatal tremor and idiopathic PAPT result from a lesion affecting the inferior olive, red nucleus, and contralateral dentate nucleus that together form the Guillain-Mollaret triangle. Common associations with this condition are progressive ataxia, cerebellar dysfunction, visual disturbance, and palatal tremor at 2 Hz. Patients with familial PAPT present with corticospinal tract symptoms associated with significant brainstem and/or cervical cord atrophy. The familial form is also known as dark dentate disease due to abnormal iron accumulation/deposition. The familial variant does not have inferior olivary hypertrophy. The exact pathophysiology underlying palatal tremor in patients suffering familial PAPT syndromes also remains unclear. In most cases, there are no identifiable imaging features on MRI. However, if present, the typical MRI features include hypertrophy and hyperintensity of the inferior olivary nuclei that are best depicted on T2-weighted and proton density sequences. In some cases, cerebellar atrophy may be noted. In addition to these findings, familial SPT is associated with brainstem and spinal cord atrophy. Recent case reports have emphasized the importance of T1-MTC pulse sequence. The bright signal on T1-MTC images is more evident as compared to T2-weighted and FLAIR sequences performed on 1.5 and 3 Tesla scanners. The signal changes are attributable to myelin loss or transsynaptic degeneration, similar to hypertrophic olivary degeneration. Few case reports with hypometabolism involving the dentate-rubro-olivary pathway have been described with dopaminergic dysfunction on FDG-PET imaging. The symptoms of PAPT are disabling with no effective treatment.

Key Points  Idiopathic progressive ataxia and palatal tremor is a rare neurodegenerative disease spectrum encompassing sporadic and familial forms. The exact pathomechanism is not fully known but partly attributable to changes occurring in the Guillain-Mollaret triangle.  Symmetric hyperintense signal changes in the bilateral inferior olivary nuclei on T2-weighted and FLAIR images are the typical findings, although absent in the familial variant.

Part I. Neurodegenerative Diseases: Case 15

 Palatal tremor can be easily missed and clinical reexamination must be recommended in correlation with imaging findings and history of ataxia.  T1-MTC is a sensitive imaging sequence that detects the subtle and early changes not evident on T2-weighted and FLAIR sequences.

Suggested Reading Brinar VV, Barun B, Zadro I, Ozretic D, Habek M. Progressive ataxia and palatal tremor. Arch Neurol 2008; 65(9): 1248–9. Cilia R, Righini A, Marotta G, et al. Clinical and imaging characterization of a patient with idiopathic progressive ataxia and palatal tremor. Eur J Neurol 2007; 14(8): 944–6.

de Jong FJ, Boon AJ. Progressive ataxia and palatal tremor–two cases with an unusual clinical presentation and course. Parkinsonism Relat Disord 2012; 18(7): 904–5. Howard KL, Hall DA, Moon M, et al. Adult-onset Alexander disease with progressive ataxia and palatal tremor. Mov Disord 2008; 23(1): 118–22. Samuel M, Torun N, Tuite PJ, Sharpe JA, Lang AE. Progressive ataxia and palatal tremor (PAPT): clinical and MRI assessment with review of palatal tremors. Brain 2004; 127: 1252–68. Yared JH, Lopes BS, Rogerio RM, Amaral LL, Ferreira NF. Progressive ataxia and palatal tremor: T1-weighted with magnetization transfer pulse hyperintensity in the inferior olivary nucleus. Arq Neuropsiquiatr 2013; 71(4): 264–5.

47

CASE

Part I

16

Neurodegenerative Diseases Lázaro Luís Faria do Amaral, Bruno Shigueo Yonekura Inada, Leonardo Furtado Freitas, Prasad B. Hanagandi

Clinical Presentation A 40-year-old, white male presented with gait and balance problems. His medical history included chronic diarrhea since birth, normal psychomotor development until four years of age, absence seizures at five years of age, and developmental delay onset with learning difficulties at eight years of age. He had swelling around his ankles and knees. He had bilateral cataract surgery at the age of 20. Neurologic examination revealed dysdiadochokinesia, dysmetria, loss of joint position sense, and gait

instability, exaggerated deep tendon reflex, and bilateral 4/5 lower extremity weakness. His physical examination was remarkable for thickening of tendons (calcaneus and patellar), and xanthomas. Electromyography indicated presence of chronic demyelinating peripheral polyneuropathy in four limbs. Hematologic analysis revealed fasting cholesterol level of 180 mg/dl (< 200 mg/dl); LDL level of 122 mg/dl (< 130 mg/dl); HDL level of 78 mg/dl (> 40 mg/dl); and a triglyceride level of 124 mg/dl (< 150 mg/dl). Cerebrospinal fluid analysis was normal.

Imaging

Fig. 16.1 Axial T2WI through the dentate nuclei.

Fig. 16.2 Axial FLAIR sequence through the cerebellar hemispheres.

Advanced Neuroradiology Cases, ed. Amaral et al. Published by Cambridge University Press. © Cambridge University Press 2016.

49

Part I. Neurodegenerative Diseases: Case 16 Fig. 16.3 Axial T2 sequence through the level of basal ganglia.

Fig. 16.4 Axial T2WI through the midcervical cord.

50

Part I. Neurodegenerative Diseases: Case 16 Fig. 16.5 Midsagittal T2WI through the cervical spine.

51

Part I. Neurodegenerative Diseases: Case 16

Cerebrotendinous Xanthomatosis Primary Diagnosis Cerebrotendinous xanthomatosis

Differential Diagnosis Refsum disease

Imaging Findings Fig. 16.1: Axial T2WI through the dentate nuclei demonstrated bilateral, symmetric T2 hyperintensity involving the dentate nuclei. Severe atrophy of the cerebellum was noted with prominent cerebellar sulci and secondary enlargement of the fourth ventricle. Subtle T2 hyperintensity in the superior aspect of the olives was also noted. Fig. 16.2: Axial FLAIR image through the cerebellar hemispheres demonstrated bilateral symmetric cerebellar white matter FLAIR abnormality. Fig. 16.3: Axial T2 image through the basal ganglia demonstrated bilateral, symmetric abnormal T2 signal in the periventricular white matter that extends posteriorly from the periatrial white matter to the globus pallidi through the posterior limb of the internal capsule. Fig. 16.4: Axial T2WI image through the midcervical spinal cord demonstrated bilateral, symmetric T2 hyperintensity restricted to the lateral and posterior columns. Fig. 16.5: Midsagittal T2WI through the cervical spine demonstrated longitudinal involvement of the posterior column.

Discussion This patient has typical clinical presentation of cerebrotendinous xanthomatosis (CTX), including cerebellar symptoms, weakness, posterior column symptoms, bilateral muscular weakness, history of cataract surgery, and characteristic tendinous xanthomas in the prominent tendons. Bilateral, symmetric T2 abnormalities involving the dentate nuclei, periventricular white matter, internal capsule, globus pallidi, and lateral and posterior column are also consistent with the diagnosis. Refsum disease typically presents with retinitis pigmentosa, peripheral neuropathy, cerebellar ataxia, and elevated CSF protein levels, without an increase in the number of cells. The typical age of presentation is late in the first decade through the third decade of life. Cataracts and CNS imaging findings are not suggestive of Refsum disease. A rare autosomal recessive disorder, CTX is characterized by the accumulation of cholesterol and cholestanol, typically in the CNS and tendons. The mutation localizes in the CYP27 gene that encodes the mitochondrial 27-hydroxylase enzyme. Most patients have onset of symptoms in childhood and often present with chronic diarrhea and bilateral cataracts. Symptoms that are more specific usually manifest late in childhood, adolescence, or even in young adulthood, delaying diagnosis. Occurring in over 90% of patients with CTX, neurologic signs and symptoms generally manifest in the second and third decades of life and include cerebellar symptoms (ataxia, tachylalia, and dysarthria), pyramidal dysfunction, spastic paraparesis, tetraparesis, and polyneuropathy. Mental retardation, epilepsy, and psychiatric disorders may also occur.

52

Tendon xanthomas typically affect the Achilles tendon, but also can occur in the tendon of the quadriceps muscle, the triceps muscle, and finger extensors. Clinical triad of CTX is cataract, tendon xanthomas, and progressive neurologic impairment; however, this classic triad of symptoms may not be present in all CTX patients. The typical CTX imaging appearance, present in the majority of patients, consists of bilateral, symmetric T2 hyperintensity involving the dentate nuclei and adjacent cerebellar white matter. Other very characteristic but less common MRI signs are bilateral, symmetric T2 abnormality involving the globus pallidi and adjacent internal capsules, cerebral peduncles, and olives. These suggestive imaging findings are frequently accompanied by additional, non-specific findings such as focal or diffuse cerebral and cerebellar white matter T2 signal abnormality with varying degree of cerebral and cerebellar atrophy. On MR spectroscopy, low NAA peak, high lactate peak, and a prominent lipid peak can be seen with short TE spectra. Laboratory findings include normal or slightly increased levels of serum cholesterol and a significant increase in urine cholestanol. Cerebrospinal fluid analysis may show increased protein levels and cholestanol. Typically, the diagnosis is suggested by demonstrating abnormally high bile alcohol levels in urine. The confirmation can be done by validating the lack of 27-hydroxylase enzyme activity in cultured fibroblasts, liver, and leukocytes, and through DNA mutational analysis. Microscopic examination of the dentate nucleus and the cerebellar white matter shows thinning with significant neuronal loss and demyelination cracks with cholesterol crystals, reactive astrocytosis, hemosiderin deposits, and calcifications. Large fat accumulation is apparent within mononuclear cells with a foamy cytoplasm, which generally accumulate around blood vessels.

Key Points  Presentation of the classic triad of cataract, tendon xanthomas, and progressive neurologic impairment should prompt the diagnosis of CTX.  Bilateral, symmetric T2 hyperintensity involving the dentate nuclei, globus pallidi, periventricular, and white matter are most common imaging findings. Abnormal T2 signal in the cerebral peduncles, olives, and lateral and posterior columns are uncommon but known imaging abnormalities associated with CTX. Association of cerebellar atrophy and chronic diarrhea are typical findings in this disease.

Suggested Reading Barkhof F, Verrips A, Wesseling P, et al. Cerebrotendinous xanthomatosis: the spectrum of imaging findings and the correlation with neuropathologic findings. Radiology 2000; 217(3): 869–76. Choksi V, Hoeffner E, Karaarslan E, et al. Infantile refsum disease: case report. AJNR Am J Neuroradiol 2003; 24: 2082–4.

Part I. Neurodegenerative Diseases: Case 16 Embiruçu EK, Otaduy MCG, Taneja AK, et al. MR spectroscopy detects lipid peaks in cerebrotendinous xanthomatosis. AJNR Am J Neuroradiol 2010; 31: 1347–9.

Pudhiavan A, Agrawal A, Sangit C, et al. Cerebrotendinous xanthomatosis - the spectrum of imaging findings. J Radiol Case Rep 2013; 7(4): 1–9.

Prayer D, Grois N, Prosch H, et al. MR imaging presentation of intracranial disease associated with Langerhans cell histiocytosis. AJNR Am J Neuroradiol 2004; 25: 880–91.

Sedrak P, Ketonen L, Hou P, et al. Erdheim-Chester disease of the central nervous system: new manifestations of a rare disease. AJNR Am J Neuroradiol 2011; 32: 2126–31.

53

CASE

Part I

17

Neurodegenerative Diseases Fabrício Guimarães Gonçalves

Clinical Presentation A 46-year-old woman, with a three-year history of subtle changes in coordination, minor involuntary movements, difficulty in mental planning, and irritable mood presented for neurologic evaluation. She reported that her symptoms progressed to non-voluntary movements (choreoathetoid

movements), dystonia, and frequent falls. Family history revealed that her grandfather died at 56 years of age, with a very similar presentation of movement disorder that lasted approximately 17 years. Physical examination revealed no significant abnormalities. Hematologic analysis was unremarkable.

Imaging Fig. 17.1 Axial FLAIR.

Fig. 17.2 Axial T2WI.

Fig. 17.3 Coronal T2-weighted MR image.

Advanced Neuroradiology Cases, ed. Amaral et al. Published by Cambridge University Press. © Cambridge University Press 2016.

55

Part I. Neurodegenerative Diseases: Case 17

Huntington Disease Primary Diagnosis Huntington disease

Differential Diagnoses Bilateral chronic caudate nuclei infarcts Multiple system atrophy type P (putaminal) Hypoxic ischemic injury

Imaging Findings Fig. 17.1: Axial FLAIR and Fig. 17.2: Axial T2WI demonstrated diffuse brain atrophy as evidenced by enlargement of the subarachnoid spaces, particularly in the frontal regions (unexpected in a patient this age), and bilateral lateral ventricular enlargement, particularly the frontal horns, due to severe atrophy of the head of the caudate nuclei (thin arrows). Putaminal atrophy (large arrows) is also noted. Fig. 17.3: Coronal T2WI better showed compensatory enlargement of the frontal horns of the lateral ventricles (arrows).

Discussion In a middle-aged patient, the progressive onset of a movement disorder and behavior changes in combination with imaging findings demonstrating bilateral caudate nucleus head and putaminal volume loss (striatal atrophy) is highly suggestive of a diagnosis of Huntington disease (HD). Caudate infarcts are rare, representing approximately 1% of stroke (ischemic and hemorrhagic). Caudate nuclei infarction could result from involvement of any of the three arterial systems: 1) Heubner’s artery, a branch of the anterior cerebral artery, supplies the inferior portion of the head of the caudate and the anterior limb of the internal capsule; 2) anterior lenticulostriate arteries originating from the proximal part of the anterior cerebral artery, which supply the anterior portions of the caudate nuclei; and 3) the lateral lenticulostriate arteries from the middle cerebral artery, which supply the major portions of the caudate and the anterior internal capsule and putamen. Although volume loss is an expected finding in chronic infarcts, bilateral symmetric caudate head infarct is very unlikely to be of vascular etiology. In addition, severe brain atrophy in this age is very suggestive of underlying neurodegenerative disease. Multiple system atrophy type P (MSA-P) can cause putaminal volume loss, but not atrophy of the caudate nuclei. Additionally, patients with MSA-P are commonly in their sixth decade and present with Parkinsonism, bradykinesia, tremor, rigidity, and gait abnormalities. Owing to the discrepancy of imaging findings and clinical findings, the diagnosis of MSA-P is less likely. Hypoxic ischemic injury (HII) is a known cause of bilateral basal ganglia volume loss, particularly in the chronic stage. It is also known that HII can involve not only the basal ganglia, but

56

also the cerebral cortex and watershed areas of the brain. In adults, the most common causes of HII are cardiac arrest, drowning, suicidal attempt, or strangulation, which can induce a variable degree of neurologic abnormalities depending on the degree of injury. Owing to the absence of a consistent history, the possibility of HII is less likely. Huntington disease is an autosomal dominant inherited neurodegenerative disorder that affects young adult patients with age ranging from 30 to 50 years. The disease is caused by a mutation in the HTT gene located in chromosome 4p16:3 that encodes the huntingtin protein. Although huntingtin is in multiple body tissues, it is most highly concentrated in the brain. Mutations to the huntingtin protein, implicated in neural transmission, protein transport, and apoptosis, lead to progressive neurodegeneration due to impairment of energy metabolism. Death usually occurs within one to two decades after the symptom onset. The following clinical findings are suggestive of HD: progressive movement disorder (chorea); mental disturbances including cognitive decline, changes in personality and/or depression; and a positive family history. The natural history of patients with HD can be divided into three different stages. Initially, patients may experience clumsiness, agitation, irritability, apathy, anxiety, disinhibition, delusions, hallucinations, abnormal eye movements, and depression. In the following stage, chorea and involuntary movements become more prominent and voluntary activity, dysarthria, and dysphagia worsens. Later signs include rigidity, bradykinesia, severe chorea (less common), and an increase in the severity of motor disabilities. The affected individual, during the last stage, is often totally dependent, mute, and incontinent. Neuropathologic features of HD primarily include a selective degeneration of GABAergic neurons in the caudate and putamen. The preferential degeneration of neurons of the indirect pathway of movement control in the basal ganglia provides the neurobiologic basis for chorea. Other regions of the brain that can be affected include the substantia nigra, hippocampus, and various regions of the cortex. Any imaging modality suitable for structural brain evaluation is useful to support the clinical diagnosis of HD. Owing to its superior spatial and contrast resolution, MRI is the imaging modality of choice. The hallmark imaging feature of HD is caudate volume loss, which begins in the dorsal caudate and proceeds ventrally and laterally to encompass the putamen. Owing to caudate atrophy, there is enlargement of the frontal horns, giving them a box-like configuration. Caudate atrophy can be quantified using the frontal horn width-tointercaudate distance ratio (FH/CC) as well as the intercaudate distance-to-inner table width ratio (CC/IT). Additionally, diffuse cortical atrophy is present, particularly in the frontal regions. Signal changes in the caudate and putamina may be noted, representing gliotic tissue. Several studies using advanced MRI techniques have demonstrated brain changes in pre-HD patients, especially in white

Part I. Neurodegenerative Diseases: Case 17

matter striatum and brain volumes. Magnetic resonance imaging can demonstrate significant striatal atrophy as many as 11 years prior to clinical onset of the disease. Spectroscopy can show increased lactate peaks in the basal ganglia and in the occipital cortex of affected individuals. Markers of neuronal degeneration, decreased NAA/creatine and increased choline/creatine levels, can also be found. At present, there is no cure, nor any effective treatment to delay symptom onset or slow progression, and most current treatments aim to minimize symptoms.

Key Points  Huntington disease should be suspected in young patients with progressive movement disorder and behavior changes in combination with bilateral caudate nucleus head and putaminal volume loss (striatal atrophy).  Neuropathologic features of HD include a selective degeneration of GABAergic neurons of the indirect pathway of movement control in the basal ganglia.

 Any imaging modality suitable for structural brain evaluation is useful to support the clinical diagnosis of HD, preferably MRI.  The hallmark imaging feature of HD is caudate volume loss, which begins in the dorsal caudate and proceeds ventrally and laterally to encompass the putamen with enlargement of the frontal horns, giving them a box-like configuration.

Suggested Reading Bohanna I, Georgiou-Karistianis N, Hannan AJ, Egan GF. Magnetic resonance imaging as an approach towards identifying neuropathological biomarkers for Huntington’s disease. Brain Res Rev 2008; 58(1): 209–25. Ha AD, Fung VSC. Huntington’s disease. Curr Opin Neurol 2012; 25(4): 491–8. Klöppel S, Henley SM, Hobbs NZ, et al. Magnetic resonance imaging of Huntington’s disease: preparing for clinical trials. Neuroscience 2009; 164(1): 205–19. Kumral E, Evyapan D, Balkir K. Acute caudate vascular lesions. Stroke 1999; 30(1): 100–8.

57

CASE

Part I

18

Neurodegenerative Diseases Asim K. Bag, Victor Hugo Rocha Marussi, Lázaro Luís Faria do Amaral

Clinical Presentation A 45-year-old man, with no known familial or past history of neurologic disorders, presented with a two-year history of progressive neurocognitive and behavioral changes. During this time, his demeanor had become more placid and less argumentative. He was less conversational and lost interest in maintaining personal hygiene. His appetite noticeably changed. Whereas previously he was particular about his diet, he began eating everything on his plate, including sweets.

He developed progressive language problems and became mute. During the course of this disease, he also developed Parkinsonian features without any chorea-like movement. On neuropsychologic evaluation, there were prominent decreases in attention, concentration, and executive function. A neurodegenerative disease was diagnosed and MRI was ordered (shown below). Hematologic blood studies demonstrated normal-shaped red blood cells, without any acanthocytes.

Imaging

Fig. 18.1 Axial T2WI sequence through the level of the basal ganglia.

Fig. 18.2 Axial FLAIR sequence through the level of the basal ganglia inferior to Fig. 18.1.

Advanced Neuroradiology Cases, ed. Amaral et al. Published by Cambridge University Press. © Cambridge University Press 2016.

59

Part I. Neurodegenerative Diseases: Case 18

Frontotemporal Lobar Degeneration Associated with Fused in Sarcoma Protein Primary Diagnosis Frontotemporal lobar degeneration associated with fused in sarcoma protein

Differential Diagnoses Frontotemporal lobar degeneration-tau Frontotemporal lobar degeneration-TDP Huntington disease Neuroacanthocytosis

Imaging Findings Fig. 18.1: Axial T2WI through the level of the basal ganglia demonstrated diffuse symmetric brain atrophy that is more severe in the bilateral frontal lobes. In addition, severe atrophy of the head of the caudate nuclei with unusual prominence of the frontal horns is noted. Fig. 18.2: Axial FLAIR sequence through the level of the basal ganglia inferior to Fig. 18.1 demonstrates diffuse symmetric brain atrophy that is more severe in the bilateral frontal and temporal lobes. In addition, the atrophy of the head of the caudate nuclei is noted.

Discussion Frontotemporal lobar degeneration-tau (FTLD-tau) and frontotemporal lobar degeneration-TDP (FTLD-TDP) are the two most common pathologic findings in frontotemporal dementia (FTD); however, they are not associated with striatal atrophy. Huntington disease (HD), an autosomal dominant neurodegenerative disease, is characterized by severe caudate atrophy and a characteristic abnormal movement. As this patient did not have any known family history or chorea-like movement, HD was excluded. Lack of blood acanthocytes rules out neuroacanthocytosis, a heterogeneous group of genetically defined HD-like neurodegenerative diseases characterized by red blood cell acanthocytosis and chorea-like movements. Frontotemporal dementia, a cluster of clinical syndromes resulting from the preferential neurodegeneration of the orbitomedial frontal lobe and temporal lobe in varying combinations, is extremely heterogeneous in terms of clinical presentation, histopathologic findings, imaging findings, and genetics. There are three well-known clinical syndromes categorized under FTD: 1) behavioral variant (bv-FTD), 2) progressive non-fluent aphasia (PNFA), and 3) semantic dementia (SD). Behavioral changes are the dominant clinical findings in bv-FTD, which preferentially affects the frontal lobe. The breakdown of spontaneous speech and difficulty understanding language are the key clinical findings in PNFA, which preferentially affects the perisylvian area of the dominant hemisphere, and fluent speech with prominent anomia, word finding difficulty, and difficulty in word recognition are the typical findings in SD due to preferential involvement of the dominant temporal lobe. Approximately 15% of patients with bv-FTD have associated motor neuron disease. This

60

variant is recognized as a special subtype of FTD (FTDMND) as it has a specific genetic linkage. In addition to these four clinical variants, progressive supranuclear palsy syndrome (PSPS) and corticobasal syndrome (CBS) are linked to a number of molecular pathologies targeting the frontal and temporal lobes that are collectively called frontotemporal lobar degeneration (FTLD). Parkinsonian features are noted in all FTD subtypes. Unlike FTD, which is a clinical term, FTLD incorporates specific aspects of the pathologic and histopathologic findings. Similar to the three well-recognized clinical syndromes, FTLD can be subclassified: 1) FTLD-tau, characterized by tubulinassociated unit immunoreactive inclusions; 2) FTLD-TDP-43, characterized by transactive response DNA binding protein of 43 kDa immunoreactive inclusions; and 3) FTLD-FUS, characterized by fused in sarcoma immunoreactive inclusions. Both the TDP and FUS proteins are ubiquitinated and part of FTLD-U. Cases that are only positive for ubiquitinimmunoreactive inclusions are categorized as FTLD-UPS (UPS, i.e., ubiquitin proteasome system). Almost all of the FTD variants can be characterized by the three different FTLD patterns. Pick disease, corticobasal degeneration, and progressive supranuclear palsy (PSP) are pathologically categorized as FLTD-tau, as are other extremely rare histologic subtypes such as argyrophilic grains disease, sporadic multisystem tauopathy, and diffuse neurofibrillary tangle dementia with calcifications. There is strong clinical association of each clinical subtype of FTD syndrome with all FTLD subtypes. Almost all cases of CBS and PSP are associated with FTLD-tau, whereas all of the FTD-MNDs are associated with FTLD-TDP. More than 80% of cases of SD are associated with FTLD-TDP, and up to 70% of PNFA cases are associated with FTLD-tau. Patients with bv-FTD are predominantly categorized as FTLD-tau or FTD-TDP. Young-onset bv-FTD is typically associated with FTLD-FUS. Originally described in association with human cancers, FUS belongs to FET/TET family of multifunctional DNA/ RNA binding proteins. Fused in sarcoma proteinopathies, linked to both amyotrophic lateral sclerosis (ALS) and FTLD, are characterized by the presence of inclusions that are immunoreactive for FUS in the cytoplasm and nuclei of both glial and neuronal cells. How FUS accumulation leads to neurodegeneration is still not well understood. FTLD-FUS clinically manifests as bv-FTD or FTD-MND characterized by severe progressive psychobehavioral alteration, negative family history, and striking striatal atrophy. FTLD-FUS encompasses approximately 10% of FTLD-U and approximately 3% of FTD syndrome. It has been shown that FTLDFUS is associated with bilateral atrophy of the caudate nuclei, which can be used as an antemortem biomarker for FTLDFUS subtype. In addition, FTLD-FUS is associated with bilateral symmetric extratemporal atrophy rather than the asymmetric atrophy of the frontal and temporal lobes in other subtypes of FTLD.

Part I. Neurodegenerative Diseases: Case 18

Key Points  Frontotemporal dementia-FUS is a rare subtype of FTLD that predominantly manifests as bv-FTD.  Frontotemporal dementia-FUS should be considered in patients with early-onset FTD, negative family history, and atrophy of the caudate head on MRI.

Suggested Reading Josephs KA, Whitwell JL, Parisi JE, et al. Caudate atrophy on MRI is a characteristic feature of FTLD-FUS. Eur J Neurol 2010; 17(7): 969–75. Karageorgiou E, Miller BL. Frontotemporal lobar degeneration: a clinical approach. Semin Neurol 2014; 34(2): 189–201.

Rohrer JD, Lashley T, Schott JM, et al. Clinical and neuroanatomical signatures of tissue pathology in frontotemporal lobar degeneration. Brain 2011; 134(Pt 9): 2565–81. Seelaar H, Klijnsma KY, de Koning I, et al. Frequency of ubiquitin and FUS-positive, TDP-43-negative frontotemporal lobar degeneration. J Neurol 2010; 257(5): 747–53. Siuda J, Lewicka T, Bujak M, et al. ALS-FTD complex disorder due to C9ORF72 gene mutation: description of first Polish family. Eur Neurol 2014; 72(1-2): 64–71. van Blitterswijk M, DeJesus-Hernandez M, Niemantsverdriet E, et al. Association between repeat sizes and clinical and pathological characteristics in carriers of C9ORF72 repeat expansions (Xpansize-72): a cross-sectional cohort study. Lancet Neurol 2013; 12(10): 978–88.

61

CASE

Part I

19

Neurodegenerative Diseases Prasad B. Hanagandi, Rahul J. Vakharia, Lázaro Luís Faria do Amaral

Clinical Presentation A 26-year-old man presented with a 14-year history of gradual onset ataxia with progressive paraparesis, and preserved higher mental functions. The patient was born of consanguineous marriage. Clinical examination revealed ataxia, hyperreflexia, and sensory neuropathy in both lower limbs. Posterior column conduction disturbances were noted in both lower extremities. Hematologic evaluation of vitamin B12, thyroid function, ceruloplasmin, creatine kinase, and very long chain fatty acid deficiency was negative. Analysis of cerebral CSF revealed presence of lactate.

Imaging

Fig. 19.1 Axial T2WI at the level of lateral ventricles. Fig. 19.2 Axial FLAIR image at the level of lateral ventricles.

Advanced Neuroradiology Cases, ed. Amaral et al. Published by Cambridge University Press. © Cambridge University Press 2016.

63

Part I. Neurodegenerative Diseases: Case 19 Fig. 19.3 Axial T2WI of the posterior fossa at the level of pons and superior and middle cerebellar peduncles.

(A)

(B)

Fig. 19.4 (A–B) Axial T2WI of the medulla at the inferior cerebellar peduncles and pyramid.

64

Part I. Neurodegenerative Diseases: Case 19

(A)

Fig. 19.5 (A) Sagittal T2WI of the upper thoracic spinal cord. (B) Axial T2WI of the same area.

(B)

Fig. 19.6 MR spectroscopy of the periventricular white matter.

65

Part I. Neurodegenerative Diseases: Case 19

Leukoencephalopathy with Brainstem and Spinal Cord Involvement and Lactate Elevation Primary Diagnosis Leukoencephalopathy with brainstem and spinal cord involvement and lactate elevation

Differential Diagnoses Vitamin B12 deficiency Spinal variant cerebrotendinous xanthomatosis Adult-onset autosomal dominant leukodystrophy Mitochondrial disorders

Imaging Findings Fig 19.1: Axial T2WI and Fig 19.2: Axial FLAIR sequences showed diffuse and nearly symmetric hyperintense white matter changes in the bilateral periventricular regions with relative sparing of subcortical U fibers. Fig 19.3: Axial T2WI of the posterior fossa revealed hyperintense signal changes in the superior cerebellar peduncles (arrowheads) and intraparenchymal trajectories of trigeminal nerves (arrow). Fig 19.4: (A) Symmetric T2-weighted hyperintense signal changes were also noted in the inferior cerebellar peduncles (arrows) and pyramidal tracts (arrowheads) and along the decussation of the pyramid and medial lemniscus (B). Fig 19.5: (A) Sagittal T2WI of the upper thoracic spinal cord demonstrated linear, hyperintense signal changes with involvement of the posterior columns. (B) Axial T2WI of the same area demonstrated hyperintense signal changes in lateral corticospinal tracts (arrows) and posterior column (arrowhead). Fig 19.6: MR spectroscopy from the periventricular white matter shows an inverted doublet lactate peak at 1.3 ppm at TE 135 ms.

Discussion The MRI findings of nearly symmetric involvement of the posterior limbs of internal capsule, pyramidal tract involvement, trajectories of the trigeminal nucleus, pyramidal involvement in the medulla, and signal changes in the dorsal and lateral corticospinal tracts are classic imaging features of leukoencephalopathy with brainstem and spinal cord involvement and lactate elevation (LBSL). Although the spinal cord imaging findings mimic vitamin B12 deficiency, the brainstem imaging characteristics and negative hematologic tests for vitamin B12 deficiency exclude this as a diagnosis. Patients with the spinal variant of cerebrotendinous xanthomatosis also exhibit similar spinal cord imaging features; however, they present with a history of diarrhea, mental retardation, and cataract – which were absent in our patient. Tendon xanthomas are another clinical pathognomonic feature of this LBSL. Mitochondrial diseases tend to have varied brainstem imaging; however, involvement of specific areas of brainstem confirms a LBSL diagnosis. Adult-onset autosomal dominant leukodystrophy (ADLD)

66

predominantly manifests with autonomic symptoms but can have similar diffuse supratentorial imaging findings and cord atrophy; however, the brainstem imaging features seen in this case are very characteristic for LBSL. Mutations in DARS2, the gene encoding mitochondrial aspartyl-tRNA synthase, cause LBSL, an autosomal recessive disorder. Van der Knaap and colleagues first described LBSL in 2003. The disease begins in early childhood or adolescence, with gradually progressive cerebellar ataxia and motor and sensory neuropathy due to involvement of the dorsal columns and lateral corticospinal tracts. Seizures and cognitive decline that progresses to mental retardation are delayed features of the disease. Based on MR imaging, the disease has major and supportive criteria. For imaging diagnosis of LBSL, all the major criteria and at least one supportive criteria have to be met along with high signal changes on T2-weighted MR images and low signal on corresponding T1-weighted images.

Major LBSL criteria  Homogeneous, spotty, or non-homogeneous white matter involvement with sparing of the U fibers  Lateral corticospinal tract and dorsal column signal abnormalities  Involvement of pyramids in the medulla oblongata

Supportive LBSL criteria include abnormal signal in:         

Posterior limbs of internal capsule Splenium of corpus callosum Superior cerebellar peduncles Medial lemniscus of brainstem Inferior cerebellar peduncles Intraparenchymal trajectories of trigeminal nerves Mesencephalic nucleus of trigeminal nerve Anterior spinocerebellar tracts in the medulla Cerebellar white matter with subcortical preponderance Very often, elevated CSF and serum lactate levels may not be detected. If the clinical presentation is compatible with spinocerebellar ataxia, and all the MR imaging criteria are fulfilled, then genetic testing for DARS2 mutation is unwarranted. Treatment options for this disease are mainly supportive care consisting of rehabilitation and physical therapy.

Key Points  Nearly symmetric involvement of the supratentorial white matter, pyramidal trajectories of the trigeminal, pyramidal involvement in the medulla, and signal changes in the dorsal and lateral corticospinal tracts of spinal cord are characteristic neuroimaging features of LBSL.  The disease is attributable to DARS2 gene mutation.  Cerebrospinal fluid lactate levels can be variable, based on the severity of the disease.

Part I. Neurodegenerative Diseases: Case 19

Suggested Reading Labauge P, Roullet E, Boespflug-Tanguy O, et al. Familial, adult onset form of leukoencephalopathy with brain stem and spinal cord involvement: inconstant high brain lactate and very slow disease progression. Eur Neurol 2007; 58(1): 59–61. Scheper GC, van der Klok T, van Andel RJ, et al. Mitochondrial aspartyl-tRNA synthetase deficiency causes leukoencephalopathy with brain stem and spinal cord involvement and lactate elevation. Nat Genet 2007; 39: 534–9. Serkov SV, Pronin IN, Bykova OV, et al. Five patients with a recently described novel leukoencephalopathy with brainstem and

spinal cord involvement and elevated lactate. Neuropediatrics 2004; 35(1): 1–5. van der Knaap MS, Valk J. Leukoencephalopathy with brain stem and spinal cord involvement and elevated white matter lactate. In: van der Knaap MS, Valk J, eds. Magnetic Resonance of Myelination and Myelin Disorders, 3rd edn. Berlin, New York: Springer; 2005: 510–18. van der Knaap MS, van der Voorn P, Barkhof F, et al. A new leukoencephalopathy with brainstem and spinal cord involvement and high lactate. Ann Neurol 2003; 53: 252–8.

67

CASE

Part I

20

Neurodegenerative Diseases Satya Patro, Prasad B. Hanagandi, Lázaro Luís Faria do Amaral, Antônio José da Rocha

Clinical Presentation A 49-year-old woman presented with a 21-year history of progressive worsening of resting and intentional tremors involving her head and upper limbs. She reported that her initial neurologic symptoms were followed by gait ataxia, incoordination, ophthalmoplegia, and dysphagia with generalized tonic-clonic seizures and altered sleep rhythm. Neurologic exam revealed generalized muscle atrophy. History of similar neurologic symptoms was also noted in her older sibling. Routine hematologic analysis for vitamin B12 level and HIV and HTLV serologies were unremarkable. Molecular genetic analysis revealed mutation in the MJD1 gene.

Imaging (A)

(B)

(C)

Fig. 20.1 (A) Axial T2-weighted, (B) Axial FLAIR, and (C) Axial GRE images at the level of pons and middle cerebellar peduncles.

Advanced Neuroradiology Cases, ed. Amaral et al. Published by Cambridge University Press. © Cambridge University Press 2016.

69

Part I. Neurodegenerative Diseases: Case 20 Fig. 20.2 Midsagittal T1WI of the brain through the corpus callosum and pons.

Fig. 20.3 Coronal T2WI of the brain through superior cerebellar peduncles.

70

Part I. Neurodegenerative Diseases: Case 20

(A)

(B)

Fig. 20.4 (A) Axial T1WI of the brain at the level of lateral ventricles and (B) Axial T1WI of the brain at the level of corona radiata.

71

Part I. Neurodegenerative Diseases: Case 20

Machado-Joseph Disease Primary Diagnosis Machado-Joseph disease

Differential Diagnoses Sporadic olivopontocerebellar atrophy Dentatorubral-pallidoluysian atrophy Spinocerebellar ataxia type-6 Fragile X-associated tremor/ataxia syndrome Cerebrotendinous xanthomatosis

Imaging Findings Fig. 20.1: (A) Axial T2WI through the brainstem and cerebellum demonstrated abnormal hyperintense signal changes in the pons giving the appearance of hot-cross-bun sign. (B) Axial FLAIR and (C) Axial GRE sequence also demonstrated hot-cross-bun sign (white arrow). Note marked atrophy of the dentate nuclei and middle cerebellar peduncles (black arrows) with significant cerebellar volume loss. Fig. 20.2: Sagittal T1WI showed pontine tegmental volume loss with flattening (arrow). Fig. 20.3: Coronal T2WI showed thinning of the superior cerebellar peduncles (arrows). Fig. 20.4: (A–B) Axial T1WI demonstrated generalized atrophy of both cerebral hemispheres.

Discussion The imaging findings in this patient’s case overlap with multiple neurodegenerative conditions such as sporadic olivopontocerebellar atrophy (sOPCA), dentatorubral-pallidoluysian atrophy (DRPLA), and spinocerebellar ataxia type-6 (SCA type-6). Sporadic olivopontocerebellar atrophy, a condition with a similar but more rapid clinical course, usually spares the dentate nuclei, red nuclei, and superior cerebellar peduncles, frequently involved in patients with Machado-Joseph disease (MJD). Thus, a diagnosis of MJD is more likely based on the characteristic imaging features. Other spinocerebellar neurodegenerative conditions such as DRPLA and SCA type-6 with similar involvement of the dentatorubral system and pallidosubthalamic system can be differentiated from MJD by the lack of ophthalmoplegia and atrophy of the facial nerve colliculus. Abnormal signal changes detected in the thalamus, brainstem, and cerebral white matter in DRPLA may be helpful in differentiating DRPLA from MJD. Fragile X-associated tremor/ataxia syndrome (FXTAS) is a sex-linked genetic disease with a male preponderance and middle-age onset (50 years of age) characterized by neuroimaging findings of atrophy and T2-weighted hyperintensity in the cerebellar hemispheres close to dentate nuclei and middle cerebellar peduncles. FXTAS demonstrates diffuse white matter hyperintensity – distinguishing it from MJD. Cerebrotendinous xanthomatosis (CTX) is an inherited metabolic condition resulting from the deposition of cholesterol and cholestanol in the nervous system. Early-onset clinical manifestations of CTX include low intellectual performance,

72

ataxia, spastic paraparesis, peripheral neuropathy, and tendon xanthomas. Unique neuroimaging features of CTX patients such as T1 hyperintensity/T2-weighted hypointensity in the dentate nucleus (hyperdense on CT) differentiate it from MJD; thus, CTX can be excluded as a potential diagnosis in this case. Machado-Joseph disease, also known as spinocerebellar ataxia type-3, is a hereditary autosomal dominant neurodegenerative disease that involves multiple systems. It was first described by Nakano et al. in a Portuguese-American family, who descended from Guilherme Machado, and emigrated from the Azores Island to Massachusetts. Since then, it has been described in other global populations. It is the most common and familial type of spinocerebellar ataxia caused by the expansion of CAG repeats in the MJD1 gene, now officially called ATXN3 gene located on chromosome 14q32.1. The disease is manifested in affected individuals between the ages of 25 and 55 years with a mean age of 40 years. The common clinical manifestations include progressive ataxia, ophthalmoplegia, extrapyramidal signs (dystonia and Parkinsonism), peripheral neuropathy, and other non-motor symptoms. Neuropathologically, selective brain areas including the dentate nucleus of the cerebellum, the nucleus dorsalis of Clarke in the spinal cord, cranial motor nerve nuclei, pontine nuclei, and substantia nigra are reported to show degeneration. The most striking imaging features demonstrated in patients with MJD are moderate cerebellar atrophy and marked brainstem atrophy, especially in the tegmentum region of the pons. Other common imaging findings include atrophy of dentate nuclei, middle, and superior cerebellar peduncles with marked dilatation of the fourth ventricle, which can be explained by the atrophy of the brainstem and involvement of the afferent cerebellar tracts from the brainstem to cerebellum. Flattening of the facial colliculus is another finding that has been described in the literature. The non-specific T2-weighted high signal intensity in the transverse pontine fibers (hotcross-bun sign) observed in various other degenerative cerebellar diseases, especially multiple system atrophy (MSA) (see Part I: Cases 8 and 9), also is described in MJD. There can be associated moderate to severe atrophy of the frontal and temporal lobes as well as marked atrophy of the globus pallidi, which correlates with the severity, and duration of the disease. Various SPECT and PET studies have demonstrated hypoperfusion and hypometabolism in the cerebral cortices involving the frontal, lateral temporal, parietal, and occipital lobes.

Key Point  The clinical features specific to MJD, early age of onset and characteristic MRI findings of significant volume loss involving the pons, dentate nuclei, middle and superior cerebellar peduncles, atrophy of the frontotemporal lobes and globus pallidi (in late stages) are helpful in differentiating this disease entity from other neurodegenerative conditions.

Part I. Neurodegenerative Diseases: Case 20

Acknowledgement The authors would like to thank personally Santa Casa de Misericórdia de São Paulo, Brazil for the images.

Suggested Reading Etchebehere EC, Cendes F, Lopes-Cendes I, et al. Brain single-photon emission computed tomography and magnetic resonance imaging in Machado-Joseph disease. Arch Neurol 2001; 58: 1257–63. Murata Y, Yamaguchi S, Kawakami H, et al. Characteristic magnetic resonance imaging findings in Machado-Joseph disease. Arch Neurol 1998; 55(1): 33–7.

Nakano KK, Dawson DM, Spence A. Machado disease: a hereditary ataxia in Portuguese immigrants to Massachusetts. Neurology 1972; 22: 49–55. Ogawa Y, Ito S, Makino T, et al. Flattened facial colliculus on magnetic resonance imaging in Machado-Joseph disease. Mov Disord 2012; 27(8): 1041–6. Tokumaru AM, Kamakura K, Maki T, et al. Magnetic resonance imaging findings of Machado-Joseph disease: histopathologic correlation. J Comput Assist Tomogr 2003; 27(2): 241–8. Yoshizawa T, Watanabe M, Frusho K, et al. Magnetic resonance imaging demonstrates differential atrophy of pontine base and tegmentum in Machado-Joseph disease. J Neurol Sci 2003; 215: 45–50.

73

CASE

Part II

21

Neurovascular Diseases Asim K. Bag, Lázaro Luís Faria do Amaral

Clinical Features An 18-year-old previously healthy woman presented complaining of a headache. She also reported a several-month history of gradually worsening seizures. She noted that seizures were gradually becoming resistant to antiepileptic medication. She does not use any illicit drugs. Routine hematologic analysis was normal. Magnetic resonance imaging was performed (images shown below).

Imaging Fig. 21.1 Axial contrast-enhanced CT scan through the level of the ventricles.

Advanced Neuroradiology Cases, ed. Amaral et al. Published by Cambridge University Press. © Cambridge University Press 2016.

75

Part II. Neurovascular Diseases: Case 21

(A)

(B)

Fig. 21.2 (A) Axial T2WI through the level of the ventricles. (B) Postcontrast axial gradient echo T1WI sequence through a slightly superior aspect of the brain.

Fig. 21.3 Maximum intensity axial projection of the 3D postcontrast axial T1WI of the head.

76

Part II. Neurovascular Diseases: Case 21

Fig. 21.4 Perfusion parametric maps with region of interest through the same level as Fig. 21.2B.

Fig. 21.5 Lateral digital subtraction angiogram of the head.

77

Part II. Neurovascular Diseases: Case 21

Cerebral Proliferative Angiopathy Primary Diagnosis Cerebral proliferative angiopathy

Differential Diagnosis Arteriovenous malformation

Imaging Findings Fig. 21.1: Axial contrast-enhanced CT scan through the level of the ventricles demonstrated numerous, abnormal-appearing blood vessels involving almost the entire right hemisphere with relative sparing of the medial right frontal lobe. Diffuse swelling of the hemisphere, with subtle subfalcine herniation, was also noted. Fig. 21.2: (A) Axial T2WI through the same level demonstrated flow voids from abnormally large blood vessels in the right hemisphere and adjacent to the septum of the ventricles. The intervening brain has a relatively normal appearance. The left hemisphere was normal. (B) Postcontrast, axial gradient echo T1WI sequence through a slightly superior aspect of the brain demonstrated similar findings. Vessels are numerous along the posterior aspect of the brain close to the dura, as well as along the falx, suggesting dural involvement. Abnormal vasculature was also noted in the right frontal scalp. Fig. 21.3: Maximum intensity axial projection of the 3D postcontrast, axial T1WI of the head demonstrated an extensive network of abnormally large blood vessels in the right side of the cranial cavity and in the right frontal scalp. A large vein was also noted along the convexity of the left hemisphere. Fig. 21.4: Perfusion parametric maps with region of interest through the same level as Fig. 21.2B demonstrated increased CBV, CBF, and mean transit time (MTT) in the relatively normal-appearing area of the right frontal lobe. Please note that there was absence of signal over the areas of abnormally enlarged blood vessels on CBV, CBF, and MTT maps (values in these areas were outside the scale showing these results). Fig. 21.5: Lateral digital subtraction angiogram of the head demonstrated a network of abnormally large blood vessels supplied by the right internal carotid artery. The morphology is bizarre: no discrete nidus, feeding artery, or prominent draining vein was noted.

Discussion Although the clinical presentation is non-specific, the imaging appearance is diagnostic of cerebral proliferative angiopathy (CPA). There are numerous enlarged blood vessels involving almost the entire right side of the cranial cavity, with no identifiable feeding artery, early draining vein, or prominent draining venous system. The adjacent brain tissue has a relatively normal appearance on T2WI sequence, but demonstrates high MTT, suggesting tissue ischemia is also a characteristic finding. There is no hemorrhage. Demonstrating both abnormal vasculature and abnormal cellular and/or endothelial proliferation, CPA is a unique vascular pathology. It accounts for approximately 3% of all arteriovenous

78

malformations (AVMs), with a female predominance (female: male: 2:1). Typically, patients present in early adulthood with a mean age of 22 years. Cerebral proliferative angiopathy differs from typical AVM in its demography, pathogenesis, clinical presentation, angiomorphology, pathology, and natural history. Unlike AVM, CPA demonstrates absence of dominant feeders, flow-related aneurysms, and presence of extensive transdural supply to healthy and pathologic tissue, stenosis, rather than enlargement of the feeding arteries, and extensive angiectasia. Compared to AVMs, CPA is usually much larger and can involve the entire hemisphere, like this case. Given the size of the nidus, the draining veins are only moderately enlarged. On perfusion imaging, the presence of increased CBV and increased MTT, within the nidus, suggest venous and capillary ectasia; profound hypoperfusion and severely impaired cerebrovascular reserve in the surrounding brain tissue can also be seen in the entire involved hemisphere. Cerebral proliferative angiopathy and AVM differ histopathologically in multiple ways. The key difference between CPA and AVM is the presence of normal brain tissue, which, in CPA, intervenes between the abnormal blood vessels. Moreover, in CPA, the severe hypoperfusion of surrounding brain tissue stimulates angiogenesis (new blood vessel formation). Segmental stenosis of the anterior and middle cerebral arteries also occurs. Angioectasia and arterial stenosis indicate abnormal vascular development. As CPA demonstrates both abnormal proliferation and abnormal vasculature, this is a unique pathology with features of both types of vascular abnormality: vascular tumor such as hemangioma, and vascular malformation such as AVM in the Mulliken classification system. The typical clinical presentation of CPA includes seizures and headache, unlike in AVM, which typically presents with acute intracranial hemorrhage. However, CPA does present with intracranial hemorrhage, albeit less commonly than with AVMs. In addition, CPA patients are also at risk of ischemia, transient ischemic attack, and stroke due to the profound hypoperfusion in the surrounding brain tissue. Typical imaging features include a diffuse network of abnormal, large, densely enhancing vascular spaces evident on both CT and MRI involving a large area of the brain that in some patients can be holohemispheric. The dominating MRI findings are the prominent flow voids from the large abnormal blood vessels seen on non-contrasted sequences. The brain tissue between the abnormal blood vessels appears normal. The frontal and parietal lobes are most commonly affected, possibly because of the size of these lobes, rather than tropism. The lesions preferentially involve the arterial border-zone areas. On perfusion imaging, abnormal perfusion is noted far beyond the boundary of the structural abnormalities. Within the nidus, there is increased CBV and MTT, whereas areas of brain remote to the nidus demonstrate low blood volume and increased transit time, suggesting hypoperfusion. Angiograms demonstrate a striking abnormality: a very large nidus supplied by multiple moderately enlarged, abnormal-looking vessels,

Part II. Neurovascular Diseases: Case 21

rather than any identifiable single feeding artery or prominent draining vein. Early filling of the veins is not a feature of CPA. Unlike in AVM, stenosis of the proximal arteries is also a characteristic feature. Therapeutic options for CPA include surgery or radiosurgery, although both options are associated with risk of hemorrhage. Embolization is also difficult, as there is no definite nidus to treat, which in turn increases the risk of embolization to the normal surrounding tissues.

Key Points  Cerebral proliferative angiopathy is manifested by numerous abnormal-appearing blood vessels involving large areas of brain on cross-sectional imaging, with evidence of normal-appearing vessels between abnormal blood vessels.

 Unlike in AVM, there is characteristic absence of any feeding artery, prominent draining vein, and early venous filling on angiogram. In addition, there is stenosis of the feeding arteries rather than enlargement.

Suggested Reading Fierstra J, Spieth S, Tran L, et al. Severely impaired cerebrovascular reserve in patients with cerebral proliferative angiopathy. J Neurosurg Pediatr 2011; 8(3): 310–15. Lasjaunias PL, Landrieu P, Rodesch G, et al. Cerebral proliferative angiopathy: clinical and angiographic description of an entity different from cerebral AVMs. Stroke 2008; 39(3): 878–85. Mulliken JB, Glowacki J. Hemangiomas and vascular malformations in infants and children: a classification based on endothelial characteristics. Plast Reconstr Surg 1982; 69(3): 412–22.

79

CASE

Part II

22

Neurovascular Diseases Satya Patro, Prasad B. Hanagandi, Salo Haratz, Lázaro Luís Faria do Amaral

Clinical Presentation A 67-year-old woman with poorly controlled type 2 diabetes mellitus presented with left hemiplegia, which resolved within 12 hours. There was no history of chronic use of medications such as neuroleptics, metoclopramide, or levodopa. She did not have a history of hyperthyroidism or family history of movement disorders. Laboratory studies did not reveal significant biochemical or metabolic abnormalities, with the exception of raised fasting and postprandial blood sugar levels. Her glycemic status was eventually controlled but she continued to have persistent hemichorea-hemiballism.

Imaging (A)

(B)

Fig. 22.1 (A) Axial non-contrast T1WI and (B) postcontrast T1WI at the level of the lentiform nuclei.

Advanced Neuroradiology Cases, ed. Amaral et al. Published by Cambridge University Press. © Cambridge University Press 2016.

81

Part II. Neurovascular Diseases: Case 22 Fig. 22.2 Axial FLAIR sequence at the level of basal ganglia.

Fig. 22.3 DWI image.

82

Part II. Neurovascular Diseases: Case 22 Fig. 22.4 Axial GRE image through the same level as that in Fig 22.2.

Fig. 22.5 MR angiography of the circle of Willis.

83

Part II. Neurovascular Diseases: Case 22

Spectacular Shrinking Deficit Primary Diagnosis Spectacular shrinking deficit

Differential Diagnoses Non-ketotic hyperglycemic hemichorea Hemorrhagic infarct Hypoglycemic coma Cardiac arrest/global hypoxia Basal ganglia calcification Hepatic failure Hyperalimentation or long-term parenteral nutrition due to manganese toxicity Wilson disease

Imaging Findings Fig. 22.1: (A) Axial T1WI showed a non-enhancing T1 hyperintense signal in the right caudate head and (B) lentiform nucleus. Fig. 22.2: FLAIR sequence demonstrated subtle focal hypointense signal. Fig. 22.3: T2 demonstrated shine through on DWI sequence and no obvious changes on ADC map (not shown). Fig. 22.4: GRE sequence did not exhibit hemorrhage. Fig. 22.5: MR angiography of the circle of Willis showed focal narrowing and irregularity of the M1 segment of right middle cerebral artery (MCA).

Discussion Spectacular shrinking deficit (SSD) is typically defined as a major, hemispheric ischemic stroke syndrome that is followed by dramatic improvement within hours and disappearance of most of the clinical manifestations. Cerebral hemispheric ischemic syndrome classically includes moderate to severe consciousness disturbance, hemiparesis, aphasia, apraxia, and amnesia. Neurologically, a decline in 8 NIHSS points or a drop to < 4 NIHSS points within 24 hours is considered a required clinical criteria for diagnosis of SSD. A temporary occlusion of the internal carotid artery or MCA, with subsequent fragmentation and distal migration of the thrombus and restoration of perfusion prior to irreversible damage, has been proposed as the underlying mechanism leading to SSD. Previously published literature has suggested that the majority of SSD cases are associated with cardioembolism, ranging from 80% to 94%. However, other recent studies have shown no specific relation to cardiac sources. Spectacular shrinking deficit has been reported to occur in 12–14% of patients with initial major hemispheric syndrome. Patients with SSD tend to be younger and have a smaller final infarct core, as compared to hemispheric syndrome patients without SSD. The presence of smaller initial DWI lesion with a large mismatch volume associated with reperfusion and small final infarct volume are characteristic MRI patterns of SSD. However, in most patients, serial MRI can reveal T1-weighted hyperintensity and T2weighted relative hypointensity in the ischemic lesions of basal ganglia and cerebral cortex, usually between one and three

84

weeks after SSD. This is described as delayed ischemic hyperintensity (DIH) on T1-weighted MRI. Thereafter over time, it gradually fades away with the appearance of atrophy of the affected structures. Magnetic resonance imaging performed in our patient after approximately two weeks showed unilateral T1-weighted hyperintensity with focal MCA narrowing, consistent with an SSD diagnosis. Two common differential diagnoses of T1-weighted hyperintensity on cranial MR images include hemorrhagic infarct and calcification. Typically, hemorrhagic transformation follows the process of hemoglobin degradation within the tissue and the MRI signal differs during various phases of degradation. For example, during acute stage hemorrhage, intracellular deoxyhemoglobin, MR signal appears isointense on T1-weighted and hypointense on T2-weighted images. In contrast, during chronic stage, hemosiderin, MR signal appears hypointense on both T1-weighted and T2-weighted images. In addition, CT can show hyperdense hemorrhage in the affected areas. Calcification can appear hyperintense on T1-weighted and hypointense on T2-weighted images and hyperdense on CT. Cardiac arrest with global hypoxia typically presents with symmetric changes in the basal ganglia, thalami, and/or substantia nigra and cortex with diffusion restriction. The changes in SSD do not represent infarction or hemorrhage on MRI. The difference in presentation is most likely related to the mechanism of ischemia (focal and brief in SSD and global and prolonged in cardiac arrest). The MRI findings of hypoglycemic coma can mimic SSD, sharing a similar time of onset following hypoglycemic injury. Diffusion imaging best demonstrates early-onset hypoglycemic effects; however, the signal changes are often bilateral and symmetric. Hemichorea-hemiballism syndrome is a condition that typically manifests in elderly diabetics with non-ketotic hyperglycemia. Usually CT images show unilateral hyperdensity involving the lentiform nuclei and caudate head, with similar hyperintense signal changes on T1-weighted images. Moreover, in hemichorea-hemiballismus, symptoms resolve with adequate glycemic control. The histopathology, which usually reveals glial tissue with abundant gemistocytes, is also seen in SSD. Improvement in hemiplegia but persistence of symptoms even after glycemic control, and angiographic evidence of stenosis of the MCA, confirm the suggested diagnosis of SSD and exclude hemichorea-hemiballism syndrome. Other conditions presenting with T1-weighted hyperintensity of basal ganglia such as hepatic encephalopathy, hepatic failure, hyperalimentation, or long-term parenteral nutrition (manganese), Wilson’s disease, and hamartomas as noted in neurofibromatosis-1 were excluded because of the discrepancy in clinical presentation and laboratory findings. Various animal and human studies have shown that a brief period of MCA occlusion causes selective neuronal necrosis and apoptosis, leaving intact glial cells and microvessels in the caudo-putamen region and cerebral cortex. This has been

Part II. Neurovascular Diseases: Case 22

Suggested Reading

termed incomplete infarction, in which the brain tissue framework is preserved, without any cavitation. Therefore, the DIH in the basal ganglia and cerebral cortex may neuroradiologically represent selective neuronal loss and gliosis without infarct or hemorrhage. The neuronal cell death matures slowly in the course of several weeks after an incomplete infarction with increasing glial reactions and deposition of paramagnetic compounds such as iron, manganese, and free radicals generated by the macrophages. This could explain the short T1-weighted and T2-weighted relaxation seen in the affected areas in SSD.

Cornelius JR, Zubkov AY, Wijdicks EF. Following the clot in spectacular shrinking deficit. Rev Neurol Dis 2008; 5(2): 92–4.

Key Points

Kraemer N, Thomalla G, Soennichsen J, et al. Magnetic resonance imaging and clinical patterns of patients with ‘spectacular shrinking deficit’ after acute middle cerebral artery stroke. Cerebrovasc Dis 2005; 20(5): 285–90.

 Brief period of hemispheric syndrome with improvement and typical unilateral MRI signal changes in the basal ganglia on MRI with focal narrowing of MCA on MR angiogram are features suggestive of SSD.  Non-ketotic hyperglycemia and several other conditions mentioned in the differential diagnosis list can have similar MR imaging appearances but differ in their clinical presentation.

Baird AE, Donnan GA, Austin MC, McKay WJ. Early reperfusion in the ‘spectacular shrinking deficit’ demonstrated by single-photon emission computed tomography. Neurology 1995; 45(7): 1335–9.

Fujioka M, Taoka T, Hiramatsu KI, Sakaguchi S, Sakaki T. Delayed ischemic hyperintensity on T1-weighted MRI in the caudoputamen and cerebral cortex of humans after spectacular shrinking deficit. Stroke 1999; 30(5): 1038–42.

Minematsu K, Yamaguchi T, Omae T. ‘Spectacular shrinking deficit’: rapid recovery from a major hemispheric syndrome by migration of an embolus. Neurology 1992; 42(1): 157–62. Sakai T, Kuzuhara S. [Diffusion-weighted magnetic resonance imagings at the acute stage in two patients with spectacular shrinking deficit due to cardioembolic stroke]. Rinsho Shinkeigaku 2006; 46(2): 122–7.

85

CASE

Part II

23

Neurovascular Diseases Fabrício Guimarães Gonçalves

Clinical Presentation A 72-year-old man presented to my facility with a long history of cognitive decline. He reported a recent episode of seizures, with transient interictal mild left-sided motor deficit.

Imaging

Fig. 23.1 Axial CT scan at the level of the lateral ventricles.

Fig. 23.2 Axial T2WI at the same level.

Advanced Neuroradiology Cases, ed. Amaral et al. Published by Cambridge University Press. © Cambridge University Press 2016.

87

Part II. Neurovascular Diseases: Case 23 Fig. 23.3 Axial FLAIR image at the same level.

Fig. 23.4 Axial T2* GRE image at the same level.

88

Part II. Neurovascular Diseases: Case 23 Fig. 23.5 Axial T2* GRE image at the level of the thalami.

89

Part II. Neurovascular Diseases: Case 23

Cerebral Amyloidoma Primary Diagnosis Cerebral amyloidoma

Differential Diagnoses Central nervous system lymphoma Oligodendroglioma Ependymoma

Imaging Findings Fig. 23.1: Axial CT scan at the level of the lateral ventricles showed a periventricular calcified nodule on the right side, with no significant edema or mass effect (arrow). Fig. 23.2: Axial T2WI at the level of the lateral ventricles showed a small focus of hyperintensity, possibly representing vasogenic edema in the right periventricular white matter (arrow). Fig. 23.3: Axial FLAIR image at the level of the lateral ventricles showed a small focus of hyperintensity possibly representing vasogenic edema in the right periventricular white matter (arrow). Fig. 23.4: Axial T2* GRE image at the level of the lateral ventricles showed a small focus of hypointensity at the same level as in Fig 23.1. Fig. 23.5: Axial T2* GRE image at the level of the thalami showed diffuse, bilateral low signal in the subarachnoid space and in the periphery of the hemispheres (arrows).

Discussion Imaging findings of a calcified nodule that is associated with a small amount of surrounding vasogenic edema and low signal material, consistent with hemosiderin deposition, covering the cerebral hemispheres and subarachnoid spaces are highly suggestive of amyloid angiopathy associated with an amyloidoma. Cerebral amyloidoma (CA) should be considered in patients demonstrating solitary or multiple intracerebral white matter masses with little or no mass effect, hyperdensity on non-enhanced CT scans, and contrast enhancement. Medial extension of the mass, up to the lateral ventricular ependymal, and a fine, irregular, radiating margin on imaging studies could possibly add specificity to this diagnosis. The most common locations of CA CNS lesions are the spinal cord, cerebral white matter, leptomeninges, and the gasserian ganglion. These lesions are frequently described as solitary, slow-growing, tumor-like masses. The majority of CNS lymphoma cases demonstrate increased density on CT, unrelated to calcification. Additionally, CNS lymphoma lesions usually strongly enhance after contrast injection on either CT or MRI. One other important imaging feature of CNS lymphoma is that they typically show restricted diffusion. The fact that CNS lymphoma is hyperdense on CT but does not demonstrate calcification, which is present in this case, makes the possibility of lymphoma less likely. Calcification can be found in some brain tumors such as oligodendrogliomas, ependymomas, and subependymomas. Oligodendrogliomas are typically found in the supratentorial compartment and can present coarse calcifications in around

90

one-half of cases. Ependymomas are more frequently found in the infratentorial compartment and can show calcification in one-half of cases. Additionally, subependymomas are capable of demonstrating calcification and are more commonly found in the elderly. The lack of significant mass effect, edema, and adjacent brain parenchyma changes – expected findings in patients with brain tumors – make the possibility of a brain tumor unlikely in this patient. Amyloidosis is a complex pathologic state secondary to intra- and extracellular space amyloid accumulation. In the brain, amyloid fibrils most commonly deposit in blood vessels, manifesting in many forms, including cerebral amyloid angiopathy, the senile plaques associated with Alzheimer dementia, and the deposits seen in Kuru and Creutzfeldt-Jakob disease encephalopathy. Tumoral deposition (amyloidoma) is the least common form of brain involvement by amyloid proteins, with no known cause. Cerebral amyloidoma is more commonly found in the adult population (mean age at presentation, 47.8 years), with a slight female preponderance. Patients may present with history of epileptic seizures, hemiparesis, gait disturbance, visual impairment, cognitive impairment, and hearing loss. The clinical course of intracerebral amyloidoma appears to be benign, although these lesions can show slow growth, and they can recur after surgical resection. Almost all cases of CA are located in the white matter of the cerebral hemispheres, more commonly in the frontal lobe, in the periventricular white matter, and in the choroid plexus. Cerebral amyloidoma can involve the peripheral nervous system, most commonly in the trigeminal (gasserian) ganglion, in the cerebellopontine angle, and in the jugular foramen. The great majority of these masses are confined to the supratentorial compartment, with very few cases reported in the brainstem or cerebellar hemispheres. Amyloidomas present as single or multiple lesions. Cerebral amyloidoma is typically hyperdense on CT and commonly presents contrast enhancement. Amyloidomas tend to present little or no mass effect with little or no perilesional edema. On MRI, CA has a variable appearance. On T1WI, they can be hypo-, iso-, or hyperintense, relative to the surrounding gray matter. On T2WI, CA signal is usually mixed, with areas of high and low signal intensity. After gadolinium injection, the great majority of amyloidomas show enhancement. Cerebral angiographic results are normal, or may reveal vascular displacement due to the presence of the lesion. The majority of patients with CA demonstrate a medial extension to the ependymal surface of the lateral ventricle. Lateral ventricle wall thickening may also be observed. Very few reports are available that discuss the potential advantages of advanced MRI methods in differentiating CA from brain tumors. Magnetic resonance spectroscopy may demonstrate non-elevated choline levels and reduced creatinine peaks, indicating the absence of increased membrane turnover. In patients with malignant tumors, one may detect elevated lipids and lactate peaks, not usually found in

Part II. Neurovascular Diseases: Case 23

amyloidomas. Fractional anisotropy in amyloidomas tends to be lower, indicating destruction of normal tissue. Moreover, diffusivity values tend to be higher, indicating a non-malignant lesion. Low diffusivity is expected in cases of malignant tumors. Perfusion-weighted images may demonstrate a slight decrease in the signal intensity in the mass, as compared to the normal contralateral side, with rCBV ratio of 0.5. This is in contrast to malignant tumors, the rCBV ratio tends to be above 1.5 because of the increase in the capillary density. Susceptibility-weighted images may demonstrate absence of significant negative phase changes in amyloidomas, which is indicative of an absence of increased vascularity, a common finding in malignant lesions. Magnetic transfer ratio maps may demonstrate low magnetization transfer contrast in the lesion, as compared to normal-appearing brain, which is consistent with previous studies for detection of amyloid plaques in patients with Alzheimer disease. Definitive diagnosis of amyloidoma is usually performed by brain biopsy and histopathologic examination. Central nervous system amyloidomas are usually slow-growing lesions with benign course and similar clinical characteristics to indolent tumors. For this reason, even though they are benign entities, they can present with the same treatment difficulties as other slow-growing tumors.

Key Points  Cerebral amyloidoma can be suspected when a calcified enhancing nodule (single or multiple) is seen in association with little or no mass effect, hyperdensity on plain CT scans, and enhancement after contrast injection.  Medial extension of the mass up to the lateral ventricular ependyma and a fine, irregular, radiating margin on imaging studies could possibly add specificity to this diagnosis.

 In the great majority of cases, cerebral amyloidomas are located in the white matter of the cerebral hemispheres or the trigeminal (gasserian) ganglion.  Cerebral amyloid lesions tend to present with little or no mass effect, or perilesional edema.

Suggested Reading Blattler T, Siegel AM, Jochum W, et al. Primary cerebral amyloidoma. Neurology 2001; 56(6): 777. Fischer B, Palkovic S, Rickert C, Weckesser M, Wassmann H. Cerebral AL λ-amyloidoma: clinical and pathomorphological characteristics. Review of the literature and of a patient. Amyloid 2007; 14(1): 11–19. Gallucci M, Caulo M, Splendiani A, et al. Neuroradiological findings in two cases of isolated amyloidoma of the central nervous system. Neuroradiology 2002; 44(4): 333–7. Gandhi D, Wee R, Goyal M. CT and MR imaging of intracerebral amyloidoma: case report and review of the literature. AJNR Am J Neuroradiol 2003; 24(3): 519–22. Kotsenas AL, Morris JM, Wald JT, Parisi JE, Campeau NG. Tumefactive cerebral amyloid angiopathy mimicking CNS neoplasm. AJR Am J Roentgenol 2013; 200(1): 50–6. Nossek E, Bashat DB, Artzi M, et al. The role of advanced MR methods in the diagnosis of cerebral amyloidoma. Amyloid 2009; 16(2): 94–8. Parmar H, Rath T, Castillo M, Gandhi D. Imaging of focal amyloid depositions in the head, neck, and spine: amyloidoma. AJNR Am J Neuroradiol 2010; 31(7): 1165–70. Symko SC, Hattab EM, Steinberg GK, Lane B. Imaging of cerebral and brain stem amyloidomas. AJNR Am J Neuroradiol 2001; 22(7): 1353–6. Tabatabai G, Baehring J, Hochberg FH. Primary amyloidoma of the brain parenchyma. Arch Neurol 2005; 62(3): 477–80.

91

CASE

Part II

24

Neurovascular Diseases Asim K. Bag, Lázaro Luís Faria do Amaral

Clinical Presentation A 65-year-old man presented to an emergency department with new-onset seizures. Family members noted that he had complained about headaches, visual abnormalities, and tingling and numbness of both lower extremities for the last several weeks. After seizure stabilization, neurologic examination revealed neurocognitive decline. Spinal tap revealed high CSF protein, elevated WBC, and autoantibodies against Aβ 1–40. An MRI of the brain was performed (shown below).

Imaging (A)

(B)

Fig. 24.2 Axial FLAIR image through the centrum semiovale.

Fig. 24.1 (A) Axial T2*-weighted sequence through the level of the centrum semiovale. (B) Sagittal left paramedian T1WI.

Fig. 24.3 Axial DWI image through the centrum semiovale.

Advanced Neuroradiology Cases, ed. Amaral et al. Published by Cambridge University Press. © Cambridge University Press 2016.

93

Part II. Neurovascular Diseases: Case 24

Cerebral Amyloid Angiopathy with Inflammation Primary Diagnosis Cerebral amyloid angiopathy with inflammation

Differential Diagnoses Cerebral amyloid angiopathy Primary angiitis of CNS

Imaging Findings Fig. 24.1: (A) Axial T2*-weighted sequence through the level of the centrum semiovale demonstrated multiple foci of microhemorrhages in the cerebral cortices in bilateral posterior parietal lobes and in the left frontal lobe. (B) Sagittal left paramedian T1WI demonstrated tiny foci of hemorrhage in the posterior parietal regions and hypointensity in the adjacent brain parenchyma. Fig. 24.2: Abnormal hypointensity can also be noted in the left occipital lobe gray-white interface. Axial FLAIR image through the centrum semiovale demonstrated abnormal FLAIR hyperintensity involving bilateral posterior parietal gray-white interface suggestive of brain edema. Subtle FLAIR hyperintensity was also noted in the left frontal subcortical white matter as well, in the region of the microhemorrhages. Increased FLAIR signal was also noted involving left occipital lobes as well (not shown). Fig. 24.3: Axial DWI image through the centrum semiovale demonstrated punctate areas of diffusion restriction (with low value on ADC map, not shown) suggestive of punctate areas of infarction involving bilateral posterior parietal lobes.

Discussion The presence of cortical microhemorrhages in an elderly patient is highly suggestive of cerebral amyloid angiopathy (CAA). However, edema in the adjacent brain tissue is not a common finding in a patient with CAA in the absence of inflammation or tiny infarcts. Subacute-onset neurologic symptoms, cortical microhemorrhages, tiny infarcts, and extensive edema in the adjacent brain parenchyma is highly suggestive of cerebral amyloid angiopathy with inflammation. Autoantibodies against Aβ 1–40 in the spinal fluid further support the diagnosis of amyloid β-related angiitis (ABRA), one of the cerebral amyloid angiopathy with inflammation variants, which was confirmed on biopsy. Cerebral amyloid angiopathy-related inflammation (CAA-RI), another variant of cerebral amyloid angiopathy with inflammation, may have similar clinical presentation and CSF findings, but the presence of hemorrhage and infarcts are not common. Cerebral amyloid angiopathy without any inflammation more commonly has neurocognitive decline and frank intraparenchymal hemorrhages, compared to the two variants of cerebral amyloid angiopathy with inflammation. In addition, CAA presents at a slightly higher age group and usually has numerous microhemorrhages involving extensive areas of superficial cortical tissue rather than a few foci of microhemorrhages, as seen in this patient. Primary angiitis of CNS (PACNS) shares many of the imaging abnormalities with

94

cerebral amyloid angiopathy with inflammation. However, clinical characteristics are different. Primary angiitis of CNS is primarily a disease of much younger patients who typically present with multifocal infarction, headache, and focal neurologic deficit. Intracranial hemorrhage is rare. Neurocognitive decline is not a dominant clinical finding of PACNS (please see Part IV: Case 51, for in-depth discussion of PACNS). Sporadic CAA is deposition of Aβ in the wall of the leptomeningeal and cortical vessels. It weakens the vessel wall and results in vessel fragility, rupture, and intraparenchymal hemorrhage. Hemorrhage can extend to the adjacent subarachnoid component. As such, this sporadic form of CAA is not associated with any vessel wall inflammation. However, some patients with CAA do demonstrate Aβ deposition-associated inflammation. Two distinct subtypes have been identified: one with perivascular inflammation, known as CAA-RI, and a more common, destructive, transmural vasculitic process, ABRA. Owing to extensive transmural vasculitis, it has been proposed that ABRA is more closely related to PACNS than CAA. Amyloid β-related angiitis is characterized by severe leptomeningeal and cortical amyloid angiopathy and mild to moderate chronic inflammatory infiltrates in the adjacent brain parenchyma that is often granulomatous. Splitting of vessel walls, fibrinoid necrosis, acute thrombosis, and chronic thrombosis with recanalization are common histopathologic findings. Quantitative immunohistochemical studies have shown that ABRA is associated with excessive immune activation against Aβ with associated enhanced clearing of the Aβ from the adjacent brain parenchyma. Cognitive decline and headache are the most common clinical ABRA presentations. The most common imaging abnormalities seen in patients with ABRA are vasogenic edema and leptomeningeal enhancement. Intraparenchymal enhancement has also been described and infarction can be seen in up to 25% of patients. Intraparenchymal hemorrhage is less common in ABRA, as compared to CAA, 18% versus 60%. A distinct clinicopathologic entity, ABRA differs from CAA in multiple ways. Patients with ABRA are younger than CAA at the time of diagnosis, and have lower frequency of neurocognitive decline and intraparenchymal hemorrhage. Leptomeningeal and intraparenchymal enhancement is more common in ABRA. It responds well to steroids, suggesting that the key underlying pathology is an inflammatory process with favorable prognosis, whereas CAA is a progressive disease and is considered untreatable. Significantly different from PACNS patients, ABRA patients are older than PACNS patients at the time of diagnosis and have an increased frequency of altered cognition and seizures. Leptomeningeal enhancement is a more common neuroimaging abnormality in ABRA, as compared to PACNS, in which infarction is a dominant feature. The response to treatment and overall prognosis is similar in both ABRA and PACNS. Autoantibodies against Aβ 1–40 and 1–42 have been associated with ABRA and CAA-RI, further suggesting an

Part II. Neurovascular Diseases: Case 24

underlying inflammatory role in these two conditions. In fact, treatments targeting amyloid load reduction and monoclonal antibody against Aβ in Alzheimer disease have been shown to simulate clinical and imaging abnormalities like that of ABRA and CAA-RI. In addition, both the spontaneous amyloidrelated inflammatory conditions and drug-induced inflammatory conditions share a common genetic associated with the APOE ε4 allele.

Suggested Reading

Key Point

Salvarani C, Hunder GG, Morris JM, et al. Aβ-related angiitis: comparison with CAA without inflammation and primary CNS vasculitis. Neurology 2013; 81(18):1596–603.

 Diagnosis of cerebral amyloid angiopathy with inflammation should be considered in elderly patients presenting with clinical features suggestive of vasculitis, especially if it is associated with neurocognitive decline,

evidence of intraparenchymal hemorrhage, small vessel infarction, significant edema of the adjacent brain parenchyma and leptomeningeal enhancement. Bogner S, Bernreuther C, Matschke J, et al. Immune activation in amyloid-beta-related angiitis correlates with decreased parenchymal amyloid-beta plaque load. Neurodegener Dis 2014; 13(1): 38–44.

Scolding NJ, Joseph F, Kirby PA, et al. Aβ-related angiitis: primary angiitis of the central nervous system associated with cerebral amyloid angiopathy. Brain 2005; 128(Pt 3): 500–15.

95

CASE

Part II

25

Neurovascular Diseases Aparna Singhal, Fabrício Guimarães Gonçalves, Asim K. Bag

Clinical Presentation A 35-year-old woman presented with acute-onset, severe thunderclap headache. She reported history of similar headaches on numerous occasions that required hospital admission for management. On examination, she had bilateral extremity weakness, visual abnormality, and aphasia. No systemic disease was noted. Hematology studies were normal. Routine CSF analysis was negative. Head CT (Fig. 25.1), MRI of the brain (Figs. 25.2–25.3), and catheter angiogram (Figs. 25.4–25.5) were obtained.

Imaging Fig. 25.1 Axial head CT through the level of central sulcus.

Advanced Neuroradiology Cases, ed. Amaral et al. Published by Cambridge University Press. © Cambridge University Press 2016.

97

Part II. Neurovascular Diseases: Case 25

Fig. 25.2 Axial FLAIR image through the level of central sulcus.

Fig. 25.3 Axial GRE image through the same level.

98

Part II. Neurovascular Diseases: Case 25 Fig. 25.4 Catheter angiogram of the left internal carotid artery, lateral projection.

Fig. 25.5 Catheter angiogram of the left internal carotid artery, oblique projection.

99

Part II. Neurovascular Diseases: Case 25

Reversible Cerebral Vasoconstriction Syndrome Primary Diagnosis Reversible cerebral vasoconstriction syndrome

Differential Diagnoses Cerebral vasculitis (primary angiitis of CNS) Systemic lupus Non-aneurysmal subarachnoid hemorrhage (dural fistula, trauma, arteriovenous malformation) Aneurysmal subarachnoid hemorrhage Migraines

Imaging Findings Fig. 25.1: Axial head CT through the level of central sulcus demonstrated subtle subarachnoid hemorrhage in the left parietal region, including the postcentral sulcus. Fig. 25.2: Axial FLAIR image through the level of central sulcus demonstrated subtle subarachnoid hemorrhage in the left parietal region, including the postcentral sulcus. Fig. 25.3: Axial GRE image through the level of central sulcus confirmed subarachnoid hemorrhage in the left postcentral sulcus. Fig. 25.4: Catheter angiogram of the left internal carotid artery on the lateral projection demonstrated multifocal beading of one of the left M3 branches (arrows). Fig. 25.5: Catheter angiogram of the left internal carotid artery on the oblique projection demonstrated focal narrowing of the left pericallosal artery (arrow).

Discussion Typical clinical presentation (recurrent acute thunderclap headache in a young female patient) associated with typical imaging findings (subarachnoid hemorrhage [SAH] close to the vertex, focal beading, and narrowing of the arteries) are consistent with the diagnosis of reversible cerebral vasoconstriction syndrome (RCVS). Central nervous system vasculitides, particularly primary angiitis of CNS (PACNS), can have a similar angiographic appearance but are more frequently associated with a gradual symptomatic onset. Moreover, thunderclap headaches have never been reported in patients with PACNS. These patients demonstrate inflammatory CSF abnormalities whereas MRI scans are abnormal in most cases of PACNS, showing small deep or superficial infarcts of different ages, with or without white matter signal abnormalities. In addition, complications including posterior reversible encephalopathy syndrome (PRES), infarctions, and hemorrhages typically occur in watershed zones in PACNS, whereas in RCVS, hemorrhages can be distributed over watershed zones. Aneurysmal SAH is more diffuse and typically involves the basilar cisterns whereas SAH is typically minimal and involves the cerebral convexity, overlying a few cortical sulci in RCVS. Delayed vasospasm in aneurysmal SAH may be a differential consideration on angiography; however, it is usually long segmental and in close proximity to the bleeding site. In contrast, RCVS has diffuse, short segmental vasoconstriction.

100

Cerebral angiography would be diagnostic for both aneurysms and entities such as arteriovenous malformation or dural fistula. Migrainous headache does not have a specific imaging abnormality. Reversible cerebral vasoconstriction syndrome refers to a group of disorders characterized by reversible, multifocal cerebral arterial vasoconstriction, most commonly manifesting as severe acute, usually recurrent thunderclap headaches, with or without other acute neurologic symptoms including nausea, vomiting, photophobia, confusion, and blurred vision. This entity has a female predominance (3:1) and most patients are middle-aged. Typically, the headaches recur for one to two weeks. Ischemic and hemorrhagic stroke are the major complications (7–54%). Other complications include PRES (9–14%), brain edema (38%), cortical SAH (up to 34%), or subdural hemorrhages (2%). Ischemic events predominantly occur during the second week, later than the hemorrhagic events that tend to occur in the first week. These complications can arise after complete resolution of headache as well. This syndrome may be spontaneous, or primary in approximately one-third of patients. It may be secondary and associated most commonly with use of vasoactive substances (e.g., recreational drugs such as cannabis or cocaine, or drugs such as SSRIs, nasal decongestants, pseudoephedrine, and occasionally immunosuppressant or cytotoxic agents) or with a postpartum state (about 9%). Additional noted associations have been with tumors such as pheochromocytoma, glomus tumors, or bronchial carcinoid tumor, cervical artery or aortic dissection, unruptured intracranial aneurysm, fibromuscular dysplasia, postcarotid endarterectomy, erythropoietin, intracranial hypotension, intracranial hemorrhage, spinal subdural hematoma, neurosurgery, head trauma, and miscellaneous states such as porphyria. Pathophysiology of RCVS remains unclear: possible causes include transient dysfunctional regulation of cerebral vascular tone that leads to segmental vasoconstriction and vasodilatation in small vessels. This triggers thunderclap headache by abruptly stretching vessel walls. Oxidative stress, sympathetic overactivity, aberrant sympathetic response of the vessels, endothelial dysfunction, and alterations in biochemical and immunologic factors regulating vascular tone, have been suggested as potential causes. Histologic studies have showed no evidence of arterial inflammation or infection. Non-contrast CT is often negative but may show small cortical SAH in 20% (presumably from minor leaks or rupture of surface vessels), with or without parenchymal hemorrhage. Typical imaging findings include convexity SAH, intracerebral hemorrhage, and intracerebral edema. Convexity SAH is usually mild, and limited to a few cerebral sulci, and appears as hyperintense on FLAIR and hypointense on GRE images. Intracerebral hemorrhage is of variable volume and is usually solitary and lobar. Infarction is typically in the arterial borderzone areas, often between the posterior cerebral artery and anterior circulation. Computed tomography or MR angiography may be normal (10%) but may show diffuse segmental

Part II. Neurovascular Diseases: Case 25

arterial vasoconstriction (90%). Occasionally, if the first study was negative, second imaging studies obtained one to two weeks later may show positive findings. Digital subtraction angiography is critical for diagnosis and demonstrates diffuse, multifocal, segmental narrowing in the large and medium-sized arteries that is frequently bilateral. Occasional dilated segments may give a string-of-beads appearance. If the angiogram is performed early, within a week of symptomatic onset, it may be completely normal, even with presence of SAH, cerebral infarction, and intracerebral hemorrhage. Angiogram should be repeated after several days if the clinical suspicion is strong, and often follow-up angiogram is positive. By definition, the abnormalities are transient, and should show complete resolution on repeat imaging one to three months later. Cerebrospinal fluid analysis is usually normal or near normal (protein concentrations < 100 mg/dl, < 15 white blood cells per μl). Reversible cerebral vasoconstriction syndrome should be considered an emergent condition and treatment includes discontinuation of the offending agent and use of vasodilators such as calcium channel blockers (e.g., nimodipine). The disease has an overall favorable long-term prognosis, influenced by the occurrence of stroke. In a series, 71% of patients in the long term had no evidence of disability and 29% had only

minor disability. Most patients with strokes gradually improve for several weeks, and few have residual deficits and 100 cells/mm3), whereas in PNS, the mean cell count is up to 50 cells/mm3. Neutrophil predominance has been reported in both listeria infection and BD. A low CSF glucose level indicates that the infectious agent is bacterial. Magnetic resonance imaging is extremely important for the etiologic diagnosis of RE. The usual MR findings are increased signal intensity in the pons, medulla, upper cervical cord, and cerebellum, more frequently than in the midbrain on T2WI or FLAIR MR scans. Listeria infection, unlike other causes, can cause ring-enhancing abscesses in these locations. In the series reported by Moragas et al., in 2011, MRI was normal in 100% of PNS cases and abnormal imaging was seen in all patients with multiple sclerosis (MS), BD, and listeriosis. Lesions caused by listeria were found in an exclusively intratentorial location. In MS, associated supratentorial and infratentorial lesions were more common. In all patients with RE, blood culture and lumbar puncture should be performed to investigate the presence of HSV and tuberculosis by PCR, Gram stain and Ziehl-Neelsen stain in CSF, and to perform conventional and mycobacterium cultures. Empirical treatment with ampicillin and acyclovir should be initiated in all patients, with the possible exception of those who are afebrile and demonstrate normal MRI findings at presentation. Antibiotics can be changed based upon MRI, culture results, PCR results, and antibody studies. The outcome of RE in all etiologic groups is similar, with the exception of individuals with PNS, who have the poorest prognosis, due to the unfavorable prognosis of the underlying disease. Exclusion of non-survivors does not improve the prognosis of Ramsay Hunt patients with PNS over those with the remaining etiologies.

Key Points  Rhombencephalitis is a syndrome of multiple causes and multiple outcomes.  Early diagnosis and treatment optimizes patient outcomes.

Part III. Neuroinfectious Diseases: Case 38

 Recognizing certain clinical characteristics, as well as CSF and radiologic findings, can guide the initial approach to the etiologic diagnosis and management of RE.

Suggested Reading Dalmau J, Rosenfeld MR. Paraneoplastic syndromes of the CNS. Lancet Neurol 2008; 7: 327–40.

Jubelt B, Mihai C, Li TM, Veerapameni P. Rhombencephalitis / brainstem encephalitis. Curr Neurol Neurosci Rep 2011; 11: 543–52. Moragas M, Martinez-Yelamos S, Majos C, et al. Rhombencephalitis: a series of 97 patients. Medicine (Baltimore) 2011; 90(4): 256–61. Smiatacz T, Kowalik MM, Hlebowicz M. Prolonged dysphagia due to Listeria-rhombencephalitis with brainstem abscess and acute polyradiculoneuritis. J Infect 2006; 52: e165–7.

161

CASE

Part III

39

Neuroinfectious Diseases Ingrid Aguiar Littig, Asim K. Bag, Lázaro Luís Faria do Amaral

Clinical Presentation A 51-year-old woman with a history of rheumatoid arthritis, methotrexate use, and progressive multifocal leukoencephalopathy (PML) presented with acute illness following discontinuation of methotrexate. Magnetic resonance studies demonstrated presence of new lesional margin enhancement. She responded favorably to intervenous steroids.

Imaging (A)

(B)

Fig. 39.1 (A) Axial T1WI and (B) Axial T2WI through the level of the middle cerebellar peduncles.

Advanced Neuroradiology Cases, ed. Amaral et al. Published by Cambridge University Press. © Cambridge University Press 2016.

163

Part III. Neuroinfectious Diseases: Case 39

(A)

(B)

164

Fig. 39.2 (A) Axial DWI and (B) ADC map through the level of the middle cerebellar peduncles.

Part III. Neuroinfectious Diseases: Case 39

(A)

Fig. 39.3 (A) Axial T1WI postcontrast and (B) Coronal T1WI postgadolinium through the level of the middle cerebellar peduncles.

(B)

165

Part III. Neuroinfectious Diseases: Case 39

Progressive Multifocal Leukoencephalopathy – Immune Reconstitution Inflammatory Syndrome Primary Diagnosis Progressive multifocal leukoencephalopathy – immune reconstitution inflammatory syndrome

Differential Diagnoses Demyelinating diseases Multiple system atrophy Spinocerebellar ataxia

Imaging Findings Fig. 39.1: (A) Axial T1WI and (B) T2WI images demonstrated bilateral, middle cerebellar peduncle lesions with hyperintensity on T2WI and predominant hypointensity on T1WI. Fig. 39.2: (A) Axial DWI and (B) the ADC map showed small areas of restricted water molecule diffusion within the lesions. Fig. 39.3: (A) Axial T1WI postcontrast and (B) Coronal T1WI postgadolinium images demonstrated enhancement at the margin of the lesions.

Discussion The combination of an existing diagnosis of rheumatoid arthritis, history of methotrexate use, and classic imaging appearance is suggestive of progressive multifocal leukoencephalopathy (PML). As classic PML typically does not demonstrate enhancement, the presence of enhancement is suggestive of immune reconstitution inflammatory syndrome (IRIS)-associated PML. The most common imaging manifestation of PML is asymmetric, multifocal, bilateral, confluent supratentorial lobar white matter, but because JC virus has tropism to oligodendrocytes, any area of the brain may be affected. In the posterior fossa, the middle cerebellar peduncle is most commonly involved. Progressive multifocal leukoencephalopathy is a subacute CNS infection caused by reactivation of the John Cunningham (JC) virus, a latent polyomavirus present in the majority of adults. Most reported cases of JC virus reactivation occurred in patients with AIDS, or hematologic or solid cancers, or in transplant recipients on immunosuppressant therapy. Less frequently, PML has been described in patients with chronic inflammatory joint diseases associated with autoimmune disorders. Among these autoimmune diseases, lupus is the most commonly described disorder. Other associated conditions susceptible to PML include rheumatoid arthritis (0.25% of PML cases), Wegener granulomatosis, dermatomyositis, polymyositis, and scleroderma. Progressive multifocal leukoencephalopathy lesions are characterized by multifocal, demyelinating changes that most likely occur because of JC virus-induced oligodendrocyte cell death – resulting in profound demyelination, and typically, a rapidly fatal outcome. Thus, MR imaging of the brain shows suggestive changes. A positive PCR finding from CSF testing

166

identifying the JC virus confirms the PML diagnosis. Better understanding of PML disease pathophysiology and imaging appearances eliminates the need for brain biopsy to confirm positively a PML diagnosis, in most patients. Typical PML lesions located at the subcortical white matter with relative sparing of the adjacent gray matter can be best evaluated on T2WI sequences. Classic PML lesions are hyperintense on T2WI and hypointense on T1WI. Low T1 lesion signal is typical of PML. As the disease progresses, lesions enlarge over time with an advancing edge that typically demonstrates restricted diffusion from swollen oligodendrocytes. The central component of a large PML lesion gradually cavitates and becomes cystic in nature, demonstrating nulling of CSF signal on FLAIR. There may be areas of microcysts within the relatively recent areas of involvement that can be best demonstrated on T2WI sequences. The key observation is absence of mass effect, given the size of PML lesions. The profound demyelination, in the absence of inflammation, gives PML lesions a visually striking histopathologic appearance. Expectedly, there is no associated enhancement in a classic PML lesion. However, immune system reconstitution can occur, either because of antiretroviral therapy use to treat AIDS, or from discontinued use of immunosuppressive drugs in non-HIV patients, allowing the body to mount an antiviral inflammatory response. As a result, an immunoregulatory response can be visualized as subtle mass effect with patchy areas of enhancement on imaging, as seen in this patient.

Key Points  A T1 hypointense, T2 hyperintense, non-enhancing lesion with linear areas of diffusion restriction at the margin with either unilateral or bilateral middle cerebellar peduncle involvement, in the correct clinical setting is almost conclusively diagnostic of PML.  Patchy areas of enhancement with subtle mass effect may be seen in PML-IRIS, particularly with an identifiable cause of immune reconstitution.

Suggested Reading Bag AK, Curé JK, Chapman PR, Roberson GH, Shah R. JC virus infection of the brain. AJNR Am J Neuroradiol 2010; 31: 1564–76. Boren EJ, Cheema GS, Naguwa SM, Ansari AA, Gershwin ME. The emergence of progressive multifocal leukoencephalopathy (PML) in rheumatic diseases. J Autoimmun 2008; 30(1–2): 90–8. Calabrese LH, Molloy ES, Huang D, Ransohoff RM. Progressive multifocal leukoencephalopathy in rheumatic diseases: evolving clinical and pathologic patterns of disease. Arthritis Rheum 2007; 56(7): 2116–28. Giacomini PS, Rozenberg A, Metz I, et al. Maraviroc and JC virusassociated immune reconstitution inflammatory syndrome. N Engl J Med 2014; 370: 486–8. Molloy ES, Calabrese LH. Progressive multifocal leukoencephalopathy: a national estimate of frequency in

Part III. Neuroinfectious Diseases: Case 39 systemic lupus erythematosus and other rheumatic diseases. Arthritis Rheum 2009; 60(12): 3761–5. Okamoto K, Tokiguchi S, Furusawa T, et al. MR features of diseases involving bilateral middle cerebellar peduncles. AJNR Am J Neuroradiol 24(10): 1946–54.

Palazzo E, Yahia SA. Progressive multifocal leukoencephalopathy in autoimmune diseases. Joint Bone Spine 2012; 79(4): 351–5. Tan CS, Koralnik IJ. Beyond progressive multifocal leukoencephalopathy: expanded pathogenesis of JC virus infection in the central nervous system. Lancet Neurol 2010; 9(4): 425–37.

167

CASE

Part III

40

Neuroinfectious Diseases Ricardo Tavares Daher, Lázaro Luís Faria do Amaral

Clinical Presentation A 52-year-old man with a history of HIV infection and high-risk sexual behavior presented with progressive dementia and behavioral disturbances. Combined presence of HIV and neurologic symptoms prompted immediate CSF evaluation. Cerebrospinal fluid analysis revealed pleocytosis and elevated proteins. Magnetic resonance imaging was performed (Fig. 40.1) and controls were made at two-year post-treatment follow-up (Fig. 40.4).

Imaging (A)

(B)

Fig. 40.1 (A–B) Coronal FLAIR through the temporal lobes.

Advanced Neuroradiology Cases, ed. Amaral et al. Published by Cambridge University Press. © Cambridge University Press 2016.

169

Part III. Neuroinfectious Diseases: Case 40

(A)

(B)

(C)

Fig. 40.2 (A–C) Axial T1WI postcontrast images through the posterior fossa.

170

Part III. Neuroinfectious Diseases: Case 40

(A)

(C)

(B)

Fig. 40.3 (A–C) Axial T1WI postgadolinium images through the lateral ventricles.

(A)

(B)

Fig. 40.4 (A) Axial FLAIR and (B) Axial T1WI postgadolinium images through the third and lateral ventricles (two-year post-treatment follow-up).

171

Part III. Neuroinfectious Diseases: Case 40

Neurosyphilis Involving Temporal Lobe Primary Diagnosis Neurosyphilis involving temporal lobe

Differential Diagnoses Meningeal syphilis Syphilitic transverse myelitis Herpes simplex infection Polyradiculitis

Imaging Findings Fig. 40.1: (A–B) Coronal FLAIR demonstrated signal hyperintensity of the hippocampus and temporal lobe white matter. Fig. 40.2: (A–C) Postcontrast axial T1WI through the posterior fossa, and Fig. 40.3: (A–C) Contrast-enhanced, axial T1WI supratentorial images showed micronodular, abnormal enhancement on the right hippocampus, amygdala, temporal lobe, and leptomeningeal enhancement in the posterior fossa and supratentorial areas. Fig. 40.4: (A) Axial FLAIR and (B) T1WI postcontrast control images (two-year post-treatment follow-up) demonstrated improvement of the lesions and hippocampal and temporal lobe atrophy.

Discussion The varied neuroradiologic findings in neurosyphilitic patients implicate numerous differential diagnoses, dependent on topographic location. For example, meningeal syphilis is difficult to differentiate from other inflammatory meningeal pathologies; syphilitic transverse myelitis, and polyradiculitis share some unspecific findings that also occur in myelitis; and the presence of high intensity signal in the temporal lobes can be difficult to differentiate from herpes simplex infections. Thus, patient clinical evaluation must encompass thorough appreciation of all presenting symptomatology in correlation with neuroradiologic findings. Neurosyphilis is a parasitic infection of the brain and spinal cord caused by a sexually transmitted spirochete, Treponema pallidum. Central nervous system involvement typically indicates tertiary syphilitic infection. The increased prevalence of patients with AIDS coincides with a significant rise in the incidence of syphilis, and consequently, neurosyphilis. Approximately 1.5% of patients with AIDS develop neurosyphilis; however, it also occurs in 5–10% of untreated patients. Many patients with neurosyphilis are asymptomatic or have non-specific symptoms, confounding diagnostic efforts. However, its symptomatic nosology can be divided into four forms based on the predominant feature: meningeal, vascular (the most prevalent), general paresis, and tabes dorsalis (a rare manifestation in post-antibiotic era). Syphilis follows a natural course of progression that has four stages: 1) early stage – chancre appears at the inoculation site with regional adenopathy; 2) secondary stage – two to four

172

weeks later, the characteristic hematologic dissemination of syphilis manifests as a rash on the palms of the hands and soles of the feet, CNS involvement may occur, producing aseptic meningitis; 3) latent stage – begins when earlier symptoms disappear (serology remains positive for infection; and 4) tertiary syphilis – occurs 5 to 10 years after the primary infection and affects the CNS, cardiovascular system, bones, joints, skin, and mucous membranes. Depending on the form, neurosyphilitic infection presents a wide variety of imaging findings. Meningeal neurosyphilis shows a focal or diffuse thickening and enhancement of the leptomeninges or pachymeninges, with or without hydrocephalus. The meningeal form can be also associated with cranial nerve involvement (commonly, CN II, and VIII), and formation of focal leptomeningeal granulomas or gummas of various sizes, which appear iso- to hypointense on T1WI, and hyperintense on T2WI. Vascular neurosyphilis shows evidence of multiple focal hyperintensities on T2WI, involving both gray and white matter associated with small foci of infarcts, secondary to vasculitis. In patients with the general paresis form, classical findings include frontal and temporal lobe cortical and subcortical atrophy, with subcortical gliosis. The tabes dorsalis form demonstrates an increase in signal intensity of the spinal cord on T2WI, a contrast enhancement of the pia and of the spinal cord nerve roots, and spinal cord atrophy. Laboratory data can help approximate the diagnosis and may include positive serum fluorescent antibody findings, pleocytosis, elevated protein levels, or a positive VDRL test. The CSF should be analyzed; analysis should include VDRL, at least one treponemal reaction (FTA abs), PCR for Treponema pallidum, as well as standard CSF tests. Although VDRL of CSF is highly specific for neurosyphilis, it is negative in approximately one-half of neurosyphilis patients. Owing to immunocompromise, the course of neurosyphilis in HIV-infected patients tends to be more aggressive than it is in the general population.

Key Points  Neurosyphilis is associated with multiple patterns of CNS involvement and multiple types of radiographic manifestations.  Thus, diagnostic neuroradiology of patients with neurosyphilis may be challenging.  Because syphilis serologies are not routinely tested in patients with seizures or amnesia, radiologists should suspect neurosyphilis, herpes encephalitis, or limbic encephalitis when mesiotemporal signal changes are seen on MRI.

Suggested Reading Bash S, Hathout GM, Cohen S. Mesiotemporal T2-weighted hyperintensity: neurosyphilis mimicking herpes encephalitis. AJNR Am J Neuroradiol 2001; 22: 314–16.

Part III. Neuroinfectious Diseases: Case 40 Brightbill TC, Ihmeidan IH, Post MJD, Berger JR, Katz DA. Neurosyphilis in HIV-positive and HIV-negative patients: neuroimaging findings. AJNR Am J Neuroradiol 1995; 16: 703–11. Gupta RK. Tuberculosis and other non-tuberculous bacterial granulomatous infections. In: Gupta RK, Lufkin RB, eds. MR Imaging and Spectroscopy of Central Nervous System Infection. New York: Kluwer Academic/Plenum Publishers, 2001; 95–145. Jeong YM, Hwang HY, Kim HS. MRI of neurosyphilis presenting as mesiotemporal abnormalities: a case report. Korean J Radiol 2009; 10: 310–12. Machado LR, Livramento JA, Vianna LS. Cerebrospinal fluid analysis in infectious diseases of the nervous system: when to

ask, what to ask, what to expect. Arq Neuropsiquiatr 2013; 71(9-B): 693–8. Pandey S. Magnetic resonance imaging of the spinal cord in a man with tabes dorsalis. J Spinal Cord Med 2011; 34(6): 609–11. Quétel C. The History of Syphilis. Baltimore: John Hopkins University Press, 1992. Smith AB, Smirniotopoulos JG, Rushing EJ. Central nervous system infections associated with human immunodeficiency virus infection: radiologic—pathologic correlation. Radiographics 2008; 28: 2033–58. Smith MM, Anderson JC. Neurosyphilis as a cause of facial and vestibulocochlear nerve dysfunction: MR imaging features. AJNR Am J Neuroradiol 2000; 21: 1673–5.

173

CASE

Part III

41

Neuroinfectious Diseases Lázaro Luís Faria do Amaral

Clinical Presentation A previously healthy 49-year-old man presented with a history of headache and seizures. He was referred to our hospital after CT scans at another hospital revealed the presence of a brain tumor. Prior to his referral, he suffered from frequent headaches and used analgesics to control the pain. He had no history of diabetes mellitus or hypertension. Although he was HIV-negative, it was noted that he had a previous history of promiscuous behavior. No other focal neurologic deficit was present and no systemic symptoms were noted. Hematologic tests were negative for all rheumatologic and autoimmune diseases. Serum ACE levels were in the normal range.

Imaging (A)

(B)

(C)

Fig. 41.1 (A–C) Axial T2WI through the level of the lateral ventricles.

Advanced Neuroradiology Cases, ed. Amaral et al. Published by Cambridge University Press. © Cambridge University Press 2016.

175

Part III. Neuroinfectious Diseases: Case 41

(A)

(B)

(C)

Fig. 41.2 (A–C) Axial T1WI postgadolinium images through the level of the lateral ventricles.

176

Part III. Neuroinfectious Diseases: Case 41

(A)

(B)

Fig. 41.3 (A–B) Sagittal T1WI postgadolinium images through the level of the tentorium and the right temporal lobe.

Fig. 41.4 Spectroscopy through the level of the lesion in the right temporal lobe.

177

Part III. Neuroinfectious Diseases: Case 41

Syphilitic Gumma Primary Diagnosis Syphilitic gumma

Differential Diagnoses Glioblastoma Bacterial, mycobacterial, and fungal abscesses Toxoplasmosis Lymphoma

Imaging Findings Fig. 41.1: (A–C) T2WI demonstrated a heterogeneous, hyperintense mass in the posterior part of the right temporal superior gyrus, surrounded by vasogenic edema. Fig. 41.2: (A–C) Axial T1WI postcontrast demonstrated a ringenhancing lesion with mass effect in the periventricular white matter, adjacent to the atrium of the right ventricle. Fig. 41.3: (A) Sagittal T1WI postgadolinium image showed thickening and enhancement of the anterior portion of the pachymeninges in the right tentorium (arrows), (B) at the same side of the ring-enhancing lesion. Fig. 41.4: Spectroscopy of the lesion showed a high choline peak, low NAA peak, and high lipid and lactate peaks, similar to spectroscopy findings of a high-grade tumor.

Discussion In general, glioblastoma is a more infiltrative lesion. Toxoplasmosis is a ring-enhancing lesion with an eccentric nodule. Pyogenic abscesses have a ring-enhancement appearance, with restricted diffusion in the central area of necrosis. Fungal lesions are hypointense on T2 images. Lymphoma shows restricted diffusion and solid areas of enhancement postgadolinium in non-HIV patients. Although syphilitic gumma is a ring-enhancing lesion found in the brain, it should be differentially diagnosed to distinguish it from other brain masses. In the case of this patient, a brain tumor was first suspected, based on initial imaging. In our case, the presence of thickening and intense enhancement of the pachymeninges of the tentorium at the same side strongly suggested an infectious disease etiology. The associated clinical history, promiscuous behavior, and clinical findings including positive CSF laboratory findings for VDRL, and MR images demonstrating a ring-enhancing lesion with pachymeningeal involvement also supported an infectious etiology. Surgical biopsy confirmed a diagnosis of cerebral gumma due to neurosyphilis. Syphilis, along with the recent increase of HIV patients, has also been on the rise. It has a broad spectrum of clinical manifestations, including cerebral gumma, a manifestation of neurosyphilis; however, it is rare and can be cured by penicillin. Thus, cerebral gumma needs to be differentially diagnosed from other brain masses that may be present in syphilis patients. Syphilis is a chronic, systemic infectious disease caused by the spirochete, Treponema pallidum, and can affect most

178

organs. It is commonly transmitted via sexual contact; however, it is vertically transmitted from mother to fetus in some cases. It was expected to be eradicated by the use of penicillin, one of the most effective antibiotics. Since the early 2000s, however, syphilis has been reported to be prevalent among HIV patients in several countries. Clinically, syphilis progresses through the following series of stages: incubation period following initial infection, primary syphilis, secondary syphilis, latent syphilis, and late syphilis (or tertiary syphilis). Late syphilis includes cardiovascular syphilis, neurosyphilis, and late benign syphilis (gummatous syphilis). Recent studies showed that syphilis is prevalent in HIV-positive patients, is characterized by a broad spectrum of symptoms, and that it can occur after 1 to 25 years following infection. Various manifestations of neurosyphilis may be observed, such as asymptomatic, meningovascular, and parenchymal neurosyphilis. Of the various types of neurosyphilis, gummatous neurosyphilis is rare; its occurrence is not associated with HIV infection, and it is commonly misdiagnosed as a brain tumor. Brain gumma is a curable disease; therefore, an appropriate diagnosis is essential for optimal patient treatment and prognosis. The etiology of brain gumma due to neurosyphilis was presumed to be an excessive response of the cell-mediated immune system to T. pallidum. Histopathologically, this case was characterized by the presence of a circumscribed mass resembling granuloma at the site where lymphocytes or plasma cells infiltrated the brain parenchyma or meninges. This granuloma underwent fibrotic or necrotic transformation over time. Spirochetes are rarely identified from tissue samples. In the present case, the spirochetes were detected on histologic examination. Brain gumma commonly develops from the dura and pia mater over the cerebral convexity or at the base of the brain. Single or multiple masses attached to the dura mater can invade brain parenchyma. The symptoms of brain gumma are similar to those of other tumors arising from brain parenchyma and are often accompanied by seizures. The radiologic findings of brain gumma are very inconsistent. On CT scans, the lesion was localized at the periphery of brain tissue. The findings of non-contrast-enhancement CT revealed a hypodense area with no mass effect. On contrast-enhanced CT scans, however, brain gumma can be observed to be accompanied by severe edema of adjacent tissue. T1WI MR scans showed a low-intensity or isointensity mass. T2WI MR scans, however, revealed a homogeneous and high-intensity mass. The adjacent area of the mass showed high intensity on T2WI and a low intensity on T1WI. Brain gumma occurs commonly in association with the meninges. Accordingly, the location of lesion and the findings of contrast-enhanced imaging are useful in making a presumptive diagnosis of brain gumma. In the present case, brain tumor was suspected, since the central portion of the lesion was necrotic. The MR spectroscopic image showed higher peaks formed by choline compounds, indicating the possibility of a tumor. The choline peaks in MR spectroscopy represent

Part III. Neuroinfectious Diseases: Case 41

the complexes of phosphorylcholine and glycerophosphorylcholine found in the membrane, and these complexes play a role in cell membrane synthesis or destruction. Choline peaks are thus regarded as cancer markers. Low NAA peaks mark neuronal populations, and higher peaks of lipids and lactate (anaerobiosis) typically indicate high-grade brain tumors.

Key Points  In this case, the patient was preoperatively suspected of having a brain tumor but was diagnosed with brain gumma, based on brain histopathology and CSF cytology.  From an empirical perspective, a high-dose penicillin therapy accompanied with MRI findings of a decrease in

mass size would be useful for the diagnosis based on the CSF examination and MRI, thereby avoiding unnecessary surgeries.

Suggested Reading Ances BM, Danish SF, Kolson DL, Judy KD, Liebeskind DS. Cerebral gumma mimicking glioblastoma multiforme. Neurocrit Care 2005; 2:300–2. Berger JR, Waskin H, Pall L, et al. Syphilitic cerebral gumma with HIV infection. Neurology 1992; 42: 1282–7. Hook EW 3rd, Marra CM. Acquired syphilis in adults. N Engl J Med 1992; 326: 1060–9. Pall HS, Williams AC, Stockley RA. Intracranial gumma presenting as a cerebral tumor. J R Soc Med 1988; 81: 603–4.

179

CASE

Part III

42

Neuroinfectious Diseases Asim K. Bag, Lázaro Luís Faria do Amaral

Clinical Presentation A 41-year-old, immunocompetent man, originally from the Great Lakes Basin, presented with acute onset of diplopia. He reported a recent history of gradually progressive, intermittent dizziness, headaches, difficulty walking down stairs, and rightsided tingling and numbness of his face, hand, and foot. Two weeks later, he returned to our clinic and reported new onset of diplopia (see imaging). His CSF studies demonstrated slightly elevated protein levels, but were otherwise unremarkable. Cerebrospinal fluid culture was negative for bacterial growth. Routine hematologic studies, paraneoplastic profile, serum ACE levels, and antibodies for common autoimmune disorders were negative. Extensive diagnostic workup did not reveal any primary malignancy or mediastinal adenopathy.

Imaging Fig. 42.1 Axial FLAIR through the level of the pons.

Fig. 42.2 Axial T2WI at the same level.

Fig. 42.3 Axial T1WI postcontrast through the level of the pons.

Fig. 42.4 DWI through the same level.

Advanced Neuroradiology Cases, ed. Amaral et al. Published by Cambridge University Press. © Cambridge University Press 2016.

181

Part III. Neuroinfectious Diseases: Case 42

Central Nervous System Blastomycosis Primary Diagnosis Central nervous system blastomycosis

Differential Diagnoses Metastasis Lymphoma Sarcoid Listeria rhombencephalitis Other causes of rhombencephalitis Brainstem glioma

Imaging Findings Fig. 42.1: Axial FLAIR image through the level of the pons demonstrated diffuse swelling of the brainstem, with FLAIR hyperintensity involving the entire pons and bilateral middle cerebellar peduncles, and mass effect to the fourth ventricular floor. Fig. 42.2: Axial T2 image through the same level demonstrated the same findings; however, prominent hypointensity at the center of the lesion was noted. Fig. 42.3: Axial postcontrast T1WI at the same level demonstrated heterogeneous ring enhancement of the T2 hypointense area. Fig. 42.4: DWI through the same level demonstrated a tiny focus of diffusion restriction (with low ADC value, not shown) at the center of the T2 hypointense and relatively less enhancing area of the lesion. Spectroscopy demonstrated high choline-to-creatinine ratio and decreased NAA peak (not shown).

Discussion In a patient from an endemic area, a gradually worsening disease process involving the brainstem, with diffuse enlargement and FLAIR hyperintensity on imaging, is suggestive for CNS blastomycosis. This is particularly true with the confirmed presence of a central T2 hypointensity that demonstrates heterogeneous ring enhancement in the absence of congruent diffusion restriction. Central T2 hypointensity can often be seen in mucinous metastasis (see Part VI: Case 102), lymphoma, and neurosarcoidosis. In an immunocompetent patient with CNS lymphoma, solid enhancement, and incongruent diffusion, restriction is typically demonstrated on imaging studies, in the absence of heterogeneous ring enhancement. In lymphoma patients, the entire T2 hypointense area usually demonstrates diffusion restriction because of hypercellularity. Ring enhancement is a feature of CNS lymphoma in an immunocompromised patient, or in a patient with prior treatment – not a treatment-naïve CNS lymphoma in an immunocompetent patient. The combination of T2 hypointensity, heterogeneous ring-like enhancement, and extensive perilesional FLAIR hyperintensity can be seen in a metastasis from a mucinous adenocarcinoma, either from the gastrointestinal tract or from the lungs. Although metastasis from these primary sites can involve any regions of the brain, involvement of isolated brainstem is uncommon. Absence of a known primary excludes diagnosis of metastatic disease.

182

Intraparenchymal CNS sarcoidosis can present with similar imaging findings. However, intraparenchymal lesions are usually associated with adjacent leptomeningeal disease, notably absent in this patient. Additionally, isolated involvement of the brainstem in CNS sarcoidosis is rare. Listeria is the most common cause of rhombencephalitis. The absence of CNS pleocytosis and negative CSF culture findings exclude a diagnosis of CNS listeriosis. Based on the imaging appearance, brainstem gliomas should be considered in the differential diagnosis, particularly in conjunction with the presence of high choline and low NAA levels, as well as the clinical presentation. However, prominent T2 hypointensity and tiny central, rather than peripheral, diffusion restriction is atypical for high-grade brainstem gliomas. Blastomyces dermatitidis is the causative agent for blastomycosis. Blastomycosis is an uncommon, but potentially serious fungal infection, endemic in the Mississippi and Ohio River basins, and in the Great Lakes and the St Lawrence River regions. Occasionally, it can be found in non-endemic areas as well. There is no predilection for age, sex, race, or occupation, but it occurs more frequently in immunocompromised hosts, particularly in patients with AIDS. It primarily affects the pulmonary system, frequently manifesting as acute or chronic pneumonia. Central nervous system dissemination accounts for 5–10% of extrapulmonary blastomycosis and usually manifests as an intracranial mass lesion, spinal cord abscess, or epidural abscess. Brain masses may be associated with adjacent leptomeningitis; however, presentation with isolated meningitis is rare. The cerebellum is frequently involved, but blastomycosis can involve any region of the brain. Multiple lesions throughout the brain mimicking multiple cerebral metastases have also been described in the literature. Cerebrospinal fluid analysis is not as sensitive as CSF culture in the diagnosis of blastomycosis. Central nervous system blastomycosis is usually treated with amphotericin B, in combination with oral azoles.

Key Points  Central nervous system involvement with blastomycosis is a rare but serious complication of systemic blastomycosis.  It should be suspected in a patient with gradual onset posterior fossa symptoms and a mass-like appearance on imaging.  Prominent T2 hypointensity can be a clue to the diagnosis.

Suggested Reading Bariola JR, Perry P, Pappas PG, et al. Blastomycosis of the central nervous system: a multicenter review of diagnosis and treatment in the modern era. Clin Infect Dis 2010; 50: 797–804. Borgia SM, Fuller JD, Sarabia A, El-Helou P. Cerebral blastomycosis: a case series incorporating voriconazole in the treatment regimen. Med Mycol 2006; 44: 659–64. Chapman SW, Dismukes WE, Proia LA, et al. Clinical practice guidelines for the management of blastomycosis: 2008 update by the Infectious Diseases Society of America. Clin Infect Dis 2008; 46(12): 1801–12.

CASE

Part III

43

Neuroinfectious Diseases Fabrício Guimarães Gonçalves, Guilherme Cássia

Clinical Presentation A 45-year-old woman with a history of rheumatoid arthritis presented to our facility with acute dyspnea and cough and acute changes in mental status, confusion, seizures, and left arm hemiparesis. Her medical history included use of highdose steroids and methotrexate. Cerebrospinal fluid laboratory studies disclosed the following values: elevated leukocytes, 17/mm3 with 56% segmented neutrophils; erythrocytes, 4/mm3; glucose, 69 mg/dl; and protein, 20 mg/dl. On admission, she had an abnormal chest X-ray that showed multiple nodular opacities bilaterally. Because of her neurologic deterioration, a CT scan of the brain was performed. Subsequent to CT, MR imaging studies were performed. Conformational diagnosis was made following brain biopsy.

Imaging

Fig. 43.1 Axial T1WI at the level of C1–C2. Fig. 43.2 Axial T2WI at the level of the lateral ventricles.

Advanced Neuroradiology Cases, ed. Amaral et al. Published by Cambridge University Press. © Cambridge University Press 2016.

183

Part III. Neuroinfectious Diseases: Case 43 Fig. 43.3 Axial proton density image at the level of the basal ganglia.

Fig. 43.4 Axial enhanced fat-saturated T1WI at the same level as Fig 43.3.

184

Part III. Neuroinfectious Diseases: Case 43

Fig. 43.5 Axial DWI at the level of the central semiovale.

185

Part III. Neuroinfectious Diseases: Case 43

Central Nervous System Aspergillosis Primary Diagnosis Central nervous system aspergillosis

Differential Diagnoses Brain metastasis Pyogenic brain abscesses

Imaging Findings Fig. 43.1: Axial T1WI at the level of C2 demonstrated excessive, subcutaneous fat accumulation in the upper neck, secondary to chronic steroid use. Fig. 43.2: Axial T2WI at the level of the lateral ventricles showed multiple rounded lesions associated with a moderate degree of vasogenic edema. Note that some lesions have a marked hypointense rim, which can represent hemorrhage, or deposition of other paramagnetic substances such as iron, manganese, or magnesium (arrow). Fig. 43.3: Axial proton density image at the level of the basal ganglia showed multiple hyperintense nodules with a moderate degree of vasogenic edema in the basal ganglia and subcortical white matter (arrows). Fig. 43.4: Axial enhanced fat-saturated T1WI at the level of the basal ganglia showed multiple, widespread ring-enhancing lesions in the brain parenchyma (arrows). Fig. 43.5: Axial DWI at the level of the central semiovale demonstrated that the multiple ring-enhancing lesions showed internal restricted diffusion of the molecules’ motion. Apparent diffusion coefficient map (not shown) demonstrated that the areas of increased signal have low values.

Discussion Imaging findings demonstrating multiple ring-enhancing lesions, with restricted diffusion, hemorrhage, and surrounding vasogenic edema, in a chronically immunosuppressed patient, with rapid neurologic deterioration, are consistent with the presence of an opportunistic infection, such as aspergillosis. Central nervous system metastases can occur in around 15–40% of patients with malignant cancer. The most common primary metastatic CNS cancers include lung, breast, skin, colon, pancreas, testes, ovary, cervix, renal cell carcinoma, and melanoma. The majority of patients with CNS metastases are asymptomatic (60–75%). When symptomatic, patients may present with headache, seizure, syncope, focal neurologic deficit, or papilledema. Metastasis tends to be located at the graywhite junction and at watershed zones between major arterial vascular territories. Up to 80% of brain metastases occur in the cerebral hemispheres, 15% in the cerebellum, and 3% in the basal ganglia. More rarely, metastatic infiltration of the choroid plexus can include the meninges, ventricular system, pituitary gland, and along blood vessels. Hemorrhagic metastasis is more commonly associated with melanoma, choriocarcinoma, renal cell carcinoma, and thyroid cancer. Lung and breast metastases are also known to

186

bleed; moreover, they are the most common hemorrhagic metastatic CNS lesions, because of their high prevalence. On MRI, the majority of these lesions are iso- or hypointense on T1, hyperintense on T2, and exhibit an avid solid, or ring of, enhancement. Diffusion-weighted images of hemorrhagic CNS metastases are usually negative, demonstrating restricted diffusion, with high values on ADC maps. Vasogenic edema, usually a common feature, tends to be disproportionate in relation to the size of the lesion. The combination of the patient’s medical history (age, absence of malignancy) and imaging findings of diffuse and multiple restricted diffusion foci makes CNS metastasis an unlikely diagnosis in this case. Pyogenic brain abscesses are known to have all of the previously mentioned imaging characteristics including ring enhancement, vasogenic edema, signal changes, and typical, internal restricted diffusion. Pyogenic brain abscesses are unquestionably an excellent probable diagnostic possibility in this patient. The clinical history of immunosuppression should raise suspicion for the possibility of an opportunistic infection. Aspergillus is a ubiquitous saprophytic fungus commonly found in soil, on plants and building materials, as well as in household dust and in decaying organic matter. There are many different species of Aspergillus, but the most common species that can cause aspergillosis are Aspergillus fumigatus and Aspergillus flavus. Affected individuals are usually infected by inhaling aspergillus spores. The most clinically important forms of aspergillosis are invasive aspergillosis, allergic bronchopulmonary aspergillosis, and aspergillomas. Invasive aspergillosis usually affects immunosuppressed individuals, in which the fungi invade and damage multiple tissues. It more commonly affects the lungs and paranasal sinuses, but can spread throughout the body and cause infection in other organs, such as the brain, its membranes, and other CNS components. Intracranial spread of aspergillosis is a severe and commonly fatal complication, occurring in 10–20% of cases with a very high mortality rate of more than 90%. Aspergillus can reach the CNS by three different routes. The first is by hematogenous spread from a remote extracranial focus, commonly from the lungs, leading to occlusion of large and middle-sized arteries. The second is by contiguous extension from an extracranial location, commonly from the paranasal sinuses (rhino-cerebral). The third route of CNS entry is iatrogenically, by direct introduction of aspergillus via a surgical intervention. Other potential sites of primary infection include the ear canal, aorta, heart, vertebral disks, and/or cranial bones following traumatic injury or from surgical contamination. Central nervous system aspergillosis is a worrisome potential complication in patients undergoing chemotherapy or on immunosuppressive therapy following solid organ or bone marrow transplantation. Additional patients at risk for opportunistic CNS aspergillosis include individuals with diabetes, autoimmune diseases requiring steroids, hepatic insufficiency,

Part III. Neuroinfectious Diseases: Case 43

chronic obstructive pulmonary disease, previous brain lesions, or HIV-AIDS. Previously injured brain tissue (glial scars) may aid the attachment and subsequent invasion of the fungal spores, possibly because of an abnormal blood-brain barrier. Various imaging patterns can be seen in patients with CNS aspergillosis, such as single or multiple infarcts, single or multiple ring-enhancing lesions (consistent with abscess formation), and dural or vascular infiltration arising from the paranasal sinuses or orbits. Other findings include meningitis, encephalitis, mycotic aneurysm, subdural/epidural abscesses, hemorrhagic transformation of infarcted areas, and sellar abscesses. Signs of hemorrhage may also be seen secondary to vascular invasion. In order to depict all relevant MRI features in CNS aspergillosis, the protocol should include T1- and T2-weighted sequences, T2*-weighted sequences for an early detection of hemorrhage, DWI to detect early signs of ischemia, and contrast-enhanced T1-weighted images for abscess identification. Spectroscopy may be helpful in analysis of amino acid and lipid levels, and lactate peaks. The presence of multiple peaks seen between 3.6 and 3.8 ppm, due to presence of the disaccharide trehalose, is a distinguishable feature of fungal abscesses. Diffusion-weighted imaging plays an important role in detecting early manifestations of ischemic CNS lesions because of the angioinvasive character of the fungi, particularly in the hematogenous spread of spores. Involvement of the basal ganglia is a characteristic finding that may indicate a predominant involvement of the lenticulostriate and thalamoperforating arteries. In the majority of the patients, an infarction is followed by a conversion to an infected area, with subsequent abscess formation. When there is extension of aspergillosis from the paranasal sinuses, initial involvement is seen, in most cases, as a nonspecific contrast-enhancing pansinusal mucosal thickening of the sinuses on CT or MRI. Serious complications may exist in cases of infiltration and erosion of the adjacent bony structures, leading to focal meningitis. In cases of invasive lesions in the brain, a well-defined granuloma can usually be seen in the basifrontal or anterior temporal region, often having continuity with the adjacent sinus disease. These granulomas are virtually indistinguishable from tuberculous granulomas or other chronic granulomas. Aspergillus granulomas are typically T1 hypointense, with a T2 hyperintense center and hypointense wall, attributable to paramagnetic substances such as iron, manganese, and magnesium that are found in the fungal hyphae. Granulomas also show mild to moderate ring-like contrast enhancement and 25% of lesions can be purely hemorrhagic. Dural enhancement is generally seen in lesions adjacent to involved sinuses and the existence of a concomitant sinonasal disease points toward fungal etiology. Aspergillus spores have a predilection for the anterior and middle cranial fossa and abscesses in the cerebellum are extremely rare.

Central nervous system aspergillosis may occur in any age group, with a mean age of 43 years, with a slight male predominance. Regarding signs and symptoms, patients commonly present with headache, fever, mental status changes, generalized seizures, focal neurologic deficits, hemiplegia, vision changes, and cranial nerve palsy. Aspergillus fumigatus is the predominant infectious species found in patients with aspergillosis. It has a propensity to invade the blood vessels following the release of the enzyme elastase, leading to coagulative necrosis, infarcts, mycotic aneurysms, and hemorrhagic lesions. Galactomannan serologic levels are also helpful diagnostic values, rapid diagnostic PCR techniques performed on the CSF and serum could facilitate a faster and simpler diagnostic approach. Voriconazole has become the standard of care in most cases of CNS aspergillosis. Mortality is high even among patients who received medical treatment, but patients who underwent neurosurgery had a better survival than those who received only medical therapy. Surgical procedures may involve abscess removal or drainage, excision of infected mass, orbital exenterations, debridement of inner ear, and debridement of the skull base. The aim of treatment should be the complete removal of the masses as early as possible. In patients with invasive aspergillosis, providing intradural extension has not yet occurred, total excision may be curative without systemic use of antifungal agents. However, in patients in whom total removal cannot be achieved or intradural invasion has already been confirmed, intensive systemic therapy with antifungal agents must be administered postoperatively to prevent subsequent lethal vasculitis or meningoencephalitis development. Incidence of invasive aspergillosis appears to be on the rise. A comprehensive understanding of the infectious process, a high index of suspicion, and advanced laboratory and radiologic diagnostic techniques, may allow early diagnosis.

Key Points  Immunosuppressed patients are at high risk of opportunistic infections such as CNS aspergillosis.  Central nervous system aspergillosis can have similar imaging findings to metastasis and pyogenic abscesses. Immunosuppression and lack of history of malignancy are important clinical features to consider when including or excluding potential diagnoses.  Intracranial spread of aspergillosis is a severe and commonly fatal complication, occurring in 10–20% of cases of aspergillosis, and it has a very high mortality rate of more than 90%.  Central nervous system aspergillosis is a worrisome potential complication in patients treated with intensive chemotherapy and immunosuppressive therapy after solid organ or bone marrow transplantation.  A comprehensive understanding of the infectious process, a high index of suspicion, and advanced laboratory

187

Part III. Neuroinfectious Diseases: Case 43

and radiologic diagnostic techniques may allow early diagnosis and favorable outcome.

Suggested Reading Antinori S, Corbellino M, Meroni L, et al. Aspergillus meningitis: a rare clinical manifestation of central nervous system aspergillosis. Case report and review of 92 cases. J Infect 2013; 66(3): 218–38.

188

Khandelwal N, Gupta V, Singh P. Central nervous system fungal infections in tropics. Neuroimaging Clin N Am 2011; 21(4): 859–66, viii. Kourkoumpetis TK, Desalermos A, Muhammed M, Mylonakis E. Central nervous system aspergillosis: a series of 14 cases from a general hospital and review of 123 cases from the literature. Medicine (Baltimore) 2012; 91(6): 328–36.

CASE

Part III

44

Neuroinfectious Diseases Bruno Shigueo Yonekura Inada, Bruno Siqueira Campos Lopes, Lázaro Luís Faria do Amaral

Clinical Presentation A 25-year-old man, born in Alagoas (northeast Brazil), presented to our facility with a three-week history of headache. Magnetic resonance imaging studies were performed; however, he was subsequently discharged several days later, without a diagnosis. He was brought back to our emergency department after two months of worsening headache and one episode of seizure.

Imaging (B)

(A)

(C)

Fig. 44.1 (A) Axial FLAIR, (B) Axial T2WI, (C) Coronal T1WI, and (D) Axial T1WI postgadolinium images through the level of the pons.

(D)

Advanced Neuroradiology Cases, ed. Amaral et al. Published by Cambridge University Press. © Cambridge University Press 2016.

189

Part III. Neuroinfectious Diseases: Case 44

(A)

(C)

(B)

(D)

Fig. 44.2 (A) Axial FLAIR, (B) Coronal T2WI, (C) Sagittal T1WI postgadolinium, (D) Coronal T1WI postgadolinium, and (E) Axial T1WI postgadolinium images (two months after initial admission imaging studies) through the level of the frontal lobes.

190

Part III. Neuroinfectious Diseases: Case 44 (E)

Fig. 44.2 (cont.)

191

Part III. Neuroinfectious Diseases: Case 44

Cerebral Schistosomiasis Primary Diagnosis Cerebral schistosomiasis

Differential Diagnoses Central nervous system neoplasm Neurosarcoid Tuberculosis Neurocysticercosis

Imaging Findings Fig. 44.1: (A) Axial FLAIR, (B) Axial T2WI, (C) Coronal T1WI, and (D) Axial T1WI postgadolinium images (MR studies from initial admission) demonstrated a hyperintense lesion on the left pons with a small, enhanced nodule in the left middle cerebellar peduncle. Fig. 44.2: (A) Axial FLAIR, (B) Coronal T2WI, (C) Sagittal T1WI postgadolinium, (D) Coronal T1WI postgadolinium, and (E) Axial T1WI postgadolinium images (two months after MR studies from initial admission) demonstrated multiple punctate, spotty, nodular, enhancing lesions, with surrounding vasogenic edema, in the left periventricular white matter and corona radiata of the left frontal lobe, and ipsilateral basal ganglia.

Discussion Symptoms of cerebral schistosomiasis are non-specific, and patients usually present with headache, seizures, and focal neurologic deficits. The association of non-specific symptoms in conjunction with imaging findings may lead inexperienced radiologists to incorrectly diagnose schistosomiasis as a CNS neoplasm. Therefore, it is important that radiologists are able to recognize the typical MRI features associated with schistosomias in order to establish the correct diagnosis and avoid unnecessary surgery. On MR imaging studies, cerebral schistosomiasis characteristically demonstrates the presence of lesions with long T1 and T2 signals that are associated with prominent, surrounding vasogenic edema and mass effect. Following intravenous contrast administration, nodular enhancement is noted and eventually demonstrates central, linear enhancement – the characteristic arborized pattern typically seen in patients with cerebral schistosomiasis. The cerebral hemisphere is more frequently affected, followed by the cerebellar hemispheres, basal ganglia, and brainstem. The enhancing lesions seen in this patient, from an endemic region in South America, are suggestive of cerebral schistosomiasis.

192

Schistosomiasis is a chronic parasitic infection caused by trematode blood flukes of the genus Schistosoma. The three main species that affect humans are S. mansoni, S. haematobium, and S. japonicum. They are responsible for infecting more than 200 million people in 77 countries, with an annual mortality rate of 200,000. Schistosoma mansoni is endemic in tropical Africa, the Near East, northeastern South America, and the eastern Caribbean islands. Schistosoma haematobium is found in Africa and the Middle East, and S. japonicum is found exclusively in Asia. The Schistosoma life cycle begins when worm eggs in human feces or urine hatch in fresh water, infecting snails (intermediate host). The snails release their larvae (cercariae) into the water, infecting humans through contact with skin or mucosa, migrating to the liver or lungs, where they mature. The mature schistosome then migrates to the intestines and bladder venous plexus, where it releases eggs that are then excreted in urine and feces, finishing the life cycle. Schistosoma usually affects the liver and intestines, causing hepatic fibrosis, portal hypertension, and intestinal and bladder complications, such as carcinogenesis. There have been a few cases reported of S. mansoni and S. haematobium ectopic worm migration to the CNS – causing spinal cord lesions and rarely, affecting the brain. Conversely, S. japonicum typically affects the brain. The most likely explanation for this exception is because its eggs are smaller and lack spines, allowing them to reach the brain via the plexus of Batson. The presentation of the unique arborized pattern on MR images characteristic of Schistosoma infestation, if seen, in a patient from a schistosomiasistic endemic zone, can be used to distinguish between a CNS neoplasm, neurosarcoid, tuberculosis, and neurocysticercosis.

Key Point  The unique arborized patterns seen on MR images in a patient from an endemic zone or a recent traveller to South America, African countries, or/and south Asian regions should help make a presumptive diagnosis of schistosomiasis.

Suggested Reading Liu H, Lim CC, Feng X, et al. MRI in cerebral schistosomiasis: characteristic nodular enhancement in 33 patients. AJR Am J Roentgenol 2008; 191(2): 582–8. Mehta A, Teoh SK, Schaefer PW, Chew FS. Cerebral schistosomiasis. AJR Am J Roentgenol 1997; 168(5): 1322. Preidler KW, Riepl T, Szolar D, Ranner G. Cerebral schistosomiasis: MR and CT appearance. AJNR Am J Neuroradiol 1996; 17(8): 1598–600.

CASE

Part III

45

Neuroinfectious Diseases Karenn Barros Bezerra, Fabrício Guimarães Gonçalves

Clinical Presentation A 19-year-old woman presented with a history of headache, neck stiffness and pain. She reported development of confusion, generalized tonic-clonic seizures, and mild left-sided motor deficit and dysarthria. Prior to admission, she experienced 45 days of cough with expectoration and fever. On chest X-ray, there were nodular opacities in the upper portions of the left lung. Additionally, sputum analysis demonstrated acid-fast bacilli on Ziehl-Neelsen stain. On day five of admission, she had an unwanted and negative progression to coma, decerebration, and death.

Imaging

Fig. 45.1 Axial T2WI at the level of suprasellar cistern.

Fig. 45.2 Axial enhanced T1WI at the level of the basal cisterns.

Advanced Neuroradiology Cases, ed. Amaral et al. Published by Cambridge University Press. © Cambridge University Press 2016.

193

Part III. Neuroinfectious Diseases: Case 45 Fig. 45.3 Sagittal enhanced T1WI at the level of the midline.

Fig. 45.4 Coronal enhanced T1WI at the level of the cavernous sinuses.

194

Part III. Neuroinfectious Diseases: Case 45 Fig. 45.5 Axial DWI at the level of the lateral ventricles.

Fig. 45.6 ADC map at the same level as Fig. 45.5.

195

Part III. Neuroinfectious Diseases: Case 45

Tubercular Vasculitis Primary Diagnosis Tubercular vasculitis

Differential Diagnoses HIV cerebrovascular disease Neurosyphilis (meningovascular disease)

Imaging Findings Fig. 45.1: Axial T2WI at the level of suprasellar cistern showed hypointense material at the suprasellar cistern surrounding the optic chiasm and tracts (black arrow). At this level there were small hyperintense foci in the midbrain (white arrows), in keeping with inflammatory changes. Fig. 45.2: Axial enhanced T1WI at the level of the basal cisterns, showed enhancing material in the suprasellar, interpeduncular, and perimesencephalic cisterns, in keeping with exudative material due to basilar meningitis (black arrow). Also, note the small, rounded ring-enhancing lesions in the basifrontal region and in the right sylvian fissure, in keeping with granulomas (white arrows). Fig. 45.3: Sagittal enhanced T1WI at the level of the midline showed the same enhancing material in the sellar, suprasellar, prepontine, and perimedullary cisterns. Note that this material also diffusely coated the surface of the spinal cord, in keeping with extensive granulomatous meningitis (black arrows). Please note the multiple, small, rounded ringenhancing lesions in the frontal region and in the suprasellar cistern, in keeping with granulomas (white arrows). Fig. 45.4: Coronal enhanced T1WI at the level of the cavernous sinuses showed enhancing material in the sellar, suprasellar cisterns, and surrounding the proximal portions of the middle cerebral arteries, and supraclinoid internal carotids, reflecting perivascular inflammatory changes (arrows). Fig. 45.5: Axial DWI and Fig. 45.6: ADC map at the level of the lateral ventricles showed a focus of true restricted diffusion at the left caudate nucleus, in keeping with acute ischemic changes.

Discussion The findings of extensive exudative enhancing material in the basal cisterns and posterior fossa cisterns, which coats the cervical spinal cord ( in keeping with basilar meningitis), in association with ring-enhancing lesions in the subarachnoid space at those same levels (granulomas), perivascular inflammatory changes, and acute ischemic changes in a patient with upper lung lesions, which are positive acid-fast bacilli on ZiehlNeelsen stain, are highly suspicious for tuberculous meningitis (TBM) associated with vasculitis complicated with parenchymal infarcts. Human immunodeficiency virus infection is a worldwide epidemic disease with commonly reported neurologic complications. Despite its universal distribution, the presence of HIVrelated cerebrovascular disease (CVD) is an uncommon event. Forty to fifty percent of AIDS patients can have a neurologic complication, but less than 2% have a stroke syndrome, more

196

commonly ischemic infarcts than intracerebral hemorrhages. Cerebral infarcts are usually associated with non-bacterial thrombotic endocarditis or concomitant opportunistic CNS infection, and intracerebral hemorrhages are usually associated with thrombocytopenia, primary CNS lymphoma, and metastatic Kaposi sarcoma. Owing to the absence of HIV infection, and the fact that the typical imaging findings of HIV/AIDSrelated ischemic changes are not associated with basilar meningitis and intracranial granulomas, this entity is less likely in this patient. Syphilis is a sexually transmitted infectious and disabling disease caused by the spirochete Treponema pallidum, with increased incidence due to the HIV/AIDS epidemic. Neurologic complications of syphilis or neurosyphilis (NS) are well known and can be quite severe. Neurosyphilis occurs in about 10% of untreated syphilitics and typically manifests during the secondary or later stages of the disease. It can compromise all components of the nervous system, including the spinal cord, as well as cranial and peripheral nerves, meninges, and CNS blood vessels. Meningovascular syphilis (MVS) is a distinct form of NS characterized by a meningo-encephalopathic syndrome with superimposed cerebrovascular or myovascular events. Meningovascular syphilis occur in 10% of NS patients and in 3% of syphilis cases. Meningovascular syphilis involves an infectionassociated inflammatory arteriopathy resulting in injury to the blood vessels of the leptomeninges, brain, and spinal cord, leading to infarctions. Meningovascular syphilis-related cerebral infarcts are non-specific and dependent upon the arterial territory involved. Acute spinal cord infarcts resulting from MVS may clinically present in various forms of myelopathy, including hemiparaplegic syndrome (Brown-Séquard hemiplegia) or transverse myelitis. The absence of a history of HIV and syphilis, and the lack of association with granulomas and basilar meningitis, makes the possibility of MVS in this particular patient a less likely diagnosis. Tuberculosis (TB) is an infectious disease caused by Mycobacterium tuberculosis bacilli. Tuberculosis is still prevalent in many regions of the world and the incidence of TB has markedly increased in recent years, particularly in conjunction with AIDS and the emergence of drug-resistant strains of the bacillus. The lungs are the most commonly affected site, but many organs and tissues may be targeted. Tuberculous involvement of the CNS is an important and serious type of extrapulmonary involvement, occurring in around 10% of TB patients. Central nervous system TB can occur in 19% of the patients with coexisting TB and HIV. It is believed that the bacilli reach the CNS hematogenously, secondary to disease elsewhere in the body. Tuberculosis develops in the CNS in two stages. Initially small tuberculous lesions (Rich’s foci) develop in the CNS, either during the stage of bacteremia of the primary TB infection or shortly afterwards. These initial granulomatous inflammatory reactions may occur in the meninges, subpial, or subependymal surface of the brain or the bone covering of the brain or the spinal cord, and may remain dormant for years after initial infection.

Part III. Neuroinfectious Diseases: Case 45

Later, rupture and growth of these small tuberculous lesions produces development of various types of CNS TB including parenchymal and leptomeningeal tuberculomas, abscesses, cerebritis, vasculitis, infarction, meningitis, and osteomyelitis. The specific stimulus for rupture or growth of Rich’s foci is not known, although immunologic mechanisms are believed to play an important role. The most common forms of intracranial TB are: meningeal (leptomeningeal and pachymeningeal) and parenchymal (tuberculomas, tuberculous abscess, tuberculous cerebritis, and tuberculous encephalopathy). Complications of TBM are tuberculous vasculitis, hydrocephalus, and cranial nerve involvement. Intracranial vasculitis is a common finding in patients dying from TBM and a major factor contributing towards residual neurologic deficits. In patients with infarcts, TBM is reportedly up to three times more fatal than in those patients without infarcts. In survivors, the extent of ischemic cerebral damage is an important determinant of disability. The pathophysiology remains unclear, but it is believed that the basal exudate of TBM causes inflammatory changes in the vessels that predominantly involve the circle of Willis. At first, the vessel wall is involved, and later the lumen, leading to complete occlusion by reactive subendothelial cellular proliferation constituting panarteritis tuberculosa. Middle cerebral and lenticulostriate arteries are the most common vessels involved. Stroke is a common complication of tuberculous vasculitis in the context of TBM. Most infarcts in TBM are because of hemodynamic hypoperfusion due to a variable combination of vasospasm, intimal proliferation, and thrombosis of cerebral blood vessel walls. It is regarded as a poor prognostic predictor of TBM. The majority of the strokes in TBM are usually small, multiple, bilateral, and located in the basal ganglia, especially in the tubercular zone, which comprises the caudate, anterior thalamus, anterior limb, and genu of the internal capsule, corresponding to the deep sylvian region. Cortical stroke can also occur because of the involvement of the proximal portion of the middle, anterior, and posterior cerebral arteries, as well as the supraclinoid portion of the internal carotid and basilar arteries. The conventional angiographic features include narrowing of arteries at the base of the brain, and narrowed or occluded small or medium-sized arteries with early draining veins. Leptomeningeal vessels over the convexities of the brain may be stretched because of internal hydrocephalus or brain

swelling. Computed tomography or MR angiogram reveal small segmental narrowing, uniform narrowing of large segments, irregular beaded appearance of vessels, or complete occlusion. Although the cerebral vasculature can be visualized by MRA, direct angiography remains the gold standard for imaging the vascular lumen. Magnetic resonance imaging is more sensitive and detects a greater number of infarcts and hemorrhagic transformation of infarcts than CT does. On MR imaging, ischemic lesions are typically seen as hyperintensities on T2-weighted images. Diffusion-weighted imaging helps in early detection of ischemic lesions and in delineating the extent of infarction, which is of value in the management and outcome of patients. Tuberculous vasculitis is an important cause of morbidity and mortality in patients with TBM. Angiographic and MR imaging plays a crucial role in diagnosis because of its inherent sensitivity and specificity in detecting vessels narrowing and CNS strokes. The knowledge of this complication may help in better management of patients with CNS TB.

Key Points  Basilar meningitis associated with acute ischemic events is quite typical for vascular complications of CNS TB.  Most of the strokes in TBM are usually small, multiple, bilateral and located in the basal ganglia especially in the tubercular zone, which comprises the caudate, anterior thalamus, anterior limb, and genu of the internal capsule, corresponding to the deep sylvian region.  Tuberculous cerebral vasculitis should be included in the differential diagnosis of any neurologic deterioration arising during the course of TBM.

Suggested Reading Helbok R, Broessner G, Pfausler B, Schmutzhard E. Chronic meningitis. J Neurol 2009; 256(2): 168–75. Javaud N, Certal RDS, Stirnemann J, et al. Tuberculous cerebral vasculitis: retrospective study of 10 cases. Eur J Intern Med 2011; 22(6): e99–104. Meenakshi-Sundaram S, Srinivasan S, Karthik SN, et al. Malignant middle-cerebral artery territory infarction in tuberculous vasculitis. Neuroimmunol Neuroinflammation 2014; 1: 95–7. Pinto AN. AIDS and cerebrovascular disease. Stroke 1996; 27(3): 538–43. Rock RB, Olin M, Baker CA, Molitor TW, Peterson PK. Central nervous system tuberculosis: pathogenesis and clinical aspects. Clin Microbiol Rev 2008; 21(2): 243–61, table of contents.

197

CASE

Part III

46

Neuroinfectious Diseases Fabríco Guimarães Gonçalves, Lázaro Luís Faria do Amaral

Clinical Presentation A 65-year-old woman presented to the emergency department, in Asa Sul, Brazil, with a history of new-onset seizures. She reported a 5-day history of facial palsy and a 20-day history of headache and decreased level of consciousness. She reported undergoing heart transplantation eight months prior to onset of neurologic symptoms. A lumbar puncture was performed and revealed a normal opening pressure; however, CSF analysis demonstrated a white blood cell count of < 100/mm3, with lymphocyte predominance, and low glucose level. During her emergency department admission, her neurologic examination was unremarkable. A CT scan was performed and findings were unremarkable. Three days after admission, magnetic resonance of the brain was performed (see below). A brain biopsy was obtained to confirm suspected diagnosis.

Imaging Fig. 46.1 Axial enhanced T1WI centered in the internal auditory canal.

Advanced Neuroradiology Cases, ed. Amaral et al. Published by Cambridge University Press. © Cambridge University Press 2016.

199

Part III. Neuroinfectious Diseases: Case 46

Fig. 46.2 Axial T2-FLAIR image at the level of the convexity.

Fig. 46.3 Axial enhanced T1WI centered on both frontal and parietal convexities.

200

Part III. Neuroinfectious Diseases: Case 46 Fig. 46.4 Axial DWI.

Fig. 46.5 Axial perfusion-weighted image.

201

Part III. Neuroinfectious Diseases: Case 46

Chagas Disease Primary Diagnosis Chagas disease

Differential Diagnoses Toxoplasmosis Pyogenic abscess Tuberculosis Neurosarcoidosis Brain metastasis

Imaging Findings Fig. 46.1: Axial enhanced T1WI of the posterior fossa (centered in the internal auditory canal) demonstrated abnormal contrast enhancement in the fundus of the internal auditory canal, which extends to the geniculate ganglion (circle) with no significant changes in the adjacent temporal bone noted. Fig. 46.2: Axial T2-FLAIR image at the level of the convexity showed bilateral parietal and right frontal subcortical hyperintensities with mild mass effect in keeping with vasogenic edema (arrows). Note the three ill-defined, hypointense nodules in the subcortical white matter, one on the right side and two on the left side. Fig. 46.3: Axial enhanced T1WI centered on both frontal and parietal convexities, showed enhancement in the previously shown hypointense nodules (Fig. 46.2). On the right side, the nodules showed solid enhancement (arrows) and on the left the lesions showed ring enhancement (circle). Fig. 46.4: Axial DWI showed faint, hyperintense foci (arrows), which match the enhancing foci observed (Fig. 46.3). Fig. 46.5: Axial perfusion-weighted image showed that the enhancing lesions are cold and do not demonstrate increased perfusion.

Discussion The findings of bilateral subcortical enhancing nodules with vasogenic edema accompanied by abnormal enhancement in the subarachnoid space (fundus of the internal auditory canal and of the facial nerve) are not specific for Chagas disease (CD). This association can be seen in various pathologies such as inflammatory processes, infectious diseases, and in cases of brain metastasis with meningeal carcinomatosis. However, two important details in this patient’s presentation are suggestive of CD. First, her origin is Brazil – an endemic region, and second, she has a previous history of heart transplantation. In this patient, CD was presumably reactivated. Reactivation usually occurs secondary to immunosuppressive treatment, particularly after heart or liver transplantation. Reactivation of CD also occurs secondary to immunodeficiency diseases, especially AIDS. Although immunosuppressive therapy prevents rejection and graft-versus-host disease, it also increases the risk of CNS infection, which occurs in 5–10% of patients (brain abscess, encephalitis, or meningitis). Aspergillus, listeria, and cryptococcus are the most common causes of post-transplantation infection. Bacteria and

202

opportunistic pathogens are more common in the first month after transplantation. Infections due to herpes viruses (cytomegalovirus or Epstein-Barr virus), fungi, atypical bacteria, or parasitic infestation develop most often five months posttransplant, with Toxoplasma gondii being the most common infectious organism. Six months after immunosuppression therapy is reduced, CNS infection becomes less common. The association of enhancing lesions in the brain parenchyma and in the subarachnoid space may be found in infectious diseases such as in viral meningoencephalitis, pyogenic meningitis with cerebritis (with or without abscess formation), toxoplasmosis, and other opportunistic infections, tuberculosis, and fungal diseases. This pattern can be seen in some inflammatory diseases such as in neurosarcoidosis. Lymphoproliferative disorders and intracranial metastasis with meningeal carcinomatosis and brain parenchymal infiltration can also display this pattern. In cases of tuberculous meningitis, the involvement of the subarachnoid space tends to be more important and evident in the basal cisterns (basilar meningitis). Intra-axial tuberculomas can occur isolated or associated with tuberculous meningitis and usually show ring of enhancement. Toxoplasmosis meningitis can manifest meningeal enhancement and typical parenchymal toxoplasmosis lesions show ring of enhancement with eccentric enhancing nodules (eccentric target sign). In cases of neurosarcoid, the extra-axial lesions tend to involve the subarachnoid spaces and the adjacent dura. The intra-axial lesions of neurosarcoidosis can have variable location such as the brain parenchyma itself, pituitary stalk, cranial nerves, and periventricular white matter typically with contrast enhancement. These lesions can be punctate, nodular, and mass-like. Chagas disease is caused by Trypanosoma cruzi, a flagellated protozoan that affects an estimated 8–10 million people worldwide. Chagas disease is endemic in most South and Central American countries, affecting mostly people in low socioeconomic or rural regions. With the increase in urbanization and emigration, CD can be found in the United States and other parts of the world. Classically the disease is transmitted by the bite of triatomine bugs. Other routes of transmission include transfusion of contaminated blood with T. cruzi, oral transmission (ingestion of fluids or food contaminated with triatomine feces containing T. cruzi), and vertical or congenital transmission (crossing the placenta). Other less common modes of transmission include transplantation of diseased organs, ingestion of contaminated human milk, and/or cross-contamination via laboratory accidents. The disease has three distinct phases: acute, indeterminate, and chronic. In the acute phase (four to eight weeks) patients are usually asymptomatic or oligosymptomatic, showing mild symptoms such as fever, weakness, lymph node enlargement, hepato- and/or splenomegaly, and pruritus at the bite site. The indeterminate phase can last for many years, with no clinical manifestation of the disease (latent period). In the chronic phase, 20–40% of the patients (in the indeterminate phase)

Part III. Neuroinfectious Diseases: Case 46

may show signs of cardiomyopathy and digestive disease (megaesophagus and/or megacolon), resulting in swallowing impairment and severe constipation. In immunocompetent patients, meningoencephalitis is the most common manifestation of CD (in the chronic phase), especially in children. Central nervous system mass lesions are not typical in immunocompetent hosts. In immunosuppressed patients, CD may manifest as meningoencephalitis or a brain mass. The most common imaging finding in patients with CNS CD is ring-enhancing lesions, which can be solitary, multiple and small, or mass-like. Neurologic symptoms are variable; depend on the number, size, and location of the lesions; and may include headache, fever, cognitive changes, seizures, tremor, or hemiparesis.

Key Points  Emigration and intercontinental trips are contributing factors in the increase of CD incidence in the United States and Europe.  Toxoplasmosis is the most common parasitic infection in patients with immunodeficiency or immunosuppression.

 Chagas disease may be included in the differential diagnosis of ring-enhancing lesions in post-transplant patients, particularly if the patient is from an endemic area, and if there is no response following toxoplasmosis treatment.

Suggested Reading Burrill J, Williams CJ, Bain G, et al. Tuberculosis: a radiologic review. Radiographics 2007; 27(5): 1255–73. Lury KM, Castillo M. Chagas’ disease involving the brain and spinal cord: MRI findings. AJR Am J Roentgenol 2005; 185(2): 550–2. Pereira PC, Navarro EC. Challenges and perspectives of Chagas disease: a review. J Venom Anim Toxins Incl Trop Dis 2013; 19(1): 34. Shah R, Roberson GH, Cure JK. Correlation of MR imaging findings and clinical manifestations in neurosarcoidosis. AJNR Am J Neuroradiol 2009; 30(5): 953–61. Walker M, Zunt JR. Parasitic central nervous system infections in immunocompromised hosts. Clin Infect Dis 2005; 40(7): 1005–15.

203

CASE

Part III

47

Neuroinfectious Diseases Prasad B. Hanagandi, Sunila Jaggi, Lázaro Luís Faria do Amaral

Clinical Presentation(s) Patient Case A. A 30-year-old woman presented with a twoweek history of intermittent high-grade fever and altered level of consciousness. Patient resides in mosquito-infested endemic zone. Patient Case B (companion case). A 36-year-old man presented with intermittent, episodic history of high-grade fever that progressed to a comatose state. Recent history of travel to mosquito-borne endemic zone. In both cases, the fever was associated with chill, rigors, and vomiting. There was no history of diarrhea or upper respiratory tract illness. Clinical examination revealed tachycardia and splenomegaly. Hematologic studies were positive for Plasmodium falciparum. Analysis of CSF for meningitis and viral encephalitis was negative in both patients.

Imaging Fig. 47.1 Axial T1WI at the level of thalamus.

Advanced Neuroradiology Cases, ed. Amaral et al. Published by Cambridge University Press. © Cambridge University Press 2016.

205

Part III. Neuroinfectious Diseases: Case 47

Fig. 47.3 Coronal gradient echo image at the level of thalamus.

Fig. 47.2 Axial FLAIR image at the level of thalamus.

Fig. 47.4 Axial diffusion image at the level of centrum semiovale.

206

Part III. Neuroinfectious Diseases: Case 47

(A)

(B)

Fig. 47.5 (A–B) Axial susceptibility-weighted images at the level of centrum semiovale and corona radiata.

207

Part III. Neuroinfectious Diseases: Case 47

Cerebral Malaria Primary Diagnosis Cerebral malaria

Differential Diagnoses Hemorrhagic viral encephalitis Hemorrhagic acute disseminated encephalomyelitis (ADEM) Toxicities, metabolic encephalopathies

Imaging Findings Patient Case A: MRI shows hemorrhagic bithalamic lesions with hyperintense signal on Fig. 47.1: T1WI, Fig. 47.2: FLAIR, and Fig. 47.3: Blooming on coronal gradient echo image. Patient Case B: Fig. 47.4: Heterogeneous foci with diffusion restriction are noted in the deep white matter and subcortical regions (arrows). Fig. 47.5: (A–B) SWI images show multiple linear foci with significant blooming in the subcortical and central deep white matter (arrows), which correspond to capillary sludging and hemorrhage caused by parasitemia.

Discussion The imaging findings of bithalamic lesions have a wide list of differential diagnoses comprising infective, metabolic, and neoplastic pathologies. The clinical history and high index of suspicion in an endemic zone with pertinent hematologic investigations helps in arriving at the diagnosis of cerebral malaria. Hemorrhagic encephalitis, especially Japanese B encephalitis, can have similar imaging features and is one of the major differential diagnoses to be excluded in a patient residing in an endemic zone. Negative hematologic and CSF analyses play a key role in the differential diagnosis. Hemorrhagic ADEM can also have the same imaging manifestations; however, the history of endemic zone as in Case A and travel to an endemic zone and clinical presentation with splenomegaly help in exclusion. Malaria is a widespread, endemic disease in most South Asian, African, and South American countries. Cerebral malaria is caused by Plasmodium falciparum and transmitted by the female Anopheles mosquito. Patients typically present with high-grade fever, altered level of consciousness, seizures, and generalized constitutional symptoms that fluctuate with the blood parasitemia. The neurologic symptoms are often nonspecific because of diffuse brain involvement. The imaging manifestations develop because of cytokine-induced damage and mechanical capillary blockade instigated by parasiteinfested erythrocytes. Cytokine release leads to increased cerebral volume due to vasodilatation, decreased cerebral perfusion, and hypoglycemia. Imaging findings correlate well with histopathology, which demonstrates sequestration of infected

208

erythrocytes in the smaller perforating blood vessels and capillaries with white matter necrosis. A wide spectrum of neuroimaging findings have been described on T2, FLAIR, DWI, GRE, and SWI sequences involving the cortical and subcortical white matter, corpus callosum, basal ganglia, thalamus, and the cerebellum due to vasogenic and cytotoxic edema, haemorrhage, and infarcts. The imaging spectrum has been broadly classified as normal brain appearance on CT and MRI, diffuse cerebral edema, bilateral thalamic changes with diffuse cerebral edema, and/or bilateral thalamic and cerebellar involvement with diffuse cerebral edema. The sludging of capillaries by infested erythrocytes can be appreciated on GRE and SWI that are highly sensitive for hemorrhage and sluggish blood flow. Imaging features similar to central pontine myelinolysis and cerebellar syndrome with demyelination have been reported. A high index of clinical suspicion is required in endemic zones as the imaging findings can be similar to viral encephalitis or hemorrhagic ADEM. Although cerebral malaria constitutes only 2% of malarial cases; it can be life-threatening, especially in the pediatric age group. Mortality ranging from 20% to 50% has been reported – even with effective antimalarial therapy.

Key Points  Imaging findings of cerebral malaria are non-specific and can mimic atypical hemorrhagic, demyelinating conditions, and viral encephalitides.  Central nervous system imaging evaluation is often negative; however, hemorrhagic lesions predominantly caused by blood parasitemia have been described in the imaging literature.  High index of clinical suspicion is important along with familiarity regarding common diseases in a given geographic location and any history of recent travel to endemic zones.

Suggested Reading Abdel Razek AA, Watcharakorn A, Castillo M. Parasitic diseases of the central nervous system. Neuroimaging tropical diseases. Neuroimag Clin N Am 2011; 21: 815–41. Gupta S, Patel K. Case series: MRI features in cerebral malaria. Indian J Radiol Imaging 2008; 18(3): 224–6. Patankar TF, Karnad DR, Shetty PG, Desai AP, Prasad SR. Adult cerebral malaria: prognostic importance of imaging findings and correlation with postmortem findings. Radiology 2002; 224(3): 811–16. Potchen MJ, Kampondeni SD, Seydel KB, et al. Acute brain MRI findings in 120 Malawian children with cerebral malaria: new insights into an ancient disease. Am J Neuroradiol 2012; 33(9): 1740–6. Rasalkar DD, Paunipagar BK, Sanghvi D, Sonawane BD, Loniker P. Magnetic resonance imaging in cerebral malaria: a report of four cases. Br J Radiol 2011; 84(1000): 380–5.

CASE

Part III

48

Neuroinfectious Diseases Sonali H. Shah, Prasad B. Hanagandi, Bernardo Jose Alves Ferreira Martins, Lázaro Luís Faria do Amaral

Clinical Presentation A 47-year-old woman presented with gradually progressive lower limb weakness, followed by chronic spastic paraparesis. It was associated with mild cognitive decline and urinary bladder dysfunction. Based on the clinical presentation, the suspected diagnosis included a neurodegenerative disorder such as amyotrophic lateral sclerosis, hereditary spastic paraparesis, and subclinical inflammatory/infective process. Cerebrospinal fluid examination revealed mild to moderate increase in the proteins, lymphocyte pleocytosis with presence of HTLV-1 antibodies in the CSF and blood. HIV and VDRL investigations were negative. Enzymatic assessment for galactocerebrosidase in leukocytes and serum very long chain fatty acid (VLCFA) levels were normal. Rest of the hematologic evaluations was unremarkable. Toxicology analysis for substance abuse was negative.

Imaging (A)

(B) (C)

Fig. 48.1 (A) Axial T2WI, (B) Axial FLAIR, and (C) Axial postgadolinium T1WI through the level of corona radiata/centrum semiovale.

Advanced Neuroradiology Cases, ed. Amaral et al. Published by Cambridge University Press. © Cambridge University Press 2016.

209

Part III. Neuroinfectious Diseases: Case 48 Fig. 48.2 Axial DWI through the level of corona radiata/centrum semiovale.

(A)

(B)

Fig. 48.3 (A–B) Axial T2W images through the level of pons and middle cerebellar peduncles.

210

Part III. Neuroinfectious Diseases: Case 48 Fig. 48.4 Axial T2W image through the level of medulla oblongata.

211

Part III. Neuroinfectious Diseases: Case 48

HTLV-1-Associated Myelopathy/Tropical Spastic Paraparesis Primary Diagnosis HTLV-1-associated myelopathy/tropical spastic paraparesis

Differential Diagnoses Amyotrophic lateral sclerosis (ALS) Adult-onset Krabbe disease Adult-onset adrenoleukodystrophy Toluene abuse

Imaging Findings Fig. 48.1: (A) Axial T2WI and (B) FLAIR MR images (arrows) revealed nearly symmetric hyperintense signal in the central deep white matter. (C) Axial postgadolinium T1WI did not show enhancement. Fig. 48.2: Axial DWI was unremarkable. Fig. 48.3: (A–B) Axial T2WI demonstrated hyperintense signal abnormality involving the corticospinal tracts and transverse pontine fibers (arrows) and central tegmental tracts (arrowheads). Fig. 48.4: Axial T2WI of the brainstem at the level of medulla showed nearly symmetric hyperintense corticospinal signal abnormality involving the pyramids (arrows).

Discussion In a patient who is seropositive for HTLV-1 antibodies, who presents with slow progression of spastic paraparesis, cognitive decline, and coexistent bladder dysfunction, a diagnosis of HTLV-1-related tropical spastic paraparesis was considered. Amyotrophic lateral sclerosis can have similar imaging features but bladder dysfunction is an uncommon association. Similar corticospinal tract involvement and clinical presentation can also be seen in adult-onset adrenoleukodystrophy, adult-onset Krabbe disease, and toluene abuse. However, normal hematologic studies for VLCFAs, galactocerebrosidase enzymatic activity, and negative toxicology analysis helped in excluding these conditions. HTLV-1 is a retrovirus, causing immune-mediated/direct invasion of the nervous system, which is endemic in welldefined geographic regions such as Japan, the Caribbean islands, South America, the Middle East, and Africa. Approximately 20 million people are infected worldwide and most are asymptomatic carriers. The classical neurologic presentation is called HTLV-1-associated myelopathy/tropical spastic paraparesis (HAM/TSP). However, HTLV-1 presents with various isolated or assorted syndromes such as myopathy, motor neuron disease, and cognitive decline, ALS-like syndrome, dysautonomia, and polyneuropathy; hence, it should be called HTLV-1-associated neurologic complex.

212

HTLV-1 neurologic complex has been described as an iceberg model with HAM/TSP forming its tip, which develops in 2–4% of the infected population. Although the exact mechanism of CNS disease in HTLV-1associated neurologic complex is unknown, it has been postulated to stem from direct viral invasion and/or autoimmune mechanisms that result in brain and spinal cord infiltration by lymphocytes and monocytes involving the gray and white matter. Patients present with gait and urinary disturbances, constipation, hyperreflexia, and impaired posterior column conduction abnormalities on neurologic examination. The diagnosis is confirmed by detecting anti-HTLV-1 antibodies in CSF and serum. There are no specific MR imaging features of HTLV infection described in the literature. Multifocal discrete and confluent hyperintense lesions can be found in the subcortical, periventricular, and central deep white matter on T2-weighted and FLAIR images. These imaging findings at times can be confused with demyelinating lesions of multiple sclerosis. Dirty white matter changes and symmetric lesions involving the corticospinal tracts have been described that could mimic ALS on FLAIR and T2W images. Long segment lesions have been described in the cervico-thoracic spinal cord that especially involve the dorsal columns, possibly explaining the posterior column conduction disturbances in these patients. Spinal cord atrophy has been reported in up to 20–70% of cases.

Key Points  HTLV-1-related neurologic complex is an iceberg model with HAM/TSP as a common presentation.  It manifests with ALS-like syndrome, myopathy, and polyneuropathy.  Tropical spastic paraparesis should be suspected in a person from an endemic region, presenting with slow progressive paraparesis, mild cognitive decline, and early bladder dysfunction.  Magnetic resonance imaging features are often non-specific and diagnosis is confirmed by detecting HTLV-1 antibodies or antigens in CSF and serum of suspected cases.

Suggested Reading Araujo AQ, Silva MT. The HTLV-1 neurological complex. Lancet Neurol 2006; 5(12): 1068–76. Bagnato F, Butman JA, Mora CA, et al. Conventional magnetic resonance imaging features in patients with tropical spastic paraparesis. J Neurovirol 2005; 11(6): 525–34. Silva MT, Leite AC, Alamy AH, et al. ALS syndrome in HTLV-I infection. Neurology 2005; 65(8): 1332–3. Yata S, Ogawa T, Sugihara S, et al. HTLV-I carrier with unusual brain MR imaging findings. Neuroradiology 2004; 46(9): 755–8.

CASE

Part III

49

Neuroinfectious Diseases Ricardo Tavares Daher, Lázaro Luís Faria do Amaral

Clinical Presentation A 34-year-old woman presented with an acute right-sided hemiplegia, right-sided facial palsy, and mild expressive aphasia. Her past medical history was marked by prior episodes of seizures, associated with headache and vomiting. Control MRI studies were performed.

Imaging (A)

(B)

(C)

Fig. 49.1 (A–B) Axial T2WI through the level of the basal cisterns. (C) Axial T2WI through the level of the third ventricle.

(A)

(B)

(C)

Fig. 49.2 (A–B) Axial T1W postgadolinium-enhanced images through the level of the basal cisterns. (C) Axial T1W postgadolinium-enhanced image through the level of the third ventricles.

Advanced Neuroradiology Cases, ed. Amaral et al. Published by Cambridge University Press. © Cambridge University Press 2016.

213

Part III. Neuroinfectious Diseases: Case 49

Racemose Neurocysticercosis Complicated with Infarction Primary Diagnosis Racemose neurocysticercosis complicated with infarction

Differential Diagnoses Tuberculous meningitis complicated by stroke Neurosarcoidosis Leptomeningeal carcinomatosis Pyogenic meningitis

Imaging Findings Fig. 49.1: (A–C) Axial T2WI showed areas of hyperintense signal. The largest hyperintensities were noted in the left temporal and occipital lobes (arrowheads), in the territory of the left middle cerebral artery. In addition, extra-axial cysts were noted, predominantly in the basal cisterns. Fig. 49.2: (A–C) Axial enhanced T1WI showed leptomeningeal enhancement in the basal cisterns and sylvian fissure, around the left middle cerebral artery (arrows), and areas of hypointense signal associated with cortical enhancement in the left temporal and occipital lobes (arrowheads).

Discussion Neurocysticercosis, the most common parasitic infection of the CNS, is caused by Taenia solium larva (port tapeworm) and can infest any CNS site. It affects approximately 50 million people around the world and is endemic in Central and South America, East Europe, Africa, and some regions in Asia. Neurocysticercosis can be classified according to cyst location and into active and non-active forms. However, in massive cerebral infections, the two forms may coexist. The presence of a fully grown cyst or a cluster of grape-like (racemose) cysts in the subarachnoid-cisternal space characterizes the racemose form. Racemose commonly manifests as arachnoiditis, hydrocephalus, or intracranial hypertension secondary to flow obstruction of the ventricular system. Tuberculous meningitis, neurosarcoidosis, leptomeningeal carcinomatosis, and pyogenic meningitis can cause leptomeningeal thickening and enhancement and can demonstrate complications of arachnoiditis, as seen in our patient case. However, these etiologies do not characteristically demonstrate subarachnoid-cisternal cystic lesions, the unique findings characteristic of the racemose form of neurocysticercosis. Magnetic resonance imaging of patients with the racemose form of neurocysticercosis usually shows cystic lesions in the sylvian fissure and the basal cisterns. In contrast to parenchymal lesions, sylvian fissure and basal cistern cysts reach more than 1.0 cm in size. Two imaging modalities are extremely useful in diagnosing racemose and determining extent of disease: three-dimensional reconstructive interference in steady state (3D-CISS) and diffusion-weighted (DW MRI) sequences. The first is the most sensitive means of detecting racemose

214

lesions, while DW MRI images can be very specific, demonstrating restriction of the scolex in the lesion. Other advanced imaging methods such as perfusion and spectroscopy are nonspecific, but help to differentiate racemose from other groups of diseases, mainly tumors. Non-invasive MR cisternography with FLAIR and inhalation of 100% oxygen may also improve the detection of cisternal lesions because of the resultant hyperintensity of the CSF. Cerebrovascular complications of neurocysticercosis include cerebral infarction, transient ischemic attacks, and brain hemorrhage. In our case, the leptomeningeal thickening and enhancement correlated pathologically with meningitis associated with an inflammatory response in the arachnoids that subsequently led to vasculitis. This process usually affects the basal perforating vessels and may determine lacunar infarcts, but in the case presented, the left middle cerebral artery was affected. Pathologic studies of this process usually show the presence of endarteritis in the vessels of tissues surrounding the parasite. The frequency of cerebral infarction related to cysticercotic arteritis varies in the literature, between 2% and 12%. However, in 1998, Barinagarrementeria and Cantú reported a case series of 28 patients with subarachnoid cysticercosis in which 53% had angiographically documented cerebral arteritis, although most patients were symptomatic (80%).

Key Points  Neurocysticercosis is the most common parasitic infection of the CNS.  Cystic lesions in the subarachnoid-cisternal space characterize the racemose form.  Rarely, the racemose form of neurocysticercosis can complicate with arachnoiditis leading to vasculitis and ultimately causing strokes.

Suggested Reading Amaral LLF, Ferreira RM, Rocha AJ, Ferreira NPDF. Neurocysticercosis - evaluation with advanced magnetic resonance techniques and atypical forms. Top Magn Reson Imaging 2005; 16: 127–44. Barinagarrementeria F, Cantú C. Frequency of cerebral arteritis in subarachnoid cysticercosis. An angiographic study. Stroke 1998; 29: 123–5. Brutto OHD. Cysticercosis and cerebrovascular disease: a review. J Neurol Neurosurg Psychiatry 1992; 55: 252–4. Kimura-Hayama ET, Higuera JA, Cedillo RC, et al. Neurocysticercosis: radiologic-pathologic correlation. Radiographics 2010; 30: 1705–19. Levy AS, Lillehei KO, Rubisttein D, et al. Subarachnoid neurocysticercosis with occlusion of the major intracranial arteries: case report. Neurosurgery 1995; 36: 183–8. Rocha MSG, Brucki SMD, Ferraz AC, Piccolo AC. Cerebrovascular disease and neurocysticercosis. Arq Neuropsiquiatr 2001; 59(3-B): 778–83. Ter Penning B, Litchman CD, Heier L. Bilateral middle cerebral artery occlusions in neurocysticercosis. Stroke 1992; 23: 280–3.

CASE

Part III

50

Neuroinfectious Diseases Prasad B. Hanagandi, Sunila Jaggi

Clinical Presentation A 30-year-old man with a known case of neurocysticercosis presented with acute-onset high-grade fever, seizures, and altered mental status. The patient originates from a known mosquito-infested endemic zone. Cerebrospinal fluid analysis revealed a normal opening pressure and lymphocytic pleocytosis with positive IgM antibodies against Japanese B encephalitis.

Imaging Fig. 50.1 Axial T2WI at the level of the basal ganglia and thalami.

Fig. 50.2 Axial T2WI at the level of the cerebral peduncles.

Fig. 50.3 Axial T2WI at the level of the anterior frontal convexities.

Fig. 50.4 Axial T1W postgadolinium image at the level of anterior frontal convexities.

Advanced Neuroradiology Cases, ed. Amaral et al. Published by Cambridge University Press. © Cambridge University Press 2016.

215

Part III. Neuroinfectious Diseases: Case 50

Neurocysticercosis and Japanese B Encephalitis Primary Diagnosis Neurocysticercosis and Japanese B encephalitis

Differential Diagnoses Cerebral malaria Toxic-metabolic etiologies involving the basal ganglia and thalami Paraneoplastic syndromes Other viral encephalitides such as Epstein-Barr virus (EBV), herpes, and West Nile

Imaging Findings Fig. 50.1: Shows nearly symmetric T2WI hyperintense signal abnormality in the bilateral basal ganglia, thalami, and substantia nigra. Fig. 50.2: Axial T2WI at the level of the cerebral peduncles and Fig. 50.3: Axial T2WI at the level of the anterior frontal convexities demonstrated a small T2 hyperintense lesion in the right frontal subcortical white matter with ring enhancement. Fig. 50.4: Axial T1W postgadolinium image demonstrates that the ring enhancement is a coexisting neurocysticercosis granuloma.

Discussion The imaging findings overlap with several toxic-metabolic leukoencephalopathies, paraneoplastic syndromes, and a wide spectrum of viral encephalitides. High index of clinical suspicion with blood and CSF investigation is the key to diagnosing symptomatic patients residing in mosquito-infested endemic zones. Japanese encephalitis (JE) is an endemic arboviral infection in most Asian countries with high incidence of outbreaks and prevalence, especially in India and China, with seasonal variations. It is primarily a zoonotic disease. Pigs are the most important reservoirs and amplifiers. Human infections occur by culicine mosquito bites and are incidental or dead-end host. The disease is commonly seen in pediatric and infant populations. A high index of suspicion is necessary as most cases present with febrile seizures and altered level of consciousness. Serum detection, antibody detection, and viral isolation from

216

CSF can confirm diagnosis. High association of coexisting NCC (neurocysticercosis) and JE is known in endemic zones and is not an incidental finding. Neurocysticercosis acts as an immunomodulator, potentiates the viral virulence, and facilitates the spread of infection by altering the immune response and blood-brain barrier. The disease burden of JE tends to lateralize to the side of the brain with NCC lesions, possibly explaining the asymmetric pattern in many cases. The prevalence rates for NCC in patients with JE are variable according to neuroimaging literature, ranging from as low as 4% to as high as approximately 19%. Mortality rates are as high as 25% with residual neurologic deficits in survivors. The virus typically involves the basal ganglia, thalamus, midbrain, hippocampus, and cerebral cortex, with unilateral or bilateral and asymmetric distribution. The lesions are hypointense on T1WI with no contrast enhancement and hyperintense on FLAIR and T2WI. Variable DWI findings have been described in the literature.

Key Points  Cultural and geographic competence, noting an awareness of common diseases and their endemic locations, is important when dealing with imaging findings associated with acute clinical presentation.  Imaging findings often fall into a broad list of differential diagnoses and laboratory evaluation is necessary.  The association of NCC in cases of JE is not incidental but synergistic.

Suggested Reading Handique SK. Viral infections of the central nervous system. Neuroimaging Clin N Am 2011; 21(4): 777–94. Handique SK, Das RR, Saharia B, et al. Coinfection of Japanese encephalitis with neurocysticercosis: an imaging study. Am J Neuroradiol 2008; 29(1): 170–5. Kennedy PGE. Viral encephalitis: causes, differential diagnosis and management J Neurol Neurosurg Psychiatry 2004; 75(Suppl 1): i10–15. Singh P, Kalra N, Ratho RK, et al. Coexistent neurocysticercosis and Japanese B encephalitis: MR imaging correlation. Am J Neuroradiol 2001; 22(6): 1131–6.

CASE

Part IV

51

Neuroinflammatory Diseases Glenn H. Roberson, Asim K. Bag

Clinical Presentation A previously healthy 45-year-old man presented to a neurology clinic with a two-week history of headache, transient right-sided motor and sensory abnormalities. He also reported two previous episodes of convulsion that were successfully treated with antiseizure medication by his primary care physician. He denied fever; no signs of meningism noted. On admission, he underwent MRI (Fig. 51.1). Cranial CT angiogram and CSF analysis were negative. Hematologic studies did not reveal any metabolic abnormality or evidence of autoimmune disease. The CT scan of chest, abdomen, and

pelvis was negative. The patient was treated non-specifically with steroids and all symptoms improved. Three weeks later, he presented to the emergency department with status epilepticus, decreased consciousness, dense right-sided hemiplegia, and hemisensory loss. Repeat imaging studies were performed (Fig. 51.2 and Fig. 51.3). Cerebrospinal fluid analysis revealed mild pleocytosis and increased CSF protein levels without evidence of ACE, CSF antibody, or monoclonal proliferation. Findings from repeat hematologic tests for autoantibodies were normal. Repeat CT scan of chest, abdomen, and pelvis was negative.

Imaging (A)

(B)

Fig. 51.1 (A) FLAIR, (B) Postcontrast T1WI.

Advanced Neuroradiology Cases, ed. Amaral et al. Published by Cambridge University Press. © Cambridge University Press 2016.

217

Part IV. Neuroinflammatory Diseases: Case 51

(A)

(B)

(C)

Fig. 51.2 (A) FLAIR, (B) Postcontrast, (C) GRE images.

218

Part IV. Neuroinflammatory Diseases: Case 51 Fig. 51.3 Composite MIP image.

219

Part IV. Neuroinflammatory Diseases: Case 51

Primary Angiitis of CNS (PACNS) Primary Diagnosis Primary angiitis of CNS (PACNS)

Differential Diagnoses Leptomeningeal carcinomatosis Meningitis Neurosarcoidosis Reversible cerebral vasoconstriction syndrome (RCVS) Central nervous system manifestations of systemic vasculitis

Imaging Findings Fig. 51.1: (A) FLAIR image demonstrated abnormal signal in the left parietal subarachnoid space (arrows) with no obvious abnormality involving the cortex or subcortical white matter. (B) Postcontrast T1WI sequence demonstrated leptomeningeal enhancement on the left parietal region, as well as linear enhancement in the subcortical white matter. Fig. 51.2: (A) FLAIR image demonstrated prominent FLAIR abnormality involving cortical and subcortical white matter of the left posterior frontal and left parietal region. (B) Sagittal postcontrast image demonstrated prominent vascular/perivascular enhancement (arrowheads) in the left posterior frontal and parietal lobe. No significant leptomeningeal enhancement was noted. (C) GRE image demonstrated numerous microhemorrhages (arrows) in the subcortical white matter. A tiny focus of diffusion restriction was noted in the left superior frontal gyrus. Fig. 51.3: Composite MIP image of the 3D TOF MRI image demonstrated focal stenosis (arrows) involving multiple M2 branches of the left middle cerebral artery (MCA). There is paucity of the left M3 branches, as compared to the right side. Note that the right MCA and its branches are unremarkable.

Discussion Although PACNS lacks characteristic imaging findings or a typical clinical presentation, a constellation including male gender, middle aged (50 years of age), subacute onset of headache, and stroke-like symptoms correlates with the profile of the PACNS patient. Although initial imaging appearance is non-specific, imaging findings from the second MRI and MRA are strongly suggestive of vasculitis. The presence of large areas of cortical and subcortical white matter FLAIR abnormality associated with a tiny focus of diffusion restriction is suggestive of critical ischemia of a large tissue with focal area of acute infarct. Presence of microhemorrhages and multifocal vascular narrowing on MRA are also suggestive of vasculitis. Extensive vascular/perivascular enhancement in the involved area suggests vessel wall disease. In the absence of systemic vasculitides, the clinical presentation and imaging findings are highly suggestive of PACNS. Although leptomeningeal enhancement in a 50-year-old is highly suggestive of leptomeningeal carcinomatosis, it was

220

ruled out because of the lack of any known primary malignancy. Mild CSF pleocytosis and the absence of fever or meningeal signs ruled out the possibility of meningitis. Absence of chest sarcoidosis, increased serum or CSF ACE levels, and punctate enhancement are not suggestive of neurosarcoidosis. Relatively subacute, rather than acute, presentation and male gender are not typical of RCVS. Primary angiitis of CNS is a rare disease with annual incidence of 2–4 cases per million people per year. It is more common in males and the average age at presentation is 50 years. The diagnostic criteria of PACNS include: 1) newonset neurologic deficit; 2) angiographic or histologic evidence of vasculitis; and 3) absence of systemic condition that can explain the findings. As the treatment of PACNS includes cytotoxic therapy, proper patient selection is very important. Diagnosis of PACNS may be definite if it is proven by biopsy, or probable if clinical, laboratory, and imaging findings are highly suggestive of diagnosis. The disease has various forms: 1) non-progressive, large- and medium-vessel disease, 2) progressive, large- and medium-vessel disease; or 3) angiographically occult small vessel disease. All PACNS variants are age-dependent. Primary angiitis of CNS typically involves small to medium-sized arteries and veins, especially those located close to the meninges and subcortical white matter. T-cell and activated macrophage infiltration of the vessel wall, with granuloma formation, and adventitia involvement (usually associated with intimal proliferation), with luminal narrowing are characteristic microscopic findings. Gradually worsening headache and cognitive impairment are the most common initial symptoms. Focal neurologic deficits appear later in the disease process. Small vessel disease, frequently occult on angiography, more commonly presents with acute or subacute encephalopathy in conjunction with headache, confusion, and cognitive impairment. In contrast, larger vessel disease more commonly presents with infarctions and focal neurologic deficits frequently demonstrated as angiographic abnormalities. Magnetic resonance imaging is sensitive but not specific for this disease and is abnormal in 90–100% of patients. The most common imaging abnormality involves T2-FLAIR hyperintensity involving the subcortical white matter, followed by deep gray matter, deep white matter, and superficial cortex. Acute infarcts can only be seen in 50% of cases. Acute presentation may have mass effect and cortical or subcortical swelling that can be confused with a superficial tumor. Diffuse, unexplained small vessel ischemic disease involving deep cerebral and periventricular white matter may be seen. These lesions may be focal and can be confused with multiple sclerosis. Chronic superficial lesions can present as laminar necrosis. Enhancement with contrast can be seen in only one-third of all cases. No specific enhancement pattern is known. Leptomeningeal enhancement can be seen in 10–15% of cases. Subarachnoid hemorrhage is a not common finding, but can be seen as was noted in this patient.

Part IV. Neuroinflammatory Diseases: Case 51

Conventional angiogram is more specific but less sensitive for the diagnosis of PACNS than MRI. The reported sensitivity of angiographic abnormalities is 20–90% and specificity is 20–60%. Sensitivity increases if medium- or larger-sized blood vessels are involved. Typical angiographic appearances include beading of blood vessels, or segmental narrowing of the small or medium-sized blood vessels. Biopsy may be required in equivocal cases and should target the enhancing meninges, if present, for maximum yield of biopsy. There is no specific management recommendation for PACNS. Usually treatment is started with glucocorticoids, with or without cyclophosphamide, or other immunosuppressive therapy.

Key Points  Typical clinical scenario includes: 1) male patient, around age 50; 2) with subacute-onset cognitive decline; 3) gradually worsening headache; and 4) gradual development of different focal neurologic deficits.  Abnormal T2 and FLAIR signal involving cortical and subcortical white matter, leptomeningeal enhancement with or without multifocal infarction, and beading/ segmental stenosis of the arteries of the small to mediumsized blood vessels are typical findings.

Suggested Reading Alba MA, Espigol-Frigole G, Prieto-Gonzalez S, et al. Central nervous system vasculitis: still more questions than answers. Curr Neuropharmacol 2011; 9(3): 437–48. Birnbaum J, Hellmann DB. Primary angiitis of the central nervous system. Arch Neurol 2009; 66(6): 704–9. Calabrese LH, Mallek JA. Primary angiitis of the central nervous system. Report of 8 new cases, review of the literature, and proposal for diagnostic criteria. Medicine (Baltimore) 1988; 67(1): 20–39. Hajj-Ali RA. Primary angiitis of the central nervous system: differential diagnosis and treatment. Best Pract Res Clin Rheumatol 2010; 24(3): 413–26. Hajj-Ali RA, Furlan A, Abou-Chebel A, Calabrese LH. Benign angiopathy of the central nervous system: cohort of 16 patients with clinical course and long-term followup. Arthritis Rheum 2002; 47(6): 662–9. Moharir M, Shroff M, Benseler SM. Childhood central nervous system vasculitis. Neuroimaging Clin N Am 2013; 23(2): 293–308. Neel A, Pagnoux C. Primary angiitis of the central nervous system. Clin Exp Rheumatol 2009; 27(1 Suppl 52): S95–107. Salvarani C, Brown RD, Jr., Hunder GG. Adult primary central nervous system vasculitis. Lancet 2012; 380(9843): 767–77.

221

CASE

Part IV

52

Neuroinflammatory Diseases Ricardo Tavares Daher, Lázaro Luís Faria do Amaral

Clinical Presentation A 44-year-old woman presented to our facility with psychiatric symptoms and behavior changes. She had no noted history of fever, seizures, or pathologic incidents. Magnetic resonance imaging was performed to exclude cerebrovascular, metastatic, or other pathologies (Fig. 52.1) and follow-up, control images were made 10 days (Fig. 52.2), two months (Fig. 52.3), and five months (Fig. 52.4) following her baseline exam. Routine hematologic assays and analyses of N-methyl-D-aspartate receptor antibodies in sera and CSF were performed.

Imaging (A)

(B)

(C)

Fig. 52.1 (A) Coronal FLAIR, (B) Axial T1WI postcontrast, and (C) Coronal T1WI postcontrast through the temporal mesial region.

Advanced Neuroradiology Cases, ed. Amaral et al. Published by Cambridge University Press. © Cambridge University Press 2016.

223

Part IV. Neuroinflammatory Diseases: Case 52

(A)

(B)

(C)

Fig. 52.2 (A) Coronal FLAIR, (B) Axial T1WI MR postcontrast, and (C) Coronal T1WI postcontrast through the temporal lobes (10-day follow-up).

224

Part IV. Neuroinflammatory Diseases: Case 52

(A)

(B)

Fig. 52.3 (A) Coronal FLAIR and (B) Axial T1WI postcontrast through the temporal lobes (2-month follow-up).

(A)

(B)

Fig. 52.4 (A) Coronal FLAIR and (B) Axial T1WI postcontrast through the temporal lobes (5-month follow-up).

225

Part IV. Neuroinflammatory Diseases: Case 52

Anti-N-Methyl-D-Aspartate Receptor (NMDAR) Encephalitis Primary Diagnosis Anti-N-methyl-D-aspartate receptor (NMDAR) encephalitis

Differential Diagnoses Cerebrovascular event Metastatic disease Subacute or chronic inflammatory disorders

Imaging Findings Fig. 52.1: (A) Coronal FLAIR images through the level of the hippocampus demonstrated swelling and hyperintense signal of the left hippocampus associated with hyperintense signal of the right hippocampus. (B) Contrast-enhanced axial T1WI and (C) Coronal T1WI postcontrast showed abnormal enhancement of the left hippocampus. Fig. 52.2: (A) Coronal FLAIR, (B) Axial, and (C) Coronal T1 postgadolinium images (10-day follow-up) showed consistent left hippocampus findings, demonstrating swelling and hyperintense signal of the right hippocampus. Fig. 52.3: (A) Coronal FLAIR, and (B) Axial T1WI postgadolinium images (two-month follow-up) showed hippocampal atrophy associated with hyperintensity signal on the mesial aspect of the temporal lobes. Fig. 52.4: (A) Coronal FLAIR, and (B) Axial contrast-enhanced T1WI demonstrated progression, with increased atrophy of the hippocampus (five-month follow-up).

Discussion Anti-N-methyl-D-aspartate receptor (NMDAR) encephalitis usually presents with psychiatric symptoms, most frequently anxiety, agitation, and visual or auditory hallucinations. It is a newly identified paraneoplastic disorder targeting NR1 and NR2 subunits of NMDARs, causing severe neurologic and psychiatric symptoms. Neurologic features such as movement disorders, seizures, and cognitive problems are also characteristic findings. Literature findings indicate that most patients experience autonomic dysfunction and persistent cognitive deficits following disease onset, predominantly executive function, and memory impairments. Positive findings for antibodies to the NR1 subunit in sera and/or CSF confirmed the diagnosis. Magnetic resonance imaging of the brain will be normal in 45% of patients. The other 55% may show non-specific T2 or FLAIR signal hyperintensity, most frequently at the hippocampus, as seen in our patient. Faint or transient contrast enhancement of the cerebral cortex, overlaying meninges, or basal ganglia may be seen, although not a predominant finding. Follow-up brain MR imaging either remains normal or shows minimal change. Magnetic resonance spectroscopy has not yet been documented to be of any proven diagnostic value. In most patients, electroencephalograms are consistently abnormal; however, findings by Schmitt et al. indicate that

226

30% of affected individuals show an extreme delta brush pattern. Since this has not been described in other neurologic conditions, it may suggest that it is a unique finding in some patients with this disorder. Although NMDAR encephalitis has been reported to be more common in females with ovarian teratoma, it has become increasingly clear that the syndrome is not restricted to adult females but can also be seen in adult males and children of either sex, in the absence of an associated systemic neoplasm. In those patients with tumor, surgical resection may resolve symptoms. For non-surgical candidates, first-line immunotherapy consisting of corticosteroids plus intravenous immunoglobulin (IVIG) or plasma exchange has been proposed.

Key Points  Anti-N-methyl-D-aspartate receptor encephalitis is more common than previously thought.  It should be included in the differential diagnosis of encephalitis, especially in patients presenting with psychiatric symptoms, even if the MRI appears normal.  It has a favorable outcome for most patients depending on early diagnosis.  Confirmatory testing of CSF or serum for antibodies to NR1 subunits of the NMDAR is required to confirm the diagnosis.

Suggested Reading Dalmau J, Gleichman AJ, Hughes EG, et al. Anti-NMDA-receptor encephalitis: case series and analysis of the effects of antibodies. Lancet Neurol 2008; 7: 1091–8. Dalmau J, Lancaster E, Martinez-Hernandez E, et al. Clinical experience and laboratory investigations in patients with antiNMDAR encephalitis. Lancet Neurol 2011; 10: 63–74. Finke C, Kopp UA, Prüss H, et al. Cognitive deficits following antiNMDA receptor encephalitis. J Neurol Neurosurg Psychiatry 2012; 83(2): 195–8. Florance NR, Davis RL, Lam C, et al. Anti-N-methyl-D-aspartate receptor (NMDAR) encephalitis in children and adolescents. Ann Neurol 2009; 66: 11–18. Gleichman AJ, Spruce LA, Dalmau J, Seeholzer SH, Lynch DR. AntiNMDA receptor encephalitis antibody binding is dependent on amino acid identity of a small region within the GluN1 amino terminal domain. J Neurosci 2012; 32: 11082–94. Ikeguchi R, Shibuya K, Akiyama S, et al. Rituximab used successfully in the treatment of anti-NMDA receptor encephalitis. Intern Med 2012; 51: 1585–9. Jones KC, Benseler SM, Moharir M. Anti-NMDA receptor encephalitis. Neuroimaging Clin N Am 2013; 23: 309–20. Pruss H, Dalmau J, Harms L, et al. Retrospective analysis of NMDA receptor antibodies in encephalitis of unknown origin. Neurology 2010; 75: 1735–9. Schmitt SE, Pargeon K, Frechette ES, et al. Extreme delta brush—a unique EEG pattern in adults with anti-NMDA receptor encephalitis. Neurology 2012; 79: 1094–100.

CASE

Part IV

53

Neuroinflammatory Diseases Asim K. Bag

Clinical Presentation A 27-year-old man from Turkey presented with headache and subacute-onset weakness of his left upper and left lower extremities. He also had a low-grade fever and nuchal rigidity. However, sensory symptoms and abnormalities in executive functions were absent. He noted presence of intermittent pain

Imaging (A)

(B)

(A)

(B)

in his left wrist joint and right ankle joint, in addition to recurrent mouth ulcerations. On examination, multiple genital aphthae and erythema nodosum were noted. Hematologic studies revealed an elevated SED and C-reactive protein. Performing a lumbar puncture was considered; however, magnetic resonance studies were conducted first (images shown). Fig. 53.1 (A) Axial FLAIR image through the level of the optic tract. (B) Axial FLAIR through the level of the cerebral peduncles.

Fig. 53.2 (A) Postcontrast coronal T1WI through the cerebral peduncles and (B) Postcontrast axial T1WI through the cerebral peduncles.

Advanced Neuroradiology Cases, ed. Amaral et al. Published by Cambridge University Press. © Cambridge University Press 2016.

227

Part IV. Neuroinflammatory Diseases: Case 53

Neuro-Behçet Disease Primary Diagnosis Neuro-Behçet disease

Differential Diagnoses Central nervous system vasculitis Multiple sclerosis Acute disseminated encephalomyelitis (ADEM) Infiltrating primary CNS neoplasm

Imaging Findings Fig. 53.1: (A) Axial FLAIR image through the level of the optic tract and (B) cerebral peduncle demonstrates patchy areas of FLAIR abnormality in the right mesodiencephalic junction (involving the posterolateral aspect of the right cerebral peduncle and adjoining brain tissue extending superiorly to the thalami regions). It was noted that the left side was normal. There was no abnormal FLAIR signal in the visualized area of the periventricular white matter. Fig. 53.2: (A) Postcontrast coronal T1WI and (B) axial sequences demonstrate subtle enhancement (arrow) within the area of FLAIR abnormality.

Discussion Multi-joint arthritis and recurrent oral and genital ulcers is suggestive of a rheumatologic condition. Predominant motor involvement with typical mesodiencephalic junction abnormality on MRI is consistent with neuro-Behçet disease (NBD). Primary CNS vasculitis is exclusively a disease of the CNS. The presence of extra-neurologic manifestations rules out the possibility of primary CNS angiitis. The presenting clinical features including oro-genital ulcers are suggestive of Behçet disease (BD) rather than other causes of systemic vasculitis. The presence of extra-CNS manifestations rules out multiple sclerosis, ADEM, and CNS neoplasm. Behçet disease is an idiopathic, relapsing, multisystem inflammatory disease of the vascular system that typically presents in the third or fourth decade of life. The disease is prevalent in Asia and Mediterranean countries. The highest prevalence of BD, 80–370 cases per 100,000 persons, is in Turkey. The disease is rare in North America, as compared to the more prevalent areas. In areas with high prevalence of the disease, there is no gender predilection. The common clinical manifestations include recurrent painful mucocutaneous ulcerations that predominantly involve the oral cavity and genital areas. Other common manifestations of BD include episodic bilateral uveitis, venous occlusive diseases such as obstruction of superior and inferior vena cava or deep vein thrombosis, vasculitis involving medium- to large-sized arteries, including the pulmonary artery, and asymmetric arthritis. The kidneys, heart, and gastrointestinal system can also invariably be involved in patients with BD. The international BD study group criteria for diagnosing BD include recurrent

228

oral ulceration and two of the following four symptoms: recurrent genital ulcers, eye lesions (anterior or posterior uveitis or retinal vasculitis), skin lesions (erythema nodosum, pseudofolliculitis, papulopustular lesions, or acneiform nodules), and a positive pathergy test. The literature variably documents the frequency of neurologic involvement in patients with BD, i.e., NBD, reporting incidence as low as 1.35% to as high as 59%. Generally, NBD is more common in men than women. The average age of onset of NBD is between 20 and 40 years. Diagnosis of NBD should be made carefully, as similar pathologic imaging features are more prevalent in elderly patients. Neurologic manifestations commonly develop several years after the onset of other systemic features; however, NBD may be the presenting symptom in up to 6% of patients, similar to this patient. Central nervous system involvement can be either parenchymal or non-parenchymal. Parenchymal NBD is usually a result of intense meningoencephalitis with penetrating inflammatory infiltration within the brainstem, thalamus, basal ganglia, and subcortical white matter. Although inflammatory infiltrates can occur in small-sized brain blood vessels, fibrinoid necrosis is not seen, differentiating BD from other cerebral vasculitis conditions. Non-parenchymal involvement, sometimes referred to as vasculo-NBD, presents with CNS vascular involvement, and may include dural venous thrombosis, intracranial hypertension, or intracranial aneurysms. Magnetic resonance is the imaging study of choice to evaluate NBD. Unilateral T2 hyperintensity in the mesodiencephalic junction with variable degree of swelling with or without enhancement is the characteristic imaging finding in parenchymal NBD. If enhancement is present, it is usually patchy. Bilateral involvement is less common than unilateral involvement. Diffusion restriction is not a feature; however, in acute lesions, diffusion restriction may be present. In patients with severe CNS involvement, T2 hyperintense lesions can also be seen in the subcortical white matter of the frontal and temporal lobes, as well as in the hypothalamus. In the chronic phase, there may be lesions present in the supratentorial brain that are difficult to differentiate from multiple sclerosis lesions, based on imaging alone. As BD predominantly targets the vascular structures, intracranial vascular complications are also common, such as dural venous thrombosis, large arterial distribution infarction, and aneurysm involving the larger-sized intracranial arteries. Although rare, intracranial arterial dissection has also been described as a manifestation of NBD. Cerebrospinal fluid studies typically demonstrate pleocytosis and elevated protein. Opening pressure is elevated in patients with intracranial hypertension. Cerebrospinal fluid is negative for elevated IgG index and oligoclonal bands. NeuroBehçet disease is typically treated with immune suppressive agents with or without tumor necrosis factor (TNF) antagonists depending upon the nature of the disease, and severity of involvement response to steroids.

Part IV. Neuroinflammatory Diseases: Case 53

Key Points

Suggested Reading

 Central nervous system manifestations of NBD typically occur after other systemic involvement.  Typical imaging finding of NBD includes unilateral T2 hyperintensity with swelling involving the mesodiencephalic junction. Other CNS manifestations include dural venous thrombosis, intracranial hypertension, and aneurysm/occlusion of large-sized arteries such as middle and posterior cerebral arteries.

Akman-Demir G, Serdaroglu P, Tasci B. Clinical patterns of neurological involvement in Behçet’s disease: evaluation of 200 patients. The Neuro-Behçet Study Group. Brain 1999; 122(Pt 11): 2171–82. Al-Araji A, Kidd DP. Neuro-Behçet’s disease: epidemiology, clinical characteristics, and management. Lancet Neurol 2009; 8(2): 192–204. Saip S, Akman-Demir G, Siva A. Neuro-Behçet syndrome. Handb Clin Neurol 2014; 121: 1703–23.

229

CASE

Part IV

54

Neuroinflammatory Diseases Ricardo Tavares Daher, Lázaro Luís Faria do Amaral

Clinical Presentation A 32-year-old woman with history of headache was referred to our hospital for diagnostic follow-up of an expansive lesion. Other than recent headaches, she did not report any notable medical conditions. Subsequent imaging studies by thorax CT demonstrated presence of enlarged mediastinal nodes.

Imaging (A)

(B)

(C)

Fig. 54.1 (A) Sagittal T1WI, (B) Axial T2WI, and (C) Axial FLAIR images at the level of the inferior parietal lobule.

Advanced Neuroradiology Cases, ed. Amaral et al. Published by Cambridge University Press. © Cambridge University Press 2016.

231

Part IV. Neuroinflammatory Diseases: Case 54

(B) (A)

(C)

Fig. 54.2 (A) Axial diffusion, (B) Axial SWI, and (C) Axial T1WI through the level of the inferior parietal lobule.

232

Part IV. Neuroinflammatory Diseases: Case 54

(B) (A)

Fig. 54.3 (A) Sagittal and (B) Coronal T1WI postgadolinium reconstructions using a MinMip program.

(B) (A)

Fig. 54.4 (A) Axial SWI, and (B) Axial T1WI postgadolinium images through the level of the inferior parietal lobule (following surgical biopsy).

233

Part IV. Neuroinflammatory Diseases: Case 54

Parenchymal Neurosarcoidosis Primary Diagnosis Parenchymal neurosarcoidosis

Differential Diagnoses Lymphoma Other granulomatous diseases Meningioangiomatosis and fungal infections

Imaging Findings Fig. 54.1: (A) Sagittal T1WI, (B) Axial T2WI, and (C) Axial FLAIR images of the brain demonstrated subcortical signal abnormality at the left inferior parietal lobule associated with vasogenic edema. Fig. 54.2: (A) Axial diffusion, (B) Axial SWI, and (C) Axial T1WI of the brain demonstrated subcortical signal abnormality, with nodular foci of enhancement at the left inferior parietal lobule. Fig. 54.3: Axial SPGR T1WI postgadolinium with multiplanar reconstruction image. (A) Sagittal and (B) Coronal reconstructed images of the brain showed multiple areas of nodular enhancement in the left inferior parietal lobule. Fig. 54.4: (A) Postsurgical, axial SWI, and (B) Axial T1 postgadolinium image showing the surgical cavity.

Discussion Neurosarcoidosis is a chronic systemic disease of unknown etiology although current opinion favors an immune mechanism. This disorder is characterized by the presence of noncaseating granulomas with proliferation of epithelioid cells in affected organs. Lesions can be seen in the lungs, lymphatic system, eyes, skin, liver, spleen, salivary glands, heart, nervous system, muscles, and bones, although the lungs and the draining mediastinal lymph nodes are the most common sites of involvement. The clinical manifestations of neurosarcoidosis depend on the site of granuloma involvement and are non-specific. Subacute loss of central vision with retrobulbar pain was the most prevalent symptom, followed by bilateral affection of optic nerves, facial nerve impairment, and hearing loss due to vestibulocochlear pair lesion. Spinal cord involvement may present clinically with lower extremity weakness and other nonspecific signs of myelopathy. The proposed criteria for the diagnosis of neurosarcoidosis are: Definite – Clinical presentation suggestive of neurosarcoidosis with exclusion of other possible diagnoses and the presence of positive nervous system histology; Probable – Clinical syndrome suggestive of neurosarcoidosis with laboratory support for CNS inflammation (elevated level of CSF protein and/or cells, the presence of oligoclonal bands and/or MRI evidence compatible with neurosarcoidosis) and exclusion of alternative diagnoses together with evidence for systemic sarcoidosis (either through positive histology, including Kveim test, and/or at least two indirect indicator from Gallium scan, chest imaging, and serum ACE); Possible – Clinical presentation suggestive of neurosarcoidosis

234

with exclusion of alternative diagnoses where the above criteria are not met. Neurosarcoidosis has a wide spectrum of imaging findings because of the capacity to involve any part of the nervous system. Most common imaging findings include multiple white matter lesions, focal dural thickening and enhancement, optic nerve enhancement, leptomeningeal involvement including involvement of the other cranial nerves, and intraparenchymal mass lesion with or without involvement around the perivascular spaces. Spinal cord involvement may also occur. Enhancing parenchymal mass lesions are a frequent finding and characteristically have low sign intensity on T2WI images. Central necrotic component is typically absent in intraparenchymal neurosarcoid granulomas. There is no definite predilection for the intraparenchymal lesions. Mass-like intraparenchymal lesion can occur in any lobe of the brain. However, spread along the prevascular spaces is more common in the basal ganglia. Intraparenchymal lesions are frequently associated with adjacent leptomeningeal affection. The leptomeningeal finding is characterized by diffuse or nodular thickening and enhancement of the leptomeninges. When the leptomeningeal involvement is around the hypothalamus and pituitary infundibulum, patients may present with diabetes insipidus and other hormonal disturbances. Cranial nerve impairment is also very common and is especially prevalent in the optic and facial cranial nerves; imaging findings are enlargement and enhancement on contrast T1-weighted images. Pachymeningeal thickening and enhancement can also occur, such as a vasculitis-like pattern of involvement of the intracranial vasculature. Spinal involvement may present as nodular or diffuse leptomeningeal and dural lesions or intramedullary impairment. A confident diagnosis of neurosarcoidosis is often difficult, particularly when the clinician is presented with an isolated CNS disorder that is likely to have an inflammatory basis. The nervous system is a relatively uncommon site for the disease to manifest and consequently, investigation to establish a diagnosis has centered on searching for histologic confirmation in other organs.

Key Points  The presence of a T2-hypointense, intensely enhancing mass without central necrosis is the typical finding of intraparenchymal neurosarcoidosis that is frequently associated with adjacent leptomeningeal involvement.  Thorax CT showing the presence of enlarged mediastinal nodes could help in the differential diagnosis.

Suggested Reading Hodge MH, Williams RL, Fukui MB. Neurosarcoidosis presenting as acute infarction on diffusion-weighted MR imaging: summary of radiologic findings. AJNR Am J Neuroradiol 2007; 28: 84–6. James DG, Sharma OP. Neurological complications of sarcoidosis. Proc R Soc Med 1967; 60: 1169–70. Oksanen V. Neurosarcoidosis: clinical presentations and course in 50 patients. Acta Neurol Scand 1986; 73: 283–90.

Part IV. Neuroinflammatory Diseases: Case 54 Pawate S, Moses H, Sriram S. Presentations and outcomes of neurosarcoidosis: a study of 54 cases. Q J Med 2009; 102: 449–60.

Smith JK, Matheus MG, Castilho M. Imaging manifestations of neurosarcoidosis. AJR Am J Roentgenol 2004; 182: 289–95.

Shah R, Roberson GH, Curé JK. Correlation of MR imaging findings and clinical manifestations in neurosarcoidosis. AJNR Am J Neuroradiol 2009; 30: 953–61.

Zajicek JP, Scolding NJ, Foster O, et al. Central nervous system sarcoidosis—diagnosis and management. Q J Med 1999; 92: 103–17.

235

CASE

Part IV

55

Neuroinflammatory Diseases Ivanildo Castro Pereira Jr., Lázaro Luís Faria do Amaral

Clinical Presentation A 23-year-old man presented to our facility with an acute illness characterized by symmetric ophthalmoplegia, ataxia, somnolence, and abnormal Babinski reflex. His medical history included two-week history of febrile, upper respiratory tract infection. A lumbar puncture was performed and laboratory analysis of CSF demonstrated albuminocytologic dissociation (protein level of 32 mg/dl), normal glucose level (63 mg/dl). No cells were present in CSF.

Imaging (A) (B)

(C)

Fig. 55.1 (A–C) Axial T2WI through the level of the brainstem and basal ganglia.

Advanced Neuroradiology Cases, ed. Amaral et al. Published by Cambridge University Press. © Cambridge University Press 2016.

237

Part IV. Neuroinflammatory Diseases: Case 55

(A)

(B) (C)

Fig. 55.2 (A–C) Axial FLAIR MR images through the level of the brainstem and basal ganglia.

(A)

(B)

Fig. 55.3 (A) Axial diffusion and (B) Axial EPI (T2*) through the level of the middle cerebellar peduncles.

238

Part IV. Neuroinflammatory Diseases: Case 55

(A)

(B)

Fig. 55.4 (A–B) Axial T1WI postcontrast images through the level of the brainstem.

239

Part IV. Neuroinflammatory Diseases: Case 55

Bickerstaff Encephalitis Primary Diagnosis Bickerstaff encephalitis

Differential Diagnoses Infectious agents: Legionnaire disease, mycoplasma, Lyme disease, tuberculosis, or viral infections Multiple sclerosis Behçet disease Progressive multifocal leukoencephalopathy Vasculitis due to systemic lupus erythematosus Acute disseminated encephalomyelitis

Imaging Findings Fig. 55.1: (A–C) Axial T2WI demonstrated signal abnormalities in the middle cerebellar peduncle, in the midbrain, and in the posterior limb of the internal capsule, bilateral and symmetric. Fig. 55.2: (A–C) Axial FLAIR MR images confirm the T2WI findings. Fig. 55.3: (A) Axial DWI showed no diffusion restriction and (B) Axial EPI (T2*) showed hypointense signal in the middle cerebellar peduncle. Fig. 55.4: (A–B) Axial T1WI postcontrast showed no enhancement in the lesions.

Discussion Acute onset of ataxia and symmetric ophthalmoplegia in conjunction with impaired reflexes, and abnormal CSF findings in a patient with a recent history of infection is suggestive of Bickerstaff brainstem encephalitis (BBE). Although MR imaging study findings from patients with BBE vary, typically, regions of high T2 signal, with little or no enhancement, may be noted. In most reported cases of BBE, MR images showed areas of hypointensity on T1-weighted and areas of hyperintensity on T2-weighted images in the brainstem, thalamus, and cerebellum. However, eventually signal alteration may also involve the basal ganglia and thalamus. Contrast enhancement is not a typical feature. Differential diagnosis in patients with suspected infectious agents such as Legionnaire disease, mycoplasma, Lyme disease, or viral infections is almost impossible without CSF analysis and blood culture results. Multiple sclerosis is a demyelinating disease, not an infectious disease, and in general, CSF analysis demonstrates presence of oligoclonal bands. Behçet disease is rare and affects the mesencephalon region. Progressive multifocal leukoencephalopathy is more common in HIV patients, in the middle cerebellar peduncle and in the supratentorial white matter. Whipple disease is more common in the hypothalamic region and has fat lymph nodes in the peritoneum. Acute disseminated encephalomyelitis affects more children and adolescents, postvaccination, rather than adults.

240

Bickerstaff brainstem encephalitis is a monophasic brainstem inflammatory disease of unknown etiology with an acute onset, usually following viral or other infections, suggestive of an immunoregulatory pathogenesis. Although it is more prevalent in Southeast Asia, its prevalence in the United States or Europe is unknown. It is characterized by an acute onset of ophthalmoplegia, ataxia, hyperreflexia, and Babinski sign with or without flaccid tetraparesis, and disturbance of consciousness. Although rapid, severe clinical deterioration can occur during ongoing disease process, most patients have a good prognosis and will not suffer or show incomplete remission or succumb from BBE. Most pathologic, clinical, and analytical characteristics of BBE are similar to those observed in Miller-Fisher syndrome (MFS) and Guillain-Barré syndrome, particularly if patients present with flaccid tetraparesis. Some physicians believe that MFS and BBE form a continuous spectrum with variable peripheral nervous system and CNS involvement. Bickerstaff brainstem encephalitis and MFS have similar laboratory findings consisting of positive anti-GQ1b antibodies, CSF albuminocytologic dissociation, and CSF pleocytosis.

Key Points  Bickerstaff brainstem encephalitis is a rare, life-threatening disease that needs to be rapidly recognized to optimize positive patient outcome.  Severe clinical deterioration can occur during active disease process, but is reversible if diagnosed and treated early.  There is no disease-specific imaging abnormality for BBE.  Bickerstaff brainstem encephalitis should be considered in the differential diagnosis in patients with altered sensorium in the setting of new-onset ataxia.  Seropositivity for anti-GQ1b antibodies is suggestive of BBE diagnosis.

Suggested Reading Mondejar RR, Santos JM, Villalba EF. MRI findings in a remittingrelapsing case of Bickerstaff encephalitis. Neuroradiology 2002; 44(5): 411–14. Odaka M, Yuki N, Yamada M, et al. Bickerstaff’s brainstem encephalitis: clinical features of 62 cases and a subgroup associated with Guillain-Barre syndrome. Brain 2003; 126(Pt 10): 2279–90. Shahrizaila N, Yuki N. Guillain-Barré syndrome, Fisher syndrome and Bickerstaff brainstem encephalitis: understanding the pathogenesis. Neurol Asia 2010; 15(3): 203–9. Weidauer S, Ziemann U,Thomalske C, et al. Vasogenic edema in Bickerstaff’s brainstem encephalitis: a serial MRI study. Neurology 2003; 61(6): 836–8. Zagardo MT, Shanholtz CB, Zoarski GH, Rothman MI. Rhombencephalitis caused by adenovirus: MR imaging appearance. AJNR Am J Neuroradiol 1998; 19(10): 1901–3.

CASE

Part IV

56

Neuroinflammatory Diseases Benson Tran, Asim K. Bag, Lázaro Luís Faria do Amaral

Clinical Presentation A previously healthy 45-year-old white woman presented with subacute-onset cerebellar and brainstem symptoms. No other focal neurologic deficit was present. There were no systemic symptoms. Neurologic examination elicited cerebellar signs. Hematologic studies were negative for all rheumatologic and

autoimmune diseases, and serum ACE levels were normal. Cerebrospinal fluid examination for tuberculosis, syphilis, multiple sclerosis, and monoclonal lymphoid population was negative. High-dose intravenous corticosteroid therapy improved the patient’s symptoms; however, her symptoms recurred at every attempt to taper steroid dose.

Imaging Fig. 56.1 Axial FLAIR through the level of the lower pons.

Fig. 56.2 Axial T2WI through the level of the lower pons.

Fig. 56.4 Axial postcontrast T1WI image through the pons. Fig. 56.3 Axial postcontrast T1WI image through the pons.

Advanced Neuroradiology Cases, ed. Amaral et al. Published by Cambridge University Press. © Cambridge University Press 2016.

241

Part IV. Neuroinflammatory Diseases: Case 56

CLIPPERS Primary Diagnosis CLIPPERS

Differential Diagnoses Lymphomatoid granulomatosis Vasculitis (primary CNS vasculitis or CNS manifestation of systemic vasculitides) Neurosarcoidosis Angiocentric lymphoma

Imaging Findings Fig. 56.1: Axial FLAIR and Fig. 56.2: Axial T2WI images through the level of the lower pons demonstrated multiple, subtle punctate areas of increased T2 signal centered in the pons and in the middle cerebellar peduncles. Fig. 56.3: Axial postcontrast T1WI image through the same level demonstrated punctate areas of enhancement in the respective areas of T2 hyperintensities. Fig. 56.4: Axial postcontrast T1WI after a high dose of contrast through the same level demonstrated complete resolution of the enhancing foci.

Discussion The presence of subacute-onset cerebellar and brainstem symptoms, responsiveness to steroids, and signature imaging findings – punctate foci of enhancement peppering in the brainstem and cerebellum – are consistent with a diagnosis of chronic lymphocytic inflammation with pontine perivascular enhancement responsive to steroids (CLIPPERS). Perivascular enhancement can be seen in lymphomatoid granulomatosis, sarcoidosis, vasculitis (including primary CNS angiitis and CNS manifestation of systemic vasculitides), and lymphoma. Predominantly involving the lungs, lymphomatoid granulomatosis is a rare Epstein-Barr virus-associated, angiodestructive, lymphoproliferative disorder that seldom involves the CNS. Diffuse punctate enhancement that may have perivascular distribution is seen on imaging. In the case of brain involvement, punctate enhancement involves the entire brain with no predilection for the pons or cerebellum. Clinical manifestation of CLIPPERS clearly differs from that of previously mentioned differential diagnoses. Dural manifestations of neurosarcoidosis are commonly seen in the lepto- and pachymeninges. Perivascular enhancement can also be seen; however, it is characteristically limited to the basal ganglia, hypothalamus, and inferior frontal lobe. In addition, tumor-like involvement of the brain parenchyma is a known manifestation of neurosarcoidosis. Sarcoid granulomas are typically hypointense on T2WI images that demonstrate intense contrast enhancement. Primary CNS angiitis does not have any associated imaging abnormalities. Focal cortical or subcortical infarcts are common manifestations with or without meningeal enhancement. Central nervous system lymphoma typically demonstrates a homogeneously enhancing mass with diffusion restriction. Perivascular spread

242

is a known manifestation of lymphoma; however, the enhancement is linear along the perivascular space, rather than punctate. Unlike CLIPPERS, isolated pontine and cerebellar involvement has not been described in perivascular lymphoma spread. Bickerstaff brainstem encephalitis (BBE) and neuro-Behçet disease (NBD) have a predilection for the posterior fossa structures. Bickerstaff brainstem encephalitis is usually preceded by systemic illness. Affected individuals commonly present with a decreased level of consciousness and brainstem and cerebellar signs. Abnormalities on MRI can be seen in up to 31% of patients and include non-enhancing T2 signal abnormality. Central nervous system involvement in Behçet disease is rare and typically occurs three to five years after disease onset. Characteristic NBD imaging anomalies include asymmetric T2 abnormalities along the white matter tract involving 1) the mesodiencephalic junction (46%); 2) pons (40%); 3) hypothalamus and thalamus (23%); and 4) basal ganglia (18%). Although enhancement is uncommon in NBD, ringlike, patchy, and nodular, non-confluent enhancement patterns without any tropism for perivascular spaces in the cerebellum or pons have been described in patients with NBD. CLIPPERS is a definable, treatable, posterior fossapredominant inflammatory disorder of unknown etiology. It has a characteristic clinical presentation: CNS lesions that are typical in their distribution, histology, and imaging morphology. Manifestation of typical clinical presentation is dependent upon the areas of brain involvement and includes sensory abnormalities, diplopia, ataxia, and dysarthria. Symptom onset is typically subacute. Diplopia and ataxia are the most common presenting symptoms. Ataxia, diplopia, ataxic dysarthria, altered facial sensation, dizziness, nausea, tremor, and pseudobulbar affect appear in varying combinations during disease progression. The age of presentation varies between 16 and 86 years of age, with no sex predilection. The distinguishing imaging features of CLIPPERS include the presence of punctate and curvilinear enhancement peppering the pons with superior extension to the midbrain, inferior extension to the medulla, and posterior extension to the middle cerebellar peduncles with or without cerebellar involvement. Most case series report varied cerebellum involvement. As a result, a revised name, chronic lymphocytic inflammation with pontocerebellar perivascular enhancement responsive to steroids has been suggested. Typically, the densest population of enhancing foci is centered at the pons with the number and size of enhancing foci decreasing as the distance from the pons increases. Most lesions are typically less than 5 mm in diameter; few may be 1 cm or more in diameter. Tiny ring enhancement has also been described in CLIPPERS. Rarely, lesions can be seen in the basal ganglia or the spinal cord; however, even in these cases the densest population remains in the pons, the characteristic imaging abnormality. Involved areas appear as patchy or confluent areas of hyperintensity on T2WI images, including FLAIR, rather than as

Part IV. Neuroinflammatory Diseases: Case 56

punctate hyperintensity. Mass effect is typically absent. There may be a variable degree of patchy diffusion restriction. Typical histopathology findings include marked perivascular CD3-predominent lymphocytic infiltration in the white matter, in addition to presence of CD20-positive B cells, and CD68-positive histiocytes without any evidence of demyelination or granuloma formation. Inflammation of the vessel wall, as characteristically seen in vasculitis, is typically absent in CLIPPERS. As the acronym suggests, CLIPPERS responds to steroids. Rapid tapering of steroid dose may cause relapse. Relapse-free intervals increase with slower steroid tapering. Low-dose methotrexate has also been recommended to increase length of relapse-free intervals. Enhancing foci gradually disappear with successful treatment and diffusion restriction reverses. Gradually, atrophy of the involved areas is readily appreciated in the cerebellum.

Key Point  Gradual onset of brainstem symptoms with punctate areas of enhancement peppered in the pons and

adjacent cerebellum, midbrain, and medulla is highly suggestive of CLIPPERS.

Suggested Reading Bag AK, Davenport JJ, Hackney JR, Roy R, Fathallah-Shaykh HM. Case 212: chronic lymphocytic inflammation with pontine perivascular enhancement responsive to steroids. Radiology 2014; 273(3): 940–7. Pittock SJ, Debruyne J, Krecke KN, et al. Chronic lymphocytic inflammation with pontine perivascular enhancement responsive to steroids (CLIPPERS). Brain 2010; 133: 2626–34. Simon NG, Parratt JD, Barnett MH, et al. Expanding the clinical, radiological and neuropathological phenotype of chronic lymphocytic inflammation with pontine perivascular enhancement responsive to steroids (CLIPPERS). J Neurol Neurosurg Psychiatry 2012; 83: 15–22. Taieb G, Duflos C, Renard D, et al. Long-term outcomes of CLIPPERS (chronic lymphocytic inflammation with pontine perivascular enhancement responsive to steroids) in a consecutive series of 12 patients. Arch Neurol 2012; 69: 847–55.

243

CASE

Part IV

57

Neuroinflammatory Diseases Fabrício Guimarães Gonçalves, Lázaro Luís Faria do Amaral

Clinical Presentation A 39-year-old male with a history of acute, progressive, sensorial aphasia and mild right-sided weakness presented for neurologic evaluation. No other neurologic deficits or abnormalities were found. Contrast-enhanced MRI (see below)

and CSF analysis, which revealed positive oligoclonal bands, were obtained. Visual evoked potentials and somatosensory evoked potentials were both normal. Hematologic studies revealed normal WBC count, and C-reactive protein levels.

Imaging Fig. 57.1 Axial T2WI at the level of the lateral ventricles.

Fig. 57.2 Axial enhanced T1WI at the same level as Fig. 57.1.

Fig. 57.3 Axial DWI.

Fig. 57.4 Axial enhanced T1-weighted image (1.5-month follow-up).

Advanced Neuroradiology Cases, ed. Amaral et al. Published by Cambridge University Press. © Cambridge University Press 2016.

245

Part IV. Neuroinflammatory Diseases: Case 57

Balo Concentric Sclerosis Primary Diagnosis Balo concentric sclerosis

Differential Diagnoses Glioma Metastasis Multiple sclerosis Encephalitis Infections (fungal and pyogenic)

Imaging Findings Fig. 57.1: Axial T2WI at the level of the lateral ventricles demonstrated two oval-rounded lesions in the deep and periventricular white matter of the left frontal lobe. Mild adjacent edema (white arrows) noted but no significant effect. Note that the larger lesion (bold arrow) showed the typical lamellar pattern with alternating hypo- and hyperintense bands. Fig. 57.2: Axial enhanced T1WI showed that the larger lesion (arrow) is associated with blood-brain barrier breakdown, which also demonstrated incomplete ring of enhancement in a horseshoe configuration. Fig. 57.3: Axial DWI showed high intensity signal with a round configuration at the same level as the enhancement, and inside the lesions proper. The ADC values were mildly lower than in the contralateral white matter (image not shown). Fig. 57.4: Axial enhanced T1-weighted image posterior to seven-day treatment with steroids (1.5month follow-up). Both lesions (arrows) no longer showed contrast enhancement, but still show the lamellar pattern with onionskin appearance.

Discussion A clinicoradiologic picture of acutely progressive neurologic deficits is not expected in patients with a primary or secondary brain tumor, which more commonly demonstrate a more subacute presentation. Patients with multiple sclerosis (MS) may show acute neurologic symptoms, which can be seen during the first attack of the relapsing-remitting form of MS. Primary progressive MS demonstrates steadily worsening neurologic symptoms, without episodic remission. Acute, progressive neurologic deficits also present in patients with intracranial infections, such as encephalitis. However, patients with encephalitis typically show a more severe symptomatology, such as confusion, loss of consciousness, seizures, hallucinations, or even agitation. Certain fungal and pyogenic infections can cause acute progressing neurologic symptoms, which are usually associated with headache, fever, and a focal neurologic deficit. Patients with abscess can also present with nausea, vomiting, and neck stiffness if the meninges are also affected. In patients with intracranial infection, hematology studies typically demonstrate elevated WBC and SED levels; C-reactive protein level may be altered. Immunodeficient pathologies (such as AIDS) can be risk

246

factors for opportunistic brain infections, such as toxoplasmosis and cryptococcosis. Primary and secondary brain tumors typically demonstrate enhancement, active demyelination, and, possibly, infection on imaging studies. A horseshoe pattern (or open ring) of enhancement, combined with the presence of oligoclonal bands in CSF corroborate existence of a demyelinating process in this patient. The lamellar pattern (onionskin configuration) seen on both T2- and T1-weighted images (Figs. 57.1, 57.2, and 57.4) is typically seen in cases of Balo concentric sclerosis (BCS), which can be considered a pathognomonic finding in patients with this disease. Balo concentric sclerosis, first described as encephalitis periaxialis concentrica by Jozsef Baló (a Hungarian neuropathologist) in 1927, is a rare form of MS. Initially, BCS was deemed comparable to the Marburg variant of MS, a monophasic disease with fast progression and a typically fatal patient outcome. Currently, BCS is no longer considered an unfailingly fatal condition, as an increasing number of authors have been reporting cases with prolonged survival and, occasionally, spontaneous resolution. Current literature reviews indicate that BCS is more comparable to the more classical form of MS (Charcot type) than Marburg variant MS disease. Clinical symptoms of BCS include acute/subacute onset of mild cognitive impairment, altered behavior of focal neurologic deficits, which can endure for weeks or months – simulating a space-occupying lesion. Patients are typically young, and commonly present with headache, aphasia, cognitive or behavioral dysfunction, and/or seizures. Mononuclear inflammatory reaction and oligoclonal bands may be demonstrated in CSF studies. On brain MRI, the imaging modality of choice, BCS findings consist of alternating hypo- and hyperintense concentric rings on T2-weighted images and contrast-enhanced T1-weighted images with a typical onionskin appearance (Figs. 57.1, 57.2, and 57.4). Postcontrast T1-weighted images superbly demonstrate the distinct rings of enhancement in a concentric pattern at the sites of blood-brain barrier breakdown. In patients with early-stage BCS, MRI studies can be used to diagnose the disease accurately – significantly affecting patient morbidity and mortality associated with the disease. Currently, there is trend among radiologists to consider the distinctive MR findings of BCS as pathognomonic. If this trend continues, it will enable clinicians to provide an earlier diagnosis and recommended course of treatment, which may decelerate the inflammatory process and perhaps improve clinical outcome.

Key Points  The striking imaging feature in BCS is the presence of alternating bands of demyelinated and myelinated white matter, seen as concentric rings in the pathognomonic onionskin configuration.

Part IV. Neuroinflammatory Diseases: Case 57

 Magnetic resonance imaging has changed the natural history of the disease, which no longer can be considered a fatal condition.

Karaarslan E, Altintas A, Senol U, et al. Balo’s concentric sclerosis: clinical and radiologic features of five cases. AJNR Am J Neuroradiol 2001; 22(7): 1362–7.

Suggested Reading

Korte JH, Bom EP, Vos LD, Breuer TJ, Wondergem JH. Balo concentric sclerosis: MR diagnosis. AJNR Am J Neuroradiol 1994; 15(7): 1284–5.

Caracciolo JT, Murtagh RD, Rojiani AM, Murtagh FR. Pathognomonic MR imaging findings in Balo concentric sclerosis. AJNR Am J Neuroradiol 2001; 22(2): 292–3. Fleming GWTH. Encephalitis Periaxialis Concentrica. (Arch. of Neur. and Psychiat., February, 1928.) Balo, J. Br J Psych 1928; 74(307): 797.

Masdeu JC, Quinto C, Olivera C, et al. Open-ring imaging sign: highly specific for atypical brain demyelination. Neurology 2000; 54(7): 1427–33.

247

CASE

Part IV

58

Neuroinflammatory Diseases Fabrício Guimarães Gonçalves, Márcio Olavo, Gomes Magalhães, Lázaro Luís Faria do Amaral

Clinical Presentation A 10-year-old boy presented with a two-day history of fever, which rapidly escalated to a fulminant decrease in the level of consciousness and neurologic deterioration accompanied by intractable seizures. Lumbar puncture was performed and CSF analysis revealed pleocytosis. After a 10-day hospitalization, he developed signs of decortication.

Imaging Fig. 58.1 Axial T2WI at the level of the basal ganglia.

Advanced Neuroradiology Cases, ed. Amaral et al. Published by Cambridge University Press. © Cambridge University Press 2016.

249

Part IV. Neuroinflammatory Diseases: Case 58 Fig. 58.2 Coronal T2WI.

Fig. 58.3 T2* GRE.

250

Part IV. Neuroinflammatory Diseases: Case 58

Fig. 58.4 Axial DWI.

Fig. 58.5 Axial postgadolinium T1WI.

251

Part IV. Neuroinflammatory Diseases: Case 58

Acute Necrotizing Encephalopathy Primary Diagnosis Acute necrotizing encephalopathy

Differential Diagnoses Acute disseminated encephalomyelitis (ADEM) Leigh syndrome Reye syndrome Cerebral venous thrombosis Occlusion of the Percheron artery Mitochondrial encephalomyopathy and lactic acidosis and stroke-like episodes (MELAS) Myoclonus epilepsy and ragged red fibers (MERRF)

Imaging Findings Fig. 58.1: Axial T2WI at the level of the basal ganglia showed bilateral, symmetric hyperintense lesions involving both basal ganglia that are associated with mild edema and mass effect. Fig. 58.2: Coronal T2WI showed bilateral insular subcortical white matter involvement. Additional lesions are seen in the brainstem and cerebellar middle peduncles. Fig. 58.3: T2* GRE imaging showed petechial hemorrhagic foci in the right thalamus and in the subcortical white matter of both insular cortexes. Fig. 58.4: Axial DWI showed no abnormal DWI restricted diffusion in the brain parenchyma. Fig. 58.5: Axial T1WI postgadolinium administration showed bilateral symmetric ring-enhancing lesions in both basal ganglia.

Discussion The previously mentioned list of differential diagnoses shares similar imaging findings and differentiation among them can be made by assessing clinical presentation, age of involvement, history, and laboratory findings. Reye syndrome is characterized by an acute non-inflammatory encephalopathy and fatty degenerative liver disease, associated with gastroenteritis or viral infection. Patients usually present with increased liver transaminase levels, hypoglycemia, and hyperammonemia, data that can be helpful in determining the correct diagnosis. Leigh syndrome is a genetic mitochondrial disorder characterized by progressive neurodegeneration, psychomotor delay, and hypotonia, which does not cause acute encephalopathy. MELAS, another mitochondrial disease, is characterized by acute onset of headache, hemianopia, seizures, psychosis, aphasia or other focal symptoms, with muscle weakness and sensorineural hearing loss. Patients can evolve to cognitive deficit and dementia in chronic stages (see Part V: Case 74). Myoclonus epilepsy and ragged red fibers (MERRF) is a progressive multisystem syndrome, which usually begins in childhood, but can also occur in adulthood. Patients usually present with myoclonus, seizures, ataxia, hearing loss, lactic acidosis, short stature, exercise intolerance, dementia, cardiac deficits, eye abnormalities, speech impairment, and a characteristic microscopic abnormality observed in muscle biopsy, the ragged red fibers.

252

Acute disseminated encephalomyelitis (ADEM) is usually a monophasic demyelinating disease, affecting the gray and the white matter of the brain and spinal cord. It has a temporal relationship to prior infections or vaccination history. Patients can present with widespread CNS dysfunction, fever, and headache, seizures, and consciousness impairment. On MRI, lesions are poorly marginated, predominantly in the subcortical/deep white matter. Inflammatory lesions can also be seen in the spinal cord. Laboratory evaluation of CSF may reveal mild pleocytosis, uncommon IgG production, and oligoclonal bands. Occlusion of the Percheron artery and cerebral venous thrombosis usually occur in adulthood, with an acute onset of headache and variable degrees of loss of consciousness, depending upon which thalamic nuclei were involved. Executive functions, language, motor skills, and spatial cognition may also be affected. Diffusion-weighted imaging is important in depicting cytotoxic edema due to ischemic changes. Susceptibility-weighted images can show the blooming artifacts arising from the occluded veins and dural sinuses. Acute necrotizing encephalopathy (ANE), also known as acute necrotizing encephalopathy of childhood, is a recently described disease by Mizuguchi et al., in 1995. The disease is a distinct form of acute encephalopathy triggered by viral infections such as human herpesvirus-6 (HHV-6), parainfluenza, influenza A and B, H1N1, swine flu, herpes simplex virus, rotavirus, measles virus, Coxsackie A9, Epstein-Barr virus, and sometimes mycoplasma. Acute necrotizing encephalopathy has a worldwide distribution, but is more common in Asian countries. This encephalopathy usually occurs among children aged 5 months to 10 years, normally beginning around 4 years of age, with no gender predilection. It is commonly precipitated after a febrile respiratory illness, followed by the onset of acute encephalopathy. Additional symptoms may include recurrent vomiting, seizures, altered mental status which rapidly progresses to coma, and variable degrees of hepatic dysfunction. The most common biochemical abnormalities described in ANE patients are metabolic acidosis, thrombocytopenia, high blood serum levels of aspartate aminotransferases, creatine kinase, liver transaminases, and increased levels of blood urea nitrogen, as well as increased CSF proteins. Acute necrotizing encephalopathy can be sporadic or recurrent familial, which is related to a dominant trait in the gene (2q13) encoding the nuclear core protein RAN binding protein 2. Genes on other loci are also involved in recurrent familial. Located on chromosome 2 (2q12.1-2q13), ANE shows incomplete penetrance (estimated about 40%). These patients tend to have recurrent ANE with more severe neurologic sequelae. Imaging features of ANE are characterized by multiple, symmetric lesions showing increased T2 signal in the thalami, frequently with accompanying lesions in the brainstem tegmentum, periventricular white matter, internal capsule, putamen, and cerebellum. The most distinctive neuroimaging

Part IV. Neuroinflammatory Diseases: Case 58

finding of ANE is symmetric, multifocal lesions that invariably involve the thalami. On CT, ANE lesions show low density on CT and T1/T2 prolongation on MRI studies. After contrast administration, ring enhancement typically develops around the hemorrhagic areas by the second week of illness. Hemorrhage has been known to occur predominantly in the central portion of the involved deep gray matter, but not in the cerebral white matter. Intracranial hemorrhage and cavitation may also be seen, both of which are associated with a worse prognosis. Diagnostic confirmation is based on clinical symptoms and signs of acute encephalopathy, in the presence of characteristic imaging patterns. Polymerase chain reaction can be positive for some of the viruses responsible for initiating ANE, and can help determine the correct diagnosis. Hepatic dysfunction is another hallmark of the disease. The main differential diagnosis includes Reye syndrome, MELAS, MERRF, and Leigh syndrome. Histologic examination can demonstrate bilateral hemorrhagic necrosis in thalami, caudate nuclei, and cortical or subcortical areas. The tegmentum and pontine nuclei, the cerebellar cortex, and the dentate nucleus can be similarly involved. The capillary network can be prominent, with small, perivascular hemorrhages; occasionally confluent cytolysis of the neurons and glial cells, lipid-laden perivascular macrophages, myelin pallor, and spongiform changes can be seen in the affected areas. The prognosis and outcome can be poor in patients who do not start early treatment with steroids. Treatment is usually supportive and the use of steroids can be helpful. Patients with brainstem lesions usually have poor outcome. Magnetic resonance imaging signs of poor prognosis are cavitated parenchymal lesions with hemorrhage. It is not clear yet whether the use of gamma globulin can improve the outcome. As complications, patients can develop weakness, spasticity on extremities, memory disturbance, and difficulties in speech, intellectual impairments, and epilepsy.

Key Points  Acute necrotizing encephalopathy is commonly precipitated after a febrile respiratory illness followed by the onset of acute encephalopathy.  The most distinctive neuroimaging finding of ANE is symmetric, multifocal lesions that invariably involve the thalami.  Diagnostic confirmation is based on clinical symptoms and signs of acute encephalopathy, in the presence of characteristic imaging patterns.

Suggested Reading Mizuguchi M, Abe J, Mikkaichi K, et al. Acute necrotising encephalopathy of childhood: a new syndrome presenting with multifocal, symmetric brain lesions. J Neurol Neurosurg Psychiatry 1995; 58(5): 555–61. Neilson DE, Adams MD, Orr CM, et al. Infection-triggered familial or recurrent cases of acute necrotizing encephalopathy caused by mutations in a component of the nuclear pore, RANBP2. Am J Hum Genet 2009; 84(1): 44–51. Okumura A, Mizuguchi M, Kidokoro H, et al. Outcome of acute necrotizing encephalopathy in relation to treatment with corticosteroids and gammaglobulin. Brain Dev 2009; 31(3): 221–7. Osborn AG. Diagnostic Imaging. Salt Lake City, Utah: Amirsys; 2004. San Millan B, Teijeira S, Penin C, Garcia JL, Navarro C. Acute necrotizing encephalopathy of childhood: report of a Spanish case. Pediatr Neurol 2007; 37(6): 438–41. Skelton BW, Hollingshead MC, Sledd AT, Phillips CD, Castillo M. Acute necrotizing encephalopathy of childhood: typical findings in an atypical disease. Pediatr Radiol 2008; 38(7): 810–13. Suri M. Genetic basis for acute necrotizing encephalopathy of childhood. Dev Med Child Neurol 2010; 52(1): 4–5. Wong AM, Simon EM, Zimmerman RA, et al. Acute necrotizing encephalopathy of childhood: correlation of MR findings and clinical outcome. AJNR Am J Neuroradiol 2006; 27(9): 1919–23.

253

CASE

Part IV

59

Neuroinflammatory Diseases Fabrício Guimarães Gonçalves, Asim K. Bag

Clinical Presentation A 59-year-old woman presented to the neurology clinic with a one-month history of numbness and tingling of her extremities and recent onset blurring of vision in the right eye. She also had intractable vomiting and hiccups for the last three months. Extensive gastrointestinal evaluation did not reveal

any abnormalities. An MRI of the brain and cervical spine was performed (Figs. 59.1–59.5). She did not report history of recent vaccination, upper respiratory tract infection, or any form of viral infection. No family history of demyelinating disease or other autoimmune disease was reported. Hematologic studies were positive for autoantibodies against aquaporin-4 protein.

Imaging Fig. 59.1 Axial T2WI through the medulla at the level of olive.

Advanced Neuroradiology Cases, ed. Amaral et al. Published by Cambridge University Press. © Cambridge University Press 2016.

255

Part IV. Neuroinflammatory Diseases: Case 59 Fig. 59.2 Axial postcontrast T1-weighted sequence through the same level.

Fig. 59.3 Coronal FLAIR image through the level of the third ventricle/ posterior optic chiasm.

256

Part IV. Neuroinflammatory Diseases: Case 59 Fig. 59.4 Midline sagittal T2WI of the cervical spine.

Fig. 59.5 Postcontrast sagittal T1WI of the cervical spine through the same level.

257

Part IV. Neuroinflammatory Diseases: Case 59

Neuromyelitis Optica (NMO) Spectrum Disorder Primary Diagnosis Neuromyelitis optica (NMO) spectrum disorder

Differential Diagnoses Multiple sclerosis Acute disseminated encephalomyelitis Paraneoplastic syndrome

Imaging Findings Fig. 59.1: Axial T2WI through the medulla at the level of olive demonstrated T2 hyperintensity surrounding the inferior fourth ventricle in the expected location of area postrema. Fig. 59.2: Axial T1WI with contrast through the medulla at the level of olive demonstrated T1 hypointensity without any enhancement surrounding the inferior fourth ventricle in the expected location of area postrema. Fig. 59.3: Coronal FLAIR image through the level of the third ventricle demonstrated FLAIR hyperintensity involving the periventricular subependymal zone around the third ventricle. Fig. 59.4: Midline sagittal T2WI demonstrated an elongated T2 hyperintense lesion involving the dorsal medulla extending into the superior cervical cord. Fig. 59.5: Midline sagittal T1WI with contrast demonstrated possible subtle enhancement of the dorsal medulla in the area of abnormal T2 hyperintensity.

Discussion Typical clinical presentation with uncontrolled hiccups and vomiting, typical MRI findings involving the periependymal tissue surrounding the third and fourth ventricles, and positive autoantibodies against aquaporin-4 protein (NMO autoantibody) is diagnostic of neuromyelitis optica spectrum disorder (NMOSD). Although multiple sclerosis (MS) can present in older adults, the clinical presentation and imaging findings are not typical for MS lesions (see the discussion below). There was no prodromal symptom to suggest acute disseminated encephalomyelitis (ADEM). Simultaneous involvement of hypothalamus and brainstem can be seen in paraneoplastic syndromes with antibodies against Ma-2, which is typically seen in young male patients with germ cell tumors. Neuromyelitis optica (previously called Devic syndrome) is an autoimmune demyelinating disease previously considered a variant of MS. However, NMO is now classified as a distinct disease entity. Discovery of the correlative relationship between the presence of autoantibodies targeting the aquaporin-4 water channel (AQP4) protein and NMO distinguish it from MS. Aquaporin-4 is a type III transmembrane protein that regulates water entry into and out of specific cells (astrocyte foot processes) in the brain and interfaces with blood vessels within the neuropil and around the ventricles. Although aquaporin-4 regulates water in multiple types of epithelia cell types, it has a critical role when expressed by astrocytes that regulate water and ion movement in parts of the brain.

258

The serum autoantibody NMO-IgG, which targets aquaporin-4, is more than 90% specific for NMO in patients presenting with an optic-spinal syndrome; however, NMOIgG is not detected in patients with classic MS. NMO-IgG seropositivity also predicts relapse and conversion to neuromyelitis in patients presenting with a single attack of longitudinally extensive myelitis. Thus, the most current neuromyelitis optica diagnostic criteria (2006) require optic neuritis, myelitis, and at least two of three supportive criteria: longitudinally extensive transverse myelitis, onset brain MRI that is not diagnostic of MS, or NMO-IgG seropositivity. Additionally, identification of anti-AQP4 antibodies indicates a broader clinical phenotype of this disorder, the so-called NMO spectrum disorder (NMOSD) that includes anti-AQP4 antibody seropositive patients with limited or early forms of NMO presenting with specific brain abnormalities. It also includes patients of systemic lupus erythematosus (SLE) and Sjögren syndrome that are also seropositive for anti-AQP4 antibodies. This rare inflammatory disease has a severe, relapsing course with a demyelinating component that has a predilection for the optic nerve and spinal cord: it often results in blindness, quadriplegia, and death. Anti-aquaporin-4 antibodies are specific and sensitive for NMO. Anti-aquaporin-4 antibody testing not only identifies individuals who are positive for NMO and present with limited symptoms (neuromyelitis optica spectrum disorder) but also permits identification of seronegative patients. It is important to differentiate patients with NMO from other forms of demyelinating diseases such as MS. Patients with NMO incorrectly diagnosed and treated for MS may experience worsening of symptoms and unwanted side effects if treated with interferon beta and natalizumab. Imaging plays a key role in the diagnosis of NMOSD. The characteristic imaging findings of NMOSD include diencephalic and dorsal brainstem lesions surrounding the third ventricle, aqueduct, and the fourth ventricle; longitudinally extensive transverse myelitis ( 3); and posterior and bilateral optic nerve involvement. Diencephalic lesions surrounding the third ventricle and the aqueduct are frequently asymptomatic but may have hypothalamic symptoms (syndrome of inappropriate antidiuretic hormone secretion, narcolepsy, temperature and satiety imbalance, etc.) in cases of severe involvement. Lesions at the dorsal brainstem surrounding the fourth ventricle in the area postrema and nucleus tractus solitarius are the most specific signs of NMOSD and can be seen in up to 46% of patients. These lesions are associated with intractable vomiting and hiccups, as seen in this patient. Optic nerve lesions tend to involve the posterior aspect of the nerves (prechiasmatic and chiasmatic segments) and are often bilateral. Periependymal lesions that surround the lateral ventricles can be seen in up to 40% of patients, particularly involving the corpus callosum, which can be confused with MS lesions. Acute

Part IV. Neuroinflammatory Diseases: Case 59

lesions prefer to involve the inferior surface of the callosum closest to the ventricle, the entire cranio-caudal dimension of the corpus callosum, or in severe cases, they may involve the adjacent cingulum. Hemispheric white matter lesions have also been described in NMOSD that are often tumefactive, have spindle-like appearances and cloud-like enhancement. Both unilateral and bilateral corticospinal tract lesional spread has also been described in NMOSD (up to 44% of patients). Spinal cord lesions preferentially involve the central gray matter area in the cervical and upper thoracic cord. Preferential central involvement is due to more abundant AQP4 expression in the gray matter and in the periependymal tissue surrounding the central canal. In clinical practice NMOSD needs to be differentiated from MS. Typical peripheral and spinal cord MS lesions demonstrate focal, rather than long segment, involvement, and involve the peripheral brainstem, not central brainstem. Additionally, typical MS lesions have Dawson figure appearance in the periventricular (lateral ventricle) and pericallosal white matter: these areas are usually spared in NMOSD. Enhancement lesions are typically ovoid or open-ring type rather than cloud-like. Additionally, cortical lesions are frequently seen in MS, unlike NMOSD.

Key Points  Definitive diagnostic criteria for NMO include optic neuritis, myelitis, and at least two of the following: MRI evidence of a contiguous spinal cord lesion three or more segments in length, onset brain MRI non-diagnostic for MS, or NMO-IgG seropositivity.  Periependymal lesions adjacent to the third ventricle, aqueduct, fourth ventricle, longitudinally extensive transverse myelitis, and posterior optic neuritis are the characteristic MRI findings seen in NMO.

Suggested Reading Khanna S, Sharma A, Huecker J, et al. Magnetic resonance imaging of optic neuritis in patients with neuromyelitis optica versus multiple sclerosis. J Neuroophthalmol 2012; 32(3): 216–20. Kim HJ, Paul F, Lana-Peixoto MA, et al. MRI characteristics of neuromyelitis optica spectrum disorder: an international update. Neurology 2015; 84(11): 1165–73. Pichiecchio A, Tavazzi E, Poloni G, et al. Advanced magnetic resonance imaging of neuromyelitis optica: a multiparametric approach. Mult Scler 2012; 18(6): 817–24. Wingerchuk DM, Lennon VA, Pittock SJ, Lucchinetti CF, Weinshenker BG. Revised diagnostic criteria for neuromyelitis optica. Neurology 2006; 66(10): 1485–9.

259

CASE

Part V

60

Metabolic Diseases Involving Central Nervous System Fabrício Guimarães Gonçalves, Asim K. Bag

Clinical Presentation A previously healthy five-year-old girl presented as an outpatient with a six-month history of progressive walking disturbance, ataxia, and repetitive falls. Symptoms began shortly after onset of a febrile, upper respiratory tract infection. The fever subsided with treatment, but the progressive walking impairment remained. Subsequently, she was unable to maintain steady balance, walk without support, or sustain her own weight when standing. She had no history of tremor, dysarthria, or

sensory impairment. On neurologic examination, generalized hypertonia, cerebellar ataxia, and spasticity were noted. Extensive metabolic workup did not reveal any metabolic abnormalities. The neurologic course gradually progressed with worsening of cerebellar ataxia and spasticity. Computed tomography of head (Fig. 60.1) and MRI of the brain was obtained (Figs. 60.2–60.4). Follow-up imaging was obtained six months after initial presentation (Fig. 60.5). She became wheelchair-bound at six years of age.

Imaging Fig. 60.1 Axial non-enhanced CT scan of the head through the level of the lateral ventricle.

Advanced Neuroradiology Cases, ed. Amaral et al. Published by Cambridge University Press. © Cambridge University Press 2016.

261

Part V. Metabolic Diseases Involving CNS: Case 60 Fig. 60.2 Axial T2WI of the brain through the same level.

Fig. 60.3 Axial FLAIR image of the brain through the centrum semiovale.

262

Part V. Metabolic Diseases Involving CNS: Case 60 Fig. 60.4 Coronal T1WI with contrast through the ventricles.

Fig. 60.5 Axial FLAIR image of the brain through level of lateral ventricle obtained six months after initial presentation.

263

Part V. Metabolic Diseases Involving CNS: Case 60

Vanishing White Matter Syndrome Primary Diagnosis Vanishing white matter syndrome

Differential Diagnoses Metachromatic leukodystrophy (MLD) Leukoencephalopathy with brainstem and spinal cord involvement and lactate elevation (LBSL) X-linked adrenoleukodystrophy (XLA) Acyl CoA enzyme deficiency (ACED) Megalencephalic leukoencephalopathy with subcortical cysts (MLSC)

Imaging Findings Fig. 60.1: Axial unenhanced CT scan demonstrated diffuse, bilateral, symmetric periventricular, deep and subcortical white matter marked hypodensity without mass effect. Fig. 60.2: Axial T2WI demonstrated diffuse, bilateral, symmetric periventricular, deep and subcortical white matter marked T2 hyperintensity without mass effect. Note that cortical gray and deep gray matters are spared. Fig. 60.3: Axial FLAIR image at the level of the centrum semiovale demonstrated thin, transversely oriented stripes within the rarefied white matter, diffuse white matter loss, and small capitations. Fig. 60.4: Enhanced coronal T1WI showed no enhancement in the abnormal white matter. Fig. 60.5: Axial FLAIR image of the brain (6-month follow-up) showed marked periventricular and deep white matter loss, with cavitation of the periventricular white matter that had progressed since the previous scan.

Discussion A history of normal neurologic development in a young child, before the appearance of relatively acute-onset cerebellar-predominant neurologic signs and symptoms, following a febrile illness, which gradually progressed over time, is a typical clinical presentation of vanishing white matter (VWM) and rules out other congenial leukodystrophies such as MLD, LBSL, XLA, and ACED. Worsening T2 hyperintensity that predominantly involves the central white matter, with progressive rarefaction of the white matter structure, as evidenced by gradual lowering of the FLAIR signal over time, is a typical imaging finding of VWM. The thin transversely oriented stripes within the rarefied white matter further confirm the diagnosis of VWM. The progressive rarefaction of white matter with central, transversely oriented stripes is not a characteristic feature of MLD, LBSL, XLA, and ACED. In addition, in MLD and LBSL, normal complete myelination is never noted, and patients present at a much earlier age. Moreover, the classic tigroid pattern of MLD is absent, further excluding MLD as a diagnostic option. The classic imaging feature of XLA, posterior, predominant white matter changes with diffusion restriction and enhancement at the advancing edge, are absent in this patient. Cyst formation is typical in MLSC but the cystic

264

changes have been described in the anterior temporal lobe and frontoparietal subcortical white matter, rather than periventricular white matter. Additionally, in MLSC, peripheral white matter is more severely involved compared to the central white matter involvement, as noted in this patient. Also known as childhood ataxia with CNS hypomyelination, VWM is a rare autosomal recessive CNS disorder. Despite affecting patients from all age groups, VWM is one of the most prevalent inherited childhood leukoencephalopathies and is caused by a mutation on chromosome 3q27. The disease classically presents between two and five years of age. Patients have a chronic, progressive, and episodic course, beginning during infancy or early childhood, with a chronic progressive cerebellopyramidal syndrome and spasticity. Cerebellopyramidal symptoms can progress to extensive neurologic deterioration that is precipitated by minor stress conditions, such as infections or mild head trauma. Along with the disease progression, individuals may experience hypotonia, irritability, vomiting, and epilepsy. Patients also may show consciousness impairment, which may vary from somnolence to coma, and occasionally death. The disease progression is heterogeneous with a wider clinical spectrum related to age at onset that is inversely related to clinical severity. Magnetic resonance imaging in VWM typically shows diffuse and symmetric signal abnormalities of the cerebral white matter, which becomes increasingly rarefied and subsequently cystic. Typically, patients present confluent and symmetric abnormalities in the periventricular and deep white matter. In young patients, myelination may be delayed. Reactive astrocytosis along the medullary veins leads to radial stripes that stretch from the ventricular wall to the subcortical regions, which is another typical imaging feature. Subcortical U fibers, the outer part of the corpus callosum, the internal capsules, and anterior commissure are typically spared. Contrast enhancement has not been reported in VWM patients. Longitudinal follow-up imaging studies usually demonstrate progressive replacement of white matter by cystic areas. In cerebral white matter, DWI increased diffusivity is apparent while proton spectroscopy demonstrates progressive reduction and eventual disappearance of all major metabolites, with the accumulation of glucose and lactate at CSF-like foci. The cerebellar white matter may also be affected in VWM (typically without cystic degeneration). In the brainstem, abnormal signal intensity may be detected, particularly in the pontine central tegmental tracts. Typically, abnormalities of the inner rim of the corpus callosum are noted. In the early stages of VWM, MRI does not necessarily display diffuse cerebral white matter involvement, rarefaction, or cystic degeneration. If MRI abnormalities do not meet the criteria for VWM, the diagnosis should be considered if the inner rim (the callosal-septal boundary) is affected. Vanishing white matter provokes widespread white matter abnormalities, not only in the anomalous-appearing white matter detected on MRI, but also in the normally appearing brain parenchyma, including the gray matter. Typical

Part V. Metabolic Diseases Involving CNS: Case 60

pathologic findings of VWM include increasing white matter rarefaction and cystic degeneration, appearance of oligodendrocytic densities with highly characteristic foamy oligodendrocytes, presence of meager astrogliosis with dysmorphic astrocytes, and loss of oligodendrocytes by apoptosis.

Key Points  Vanishing white matter is one of the most common inherited leukodystrophies.  Progressive loss of central white matter in the periventricular region without any enhancement but with relative sparing of the subcortical U fibers is the key abnormality.  During late-stage VWM, there may be cyst formation at the periventricular white matter.

Suggested Reading Kim HJ, Paul F, Lana-Peixoto MA, et al. MRI characteristics of neuromyelitis optica spectrum disorder: an international update. Neurology 2015; 84(11): 1165–73. Meoded A, Poretti A, Yoshida S, Huisman TAGM. Leukoencephalopathy with vanishing white matter: serial MRI of the brain and spinal cord including diffusion tensor imaging. Neuropediatrics 2011; 42(2): 82–5. Pronk JC, van Kollenburg B, Scheper GC, van der Knaap MS. Vanishing white matter disease: a review with focus on its genetics. Ment Retard Dev Disabil Res Rev 2006; 12(2): 123–8. Yang E, Prabhu SP. Imaging manifestations of the leukodystrophies, inherited disorders of white matter. Radiol Clin North Am 2014; 52(2): 279–319.

265

CASE

Part V

61

Metabolic Diseases Involving Central Nervous System Márcio Olavo Gomes Magalhães, Fabrício Guimarães Gonçalves

Clinical Presentation A 63-year-old woman presented with a long history of hoarseness that began during adolescence. She had no history of seizures, focal neurologic deficits, or behavior abnormalities.

Imaging Fig. 61.1 Photograph of patient demonstrating facial features.

Fig. 61.2 CT scanogram of the skull.

Fig. 61.3 Threedimensional volume rendering reformation of the skull base.

Fig. 61.4 CT image at the level of the cavernous sinuses.

Fig. 61.5 Axial T2WI at the level of the mesial temporal lobes.

Advanced Neuroradiology Cases, ed. Amaral et al. Published by Cambridge University Press. © Cambridge University Press 2016.

267

Part V. Metabolic Diseases Involving CNS: Case 61

Lipoid Proteinosis Primary Diagnosis Lipoid proteinosis

Differential Diagnoses Ganglioglioma Dysembryoplastic neuroepithelial tumor Oligodendroglioma Cavernous angioma (cavernoma)

Imaging Findings Fig. 61.1: Papular lesions in the eyelids, paucity of hair follicles in the eyebrow (black arrow), and complete absence of eye lashes (white arrow). Fig. 61.2: CT scanogram showed oval-shaped calcification projected at the sellar region (circle). Fig. 61.3: Three-dimensional volume rendering reformation from a volume CD acquisition showed bilateral symmetric triangularshaped bilateral calcifications adjacent to the cavernous sinuses (arrows). Fig. 61.4: CT image at the level of the cavernous sinuses showed bilateral and symmetric, curvilinear commashaped calcifications at both mesial temporal lobes (arrows). Fig. 61.5: Axial T2WI at the level of the mesial temporal lobes showed bilateral, symmetric T2 hypointense curvilinear structures with no mass effect or edema in both unci (arrows).

Discussion The long history of hoarseness, skin changes, and imaging findings are nearly pathognomonic for lipoid proteinosis. Ganglioglioma is a benign neoplasm that commonly involves the temporal lobes. Although its lesions commonly calcify and present enhancement after contrast administration, they are rarely bilateral or present with a comma-shaped configuration. Dysembryoplastic neuroepithelial tumor is also a benign neoplasm typically located in the temporal lobes. Comparatively, its lesions show calcification less frequently than gangliogliomas and less commonly present contrast enhancement. Oligodendroglioma tumors show calcification in approximately 70% of the patients but are known to present enhancement. Patients with the above-mentioned neoplasms can present with convulsions, particularly if their lesions are located in the temporal lobes. However, none of these lesions is associated with hoarseness or eyelid changes, or presents with bilateral symmetric calcification, calcification without mass effect, or adjacent parenchymal abnormalities. Bilateral comma-shaped calcification involving the amygdala, without enhancement or adjacent parenchymal abnormalities, is virtually pathognomonic of lipoid proteinosis. Lipoid proteinosis (also known as hyalinosis cutis et mucosae or Urbach-Wiethe disease) is a rare autosomal recessive disorder characterized by generalized intracellular deposition of amorphous hyaline material, causing thickening and scarring of the skin, vocal cords, eyelids, mucosa, and viscera. The lungs, lymph nodes, striated muscles, and CNS may also

268

be involved. Lipoid proteinosis was first described in 1908 by Siebenmann and further described in detail by Urbach and Wiethe in 1929. Clinically, the most significant characteristics are hoarseness and moniliform blepharitis. Infantile hoarseness is a common presenting feature of the disease, due to infiltration of larynx, particularly the vocal cords. In two-thirds of the cases, voice changes are present at birth or in early infancy as the first manifestation. Moniliform blepharitis is characterized by nodules and papules in the eyelids with a beaded appearance that is pathognomonic of this condition. Other clinical features include cutaneous changes including waxy-textured skin, yellow papules, generalized skin nodules, and thickening. Hyperkeratosis may appear in regions exposed to mechanical friction, such as the hands, elbows, buttocks, axillae, and knees. The skin can be damaged by minor trauma or friction, resulting in blisters and scar formation. Scalp involvement may lead to hair loss and areas of alopecia. The mucosa of the pharynx, tongue, and soft palate may be hypertrophied and patients may suffer from dyspnea and difficulties in speech, requiring tracheotomy in severe cases. The responsible gene involved in lipoid proteinosis encodes the extracellular matrix protein 1 (ECM1) on band 1q21. Patients with exon 7 mutations display slightly milder clinical features, while those with mutations on exon 6 manifest a severe phenotype. Polymerase chain amplification and direct nucleotide sequencing of the ECM1 gene can confirm the diagnosis. The neuroimaging hallmark of lipoid proteinosis is the presence of CNS calcifications, secondary to glycoprotein material deposition in small arteries and vessels, especially in long-term patients. Calcifications mostly occur in the amygdala, hippocampus, parahippocampal gyri, and the striatum. Because of the temporal lobe calcifications, some patients develop epilepsy. The pathognomonic pattern is bilateral, symmetric, comma-shaped calcifications in the amygdala. In spite of being easily depicted, not all patients with lipoid proteinosis will show CNS involvement. Computed tomography is an excellent imaging modality to show clearly the calcifications, their distribution, and pattern. On MRI, the most useful sequences are T2 gradient echo (T2*) and SWI. Usually, the calcifications tend to have low signal intensity in all sequences. Moreover, MRI is useful to rule out other differential diagnoses. Usually, symptoms improve over time. Nevertheless, voice alterations, speech difficulties, and airway obstructions tend to worsen, and older patients may need surgical procedures to maintain unobstructed airways.

Key Points  Lipoid proteinosis is a rare disorder with typical clinical features characterized by early-onset hoarseness associated with cutaneous manifestations, such as acneiform scarring, waxy papules, and eyelid beading (moniliform blepharitis).  Nevertheless, patients with lipoid proteinosis may remain undiagnosed for many years: imaging may play a key role in the diagnosis of these patients.

Part V. Metabolic Diseases Involving CNS: Case 61

 Lipoid proteinosis can be confidently confirmed in patients with a long history of hoarseness and skin changes, in the presence of typical imaging findings.

Suggested Reading Al Dousary SA, Al Anazy FH. Long-term follow-up of lipoid proteinosis laryngeal manifestations. Int J Pediatr Otorhinolaryngol 2011; 75(5): 728–9. Aroni K, Lazaris AC, Papadimitriou K, Paraskevakou H, Davaris PS. Lipoid proteinosis of the oral mucosa: case report and review of the literature. Pathol Res Pract 1998; 194(12): 855–9.

Goncalves FG, de Melo MB, de L Matos V, Barra FR, Figueroa RE. Amygdalae and striatum calcification in lipoid proteinosis. AJNR Am J Neuroradiol 2010; 31(1): 88–90. Hamada T, McLean WH, Ramsay M, et al. Lipoid proteinosis maps to 1q21 and is caused by mutations in the extracellular matrix protein 1 gene (ECM1). Hum Mol Genet 2002; 11(7): 833–40. Muda AO, Paradisi M, Angelo C, et al. Lipoid proteinosis: clinical, histologic, and ultrastructural investigations. Cutis 1995; 56(4): 220–4. Takahiro H. Lipoid proteinosis. February 2005 edn: Orphanet Encyclopedia; 2005.

269

CASE

Part V

62

Metabolic Diseases Involving Central Nervous System Rasmoni Roy, Asim K. Bag

Clinical Presentation A 42-year-old man presented to our emergency department with an acute-onset, severe headache, ataxia, vision difficulties, left-sided weakness, and numbness in his right fingertips and

right side of his face in a V2 distribution. He denied nausea, vomiting, dysphagia, or hoarseness. At presentation, his blood pressure was 235/150. Diagnostic MR images obtained at admission are shown below.

Imaging Fig. 62.1 Axial FLAIR image through the level of the pons.

Fig. 62.2 Axial T2 image through the same level.

Fig. 62.3 DWI image through the same level.

Fig. 62.4 Axial postcontrast image through the same level.

Advanced Neuroradiology Cases, ed. Amaral et al. Published by Cambridge University Press. © Cambridge University Press 2016.

271

Part V. Metabolic Diseases Involving CNS: Case 62

Central Variant of Posterior Reversible Encephalopathy Syndrome (CV-PRES) Primary Diagnosis Central variant of posterior reversible encephalopathy syndrome (CV-PRES)

Differential Diagnoses Rhombencephalitis Brainstem gliomas Osmotic demyelination syndrome Tumefactive demyelination

Imaging Findings Fig. 62.1: Axial FLAIR image and Fig. 62.2: Axial T2 image through the level of the pons. Images demonstrated diffuse swelling of the pons associated with T2 hyperintensity that caused minimal mass effect on the fourth ventricle floor. Fig. 62.3: DWI image through the same level does not demonstrate any diffusion restriction or facilitation. Fig. 62.4: Axial postcontrast image through the same level does not demonstrate any abnormal enhancement. On follow-up imaging, all the abnormalities were resolved.

Discussion This clinical presentation is classic for a hypertensive emergency. Central variant of posterior reversible encephalopathy syndrome (CV-PRES) is a known variant of PRES. It involves the central brain structures such as basal ganglia, thalami, periventricular white matter, and brainstem instead of the classic posterior predominant supratentorial brain involvement seen in typical PRES. In the given clinical context, this imaging manifestation is diagnostic of CV-PRES. Rhombencephalitis (RE) or brainstem encephalitis is a syndrome of multiple causes and outcomes. Although RE can have an identical imaging appearance, it has a more subacute onset. Most common infectious causes of rhombencephalitis stem from infectious etiologies such as Listeria monocytogenes, enterovirus, or herpes simplex virus type 1. In addition to infectious causes, paraneoplastic syndrome and autoimmune diseases such as Behçet disease, may also present as rhombencephalitis. Although diffuse pontine glioma can have a similar imaging appearance, its clinical presentation is different. Osmotic demyelination syndrome (ODS) typically presents acutely after rapid correction of hyponatremia. Acute demyelination in ODS involves the central pontine white matter, and spares the peripheral pontine structures, rather than involving the global pontine. Rarely, tumefactive demyelination involves the brainstem and in such cases, presentation is different. Typically, acute tumefactive lesion presents with enhancement, which is absent in this patient. Posterior reversible encephalopathy syndrome is a clinicoradiologic diagnosis with typical predisposing factors and

272

typical imaging appearances. It typically occurs in two different clinical contexts: 1) as a manifestation of hypertensive urgency or emergency, and 2) as a complication of CNS toxicity from different drug therapies such as immunomodulator, chemotherapeutic, or monoclonal antibody therapy. The pathophysiologic manifestation of PRES is still not fully understood. In patients with hypertension, it is thought to arise from failed autoregulation, resulting in breakthrough brain edema versus severe autoregulatory vasoconstriction induced by hypertension, resulting in brain ischemia and edema. T-cell activation with inflammatory cytokine production or endothelial activation (with or without leukocyte trafficking) are common underlying biologic processes manifested in PRES. Clinical presentation of PRES is variable, and depends largely upon the areas of brain involvement and disease severity. Common symptoms include headache, vision change, extremity weakness, nausea, and altered mental status. Symptoms may develop acutely or sometimes over several days. In severe widespread brain involvement, generalized seizures and coma may develop. The typical imaging manifestations of PRES include abnormal FLAIR signal in the cortical and subcortical white matter of the occipital lobes and posterior parietal lobe. Involvement can be symmetric or asymmetric. These findings are typically reversible with treatment of hypertension or withdrawal of the offending drug. Diffusion restriction, hemorrhage, and enhancement are uncommon imaging manifestations and usually signify poorer prognosis. In severe cases, the deep brain structures such as basal ganglia and thalami can also be involved, in addition to the typical cortical and subcortical white matter involvement. Central variant of PRES is a known variant of PRES and accounts for approximately 10–20% of all PRES cases. In this variant, geographic distribution of PRES is different from the typical PRES. The deeper brain structures such as basal ganglia, thalami, periventricular white matter, and brainstem structures are involved. In addition to the typical predisposing factors, systemic lupus erythematosus and other autoimmune diseases can also present as CV-PRES. If the clinical presentation of the patient is not typical for PRES and in the absence of specific predisposing factors, the diagnosis of CV-PRES can be challenging as other diseases as discussed above can have identical imaging presentations.

Key Points  Central variant of PRES should be considered when the clinical profile of a patient is typical for PRES, such as hypertensive emergency, eclampsia, solid organ transplantation, etc., and central brain structures and brainstem are involved rather than posterior predominant supratentorial brain involvement.

Part V. Metabolic Diseases Involving CNS: Case 62

 Central variant of PRES can involve only the thalami or the brainstem (as in this patient) or combination of the two.

Bartynski WS. Posterior reversible encephalopathy syndrome, part 2: controversies surrounding pathophysiology of vasogenic edema. AJNR Am J Neuroradiol 2008; 29(6): 1043–9.

Suggested Reading

McKinney AM, Jagadeesan BD, Truwit CL. Central-variant posterior reversible encephalopathy syndrome: brainstem or basal ganglia involvement lacking cortical or subcortical cerebral edema. AJR Am J Roentgenol 2013; 201(3): 631–8.

Bag AK, Cure JK, Sullivan JC, Roberson GH. Central variant of posterior reversible encephalopathy syndrome in systemic lupus erythematosus: new associations? Lupus 2010; 19(2): 225–6.

273

CASE

Part V

63

Metabolic Diseases Involving Central Nervous System Fabrício Guimarães Gonçalves, Lázaro Luís Faria do Amaral

Clinical Presentation A 32-year-old pregnant woman with a three-day history of acute progressive headache, confusion, and decreased mental status reported to the emergency department after a recent episode of seizure and left-sided hemiparesis. Following clinical evaluation, she was diagnosed with pregnancy-induced hypertension (gestational hypertension), with levels of arterial blood pressure of 197/110 mm Hg. To evaluate neurologic

symptoms, a non-enhanced CT scan of the brain was obtained, which demonstrated a focus of hyperdensity in the subcortical right frontal region. Because of the possibility of parenchymal hemorrhage, MRI studies with venous and arterial venous MRI angiogram were obtained. Catheter angiography was not suggested because of the pregnancy status. Both angiographic examinations were unremarkable (not shown).

Imaging Fig. 63.1 Axial FLAIR image.

Advanced Neuroradiology Cases, ed. Amaral et al. Published by Cambridge University Press. © Cambridge University Press 2016.

275

Part V. Metabolic Diseases Involving CNS: Case 63 Fig. 63.2 Axial T1-weighted image.

Fig. 63.3 Axial T2-weighted image.

276

Part V. Metabolic Diseases Involving CNS: Case 63 Fig. 63.4 Axial FLAIR image.

Fig. 63.5 Axial T2* image.

277

Part V. Metabolic Diseases Involving CNS: Case 63

Posterior Reversible Encephalopathy Syndrome with Hemorrhage Primary Diagnosis Posterior reversible encephalopathy syndrome with hemorrhage

Differential Diagnoses Venous sinus thrombosis with venous infarct and hemorrhagic transformation Reversible cerebral vasoconstriction syndrome Hypertensive parenchymal hemorrhage

Imaging Findings Fig. 63.1: Axial FLAIR image showed multifocal cortical and subcortical hyperintensities (open arrows) representing vasogenic edema with mild mass effect in the posterior aspects in both parietal lobes and in the right frontal region. Fig. 63.2: Axial T1-weighted image demonstrated a subcortical slightly hyperintense focus (open arrow) with nodular appearance and mild mass effect in the right frontal lobe. Fig. 63.3: Axial T2-weighted image demonstrated that the right frontal lobe lesion was associated with fluid-fluid level material (open arrow). The material has hyperintense signal superiorly and low signal in its dependent portion with mild surrounding edema (solid arrow), which is consistent with recent acute hemorrhage. Fig. 63.4: Axial FLAIR image showed a nodular lesion with increased internal signal, more prominent than the brain parenchyma and the CSF. Note that the lesion showed mild perilesional vasogenic edema (open arrow). The increased signal in the subarachnoid spaces of the right frontal gyrus is compatible with hyperproteinaceous material, most likely representing subarachnoid hemorrhage (small solid arrows). Fig. 63.5: Axial T2* image showed a nodular lesion with mixed internal heterogeneous signal with low signal in its periphery (open arrow), consistent with hemosiderin deposition.

Discussion The findings of bilateral cortical and subcortical vasogenic edema in the posterior portions of the brain parenchyma in a pregnant woman diagnosed with pregnancy-induced hypertension who presented with altered mental status, seizures, and neurologic deficit are highly consistent with posterior reversible encephalopathy syndrome (PRES). The association of PRES with intraparenchymal and subarachnoid hemorrhage is not a typical PRES presentation; however, it has been described as a potential complication of PRES. Approximately 15% of patients with PRES demonstrate hemorrhage. Intracranial hemorrhage can consist of minute focal hemorrhages, sulcal subarachnoid hemorrhage, and focal hematoma, all of which may present in isolation or in combination. Association of PRES with hemorrhage is more typically seen in patients after allogenic marrow transplant and in patients undergoing therapeutic anticoagulation.

278

Parenchymal changes as described above can be seen as a complication of cerebral venous thrombosis (CVT), particularly if the superior sagittal sinus or cortical veins are involved. Approximately one-third of CVT patients developed intracerebral hemorrhage, which can occur in the brain parenchyma and extra-axial spaces such as the subarachnoid. Owing to local decreased cerebral perfusion, there could be ischemic injury and cytotoxic edema, disruption of the blood-brain barrier leading to vasogenic edema, and venous and capillary rupture culminating in parenchymal hemorrhage. Negative MRI venography findings (not shown) and absence of signal changes in the flow voids make the possibility of venous thrombosis less likely in this patient. Reversible cerebral vasoconstriction syndrome (RCVS) is a fairly recently described entity that occurs with varied clinical and imaging findings. Patients usually experience a sudden history of severe headache associated with reversible multifocal segmental vasoconstriction of intracranial arteries (vasospasm), and rarely focal neurologic deficit. Reversible cerebral vasoconstriction syndrome can be associated with medication use (sympathomimetic, serotonergic, or other drugs), pregnancy, and puerperium. Intracranial hemorrhage is a common finding in PRES and is most frequently subarachnoid, occurring secondary to vasospasm. Magnetic resonance images of the brain during the first week appear normal in most patients, with intracerebral hemorrhage occurring in up to 10% of cases. Ten percent of RCVS patients will have MRI abnormalities consistent with PRES, which may have a possible common pathogenetic mechanism with disturbance of vascular tone. Radiographic evidence of infarction, often in the arterial borderzone regions, may be seen during the second week. Magnetic resonance angiography reveals diffuse segmental arterial constriction in up to 90% of cases. In addition, the large- and medium-sized arteries are more commonly affected. The history of headache, pregnancy, and subarachnoid hemorrhage are good reasons to include RCVS in the list of differential diagnoses. The lack of vasoactive agent exposure, a common association with RCVS, the presence of seizures and focal neurologic deficit, uncommon in typical cases of RCVS, and the fact that the MR arterial angiogram was normal (patient could not undergo catheter angiography) make the possibility of RCVS less likely. Hypertensive intracranial hemorrhage (HIC) is a result of increased hypertension, which causes vessel wall degeneration, atherosclerosis, fibrinoid necrosis, and wall rupture. It is the second most common cause of stroke, which is seen as hyperdense lesions on non-enhanced CT. Hypertensive intracranial hemorrhage is more commonly located in the putamen, external capsule, thalamus, pons, and cerebellum. It manifests as macroscopic hematomas or micro-bleeds. Occasionally, the subarachnoid and the ventricular system can be involved as an extension of the parenchymal hemorrhage. As the majority of the patients with HIC are more than 50 years old and the hemorrhage is lobular (not the usual location), HIC is not likely the definitive diagnosis of this patient.

Part V. Metabolic Diseases Involving CNS: Case 63

Posterior reversible encephalopathy syndrome is a clinical and radiologic entity characterized by multiple symptoms such as headache, altered mental status, visual loss, seizures, focal neurologic defects, and loss of consciousness. It typically involves the posterior cerebral vasculature, affecting the parieto-occipital regions, which may be a consequence of reduced sympathetic innervation and impaired vascular autoregulation in this area. Hypertension is the most common cause of PRES, followed by cytotoxic medications (particularly immunosuppressive drugs), preeclampsia or eclampsia, and autoimmune and systemic conditions, including sepsis. Intracranial hemorrhage is a known complication of PRES, occurring in 5–20% of cases, which can occur secondarily to non-aneurysmal sulcal subarachnoid hemorrhage, to the rupture of pial vessels in the presence of severe hypertension, to impaired cerebral autoregulation, and also secondary to brain reperfusion after ischemic injuries, leading to multifocal brain hemorrhages (hemorrhagic transformation). Hemorrhage is not common in cases of eclampsia, with the highest rate among patients under immunosuppressive treatment. In immunosuppressed patients, hemorrhage is thought to occur more commonly in patients with a history of allogenic bone marrow transplant, as compared to patients with history of solid organ transplant. Hemorrhage following brain ischemia and infarction is also known to develop in the setting of cerebral vasoconstriction or vasospasm. Hemorrhage in PRES is also more frequent in patients on anticoagulation. T2-weighted images (TSE and FLAIR) are the MRI sequences of choice to evaluate PRES with hemorrhage. Abnormalities of the subcortical white matter in keeping with vasogenic edema with no enhancement are the rule, but the cortex and the basal ganglia can be eventually affected. Other structures such as the brainstem, cerebellum, and frontal and temporal lobes may also be involved. Perfusion images may play a significant role and can show cerebral hemodynamics related to this condition. As there is hyperperfusion, CBF and CBV are elevated, and time to peak is reduced. Diffusion-weighted positive images (with water molecule motion restriction) indicate that

there is cytotoxic edema, pointing towards an unfavorable outcome. Routine T2* gradient echo sequences should always be included in MRI protocols for early detection of blood byproducts. If available, more advanced susceptibility-weighted sequences are at least 3–6 times more sensitive than conventional T2*-weighted gradient echo images in detecting subtle minute hemorrhages. Treatment of the causative factor is typically sufficient to reverse the imaging findings and symptomatology. If prompt diagnosis is not made and treatment is delayed, changes in brain parenchyma (such as brain infarction/hemorrhage) and clinical manifestations may not be reversible. Blood pressure needs to be promptly and cautiously reduced. When immunosuppressive drugs are thought to be the cause, they should be withdrawn quickly to remove the insulting factor to the bloodbrain barrier. When hemorrhage is present, surgical treatment is not always necessary.

Key Points  Hemorrhage is a complication in 5–20% of patients with PRES.  Patients on immunosuppressive therapy are at higher risk for hemorrhage.  Three different patterns of hemorrhage can be seen: minute, subarachnoid, and intraparenchymal.

Suggested Reading Ferro JM. Update on intracerebral haemorrhage. J Neurol 2006; 253 (8): 985–99. Hefzy HM, Bartynski WS, Boardman JF, Lacomis D. Hemorrhage in posterior reversible encephalopathy syndrome: imaging and clinical features. AJNR Am J Neuroradiol 2009; 30(7): 1371–9. Piazza G. Cerebral venous thrombosis. Circulation 2012; 125(13): 1704–9. Pongmoragot J, Saposnik G. Intracerebral hemorrhage from cerebral venous thrombosis. Curr Atheroscler Rep 2012; 14(4): 382–9. Sattar A, Manousakis G, Jensen MB. Systematic review of reversible cerebral vasoconstriction syndrome. Expert Rev Cardiovasc Ther 2010; 8(10): 1417–21.

279

CASE

Part V

64

Metabolic Diseases Involving Central Nervous System Aparna Singhal, Asim K. Bag

Clinical Presentation A 60-year-old Asian woman with poorly controlled diabetes mellitus presented with relatively acute-onset abnormal movement of the proximal muscles of the upper extremity that developed over several days. Medical history includes hypertension that is well controlled with antihypertensive medication

use and lifestyle modification. She denied any prior history of neurologic disorder, substance abuse, critical illness, or prior hospital admission. Routine hematologic studies demonstrated blood glucose level of 420 mg/dl, elevated HBA1c levels, normal total and differential WBC counts, normal coagulation profile, and normal hepatic enzyme profile.

Imaging Fig. 64.1 Axial T1WI through the basal ganglia.

Fig. 64.2 Axial FLAIR through the basal ganglia.

Fig. 64.3 Axial GRE through the basal ganglia.

Advanced Neuroradiology Cases, ed. Amaral et al. Published by Cambridge University Press. © Cambridge University Press 2016.

281

Part V. Metabolic Diseases Involving CNS: Case 64

Hyperglycemic Hemiballismus and Hemichorea Primary Diagnosis Hyperglycemic hemiballismus and hemichorea

Differential Diagnoses Chronic hepatic encephalopathy Long-term total parenteral nutrition/hyperalimentation Methanol toxicity Metabolic abnormalities

Imaging Findings Fig. 64.1: Axial T1WI sequence through the level of the basal ganglia demonstrates T1 hyperintensity, predominantly involving the striatum (caudate nucleus and the putamen) on the left side. Subtle T1 hyperintensity is noted in the left globus pallidus. The left thalamus is normal. No abnormality is noted in the right basal ganglia or in the right thalamus. Fig. 64.2: Axial FLAIR image through the same level demonstrates abnormal T2 signal involving the left striatum, in the areas of T1 hyperintensity. Fig. 64.3: Axial GRE image through the same level demonstrates subtle punctate areas of hypointensity at the head of the left caudate nucleus (arrows). No hypointense signal is noted in the left putamen or globus pallidus.

Discussion Presentation of a patient with a history of uncontrolled diabetes mellitus, in conjunction with typical, relatively acuteonset hemichorea, hemiballismus movements, and the presence of unilateral T1 hyperintensity involving the striatum on imaging is diagnostic of hyperglycemic hemiballismus and hemichorea (HHH). The demonstrated imaging unilaterality and laboratory findings in this case exclude other processes such as hepatic encephalopathy (HE), hyperalimentation, or methanol toxicity. In addition, in hyperalimentation and chronic HE, T1 shortening and hyperintensity is typically seen in the globus pallidus, not the striatum, as seen in this patient. Methanol toxicity predominantly or exclusively involves bilateral putamina, but with infarction and diffusion restriction, which is absent in this case. T1 hyperintensity in the basal ganglia can also be seen in chronic HE, and in manganese toxicity secondary to long-term parenteral nutrition. Usually in these conditions, the abnormality is bilateral, often symmetric, and more commonly involves the putamen, in contrast to the caudate nuclei. Hemiballism-hemichorea (HH) is a clinical spectrum of continuous, non-patterned, and involuntary movements involving one side of the body. A focal structural or vascular lesion in the contralateral basal ganglia, particularly striatum, is the most common cause. Metabolic derangements (such as non-ketotic hyperglycemia or hyperthyroidism) and infectious diseases (e.g., HIV infection) can also present with HH, in the

282

absence of an identifiable lesion at the basal ganglia on imaging. Hyperglycemic hemiballismus and hemichorea is a recently described clinicoradiologic syndrome characterized by striatal (caudate and putaminal) T1 hyperintensity (contralateral to the abnormal movements) and is typically seen in older females of Asian descent with a history of uncontrolled diabetes mellitus. Although HHH has been described in patients with high blood glucose at presentation, presentation with normal glucose level has also been described. Hyperglycemic hemiballismus and hemichorea may be the presenting symptom of type 2 diabetes mellitus. The symptoms often reverse on achieving a euglycemic state; however, they may persist for years. Typical imaging findings of HHH include unilateral T1WI hyperintensity in the striatum, hypointensity on T2WI, and an unremarkable GRE sequence. Hyperdensity in the involved areas on CT and T2 isointensity have also been described. The pathophysiologic basis of the T1 hyperintensity is unclear. Many hypotheses have been proposed including petechial hemorrhage, myelinolysis, and calcium and manganese deposition. Bilateral involvement has also been reported in the literature. The striatal hyperintensity has been reported to be reversible in some patients. There are two atypical features in this patient on imaging: T2 hyperintensity and hemorrhage on GRE. Magnetic resonance spectropscopy demonstrates low NAA/Cr ratio, high choline/Cr ratio, and associated lactate peak, indicating energy depletion and neuronal dysfunction. Single photon emission computed tomography and PET studies have revealed reduction of striatal blood flow and metabolism. The HH pathogenesis triggered by hyperglycemia is unclear. Possible etiologic explanations including: 1) hyperviscosity due to hyperglycemia (causing disruption of the blood-brain barrier and leading to intracellular acidosis); 2) lack of acetoacetate in a non-ketotic state to form GABA in thalami or striatum; 3) increased sensitivity of dopamine receptors in postmenopausal women; 4) petechial hemorrhages; and rarely, 5) striatal infarction have been described by some in the literature. Typical histopathologic findings are gliosis, selective neuronal loss, and gemistocyte accumulation without any evidence of hemorrhage or infarction. Symptoms mostly resolve in response to treatment of the hyperglycemia. In up to 20% of cases, the symptoms persist even after three months. Although severity of the symptoms decrease, compared to severity at presentation, in some patients the symptoms may persist. In patients refractory to treatment of hyperglycemia, treatment with dopamineblocking agents, such as topiramate, may prove efficacious.

Key Point  Abnormal T1 signal involving unilateral striatum with or without abnormal hyperdensity on CT images (in the same areas) in a patient with poorly controlled diabetes mellitus

Part V. Metabolic Diseases Involving CNS: Case 64

presenting with contralateral hemiballismus and hemichorea is diagnostic of HHH.

Suggested Reading Lin JJ, Lin GY, Shih C, Shen WC. Presentation of striatal hyperintensity on T1-weighted MRI in patients with hemiballismhemichorea caused by non-ketotic hyperglycemia: report of seven new cases and a review of literature. J Neurol 2001; 248(9): 750–5.

Narayanan S. Hyperglycemia-induced hemiballismus hemichorea: a case report and brief review of the literature. J Emerg Med 2012; 43(3): 442–4. Nath J, Jambhekar K, Rao C, Armitano EJ. Radiological and pathological changes in hemiballism-hemichorea with striatal hyperintensity. Magn Reson Imaging 2006; 23: 564–8. Priola AM, Gned D, Veltri A, Priola SM. Case 204: nonketotic hyperglycemia-induced hemiballism-hemichorea. Radiology 2014; 271(1): 304–8.

283

CASE

Part V

65

Metabolic Diseases Involving Central Nervous System Prasad B. Hanagandi, Rahul J. Vakharia

Clinical Presentation A 50-year-old woman was rescued from an accidental fire. The patient was conscious and well oriented after the incident. Two weeks after the incident, she presented with tremors, and ataxia resembling Parkinsonian symptoms that were accompanied by cognitive decline, which rapidly progressed to quadriparesis and comatose state by the end of the third week. Laboratory studies including CSF analysis and hematologic evaluations were unremarkable. Toxicology analysis for substance abuse was negative.

Imaging Fig. 65.1 Axial T2WI image at the level of corona radiata and centrum semiovale.

Advanced Neuroradiology Cases, ed. Amaral et al. Published by Cambridge University Press. © Cambridge University Press 2016.

285

Part V. Metabolic Diseases Involving CNS: Case 65 Fig. 65.2 Axial FLAIR image at the level of corona radiata and centrum semiovale.

(B) (A)

Fig. 65.3 (A–B) Axial DWI image and corresponding ADC map at the level of corona radiata.

286

Part V. Metabolic Diseases Involving CNS: Case 65

(B) (A)

Fig. 65.4 Axial DWI image and corresponding ADC map at the level of centrum semiovale.

287

Part V. Metabolic Diseases Involving CNS: Case 65

Delayed Encephalopathy due to Carbon Monoxide Poisoning Primary Diagnosis Delayed encephalopathy due to carbon monoxide poisoning

Differential Diagnoses Heroin toxicity Cocaine abuse Postradiation changes Toxic leukoencephalopathies due to solvent abuse

Imaging Findings Fig. 65.1: Axial T2WI and Fig. 65.2: Axial FLAIR demonstrated diffuse, confluent, symmetric hyperintense signal abnormalities. Fig. 65.3: (A–B) Axial DWI and corresponding ADC maps at the level of the corona radiata and Fig. 65.4: (A–B) Axial DWI and corresponding ADC maps at the level of centrum semiovale demonstrated diffuse white matter diffusion restriction changes (arrows) reflective of delayed hypoxic demyelination.

Discussion The differential diagnosis list for the patient images showing diffuse white matter signal changes and diffusion restriction is often wide. In a patient with history of exposure to toxic fumes, no initial loss of consciousness, and negative toxicology workup, one should consider the possibility of delayed encephalopathy due to carbon monoxide poisoning. Carbon monoxide (CO) is both a colorless and odorless gas and poisoning usually occurs after inhalation of exhaust from automobiles or faulty home heating mechanisms. It is one of the preferred suicidal methods because of high success rate of approximately 30%. It has been broadly classified into acute and chronic presentations. The delayed presentation may be a progression of a known acute event; however, in 10% of cases patients may not have symptomatic loss of consciousness, as seen in our case. Patients may have neuropsychiatric and Parkinsonian features accompanied by a symptom-free or lucid interval ranging from a 7–21-day delay in onset to a 40day delay in onset. It is imperative to understand the various mechanisms underlying CO poisoning in order to correlate the imaging features of delayed encephalopathy. Carbon monoxide has 250 times more affinity than oxygen for the heme moiety and forms carboxyhemoglobin thereby reducing the oxygen-carrying capacity of the blood and triggering a cascade of events interfering with the mitochondrial respiratory chain. The pathophysiology of delayed effects is not completely understood and can be explained by multifactorial mechanisms stemming from neutrophilic aggregation and degranulation, release of myeloperoxidase and protease enzymes causing lipid peroxidation, increased oxidative stress, and apoptosis. Lipid peroxidation alters the structure of myelin basic protein (MBP), inciting a lymphocytic immune reaction causing progressive demyelination and chronic white

288

matter inflammation. Interactions of CO with cytochrome A and A3 contribute by inducing mitochondrial metabolism inhibition. The neuropathology of delayed CO poisoning predominantly consists of white matter changes, in contrast to deep gray nuclei involvement in acute toxicity, and has been grouped into three categories including: 1) necrotic lesions predominantly involving the centrum semiovale and commissures, 2) widespread necrosis with axonal damage in the periventricular white matter with extension across the corpus callosum and commissures, and 3) patchy, extensive demyelination of the periventricular and central deep white matter with relative preservation of the axons. Necrotic lesions are monophasic and non-relapsing variant while the third category comprising non-necrotic lesions is associated with delayed encephalopathy and referred to as biphasic myelinopathy of Grinker. Computed tomography scans are the initial modality for evaluating patients presenting with altered mental status and may show diffuse and confluent centrum semiovale and periventricular hypodensities. In severe cases, involvement of the external and internal capsules, corpus callosum, and subcortical white matter has also been described in the literature. However, CT studies can be normal in up to 54% of acute presentation. In 20% of the cases with no findings on initial CT, patients develop delayed neuropsychiatric sequelae (DNS). On MRI, the white matter changes appear hypointense on T1WI images and hyperintense on T2WI and FLAIR images, reflecting the demyelinating process. The lesion distribution is often symmetric or can be asymmetric. On DWI, cytotoxic and vasogenic edema can be identified by correlating the ADC values. Diffusion changes develop late after initial CO exposure and persist for approximately two to three weeks. The low ADC values can be attributable to acute myelinopathy with cytotoxic edema caused by energy failure. The imaging appearances in DNS can improve in 60–70% of cases over a period of one to two years. In 80% of the cases, marked brain atrophy changes have been documented in the first six months after exposure. Magnetic resonance spectroscopy shows elevated choline and decreased NAA peaks and increase in choline/creatine ratio in the early stages. Elevated lactate peak often serves as a marker for irreversible damage. Alterations in ADC and fractional anisotropy are also seen on diffusion tensor imaging. Administration of hyperbaric oxygen (HBO) therapy and corticosteroids can alter the morbidity in patients developing DNS. Rehabilitation with occupational, speech, and respiratory therapy is vital in improving quality of life in cases with severe DNS.

Key Points  Delayed effects of carbon monoxide poisoning should be considered when there is history of exposure to toxic fumes and initial lucid interval followed by clinical deterioration.  Confluent deep white matter changes with diffusion restriction are the pertinent imaging features and correspond to areas of toxic demyelination.

Part V. Metabolic Diseases Involving CNS: Case 65

Suggested Reading Beppu T. The role of MR imaging in assessment of brain damage from carbon monoxide poisoning: a review of the literature. Am J Neuroradiol 2014; 35(4): 25–31. Kim JH, Chang KH, Song IC, et al. Delayed encephalopathy of acute carbon monoxide intoxication: diffusivity of cerebral white matter lesions. AJNR Am J Neuroradiol 2003; 24(8): 1592–7. Kudo K, Otsuka K, Yagi J, et al. Predictors for delayed encephalopathy following acute carbon monoxide poisoning. BMC Emerg Med 2014; 14: 3.

Lo CP, Chen SY, Lee KW, et al. Brain injury after acute carbon monoxide poisoning: early and late complications. AJR Am J Roentgenol 2007; 189(4): W205–11. Molloy S, Soh C, Williams TL. Reversible delayed posthypoxic leukoencephalopathy. Am J Neuroradiol 2006; 27(8): 763–5. Shprecher D, Mehta L. The syndrome of delayed post-hypoxic leukoencephalopathy. NeuroRehabilitation 2010; 26(1): 65–72.

289

CASE

Part V

66

Metabolic Diseases Involving Central Nervous System Harry S. Hardin, Asim K. Bag

Clinical Presentation A 45-year-old man with a history of chronic HCV infection was found unconscious and was brought to the emergency department. His family reports presence of abnormal movement of his extremities during the last several months. They also noted that the patient previously complained of chronic fatigue and was not under any active treatment. His total and differential blood counts were normal, plasma PT and INR levels were elevated. He is jaundiced, serum liver function studies demonstrated elevated conjugated and unconjugated

bilirubin levels, AST and ALT; however, serum albumin levels were low. Axial head CT through the centrum semiovale and axial and sagittal T1-weighted sequences are shown. Remainder of the brain was unremarkable without any other area of hemorrhage. The remainder of the CT scan did not demonstrate any evidence of intra- or extracranial injury or evidence of physiologic basal ganglia calcification. A CT angiogram did not demonstrate any arteriovenous malformation or aneurysm in the area of hemorrhage. Gradient echo sequence did not reveal any microhemorrhage.

Imaging Fig. 66.1 Axial noncontrast head CT through the centrum semiovale.

(A)

(B)

Fig. 66.2 (A) Sagittal T1WI through the left basal ganglia. (B) Axial T1WI through the basal ganglia.

Advanced Neuroradiology Cases, ed. Amaral et al. Published by Cambridge University Press. © Cambridge University Press 2016.

291

Part V. Metabolic Diseases Involving CNS: Case 66

Chronic Hepatic Encephalopathy Primary Diagnosis Chronic hepatic encephalopathy

Differential Diagnoses Trauma with physiologic basal ganglia calcification Non-cirrhotic portal hypertension Occupational exposure to manganese

Imaging Findings Fig. 66.1: Axial non-contrast head CT through the centrum semiovale demonstrated a large intraparenchymal hematoma with minimal perihemorrhagic edema. Fig. 66.2: (A) Sagittal T1WI through the left basal ganglia demonstrated a large hematoma in the posterior aspect of the left frontal lobe, with hematocrit level (small white arrows). There is prominent T1 hyperintensity in the basal ganglia area (black arrow). (B) Axial T1WI through the basal ganglia demonstrated bilateral symmetric basal ganglia T1 hyperintensity.

Discussion Bilaterally symmetric putaminal T1 hyperintensity is highly suggestive of chronic hepatic encephalopathy. The presence of abnormal liver function tests and ethanol in the urine confirm the diagnosis. Extrahepatic hemorrhage is a known complication of chronic liver disease, secondary to coagulopathy. T1 signal, characteristic of calcified lesions, depends upon the ionic state of the calcium and the presence of other minerals, in addition to the calcium in the calcified lesion. Physiologic basal ganglia calcification can cause T1 hyperintensity; however, the absence of physiologic calcification on CT scan excludes this possibility. Isolated intraparenchymal hematoma in this location is extremely unlikely to be trauma-related. In addition, CT imaging did not reveal any evidence of head injury. Large lobar hematoma can be a complication of amyloid angiopathy; however, amyloid angiopathy is very uncommon in adults less than 65 years of age and MRI did not reveal any characteristic microhemorrhage. Hepatic encephalopathy (HE) consists of a series of neuropsychiatric, cognitive, and neuromuscular disorders in patients with comorbid liver disease. It is an important cause of mortality and morbidity in these patients. Hepatic encephalopathy can manifest as episodic, chronic HE, and clinical or subclinical HE. Chronic HE can be relapsing or persistent. Relapsing chronic HE consists of frequent episodes of acute HE due to other complications such as gastrointestinal tract bleeding, uremia, discontinuation of medication, or metabolic derangements, etc. In between two episodes, patients are usually cognitive and cooperative. Persistent chronic HE is irreversible and typically includes mental status change with or without abnormal movements. In the setting of chronic liver disease, several metabolic alterations can cause CNS symptoms. Hepatic dysfunction

292

causes an imbalance in the excitatory and inhibitory CNS neurotransmitters, and toxic buildup of many metabolites normally detoxified and excreted through the liver. For example, manganese (Mn) is one of the metabolites normally excreted through the liver. In hepatic failure, increased plasma Mn levels have a toxic effect on brain tissue, inducing selective neuronal loss and reactive gliosis in the globus pallidus (more prominently the medial segment) and substantia nigra. Patients with chronic HE frequently present with Parkinsonian syndromes secondary to Mn-induced selective basal ganglia injury. Abnormal CNS Mn deposition can be readily identified on MRI. Bilateral symmetric T1 hyperintensity in the basal ganglia and in the substantia nigra are well-known findings of abnormal CNS Mn deposition in patients with chronic hepatic failure. Signal intensity increases after transjugular intrahepatic portosystemic shunt placement as the toxic metabolites are directly shunted from portal circulation to systemic circulation. Signal intensity may reverse with hepatic transplantation. There is a disconnect between CNS Mn deposition and HE in patients with chronic liver disease. Almost all of the cirrhotic patients have abnormal pallidal T1 signal but not all chronic liver disease patients with abnormal pallidal T1 signal have chronic HE. Similarly, neuropsychiatric symptoms reverse immediately after liver transplantation; however, it takes up to a year to normalize the abnormal pallidal T1 signal. Although patients with chronic liver disease are coagulopathic, studies found intracerebral hemorrhage is very uncommon in chronic liver disease. The exact etiology of hemorrhage remained unknown in this patient as no obvious cause of hemorrhage could be found.

Key Point  Diagnosis of chronic liver disease should be suspected in any patient with bilateral symmetric putaminal T1 hyperintensity. The suspicion should be confirmed by the liver function studies, if presence of hepatic disease is unknown.

Suggested Reading Al-Okaili RN, Krejza J, Wang S, Woo JH, Melhem ER. Advanced MR imaging techniques in the diagnosis of intraaxial brain tumors in adults. Radiographics 2006; 26(Suppl 1): S173–89. Bulakbasi N, Kocaoglu M, Farzaliyev A, et al. Assessment of diagnostic accuracy of perfusion MR imaging in primary and metastatic solitary malignant brain tumors. AJNR Am J Neuroradiol 2005; 26(9): 2187–99. Chen XZ, Yin XM, Ai L, et al. Differentiation between brain glioblastoma multiforme and solitary metastasis: qualitative and quantitative analysis based on routine MR imaging. AJNR Am J Neuroradiol 2012; 33(10): 1907–12. Genovese E, Maghnie M, Maggiore G, et al. MR imaging of CNS involvement in children affected by chronic liver disease. AJNR Am J Neuroradiol 2000; 21(5): 845–51.

Part V. Metabolic Diseases Involving CNS: Case 66 Hegde AN, Mohan S, Lath N, Lim CC. Differential diagnosis for bilateral abnormalities of the basal ganglia and thalamus. Radiographics 2011; 31(1): 5–30.

Lin WC, Chou KH, Chen CL, et al. Significant volume reduction and shape abnormalities of the basal ganglia in cases of chronic liver cirrhosis. AJNR Am J Neuroradiol 2012; 33(2): 239–45.

Lassman AB, DeAngelis LM. Brain metastases. Neurol Clin 2003; 21(1): 1–23, vii.

Rovira A, Alonso J, Cordoba J. MR imaging findings in hepatic encephalopathy. AJNR Am J Neuroradiol 2008; 29(9):1612–21.

Lee HJ, Hinrichs CR. Is coagulopathic liver disease a factor in spontaneous cerebral hemorrhage? J Comput Assist Tomogr 2002; 26(1): 69–72.

Smirniotopoulos JG, Murphy FM, Rushing EJ, Rees JH, Schroeder JW. Patterns of contrast enhancement in the brain and meninges. Radiographics 2007; 27(2): 525–51.

293

CASE

Part V

67

Metabolic Diseases Involving Central Nervous System Prasad B. Hanagandi, Rahul J. Vakharia, Lázaro Luís Faria do Amaral

Clinical Presentation A 7-month-old boy presented with failure to thrive, seizures, and developmental delay. Physical examination revealed unusual coarse, brittle, sparse scalp hair with wooly texture, and hypopigmented skin. Neurologic examination revealed generalized hypotonia. Hematologic analysis showed increased serum lactate levels, decreased serum copper levels, and decreased copper oxidase levels.

Imaging (A) (B)

Fig. 67.1 (A) Axial T1WI of the brain at the level of corona radiata. (B) Axial T1WI of the brain at the level of lateral ventricles.

Advanced Neuroradiology Cases, ed. Amaral et al. Published by Cambridge University Press. © Cambridge University Press 2016.

295

Part V. Metabolic Diseases Involving CNS: Case 67

(A)

(B)

(C)

Fig. 67.2 (A) Axial T2WI of the brain at the level of corona radiata. (B) Axial T2WI of the brain at the level of lateral ventricles. (C) Coronal T2WI at the level of frontal horns and anterior temporal lobes.

296

Part V. Metabolic Diseases Involving CNS: Case 67

(B)

(A)

Fig. 67.3 (A–B) Time of flight MRA circle of Willis.

297

Part V. Metabolic Diseases Involving CNS: Case 67

Menkes Disease Primary Diagnosis Menkes disease

Differential Diagnoses Non-accidental trauma Glutaric aciduria type 1 Leigh disease (subacute necrotizing encephalopathy) Phenylketonuria

Imaging Findings Fig. 67.1: (A) Axial T1WI of the brain at the level of corona radiata and (B) Axial T1WI at the level of lateral ventricles demonstrated bilateral subdural collections with varying stages of hemorrhage along the cerebral convexities. Fig. 67.2: Axial T2W images (A) and (B) at the same levels as those in Fig. 67.1 and (C) at the level of anterior temporal lobes and frontal horns demonstrates generalized cerebral atrophy and white matter changes. Fig. 67.3: (A–B) Magnetic resonance angiography demonstrates tortuous intracranial vasculature.

Discussion The most important differential consideration in an infant presenting with failure to thrive and suspicious cranial imaging is child abuse, which can mimic Menkes disease. The combination of calvarial fractures on X-rays with subdural hygromas and hematomas of varying ages, coupled with bone fractures favors abuse. Clinical correlation with skin and hair abnormalities, serum copper levels, and tortuous intracranial vessels on MRI favor Menkes disease. Glutaric aciduria type 1 is an inborn autosomal recessive metabolic disorder resulting from deficiency of glutaryl-CoA dehydrogenase. Clinically it presents with macrocephaly, seizures, dystonia, encephalopathy, and developmental delay. On MR imaging, widening of the sylvian fissures is the most noted, characteristic feature in confirmed cases. Other findings include signal abnormalities involving the basal ganglia, dentate nuclei, and frontotemporal atrophy. Subdural hygromas can also occur in patients with glutaric aciduria, which can mimic Menkes disease, but can be differentiated by other MR imaging findings. Differential diagnoses also include Leigh disease and phenylketonuria: Unusual pathologies with associated signal abnormalities on diffusion imaging that mimic mitochondrial disease have been described in the literature. However, serum copper and ceruloplasmin concentrations would be normal in other metabolic disorders presenting with cutaneous manifestations, macrocephaly, and other neurologic symptoms, whereas they are low in Menkes disease. Menkes disease, also known as trichothiodystrophy or kinky hair syndrome, is a rare, X-linked recessive, lethal neurodegenerative disorder of impaired copper transport. It is estimated to occur in 1/300,000 people. Initial presentation during the first

298

few months of life is non-specific and may include lethargy, hypotonia, cephalohematomas, jaundice, and hypoglycemia. As the condition progresses, patients develop the characteristic, short, coarse, wiry hair, often by four to five months of age. This is accompanied by progressive neurologic deterioration. Seizures, failure to thrive, loose skin, pectus excavatum, urinary bladder diverticula, retinal hemorrhages, and developmental delay occurring by one year of age can also be seen. Menkes disease is usually lethal by two or three years of age, with respiratory failure cited to be the most common cause of death. The X-linked recessive disorder is caused by a mutation in the ATP7A gene, located on chromosome Xq21.1, which results in abnormal functioning of the copper-dependent enzyme adenosine triphosphate, thus leading to dysfunctional cellular transport and metabolism of copper, causing lower serum levels of copper. At birth, neuroimaging is often normal. As the disease progresses, symmetric cerebral and cerebellar volume loss occurs, often in association with bilateral subdural collections, both apparent on CT and MRI. Magnetic resonance imaging T1-weighted images can show hyperintense signal in the basal ganglia, similar to that of chronic hepatic encephalopathy. Magnetic resonance angiography reveals tortuous, elongated vessels. As the diagnosis is not often clear, skeletal radiographs are typically obtained, which show long bone osteoporosis, metaphyseal spurring, periosteal reaction, and scalloping of the vertebral bodies posteriorly.

Key Points  Menkes disease should be suspected in any male infant presenting with seizure disorder, hypopigmented skin, and characteristic coarse and sparse wooly, kinky, or wiry textured scalp hair.  Low copper levels on laboratory evaluation, brain atrophy, subdural collections, and tortuous intracranial vessels on imaging are major findings.  Non-accidental injury (child abuse) closely mimics Menkes disease.

Suggested Reading Bacopoulou F, Henderson L, Philip SG. Menkes disease mimicking non-accidental injury. Arch Dis Child 2006; 91(11): 919. Barkovich AJ, Good WV, Koch TK, et al. Mitochondrial disorders: analysis of their clinical and imaging characteristics. AJNR Am J Neuroradiol 1993; 14(5): 1119–37. Bindu PS, Taly AB, Kothari S, et al. Electro-clinical features and magnetic resonance imaging correlates in Menkes disease. Brain Dev 2013; 35(5): 398–405. Cronin H, Fussell JN, Pride H, Bellino P. Menkes syndrome presenting as possible child abuse. Cutis 2012; 90(4): 170–2. Datta AK, Ghosh T, Nayak K, Ghosh M. Menkes kinky hair disease: a case report. Cases J 2008; 1(1): 158.

Part V. Metabolic Diseases Involving CNS: Case 67 Jacobs DS, Smith AS, Finelli DA, et al. Menkes kinky hair disease: characteristic MR angiographic findings. AJNR Am J Neuroradiol 1993; 14(5): 1160–3. Kaler SG, Holmes CS, Goldstein DS, et al. Neonatal diagnosis and treatment of Menkes disease. N Engl J Med 2008; 358: 605–14. Leventer RJ, Kornberg AJ, Phelan EM, et al. Early magnetic resonance imaging findings in Menkes disease. J Child Neurol 1997; 12(3): 222–4.

Prasad AN, Levin S, Rupar CA, Prasad C. Menkes disease and infantile epilepsy. Brain Dev 2011; 33(10): 866–76. Seshadri R, Bindu PS, Gupta AK. Teaching NeuroImages: Menkes kinky hair syndrome. Neurology 2011; 81(2): e12–13. Takahashi S, Ishii K, Matsumoto K, et al. Cranial MRI and MR angiography in Menkes syndrome. Neuroradiology 1993; 35(7): 556–8.

299

CASE

Part V

68

Metabolic Diseases Involving Central Nervous System Taleb Al Mansoori, Prasad B. Hanagandi, Jeffrey Chankowsky

Clinical Presentation A 50-year-old woman, with a history of liver transplantation, presented with high-grade fever, and altered level of consciousness. Hematologic studies revealed elevated WBC count, C-reactive protein, and SED. Liver enzyme tests were grossly abnormal with increased serum bilirubin (60 μmol/l), ALT (856 IU/l), AST (1375 IU/l), and ALP (190 IU/l) levels, and elevated plasma ammonia level (119 μmol/l: reference range 11–32 μmol/l). Cerebrospinal fluid analysis was negative for presence of infectious agents.

Imaging (A)

(B)

Fig. 68.1 (A–B) Axial FLAIR through the level of insular cortex, basal ganglia, and anterior temporal horns.

Advanced Neuroradiology Cases, ed. Amaral et al. Published by Cambridge University Press. © Cambridge University Press 2016.

301

Part V. Metabolic Diseases Involving CNS: Case 68

(B) (A)

Fig. 68.2 (A–B) Axial DWI through the same level.

(A)

Fig. 68.3 (A–B) Axial ADC maps through the same level.

302

(B)

Part V. Metabolic Diseases Involving CNS: Case 68

Fig. 68.4 Axial postcontrast T1WI at the level of interpeduncular and suprasellar cisterns. Fig. 68.5 Axial DWI through the level of insular cortex and lateral ventricles. Fig. 68.6 Axial FLAIR through the level of insular cortex, basal ganglia, and thalami.

303

Part V. Metabolic Diseases Involving CNS: Case 68

Acute Hyperammonemic Encephalopathy Primary Diagnosis Acute hyperammonemic encephalopathy

Differential Diagnoses Posterior reversible encephalopathy syndrome (PRES) Paraneoplastic (autoimmune)/infectious encephalitis Creutzfeldt-Jakob disease Hypoxic ischemic injury

Imaging Findings Fig. 68.1: (A–B) Axial FLAIR images of the brain showed hyperintense signal changes in the bilateral insular and cingulate cortices, thalami, and frontotemporal cortices (arrows). Fig. 68.2: (A–B) Axial DWI demonstrated restriction in the areas of hyperintensity noted on Fig. 68.1. Fig. 68.3: (A–B) Corresponding axial ADC map demonstrates changes. Fig. 68.4: Axial postgadolinium image did not show abnormal enhancement. Fig. 68.5: Axial follow-up DWI showed improvement in diffusion restriction signal changes. Fig. 68.6: Axial FLAIR image showed volume loss and residual FLAIR hyperintense signal abnormality; especially the thalami.

Discussion Altered mental status, abnormal liver enzyme and ammonia levels, with involvement of the insular, cingulate, bilateral frontotemporal cortices, and thalami, with relative sparing of the basal ganglia on MRI are suggestive of hyperammonemic encephalopathy. Although imaging findings of hyperammonemic encephalopathy are well described in pediatric imaging literature, adult incidences are less prevalent. However, similar to this patient, brain MR images of affected individuals often demonstrate symmetric high signal on T2-weighted and FLAIR sequences, with diffusion restriction in the cingulate gyrus and insular cortex, sparing the perirolandic and occipital cortices. The cause of the cingulate gyrus and insular cortex susceptibility has yet to be elucidated. The perirolandic and occipital cortices are relatively resistant to injury, with no clear explanation. Hyperammonemic encephalopathy is a potentially reversible condition, if the specific signal changes are identified at early symptom onset. High signal on T2-weighted and FLAIR sequences involving the subcortical white matter, basal ganglia, thalami, and brainstem can occur in severe cases. Pseudonormalization changes on DWI and ADC occur approximately eight days after onset. Elevated glutamine peak can be seen on MR spectroscopy and correlates with the increased levels on CSF analysis. Later in the disease process, diffuse frontotemporoparietal cortical atrophy and high T1 signal intensity in the temporal cortex can be seen, indicating cortical laminar necrosis, a known consequence of hypoxic encephalopathy. The distribution of the abnormal cortical signal can be used to distinguish between hypoxic encephalopathy and hyperammonemic encephalopathy. The cortical laminar necrosis of

304

hypoxic encephalopathy has a predilection for watershed areas and/or the parieto-occipital regions, whereas cortical laminar necrosis in hyperammonemic encephalopathy mainly involves the insular, cingulate, and frontotemporal cortex. High T1 signal of the basal ganglia is prominent, which may be attributable to underlying chronic liver disease or prolonged ammonia toxicity. Seizure activity and diffuse hypoxic ischemic injury as potential diagnoses can be eliminated based on clinical history. Hematologic and CSF studies exclude paraneoplastic/ infectious encephalitis, and Creutzfeldt-Jakob disease (CJD). On imaging, CJD also tends to involve the cingulate and frontal cortices. However, the lack of basal ganglia involvement, which is typical for CJD, and the acute onset of disease, does not correlate with the natural course of CJD. In addition, CSF analysis was negative for prion disease. Severe forms of PRES tend to involve the cortex and can show diffusion restriction, but the absence of white matter changes and lack of obvious precipitating events make it an unlikely cause. Hyperammonemic encephalopathy can occur because of urea cycle defects, severe liver diseases, portosystemic shunting, sodium valproate or L-asparaginase therapy, infections, hypothyroidism, multiple myeloma, and in solid organ transplantations. Ammonia is mainly produced in the gastrointestinal tract and is metabolized to form urea. The liver is the main source for ammonia metabolism. Disruption of normal liver function results in accumulation of ammonia, which is then metabolized by the kidneys, brain, and skeletal muscle. The increased level of ammonia in the CNS is directly proportional to systemic hyperammonemia. Imaging and clinical symptoms can be seen at blood ammonia levels  60 μmol/l and in the form of lethargy, irritability, vomiting, and somnolence. Rapid synthesis of glutamine from ammonia and glutamate through glutamine synthetase leads to astrocyte swelling, oxidative and nitrosative damage, defective neurotransmitter synthesis, disruption of glucose metabolism, and increased blood-brain barrier permeability. Hyperammonemia eventually affects multiple brain neurotransmitters, especially affecting glutaminergic NMDA receptors and modulating GABA receptors. It has been shown that glutamine levels correlate with encephalopathy severity. The metabolic disturbance of ammonia and glutamine leads to cytotoxic brain edema, intracranial hypertension, cerebral hypoperfusion, and decreased consciousness. Patients usually present with progressive drowsiness, seizures, and often become comatose because of the increasing effect of ammonia toxicity on the brain. Prolonged ammonia exposure can lead to serious brain damage, including intellectual impairment and irreversible sequelae.

Key Points  High signal changes on T2-weighted and FLAIR sequences with diffusion restriction that predominantly involves the insular and frontotemporal cortices and cingulate gyri with

Part V. Metabolic Diseases Involving CNS: Case 68

relative sparing of basal ganglia are the striking imaging features of acute hyperammonemic encephalopathy.  Early detection and diagnosis is key to potentially reversing this condition.  Hyperammonemic encephalopathy can occur because of urea cycle disorders, severe liver diseases, portosystemic shunting, sodium valproate and L-asparaginase therapy toxicity, infections, hypothyroidism, multiple myeloma, and solid organ transplantations.

Suggested Reading Atluri DK, Prakash R, Mullen KD. Pathogenesis, diagnosis, and treatment of hepatic encephalopathy. J Clin Exp Hepatol 2011; 1(2): 77–86. Bindu PS, Sinha S, Taly AB, Christopher R, Kovoor JM. Cranial MRI in acute hyperammonemic encephalopathy. Pediatr Neurol 2009; 41(2): 139–42. Choi JM, Kim YH, Roh SY. Acute hepatic encephalopathy presenting as cortical laminar necrosis: case report. Korean J Radiol 2013; 14(2): 324–8.

Gomceli YB, Kutlu G, Cavdar L, Sanivar F, Inan LE. Different clinical manifestations of hyperammonemic encephalopathy. Epilepsy Behav 2007; 10(4): 583–7. Ikushima I, Korogi Y, Makita O, et al. MRI of arachnoid granulations within the dural sinuses using a FLAIR pulse sequence. Br J Radiol 1999; 72(863): 1046–51. Rosario M, McMahon K, Finelli PF. Diffusion-weighted imaging in acute hyperammonemic encephalopathy. Neurohospitalist 2013; 3(3): 125–30. Sato S, Yokota C, Toyoda K, Naganuma M, Minematsu K. Hyperammonemic encephalopathy caused by urinary tract infection with urinary retention. Eur J Intern Med 2008; 19(8): e78–9. Sureka J, Jakkani RK, Panwar S. MRI findings in acute hyperammonemic encephalopathy resulting from decompensated chronic liver disease. Acta Neurol Belg 2012; 112(2): 221–3. Takanashi J, Barkovich AJ, Cheng SF, et al. Brain MR imaging in acute hyperammonemic encephalopathy arising from late-onset ornithine transcarbamylase deficiency. AJNR Am J Neuroradiol 2003; 24(3): 390–3.

305

CASE

Part V

69

Metabolic Diseases Involving Central Nervous System Fabrício Guimarães Gonçalves

Clinical Presentation A 47-year-old man presented with a history of progressive cognitive decline, and a history of consuming one-half to one liter of vodka daily for the last 25 years. He had normal arterial blood pressure. Medical history did not include history of diabetes or other cardiovascular disease.

Imaging

Fig. 69.1 Midsagittal T2WI. Fig. 69.3 Zoomed in axial T2WI.

Fig. 69.2 Axial T2WI.

Advanced Neuroradiology Cases, ed. Amaral et al. Published by Cambridge University Press. © Cambridge University Press 2016.

307

Part V. Metabolic Diseases Involving CNS: Case 69

Marchiafava-Bignami Disease Primary Diagnosis Marchiafava-Bignami disease

Differential Diagnoses Partial callosal agenesis Chronic anterior cerebral artery infarction Susac syndrome Multiple sclerosis

Imaging Findings Fig. 69.1: Midsagittal T2WI showed marked thinning of anterior corpus callosum (CC), particularly involving the genu and the body (arrow). The remainder of the CC portions is normal. Fig. 69.2: Axial T2WI demonstrated marked anterior CC thinning with normal-appearing posterior portions. Fig. 69.3: Zoomed in axial T2WI showed that the thinned genu of the CC has a three-layered appearance. The outer layers indicated that the gliotic white matter and the inner portion are as hyperintense as the CSF.

Discussion The presence of anterior callosal thinning and the threelayered appearance of the CC are typical for MarchiafavaBignami (MB), particularly if there is a history of chronic alcoholism/alcohol consumption. In cases of partial callosal agenesis, the missing portions of the CC are typically posterior, usually the splenium and/or rostrum. In such situations, the third ventricle is usually high-riding, the lateral ventricles are parallel (non-converging) and colpocephaly, and Probst bundles are evident. In this patient, the diagnosis of partial callosal agenesis is unlikely because the abnormal portions of the CC are anterior, and thinned as opposed to missing. The remaining accompanying findings compatible with partial callosal agenesis are absent, excluding it as the primary diagnosis. Isolated CC infarcts are rare, possibly because of the rich blood supply from three main arterial systems: the anterior cerebral, anterior communicating, and posterior cerebral arteries. The body of the CC is usually supplied by the pericallosal branch of the anterior cerebral artery. The subcallosal and medial callosal arteries, branches of the anterior communicating artery, usually supply the anterior portion of the CC. The posterior pericallosal artery, a branch of the posterior cerebral artery, usually supplies the splenium of the CC. If cerebral infarct was the causative agent of the imaging findings in this patient, two different arterial territories would have been involved, which makes the diagnosis of CC chronic infarct less likely. Moreover, isolated CC infarct would be an unlikely event in a patient with no cardiovascular risk. Susac syndrome (SS) (see Part II: Case 26) and multiple sclerosis (MS) are diseases that typically involve the CC and therefore can cause callosal thinning. Susac syndrome virtually always involves the CC and patients have a typical clinical

308

presentation of encephalopathy, bilateral hearing loss, and branch retinal artery occlusions. Susac syndrome callosal lesions are T2 hyperintense and typically involve the body and splenium of the CC, particularly its middle layers. Typical MS patients have a less severe clinical evolution, when compared to SS patients. Imaging findings include multiple perpendicular callososeptal T2 hyperintensities, and Dawson fingers (areas of dysmyelination, glial scars, or sclerosis) can be seen. Additionally, periventricular, subcortical and deep white matter hyperintensities as well as brainstem, cerebellum, and spinal lesions can be seen. In more advanced cases, diffuse CC thinning can be seen. Owing to the lack of clinical and imaging concordant findings, the possibility of SS and MS are less likely in this case. Marchiafava-Bignami is a rare, progressive neurologic disease first described in Italian red wine drinkers by Carducci in 1898 and by Marchiafava and Bignami in 1903. It is more commonly seen in middle-aged or elderly alcoholic males. Rarely is it associated with various nutritional deficiencies, as opposed to alcoholism or alcohol abuse. It has been reported that MB individuals are typically heavy wine drinkers, with daily consumption of at least two liters per day for more than 20 years. The blueprint of MB is demyelination and necrosis of the CC with subsequent atrophy. Clinically, MB patients can present during one of three different stages (acute, subacute, or chronic). In the acute stage, patients may present seizures, impaired consciousness and cognition, gait disturbance, hemiparesis, stupor, coma, and death. Subacute stage features include variable mental confusion, dysarthria, behavioral abnormalities, memory deficits, interhemispheric disconnection states, and impaired gait. Chronic MB is less common and presents as mild dementia, which progresses over a period of several years. As patients in the acute stage usually have a fatal outcome (if left untreated), early diagnosis and treatment are crucial. The CC is usually affected in the middle portion (middle lamina), its highest myelinated component. Extracallosal structures can also be involved, in particular: the anterior and posterior commissures, the central semiovale, and other major white matter tracts. Other less commonly involved structures are the optic chiasm and tracts, putamen, cerebellar peduncles, and rarely, the cortical gray matter with laminar necrosis, and U fibers. Antemortem recognition of MB is mainly a neuroradiology diagnosis, since the clinical picture is often quite variable. It is important to remember that patients with acute MB can rapidly progress to death. On imaging, one can appreciate the sparing of the external layers of the CC and destruction of its middle component that gives rise to the three-layered sandwich sign, the characteristic imaging feature of MB. Cystic-necrotic changes are usually seen in the genu and splenium of the CC. On CT, the involved portions of the CC are hypodense. Occasionally, in the presence of hemorrhage in the subacute stage, lesions can appear hyperdense. On MRI, the inner

Part V. Metabolic Diseases Involving CNS: Case 69

portion of the CC is hypointense on T1 and hyperintense on T2-weighted and on FLAIR images. In the acute and subacute stages, one can appreciate thickening of the CC due to edema. In those stages, T2-weighted imaging reveals increased signal in the CC, due to edema and myelin damage. When myelin injury persists with subsequent myelin loss, the CC remains hyperintense in T2WI. In some patients, signal changes can recover to a normal appearance, despite total demyelination. In the chronic stage, there may be development of cystic changes that are T1 hypointense and T2 hyperintense. Sagittal FLAIR images are more sensitive to depict these chronic changes. The middle portion of the CC shows hypointense signal, reflecting white matter necrosis and leukomalacia; external layers show hyperintense gliotic rim. Diffusion-weighted imaging is useful to detect areas of active demyelination, which are seen as areas of restricted diffusion. Positive restricted diffusion during the acute stage does not always indicate permanent, irreversible tissue damage, as sometimes those signals may regress with no apparent permanent damage. Owing to acute stage myelin loss, the expected findings on spectroscopy are an increase in choline peaks and an increase in the choline/creatine ratio. Lactate peaks are expected in the acute/subacute stages of demyelination. Choline and lactate peaks and choline/creatine ratios can return to normal after clinical improvement. Diffusion tensor imaging is important to demonstrate regional abnormalities in the CC when there are no evident changes demonstrated on conventional MRI. Fiber tracking can demonstrate disruption of the axonal fibers within the CC, particularly in the middle portion of the body. Currently, there is no standard protocol for MB treatment; however, the patients are usually treated with thiamine, folate, and vitamin B complex, with good clinical improvement in many cases.

Acute MB can be considered a neuroradiology emergency in which early recognition and treatment are critical for satisfactory outcome.

Key Points  The presence of anterior callosal thinning and the threelayered appearance of the corpus callosum in a patient with known chronic alcohol use is suggestive of MB.  Early diagnosis and appropriate intervention may result in improved patient outcome.

Suggested Reading Arbelaez A, Pajon A, Castillo M. Acute Marchiafava-Bignami disease: MR findings in two patients. AJNR Am J Neuroradiol 2003; 24(10): 1955–7. Bano S, Mehra S, Yadav SN, Chaudhary V. Marchiafava-Bignami disease: role of neuroimaging in the diagnosis and management of acute disease. Neurol India 2009; 57(5): 649–52. Carrilho PEM, Santos MBMd, Piasecki L, Jorge AC. Doença de Marchiafava-Bignami: uma rara entidade com prognóstico sombrio. Rev Bras Ter Intensiva 2013; 25: 68–72. Kasow DL, Destian S, Braun C, et al. Corpus callosum infarcts with atypical clinical and radiologic presentations. AJNR Am J Neuroradiol 2000; 21(10): 1876–80. Kazi AZ, Joshi PC, Kelkar AB, Mahajan MS, Ghawate AS. MRI evaluation of pathologies affecting the corpus callosum: a pictorial essay. Indian J Radiol Imaging 2013; 23(4): 321–32. Nalini A, Kovoor JME, Dawn R, Kallur KG. Marchiafava-Bignami disease: two cases with magnetic resonance imaging and positron emission tomography scan findings. Neurol India 2009; 57(5): 644–8. Yoshizaki T, Hashimoto T, Fujimoto K, Oguchi K. Evolution of callosal and cortical lesions on MRI in Marchiafava-Bignami disease. Case Rep Neurol 2010; 2(1): 19–23.

309

CASE

Part V

70

Metabolic Diseases Involving Central Nervous System Prasad B. Hanagandi, Rahul J. Vakharia, Lázaro Luís Faria do Amaral

Clinical Presentation A 22-year-old man presented with a long-standing history of mental retardation and seizures, which began when he was 8 years of age. Seizures were well controlled with carbamazepine. Birth history was unremarkable; in particular, there was no evidence of birth asphyxia. Investigations for TORCH group of infections were negative at birth. Clinical examination revealed macrocephaly and hypopigmented skin macules on trunk and limbs extending along the lines of Blaschko. Ictal EEG revealed generalized spike and wave discharges at 2–3 Hz. Family history was not contributory.

Imaging Fig. 70.1 Axial T2W image at the level of lateral ventricles and corona radiata.

Advanced Neuroradiology Cases, ed. Amaral et al. Published by Cambridge University Press. © Cambridge University Press 2016.

311

Part V. Metabolic Diseases Involving CNS: Case 70

(B) (A)

(C)

Fig. 70.2 (A–B) Axial FLAIR images at the level of lateral ventricles and corona radiata. (C) Midsagittal FLAIR image through the corpus callosum.

312

Part V. Metabolic Diseases Involving CNS: Case 70 Fig. 70.3 Axial postgadolinium image at the level of lateral ventricles and corona radiata.

(A)

(B)

Fig. 70.4 (A) Picture of the abdomen and trunk. (B) Picture of the right thigh and leg.

313

Part V. Metabolic Diseases Involving CNS: Case 70

Hypomelanosis of Ito Primary Diagnosis Hypomelanosis of Ito

Differential Diagnoses Incontentia pigmenti Tuberous sclerosis Periventricular leukomalacia TORCH group of infections

Imaging Findings Fig. 70.1: Axial T2W image at the level of corona radiata and lateral ventricles showed multiple dilated perivascular spaces in the genu of corpus callosum, left periventricular, and left parietal subcortical white matter. Fig. 70.2: (A–B) Axial and (C) Midsagittal FLAIR images showed dilated perivascular spaces in the corpus callosum, periventricular, and left parietal subcortical white matter. Fig. 70.3: Axial postgadolinium image did not show enhancement. Fig. 70.4: (A–B) Diffuse hypopigmented skin lesions were noted along the lines of Blaschko on the abdominal wall, trunk, right thigh, and leg.

Discussion Hypopigmented skin patches along the lines of Blaschko, and dilated perivascular spaces in a patient with mental retardation and seizures clinch the diagnosis of hypomelanosis of Ito. Incontentia pigmenti is a closely mimicking diagnosis that also presents with dermal manifestations; however, the hyperpigmented skin lesions help in differentiating it from hypomelanosis of Ito. Lack of cortical tubers and other migration anomalies on neuroimaging make tuberous sclerosis a less likely diagnosis. The typical cutaneous manifestations in tuberous sclerosis include adenoma sebaceum and not hypopigmented lesions. Absent history of birth, asphyxia, and negative TORCH group of infections in a patient with hypopigmented skin lesions helps in excluding these options as the primary diagnosis. Hypomelanosis of Ito is a multisystem neurocutaneous syndrome involving the scalp, hair, nervous system, eyes, dentition, and skeletal system. It results from sporadic chromosomal mosaicism and affects neural migration. The cellular migration defect usually takes place in the second trimester and involves melanoblasts and neuroblasts, resulting in various cutaneous and nervous system abnormalities. Microscopic examination of the involved skin shows decreased number of melanocytes and pigmented melanosomes. The clinical presentation is varied, presenting in the form of unilateral or bilateral hypopigmented linear white streaks, patches, or whorls with irregular borders along the line of Blaschko. Focal alopecia, trichorrhexis nodosa, dysmorphic facies with flattened face, frontal bossing, macrocephaly/microcephaly, and hypertelorism, cup-shaped ears with external auditory canal stenosis are the craniofacial features. Skeletal deformities include short stature, scoliosis, syndactyly, polydactyly, and brachydactyly.

314

Association with a wide range of cardiac, genitourinary, and ophthalmologic abnormalities are also noted. According to neuroradiologic literature, the frequency of neurologic involvement is variable and ranges from 10% to 100%. Mental retardation is often seen in more than 60% of cases with a varying degree of severity. Magnetic resonance imaging findings include hemimegalencephaly, cerebellar atrophy or hypoplasia, pachygyria, heterotopia, cortical dysplasia, dilated perivascular spaces, bilateral periventricular cysts, and indistinct cortical gray-white matter differentiation. The wide range of migrational anomalies account for the seizures and mental retardation. Other infrequent findings include agenesis of corpus callosum, focal cerebral atrophy, and in rare instances, medulloblastoma, choroid plexus papilloma, and arachnoid cyst are some of the associated pathologies. Seizure control with antiepileptic medications is the mainstay of treatment for hypomelanosis of Ito. Ruiz-Maldonado et al. have proposed diagnostic criteria for hypomelanosis of Ito: A) Sine qua non-criterion: congenital or early-acquired nonhereditary, cutaneous hypopigmentation in linear streaks or patches involving more than two body segments. B) Major criterion i) One or more nervous system anomalies ii) One or more musculoskeletal anomalies C) Minor criterion i) Two or more congenital malformations other than nervous system or musculoskeletal anomalies ii) Chromosomal anomalies. D) Definitive diagnosis sine qua non-criterion plus one or more major criteria or two or more minor criteria. E) Presumptive diagnosis. Sine qua non-criterion alone or in association with one minor criterion.

Key Points  Hypomelanosis of Ito is a rare neurocutaneous syndrome with multisystem involvement.  Hypopigmented skin lesions are noted along the lines of Blaschko.  Varied CNS manifestations ranging from neuronal migrational disorders, dilated perivascular spaces, and white matter changes can be seen on neuroimaging.  The diverse spectrum of findings on imaging explains the varied neurologic symptoms.

Suggested Reading Almeida AS, Cechin WE, Ferraz J, et al. [Hypomelanosis of Ito - case report]. J Pediatr (Rio J) 2001; 77(1): 59–62. Battistella PA, Peserico A, Bertoli P, et al. Hypomelanosis of Ito and hemimegalencephaly. Childs Nerv Syst 1990; 6(7): 421–3. Bhushan V, Gupta RR, Weinreb J, Kairam R. Unusual brain MRI findings in a patient with hypomelanosis of Ito. Pediatr Radiol 1989; 20(1–2): 104–6.

Part V. Metabolic Diseases Involving CNS: Case 70 Malherbe V, Pariente D, Tardieu M, et al. Central nervous system lesions in hypomelanosis of Ito: an MRI and pathological study. J Neurol 1993; 240(5): 302–4.

Steiner J, Adamsbaum C, Desguerres I, et al. Hypomelanosis of Ito and brain abnormalities: MRI findings and literature review. Pediatr Radiol 1996; 26(11): 763–8.

Ruiz-Maldonado R, Toussaint S, Tamayo L, Laterza A, del Castillo V. Hypomelanosis of Ito: diagnostic criteria and report of 41 cases. Pediatr Dermatol 1992; 9(1): 1–10.

von Aster M, Zachmann M, Brandeis D, et al. Psychiatric, neuropediatric, and neuropsychological symptoms in a case of hypomelanosis of Ito. Eur Child Adolesc Psychiatry 1997; 6(4): 227–33.

315

CASE

Part V

71

Metabolic Diseases Involving Central Nervous System Rasmoni Roy, Lázaro Luís Faria do Amaral, Asim K. Bag

Clinical Presentation A 55-year-old man presented with subacute-onset somnolence, confusion, and left-sided hemiparesis two months after starting intravenous, high-dose methotrexate-based induction chemotherapy for right frontal lobe primary CNS lymphoma.

After hospital admission, he had two episodes of generalized seizures. An EEG demonstrated generalized slowing. A lumbar puncture was performed, but CSF studies demonstrated no abnormalities. Magnetic resonance imaging was repeated (images shown below).

Imaging Fig. 71.1 Axial FLAIR image through the level of the centrum semiovale.

(A)

(B)

Fig. 71.3 Axial FLAIR image through the level of the centrum semiovale (before starting chemotherapy treatment).

Fig. 71.2 (A) Axial DWI and (B) ADC map through the level of the centrum semiovale.

Advanced Neuroradiology Cases, ed. Amaral et al. Published by Cambridge University Press. © Cambridge University Press 2016.

317

Part V. Metabolic Diseases Involving CNS: Case 71

Methotrexate-Induced Subacute Encephalopathy Primary Diagnosis Methotrexate-induced subacute encephalopathy

Differential Diagnoses Diffuse white matter small vessel ischemic disease Posterior reversible encephalopathy syndrome (PRES) Subacute infarction

Imaging Findings Fig. 71.1: Axial FLAIR image through the level of the centrum semiovale demonstrated bilateral, almost symmetric diffuse confluent areas of FLAIR hyperintensity involving the centrum semiovale. Note the right frontal intra- and extra-axial FLAIR abnormality due to the frontal region primary CNS lymphoma. Fig. 71.2: (A) Axial DWI and (B) ADC map through the same level demonstrated areas of subtle diffusion restriction in the areas of FLAIR abnormality, as evidenced by increased signal on DWI sequence associated with low ADC value. Fig. 71.3: Axial FLAIR image through the same level before starting chemotherapy did not demonstrate any abnormal FLAIR signal in bilateral centrum semiovale. A large, right frontal lobe lymphoma was noted.

Discussion The patient, undergoing high-dose methotrexate (MTX)-based chemotherapy, developed subacute-onset neurologic symptoms with development of new, bilateral symmetric deep white matter FLAIR signal abnormality and diffusion restriction on imaging studies. In this clinical context, these imaging abnormalities are consistent with subacute MTX-induced encephalopathy. In isolation, FLAIR findings are suggestive of non-specific small vessel ischemic disease, a common imaging finding in elderly patients, particularly with a history of hypertension and chronic renal disease. However, in ischemic disease, abnormal FLAIR signal develops very slowly over years, as compared to developing over a few weeks, as in this patient. In addition, diffusion facilitation is present in the deep cerebral white matter in patients with small vessel ischemic disease, not diffusion restriction. Posterior reversible encephalopathy syndrome is a known complication of MTX therapy. However, imaging findings in this case are not typical for PRES – the typical imaging findings of PRES include patchy areas of posteriorpredominant cortical/subcortical FLAIR abnormality, usually without any diffusion restriction. Abnormal FLAIR signal can be seen in the parietal and frontal lobes as well, but usually the cortical/subcortical white matters are involved instead of deep cerebral white matter (centrum semiovale). Although the imaging features are suggestive of subacute infarction, the imaging abnormality is diffuse, rather than focal. Methotrexate is a dihydrofolate reductase inhibitor and is a commonly used chemotherapy in many different types of cancer, including lymphoma, both in a standard dose and as

318

high dose with leucovorin rescue. It is usually administered intravenously, although intrathecal administration is used to treat leptomeningeal disease and in children, it is used prophylactically to treat hematologic malignancies. Methotrexate has several neurotoxicities that can develop following systemic and intrathecal MTX administration. Acute or subacute encephalopathy and leukoencephalopathy are common complications. Transverse myelitis has also been described as a complication of MTX treatment. The mechanism of MTX neurotoxicity is poorly understood. High CSF levels of MTX may produce direct acute/subacute toxicities. Other causes may be associated with the release of adenosine, resulting in the dilation of cerebral blood vessels and modified release of both pre- and postsynaptic neurotransmitters. These other mechanisms include altered homeostasis of homocysteine and folate metabolism, as well as single nucleotide polymorphism for the methylenetetrahydrofolate reductase (MTHFR) gene. Acute or subacute encephalopathy is most commonly seen in patients treated with high-dose MTX and is characterized by somnolence, confusion, seizure, and focal neurologic deficit of varying severity. Usually this is reversible, resolves spontaneously without any sequelae, and retreatment with MTX is often an option. Symptoms may develop 2–14 days after starting therapy and can last up to three days. Imaging studies may be normal. Cortical/subcortical involvement is common and is described as PRES. Leukoencephalopathy is another common MTX neurotoxicity and can be seen during any phase of MTX treatment. Leukoencephalopathy can be of two types, acute or subacute, with different presentations. Leukoencephalopathy presents similar to PRES but the abnormalities are limited to deep white matter, rather than cortical/subcortical areas. Known imaging findings include areas of abnormal T2 signal limited to the white matter. Unlike PRES, areas of abnormal FLAIR signal demonstrate diffusion. White matter involvement can be focal or diffuse. Focal abnormalities are associated with focal neurologic symptoms. There is usually no enhancement. Unlike in infarction, there is no change on perfusion imaging. Analysis of CSF is usually normal. Chronic leukoencephalopathy usually develops slowly and presents with gradual cognitive decline. In children, learning disabilities present, sometimes in association with seizures, ataxia, and focal neurologic deficit. In many patients, symptoms may stabilize after discontinuation of MTX, but it may have a progressive course in others. Typically, MTX-induced leukoencephalopathy appears as diffuse deep cerebral white matter FLAIR hyperintensity. Diffusion imaging is usually normal but may demonstrate facilitated diffusion, rather than diffusion restriction. No abnormality is noted on perfusion imaging. These findings may be seen in addition to brain atrophy. A rare form of disseminated necrotizing leukoencephalopathy has been described in association with MTX therapy with a rapidly fatal course.

Part V. Metabolic Diseases Involving CNS: Case 71

Principles for treatment of acute/subacute CNS toxicity of MTX are either rapid removal of CSF MTX levels mechanically or with intrathecal administration of glucarpidase in association with systemic leucovorin therapy.

Key Points  Acute/subacute focal neurologic symptoms in patients being treated with systemic or intrathecal MTX is a red flag for MTX-induced encephalopathy and can manifest either as PRES or as acute/subacute leukoencephalopathy.  Acute/subacute leukoencephalopathy can be confidently diagnosed with T2 hyperintensity and diffusion restriction limited to deep white matter.  Methotrexate-induced chronic leukoencephalopathy is a chronic neurotoxicity and manifested as gradually worsening cognitive decline, and learning abilities

associated with diffuse deep cerebral white matter T2 hyperintensity and brain atrophy.

Suggested Reading Haykin ME, Gorman M, van Hoff J, Fulbright RK, Baehring JM. Diffusion-weighted MRI correlates of subacute methotrexaterelated neurotoxicity. J Neurooncol 2006; 76(2): 153–7. Rollins N, Winick N, Bash R, Booth T. Acute methotrexate neurotoxicity: findings on diffusion-weighted imaging and correlation with clinical outcome. AJNR Am J Neuroradiol 2004; 25(10): 1688–95. Sioka C, Kyritsis AP. Central and peripheral nervous system toxicity of common chemotherapeutic agents. Cancer Chemother Pharmacol 2009; 63(5): 761–7. Ziereisen F, Dan B, Azzi N, et al. Reversible acute methotrexate leukoencephalopathy: atypical brain MR imaging features. Pediatr Radiol 2006; 36(3): 205–12.

319

CASE

Part V

72

Metabolic Diseases Involving Central Nervous System Prasad B. Hanagandi, Jeffrey Chankowsky, Raquel del Carpio-O’Donovan

Clinical Presentation A 16-year-old-adolescent woman with confirmed history of inflammatory bowel disease presented with subacute onset of dysarthria, ataxia, and visual disturbances. Based on her clinical presentation, suspected diagnoses included demyelinating disease, infarct secondary to undiagnosed vasculitis, or subclinical meningitis/inflammatory process. The patient was on several broad-spectrum antibiotic medications, including metronidazole (2 g/day), yet reported recent worsening of colitis symptoms. Her hematologic studies revealed normal WBC count; however, SED and C-reactive protein were minimally elevated. Serum vitamin B12 level was normal, but borderline elevation of liver enzymes was noted.

Imaging (A)

(B)

(C)

Fig. 72.1 (A) Axial T2WI, (B) FLAIR, and (C) DWI through the level of lateral ventricles and splenium of corpus callosum.

Advanced Neuroradiology Cases, ed. Amaral et al. Published by Cambridge University Press. © Cambridge University Press 2016.

321

Part V. Metabolic Diseases Involving CNS: Case 72

(B)

(A)

Fig. 72.2 (A) Axial T2WI and (B) DWI through the level of midpons and cerebellar dentate nuclei.

(A)

Fig. 72.3 (A) Axial T2WI and (B) DWI through the level of inferior olivary nuclei.

322

(B)

Part V. Metabolic Diseases Involving CNS: Case 72

(A)

(B)

Fig. 72.4 (A) Axial T2WI and (B) DWI at the level of lateral ventricles and splenium of corpus callosum.

(A)

(B)

Fig. 72.5 (A) Axial T2WI at the level of cerebellar dentate nuclei and midpons and (B) Axial T2WI at the level of inferior olivary nuclei.

323

Part V. Metabolic Diseases Involving CNS: Case 72

Metronidazole-Induced Encephalopathy Primary Diagnosis Metronidazole-induced encephalopathy

Differential Diagnoses Non-alcoholic Wernicke encephalopathy Methylbromide intoxication Maple syrup urine disease (MSUD) Enteroviral encephalitis

Imaging Findings Fig. 72.1: (A) T2WI and (B) FLAIR showed focal hyperintense lesion in the splenium of corpus callosum (arrow). (C) DWI image demonstrated diffusion restriction in the same area. Fig. 72.2: (A) T2WI and (B) DWI demonstrated symmetric T2 hyperintense lesions with diffusion restriction in the bilateral dentate nuclei (arrows) and tegmentum (arrowheads). Fig. 72.3: (A) T2WI demonstrated similar symmetric signal abnormality in the inferior olivary nuclei (arrows) and (B) DWI with restricted diffusion. Fig. 72.4: (A) T2WI and (B) DWI demonstrate reversal of signal abnormality in the splenium of corpus callosum (8-week follow-up). Fig. 72.5: (A) Axial T2WI demonstrates resolution of T2 signal abnormality in (A) bilateral dentate nuclei and tegmentum and (B) inferior olivary nuclei.

Discussion The diagnosis of metronidazole-induced encephalopathy (MIE) is characterized, in a given context of suspected drug toxicity, by a symmetric topographic MR imaging pattern involving the bilateral cerebellar dentate nuclei, dorsal pons, tegmentum, inferior olivary nuclei, and splenium of corpus callosum. Similar clinical presentation and imaging features can be seen in acute Wernicke encephalopathy but it tends to symmetrically involve the mammillary bodies, medial thalami, midbrain, floor of third and fourth ventricles and periaqueductal region. The subsequent reversal of imaging findings after drug cessation confirms the diagnosis, eliminating the other differential diagnoses options. Metronidazole is a synthetic, 5-nitroimidazole antibiotic used for protozoal and anaerobic infections. It is widely used for surgical prophylaxis, to treat inflammatory bowel disease, and to treat non-infectious conditions, such as hepatic encephalopathy. The drug has high CNS penetrability, reaching close to serum level concentration in brain parenchyma. Commonly reported side effects include nausea, vomiting, headache, abdominal discomfort, and metallic taste. Less common adverse effects including peripheral neuropathy, ataxia, and seizures have been reported in the pharmacology literature. Metronidazole-induced encephalopathy is a poorly understood entity with conflicting hypotheses for its cause; hence, the exact mechanism of CNS drug toxicity has not been completely elucidated. The proposed causal mechanisms include Purkinje cell damage, inhibition of protein synthesis and axonal degeneration induced by neuronal RNA binding, or

324

vasogenic edema induced by the generation of semiquinone and nitro anion radicals. DNA binding of intermediate metronidazole compounds and modulation of cerebellar and vestibular nuclei GABA receptors has been found in animal studies. However, data for most of these explanations stems from animal model studies. Metronidazole-induced encephalopathy exhibits a topographic brainstem distribution pattern comprising of symmetric bilateral cerebellar dentate nuclei, midbrain (tectum, red nucleus, tegmentum, and periaqueductal gray matter), dorsal pons (vestibular, superior olivary, and abducens nuclei), and dorsal medulla. Other areas that can be involved are splenium of corpus callosum, internal capsule, and anterior commissure. Bilateral inferior olivary nuclei changes, as seen in our patient, are uncommon. This may be the result of drug toxicity or a manifestation of hypertrophic olivary degeneration due to interruption of the Guillain-Mollaret triangle. On MR imaging, the signal changes in MIE are best appreciated on T2WI and FLAIR sequences. The lesions are hyperintense on DWI and if based on high or low ADC values, can reflect either vasogenic or cytotoxic edema changes. The diffusion restriction changes in the splenium of corpus callosum are attributable to cytotoxic edema. Elsewhere in the brain, the parenchyma, for example, these diffusion changes are due to vasogenic edema. T2WI symmetric lesions are also reported in the periventricular regions of mammillary bodies, medial thalamus, floors of the third ventricle, and tectum; thus, Wernicke encephalopathy is an important differential diagnosis. Etiologies characterized by the presence of a T2 hyperintense signal involving the splenium of corpus callosum vary widely, necessitating careful consideration of differential diagnoses in the context of imaging findings and clinical data. For example, antiepileptic drug withdrawal, demyelinating diseases, a spectrum of viral encephalitides, and multiple toxic and metabolic diseases should be considered in light of a patient’s clinical presentation. Osmotic myelinolysis involves the basis pontis and often shows diffusion restriction. In contrast, MIE has an affinity for the dorsal pons and rarely shows changes on diffusion imaging which are quite specific in topographic distribution. Imaging studies obtained post-metronidazole therapy cessation show complete symptomatic recovery. Resolution of cytotoxic and vasogenic edema has been reported on follow-up MR imaging after six to eight weeks.

Key Points  Metronidazole-induced encephalopathy is a clinicoradiologic diagnosis.  It should be strongly suspected in patients with a history of metronidazole consumption exceeding 2 g/day.  Bilateral symmetric T2WI and FLAIR hyperintensities with diffusion restriction involving the cerebellar dentate nuclei, dorsal medulla, pons, inferior olivary nuclei, and splenium of corpus callosum that show reversal on follow-up imaging after drug cessation are the characteristic features of this entity.

Part V. Metabolic Diseases Involving CNS: Case 72

Suggested Reading Bottenberg MM, Hegge KA, Eastman DK, Kumar R. Metronidazole-induced encephalopathy: a case report and review of the literature. J Clin Pharmacol 2011; 51(1): 112–16. Hammami N, Drissi C, Sebai R, et al. Reversible metronidazoleinduced encephalopathy. J Neuroradiol 2007; 34(2): 133–6. Heaney CJ, Campeau NG, Lindell EP. MR imaging and diffusion-weighted imaging changes in metronidazole

(Flagyl)-induced cerebellar toxicity. Am J Neuroradiol 2003; 24(8): 1615–17. Kim E, Na DG, Kim EY, et al. MR imaging of metronidazole-induced encephalopathy: lesion distribution and diffusion-weighted imaging findings. AJNR Am J Neuroradiol 2007; 28(9): 1652–8. Seok JI, Yi H, Song YM, Lee WY. Metronidazole-induced encephalopathy, and inferior olivary hypertrophy: lesion analysis with diffusion-weighted imaging and apparent diffusion coefficient maps. Arch Neurol 2003; 60(12): 1796–800.

325

CASE

Part V

73

Metabolic Diseases Involving Central Nervous System Prasad B. Hanagandi, Jeffrey Chankowsky, Raquel del Carpio-O’Donovan

Clinical Presentation A 34-year-old woman presented with a history of cognitive dysfunction that progressed from abnormal behavior to an altered level of consciousness over a period of five to six days. The patient’s medical history included seizures and previous use of antiepileptic medication. At admission, it was noted that

antiepileptic drug use had discontinued approximately three weeks prior to symptom onset. No history of fever or neck rigidity was noted on clinical examination. Analysis of CSF for clinically suspected meningitis/encephalitis was negative. Hematologic studies revealed normal WBC count, SED, C-reactive protein, blood glucose level, and serum electrolytes.

Imaging Fig. 73.1 Midsagittal T1WI showing the corpus callosum.

(A)

(B)

Fig. 73.2 (A) Axial T2WI and (B) FLAIR images at the level of lateral ventricles and splenium of corpus callosum.

Advanced Neuroradiology Cases, ed. Amaral et al. Published by Cambridge University Press. © Cambridge University Press 2016.

327

Part V. Metabolic Diseases Involving CNS: Case 73

(A)

(B)

Fig. 73.3 (A) Axial diffusion and (B) ADC images at the level of lateral ventricles and splenium of corpus callosum.

Fig. 73.4 Axial T2W image at the level of lateral ventricles and splenium of corpus callosum.

328

Part V. Metabolic Diseases Involving CNS: Case 73 Fig. 73.5 Axial diffusion image at the level of lateral ventricles and splenium of corpus callosum.

329

Part V. Metabolic Diseases Involving CNS: Case 73

Transient Splenial Lesion Primary Diagnosis Transient splenial lesion

Differential Diagnoses Influenza-associated encephalitis/encephalopathy Infectious diseases: influenza, rotavirus, herpes, measles, mumps, varicella, and cerebral malaria Hypoglycemia, hypernatremia Demyelinating diseases: diffuse axonal injury and Marchiafava-Bignami disease Drug toxicities: cisplatinum, 5-fluorouracil, carboplatin, olanzapine, citalopram, and metronidazole Methylbromide poisoning Adrenoleukodystrophy High altitude cerebral edema

Imaging Findings Fig. 73.1: Sagittal T1WI showed a well-defined, hypointense, focal lesion in the splenium of corpus callosum (arrow). Fig. 73.2: (A) T2WI and (B) FLAIR images showed presence of a hyperintense lesion (arrows). Fig. 73.3: (A) Axial DWI showed diffusion restriction and (B) ADC showed low ADC values (arrows). Fig. 73.4: Axial T2WI shows complete resolution of the splenial lesion (6-week follow-up). Fig. 73.5: Axial DWI image also shows resolution of the lesion.

Discussion In the given context of antiepileptic drug (AED) withdrawal as noted in the patient’s history, the identification of a focal splenial lesion with reversible signal changes on follow-up MRI scan that correlate with improved clinical condition of the patient confirm a diagnosis of antiepileptic drug withdrawal–reversible/transient splenial lesion. Reversible splenial lesion syndrome (RESLES) or transient splenial lesion is an entity that has been extensively illustrated in the recent neuroradiology literature. Of the several etiologies described in the wide list of differential diagnoses, a few conditions merit special consideration, especially AED withdrawal, infectious disease, or metabolic conditions. The other entities mentioned in the list of differential diagnoses can have similar imaging features but are out of context, based on the presenting history. Cerebrospinal fluid analysis excludes most of the infectious and inflammatory conditions. Metabolic and other drug-induced conditions, and the toxic conditions are excluded by hematologic evaluation. Several causal mechanisms for RESLES have been proposed based on its etiology. They vary from excitotoxic intramyelinic edema, due to extracellular glutamate by increased neuronal activity, to cytotoxic edema, due to energy failure. In cases of

330

viral infections, a toxin-mediated immune response or cytokine-induced axonal damage has been hypothesized. A few authors have also speculated that arginine-vasopressin or rapid correction of hyponatremia caused by AED withdrawal may play a role in RESLES. The corpus callosum consists of several distinct interhemispheric fibers with an increased splenic density; however, the resulting impact of RESLES on the corpus callosum is unclear. However, some studies attribute the probable cause to a lack of adrenergic tone, which makes the corpus callosum vulnerable to hypoxic vasodilatation and autoregulation failure (leading to overperfusion). Increased frequency of seizure activity or the type of epilepsy (partial versus generalized) has no relevance or correlation with the development of the splenial lesion. Use of approximately 14 AED drugs are associated with RESLES, including those most commonly prescribed (carbamazepine, phenytoin, and lamotrigine). Patients often present with encephalopathy or non-specific symptoms. Magnetic resonance imaging demonstrates a nonenhancing round- to ovoid-shaped T2-FLAIR hyperintense lesion with diffusion restriction and low ADC values. On imaging, the lesion has been identified as early as 24 hours to 1 week after AED withdrawal. The overall incidence of reversible splenial lesion has not been reported; however, it occurs in 0.7% of patients undergoing presurgical evaluation that require AED withdrawal to provoke seizures and assess ictal EEG discharges. The signal changes usually resolve in a few months with complete reversal on follow-up MR imaging. The diversity of etiologies and multifactorial hypothesis make this condition a clinicoradiologic entity. If a history of AED withdrawal is absent, imaging features must be interpreted in the relevant clinical context and laboratory correlations to narrow down the possible list of differential diagnoses.

Key Points  Reversible splenial lesion syndrome (RESLES) is a clinicoradiologic entity with a wide list of differential diagnoses and non-specific imaging findings.  Antiepileptic drug withdrawal is one of the recognized conditions presenting with this entity.

Suggested Reading Bulakbasi N, Kocaoglu M, Tayfun C, Ucoz T. Transient splenial lesion of the corpus callosum in clinically mild influenza-associated encephalitis/encephalopathy. AJNR Am J Neuroradiol 2006; 27(9): 1983–6. Garcia-Monco JC, Cortina IE, Ferreira E, et al. Reversible splenial lesion syndrome (RESLES): what’s in a name? J Neuroimaging 2011; 21(2): e1–14. Garcia-Monco JC, Martinez A, Brochado AP, et al. Isolated and reversible lesions of the corpus callosum: a distinct entity. J Neuroimaging 2010; 20(1): 1–2.

Part V. Metabolic Diseases Involving CNS: Case 73 Hantson P, Hernalsteen D, Cosnard G. Reversible splenial lesion syndrome in cerebral malaria. J Neuroradiol 2010; 37(4): 243–6.

Maeda M, Tsukahara H, Terada H, et al. Reversible splenial lesion with restricted diffusion in a wide spectrum of diseases and conditions. J Neuroradiol 2006; 33(4): 229–36.

Lin YW, Yu CY. Reversible focal splenium lesion–MRS study of a different etiology. Acta Neurol Taiwan 2009; 18(3): 203–6.

Takanashi J, Barkovich A, Shiihara T, et al. Widening spectrum of a reversible splenial lesion with transiently reduced diffusion. AJNR Am J Neuroradiol 2006; 27(4): 836–8.

331

CASE

Part V

74

Metabolic Diseases Involving Central Nervous System Ricardo Tavares Daher, Lázaro Luís Faria do Amaral

Clinical Presentation A 28-year-old man presented to our emergency department with a four-day history of headache, ocular pain, and paroxysms of left visual phenomena (described as intensely shiny). Visual disturbance progressed and he developed hemianopia and palpebral myoclonus. Magnetic resonance imaging of his brain was performed (Figs. 74.1 and 74.2). Two months later, following symptom improvement, follow-up MRI was performed (Fig. 74.4) One week later, he developed right hemianopia and headache (Fig. 74.5). Complementary laboratory studies demonstrated increased lactic acid and creatine phosphokinase levels.

Imaging (A) (B)

Fig. 74.1 (A–B) Axial FLAIR through the level of the third and lateral ventricles.

Advanced Neuroradiology Cases, ed. Amaral et al. Published by Cambridge University Press. © Cambridge University Press 2016.

333

Part V. Metabolic Diseases Involving CNS: Case 74

(B) (A)

Fig. 74.2 (A–B) DWI through the level of the third and lateral ventricles.

(B) (A)

Fig. 74.3 (A–B) Axial FLAIR through the level of the third ventricle.

334

Part V. Metabolic Diseases Involving CNS: Case 74

(A)

(B)

Fig. 74.4 (A) Axial FLAIR and (B) Axial diffusion through the level of the occipital lobes.

(A)

(B)

Fig. 74.5 (A–B) Axial FLAIR through the level of the occipital lobes.

335

Part V. Metabolic Diseases Involving CNS: Case 74

Mitochondrial Encephalomyopathy, Lactic Acidosis, and Stroke-Like Episodes Syndrome: MELAS Syndrome Primary Diagnosis Mitochondrial encephalomyopathy, lactic acidosis, and stroke-like episodes syndrome: MELAS syndrome

Differential Diagnoses Ischemic stroke Herpes simplex encephalitis Viral infection Vasculitis (Moyamoya disease) Kawasaki disease Kearns-Sayre syndrome Wilson disease Status epilepticus

Imaging Findings Fig. 74.1: (A–B) Axial FLAIR MR images showed signal abnormality involving cortical and subcortical right lingual gyrus. Fig. 74.2: (A–B) Diffusion-weighted MR images demonstrated a tenuous, hyperintense lesion in the right occipital lobe. Fig. 74.3: (A–B) Axial FLAIR demonstrated complete resolution of the signal abnormality involving the left lingual gyrus (1-week follow-up). Fig. 74.4: (A) Axial FLAIR demonstrated new hyperintensity lesion in the left occipital lobe (2-month follow-up). (B) Axial diffusion confirms the hyperintensity lesion in the left occipital lobe. Fig. 74.5: (A–B) Axial FLAIR showing complete resolution of the lesion (1 week later follow-up).

Discussion The combination of mitochondrial encephalomyopathy, lactic acidosis, and stroke-like episodes constitute MELAS syndrome. The constellation of clinical and radiologic findings seen in this patient case is typical of MELAS. MELAS syndrome is a hereditary metabolic disorder and one of the most common multisystem mitochondrial diseases with a special predilection for the nervous system and muscles. Although the age at onset varies between 3 months and 40 years of age, in most cases the initial signs and symptoms occur before adulthood. Clinical symptoms originating from all organ systems have been described, but tissues with highenergy requirements, such as muscle and brain, are particularly vulnerable. In adults, muscle weakness, exercise intolerance, and ophthalmoplegia dominate, together with slowly progressive cognitive dysfunction. In children, the first manifestations usually belong to encephalomyopathies such as growth disturbance and epileptic seizures, with a progressive symptomatology. Learning disabilities, cognitive regression, exercise intolerance, and limb weakness are the most frequent disease manifestations.

336

The mitochondrial abnormality found in MELAS is due to point mutations of mitochondrial DNA. The most common mutation is an A>G transition at the tRNA Leu (UUR) 3243, which causes a defect in the mitochondrial protein synthesis, impairing adenosine triphosphate (ATP) production. Pavlakis et al. described the syndrome in 1984, after observing children who were usually normal at birth and during early infancy, but who were subsequently evaluated with delayed growth, episodic vomiting, seizures, and recurrent cerebral injuries resembling strokes. The stroke-like events occur in almost all patients with MELAS and most frequently give rise to neurologic deficits such as hemiparesis, hemianopia, or cortical blindness. The proposed clinical diagnosis of MELAS is based on three major features: 1) occurrence of stroke-like episodes before 40 years of age; 2) presence of encephalopathy with seizures and/or dementia; and 3) the presence of lactic acidosis, ragged red muscle fibers as well as the presence of additional symptoms such as recurrent headaches and recurrent vomiting. Clinical features that help distinguish MELAS syndrome from typical herpes simplex encephalitis (HSE) or other viral infection include the absence of fever, lack of prominent alteration in consciousness, and a normal CSF cell count. Imaging findings have been reported as multiple strokelike lesions, calcification of the basal ganglia, and diffuse atrophy. On standard MR images, the imaging hallmark of MELAS is the presence of infarct-like, often transient lesions that are not confined to the vascular territories but have a predilection to the posterior part of the cerebral hemispheres. Diffusion-weighted imaging can show variable results; however, during an acute stroke-like episode, ADC maps usually show low values, demonstrating cytotoxic edema. Since ADC values are higher in the non-affected areas than the controls, it can be inferred that MELAS involves the entire brain. Proton MR spectroscopy classically shows an increased lactate peak at 1.3 ppm, better seen at an intermediate echo time when it inverts, which can also be seen over the ventricles. Invasive procedures may help to diagnose this condition including the quantification of the lactic acid in the serum and CSF, presence of ragged red fibers on muscle biopsy, and an increased number of mitochondria in endothelial cells of small cerebral arteries.

Key Points  Muscle and brain symptoms associated with findings of multiple migrating infarct-like lesions not limited to a specific vascular territory, especially in the basal ganglia and posterior part of the cerebral hemisphere, suggest MELAS syndrome.  Magnetic resonance imaging and MR spectroscopy are very useful to approximate the diagnosis and to distinguish between MELAS syndrome and ischemic stroke.

Part V. Metabolic Diseases Involving CNS: Case 74

Suggested Reading Castillo M, Kwock L, Green C. MELAS syndrome: imaging and proton MR spectroscopic findings. AJNR Am J Neuroradiol 1995; 16: 233–9. Goto Y, Nonaka I, Horai S. A mutation in the tRNA Leu (UUR) gene associated with the MELAS subgroup of mitochondrial encephalomyopathies. Nature 1990; 348: 651–3. Kim I, Kim J, Kim W, et al. Mitochondrial myopathy, encephalopathy, lactic acidosis and stroke-like episodes (MELAS) syndrome: CT and MR findings in seven children. AJR Am J Roentgenol 1996; 166: 641–5. Kim JH, Lim MK, Jeon TY, et al. Diffusion and perfusion characteristics of MELAS (mitochondrial myopathy, encephalopathy, lactic acidosis, and stroke-like episode) in thirteen patients. Korean J Radiol 2011; 12(1): 15–24. Liu Z, Liu X, Hui L, et al. The appearance of ADCs in the non-affected areas of the patients with MELAS. Neuroradiology 2011; 53: 227–32.

Liu Z, Zheng D, Wang X, et al. Apparent diffusion coefficients of metabolites in patients with MELAS using diffusion-weighted MR spectroscopy. AJNR Am J Neuroradiol 2011; 32: 898–902. Pavlakis SG, Phillips PC, DiMauro S, DeVivo DC, Rowland LP. Mitochondrial myopathy, encephalopathy, lactic acidosis, and stroke-like episodes: a distinct clinical syndrome. Ann Neurol 1984; 16: 481–8. Valanne L, Ketonen L, Majander A, Suomalainen A, Pihko H. Neuroradiologic findings in children with mitochondrial disorders. AJNR Am J Neuroradiol 1998; 19: 369–77. van der Knaap MS, Valk J. Mitochondrial encephalopathy with lactic acidosis and stroke-like episodes. In: van der Knaap MS, Valk J, eds. Magnetic Resonance of Myelination and Myelin Disorders, 3rd edn. Amsterdam: Springer; 2005: 204–11. Vedolin L. Doenças neurometabólicas hereditárias. In: Rocha AJ, Vedolin L, Mendonça RA, eds. Encéfalo. Colégio Brasileiro de Radiologia e Diagnóstico por Imagem. Rio de Janeiro: Elsevier; 2012: 227–46.

337

CASE

Part V

75

Metabolic Diseases Involving Central Nervous System Anderson B. Belezia, Lázaro Luís Faria do Amaral

Clinical Presentation A 22-year-old man was evaluated for jaundice, mental confusion, visual changes, dysarthria, dystonia, and tremor of the extremities. There was no history of alcohol or drug abuse. Serology tests were negative for hepatitis A, B, and C. Hematologic analysis showed increased serum aminotransferase, and bilirubin. Serum ceruloplasmin level was 5 mg/dl. Corneal abnormalities were noted during slit-lamp examination. Pyramidal function remained intact and there were no sensory disturbances noted.

Imaging (A)

(B)

Fig. 75.1 (A–B) Axial FLAIR MR images through the level of the lateral ventricles.

Advanced Neuroradiology Cases, ed. Amaral et al. Published by Cambridge University Press. © Cambridge University Press 2016.

339

Part V. Metabolic Diseases Involving CNS: Case 75

(A)

(B)

Fig. 75.2 (A–B) Axial MPGR-T2*-weighted MR images through the level of the basal ganglia.

(A)

(B)

Fig. 75.3 (A) Axial T2WI, and (B) Axial T1-weighted MR postgadolinium images through the level of the middle cerebellar peduncles.

340

Part V. Metabolic Diseases Involving CNS: Case 75

(A)

(B)

Fig. 75.4 (A) Diffusion and (B) ADC maps through the level of the middle cerebellar peduncles.

(A)

(B)

Fig. 75.5 (A–B) Axial FLAIR MR images through the level of the midbrain.

341

Part V. Metabolic Diseases Involving CNS: Case 75

Wilson Disease Primary Diagnosis Wilson disease

Differential Diagnoses Methanol encephalopathy Leigh disease Carbon monoxide intoxication Hepatic encephalopathy Iron deposition disorders, including pantothenate kinaseassociated neurodegeneration 2 (PKAN2) and neuroferritinopathy

Imaging Findings Fig. 75.1 (A–B) Axial FLAIR MR images at the level of the deep nuclei showed hyperintensity in the outer rim of the deep gray matter. Fig. 75.2 (A–B) Axial MPGR-T2*-weighted images demonstrated marked hypointensity in both putamen and caudate nuclei. Fig. 75.3 (A) Axial T2-weighted MR image showed hyperintense areas on both median cerebellar peduncles. (B) Axial T1WI after gadolinium injection showed no enhancement. Fig. 75.4: (A) Axial diffusion and (B) ADC map showed no restriction in the pons or middle cerebellar peduncle lesions. Fig. 75.5: (A–B) Axial FLAIR showed presence of signal abnormalities in the pons and midbrain.

Discussion Wilson disease (WD) or hepatolenticular degeneration is a rare autosomal recessive disorder of copper metabolism that affects multiple organs, especially the liver and the CNS. A chromosomal 13 mutation in the ATP7B gene causes the dysfunction or absence of the copper intracellular transport protein, ATP7B. As a result, copper deposition occurs in susceptible tissues. The absence of copper in the ceruloplasmin protein produces an unstable molecule, resulting in low serum copper levels. The presence of altered mental status and coronal abnormalities, in combination with distinctive imaging findings, suggests a diagnosis of WD. Patients with WD usually present in young adulthood (peak age of presentation between 8 and 16 years of age), with dysarthria, dystonia, and tremors that may be followed by rigidity and ataxia. Psychiatric problems may develop later in life. In neurologically symptomatic patients, dysarthria is the most common symptom, followed by tremor, dystonia, seizure, chorea, and psychiatric disturbance. Typically, a Kayser-Fleischer (KF) ring, caused by copper deposition in the cornea, may be seen on slit-lamp ophthalmologic examination. Potential WD diagnosis should be considered in patients with unexplained liver disease, cirrhosis, liver failure, or other neurologic symptoms suggestive of WD. The presentation of neurologic or psychiatric symptoms concurrently with evidence of liver disease is suggestive of WD. The disease rarely manifests clinically before five years of age, unless there is a

342

secondary problem exacerbating hepatic disease. Hepatic presentations predominate in the first two decades of life, while neurologic or psychiatric symptoms typically appear in the second or third decade of life. A combination of a low serum ceruloplasmin and the presence of KF rings on slit-lamp exam confirm the diagnosis of WD. In individuals without KF rings, a low ceruloplasmin and elevated hepatic copper content (with exclusion of chronic cholestatic disorders) suggests a diagnosis of WD. The clinical history and the image abnormalities can exclude most of the major differential diagnoses described above. Methanol encephalopathy causes hemorrhagic necrosis of the putamen characterized by marked hypointensity on T2*-weighted MR images and does not usually involve the brainstem like WD. Patients with carbon monoxide intoxication usually have a prior history of suicide attempt or smoke exposure from a fire accident. On neuroimaging there is involvement of both globus pallidi characterized by hyperintensity with an external rim of hypointensity on T2-weighted MR images probably due to hemosiderin deposition, the hemispheric white matter can also be affected, and like methanol encephalopathy, usually there is no involvement of the brainstem. The symptoms of Leigh disease typically become evident before two years of age and include psychomotor delay or regression, ataxia, ophthalmoplegia, dystonia, abnormal respiratory rhythm, and cranial nerve palsy. The brainstem, periaqueductal gray matter, midbrain, corpus striatum, and thalami are usually affected, characterized by hyperintensity on T2-weighted images, with restricted diffusion in the acute setting. Chronic hepatic encephalopathy should also be considered in the differential diagnosis. Neuroimaging findings consist of T1-weighted hyperintensity on globus pallidus due to manganese deposition. In the acute setting, there is hyperintensity on T2-weighted images involving the cerebral cortex, preserving the perirolandic region. Wilson disease can cause hepatic disease, making a differential diagnosis difficult without clinical data. The eye-of-the-tiger sign is relatively specific to PKAN2; however, it can also be found in patients with neuroferritinopathy, but is very uncommon in patients with WD. Despite the ubiquitous presence of toxic copper within the brain, pathologic findings are primarily limited to the basal ganglia, thalamus, and brainstem. These abnormalities include atrophy, spongy softening, cavitation, a neuronal reduction, increased cellularity, and the presence of Opalski cells (an altered glial cell, originated from degenerating astrocytes in WD). These changes presumably result from an increased amount of extracellular copper, which causes oxidative stress, resulting in cell destruction, chronic ischemia, vasculopathy, or demyelination. Neuroimaging studies may show evidence of diffuse or focal atrophy of the cerebrum, brainstem, and/or cerebellum. Cerebral MRI signal abnormalities are typically seen in deep gray matter sites such as the putamen, caudate nuclei, and

Part V. Metabolic Diseases Involving CNS: Case 75

thalami (more common and usually bilateral and symmetric), central white matter (usually asymmetric), and other sites such as the pons, medulla, cerebellum, and cerebral cortex. Both pyramidal and extrapyramidal white matter tracts may be involved in WD. These abnormalities are characterized on CT as cerebral hypodense lesions, and may show variable signal on MRI. TI hypointense-T2 hyperintense lesional imaging is presumably due to gliosis, edema, and variable necrosis (with or without cavitation), and may be hypodense on CT studies as described above. Hypointense lesions on T2 images are either due to the paramagnetic effects of copper deposition, associated iron deposition, or other agents resulting in preferential T2-proton-relaxation enhancement. On T1 sequences, cerebral lesions are hypointense, if there is no significant hepatic disease. In patients with hepatic failure due to WD, T1 hyperintensity is noted in the basal ganglia (see Part V: Cases 66 and 68). Magnetic resonance diffusion may be abnormal in recently developed lesions, characterized by hyperintensity and low ADC values in affected sites. Since excess copper causes cell injury (leading to inflammation and cell death) it is likely that this finding represents cell swelling associated with inflammation, hence the diffusion restriction. Wilson disease has two specific MR signaling signs. The first is the face of the giant panda sign at the level of the midbrain that corresponds to high signal in the tegmentum, normal signal in the red nuclei and lateral portion of the pars reticulata of the substantia nigra, and hypointensity of the superior colliculus on T2 images. Dorsal pontine signal abnormalities resemble the face of a panda cub. The second sign is known as the bright claustral sign, and correlates with focal hyperintense signal of the claustrum on long TR sequences. Although specific, these findings are only visible in a minority of patients. In WD, there is a significant correlation between clinical course and follow-up MR imaging, thus MR imaging studies may be useful for documenting the treatment effect. Most WD patients without neurologic symptoms have normal MR scans; in contrast, most patients with neurologic symptoms have abnormal imaging studies. Clinical findings are correlative with lesion location: 1) patients with bradykinesia and dystonia

may have putaminal lesions, 2) dysarthria correlates with both caudate and putaminal lesions, and 3) distractibility of gaze fixation correlates with frontal lobe involvement. Finally, localization and severity of brainstem-evoked potential abnormalities correspond closely to MRI abnormalities of the midbrain and pons. Magnetic resonance images may demonstrate a significant decrease in the NAA/Cr ratio and an increase in the mI/Cr ratio in basal ganglia. The findings could possibly be assigned to neuronal loss (in the three studied areas), to gliosis, and to iron and/or copper deposition in the basal ganglia. Treatments for WD include administration of zinc, which acts by blocking intestinal copper absorption, and chelating agents, which remove copper from the body, increasing urinary copper excretion.

Key Points  There are effective treatments available for WD; thus, familiarity with its clinical and imaging findings for diagnostic, treatment, and follow-up purposes is paramount.  Panda sign in the midbrain is rare, but very typical of WD.  Key imaging findings of WD include volume loss and a hyperintense rim in the putamen.

Suggested Reading Kim TJ, Kim IO, Kim WS, et al. MR imaging of the brain in Wilson disease of childhood: findings before and after treatment with clinical correlation. AJNR Am J Neuroradiol 2006; 27(6): 1373–8. Lucato LT, Otaduy MCG, Barbosa ER, et al. Proton MR spectroscopy in Wilson disease: analysis of 36 cases. AJNR Am J Neuroradiol 2005; 26(5): 1066–71. Sener RN. Diffusion MR imaging changes associated with Wilson disease. AJNR Am J Neuroradiol 2003; 24(5): 965–7. Shivakumar R, Thomas RSV. Teaching NeuroImages: face of the giant panda and her cub: MRI correlates of Wilson disease. Neurology 2009; 72(11): 613–21. Sinha S, Taly, AB, Ravishankar S, et al. Wilson’s disease: cranial MRI observations and clinical correlation. J Neuroradiol 2006; 48(9): 613–21.

343

CASE

Part V

76

Metabolic Diseases Involving Central Nervous System Renato Hoffmann Nunes, Lázaro Luís Faria do Amaral

Clinical Presentation A 52-year-old man presented with a recent history of evaluation for hypogonadotropic hypogonadism. He reported a sixmonth history of worsening fatigue and diminished libido. Physical examination revealed a fatigued man without evidence of jaundice. Neurologic examination was unremarkable. He did not have any children or history of previous blood transfusion. Hematologic studies revealed elevated serum ferritin level (2559 μg/l: normal 25–400 μg/l). Anterior pituitary and liver function studies were otherwise normal.

Imaging (A)

(B)

Fig. 76.1 (A) Sagittal T1WI and (B) Sagittal T2WI through the midline.

Advanced Neuroradiology Cases, ed. Amaral et al. Published by Cambridge University Press. © Cambridge University Press 2016.

345

Part V. Metabolic Diseases Involving CNS: Case 76

(A)

(B)

(C)

(D)

Fig. 76.2 (A–D) Coronal DP and T2WI SE – TE = 60, 80, 120, and 160 ms – through the level of the pituitary gland.

346

Part V. Metabolic Diseases Involving CNS: Case 76

(A)

(B)

Fig. 76.3 (A) Axial MPGR-T2* and (B) Coronal T1WI FSE postcontrast through the level of the lateral ventricles and pituitary gland.

(A)

(B)

Fig. 76.4 (A–B) Abdominal imaging of the liver. Axial T1WI in and out of phase.

347

Part V. Metabolic Diseases Involving CNS: Case 76

Hereditary Hemochromatosis Primary Diagnosis Hereditary hemochromatosis

Differential Diagnoses Hemochromatosis Calcification Melanoma Pituitary hemorrhage Flow voids (aneurysm) Rathke cleft cysts Craniopharyngioma

Imaging Findings Fig. 76.1 (A) Sagittal T1WI showed a spontaneous hyperintense signal in the topography of the pituitary gland. (B) Sagittal T2WI showed a marked hypointense signal in this topography. Fig. 76.2: (A–D) Coronal DP and T2WI SE, with TE = 60, 80, 120, and 160 ms, demonstrated progressive hypointense signal of the pituitary gland. Fig. 76.3 (A): Axial MPGR-T2* image showed marked hypointense signal in the choroid plexus. (B) Coronal T1WI FSE postgadolinium enhancement of the pituitary gland. Fig. 76.4: (A–B) Upper abdomen MR images showing marked hypointense signal in the liver in the sequences. Axial T1WI in and out of phase.

Discussion Although hemochromatosis is the only clinical entity associated with both decreased pituitary T2WI signal intensity and serum ferritin values greater than 1000 ng/ml (1000 mg/l), this diagnosis can also be made by exclusion. Other causes of T2 shortening such as primary melanoma are unlikely, because it is exceedingly rare and usually associated with mass effect. An aneurysm is less consistent radiographically, given the lack of signal (abnormal flow void) or structural abnormalities observed in the internal carotid artery. A Rathke cleft cyst or other mass lesion associated with pituitary calcification (craniopharyngioma, chordoma, or pituitary stone) would likely manifest as a focal lesion or a mass effect on adjacent structures, none of which was observed in this patient. Hemochromatosis is a pathologic state of intracellular iron accumulation in parenchymal tissues. Primary (hereditary) hemochromatosis results from a genetic defect in iron transport, while secondary hemochromatosis results from a variety of acquired etiologies (frequent blood transfusions, chronic hemolytic anemia, or excessive dietary iron intake). Clinical manifestations include fatigue, bronzing of the skin, cirrhosis, cardiomyopathy, diabetes mellitus, and hypogonadism. Hemochromatosis pathology is based on iron overload resulting from a hereditary/primary cause or from a metabolic/hematologic disorder. Hemochromatosis commonly affects the liver, heart, and endocrine glands. In the brain, iron

348

deposition is typically seen at sites outside the blood-brain barrier, which include the pituitary gland, choroid plexus, pineal gland, and area postrema. Hypogonadotropic hypogonadism can occur early in hereditary hemochromatosis owing to the selective expression of the transferrin receptor by pituitary gonadotrophs, leading to accumulation of intracellular ferritin early in the natural history of the disease. The reduced signal intensity of the liver on images is caused by the paramagnetic effect of intracellular iron deposits (ferritin and hemosiderin), as occurs in hemochromatosis. This increased iron load results in increased susceptibility. This can be demonstrated on T2 FSE-weighted imaging (Fig. 76.1B), especially in spin echo T2 with progressive increasing echo times (TE) (Fig. 76.2). This feature is more evident on gradient echo or susceptibility-weighted images. The inherent dependence of gradient echo sequences on the effective transverse relaxation time, compared with spin echo sequences, induces a reduction in signal intensity with iron tissue storage, because magnetic field inhomogeneities are not erased with a 180° pulse. In addition, the typical distribution of increased susceptibility involving the following structures, the pituitary gland, choroid plexus, pineal gland, and area postrema, should strongly suggest iron overload. Upper abdomen MRI (Fig. 76.4 A and B) demonstrated findings consistent with hereditary hemochromatosis. The MR signal from the pancreas may help differentiate between primary and secondary hemochromatosis, because the first usually affects the liver, adrenals, and the pancreas, while the second commonly affects the reticuloendothelial system. However, it is well known that when the amount of iron in the blood system is higher than the capacity that can be stored by the reticuloendothelial system, such as in advanced stages of hemochromatosis or in transfusional siderosis, the pancreas is also involved. Given the patient’s clinical presentation, laboratory values, and imaging findings, hemosiderin deposition in the anterior pituitary gland secondary to hemochromatosis is the most likely cause of the hypogonadism observed in this patient.

Key Points  Possibility of hereditary hemochromatosis should be strongly considered in patients with diffuse low signal in the pituitary gland on T2WI as well as gradient echo sequences, particularly if associated with the right clinical presentation.  Associated low signal in the choroid plexus and pineal glands on the same sequences further suggest hereditary hemochromatosis.

Suggested Reading Bonkovsky HL, Rubin RB, Cable EE, et al. Hepatic iron concentration: noninvasive estimation by means of MR imaging techniques. Radiology 1999; 212(1): 227–34.

Part V. Metabolic Diseases Involving CNS: Case 76 Caruso RD, Rosenbaum AE, Sherry RG, et al. Pituitary gland. Variable signal intensities on MRI. A pictorial essay. Clin Imaging 1998; 22(5): 327–32. Fujisawa I, Morikawa M, Nakano Y, Konishi J. Hemochromatosis of the pituitary gland: MR imaging. Radiology 1988; 168(1): 213–14. Pietrangelo A. Hereditary hemochromatosis: pathogenesis, diagnosis, and treatment. Gastroenterology 2010; 139(2): 393–408, 408.e1-2. Rodriguez y Baena R, Gaetani P, Danova M, Bosi F, Zappoli F. Primary solitary intracranial melanoma: case report and review of the literature. Surg Neurol 1992; 38(1): 26–37. Siegelman ES, Mitchell DG, Semelka RC. Abdominal iron deposition: metabolism, MR findings, and clinical importance. Radiology 1996; 199(1): 13–22.

Sondag MJ, Wattamwar AS, Aleppo G, Nemeth AJ. Case 179: Hereditary hemochromatosis. Radiology 2012; 262(3):1037–41. Sparacia G, Banco A, Midiri M, Iaia A. MR imaging technique for the diagnosis of pituitary iron overload in patients with transfusiondependent beta-thalassemia major. AJNR Am J Neuroradiol 1998; 19(10):1905–7. Wahid S, Ball S. The pituitary gland and hereditary haemochromatosis. Lancet 2001; 357(9250): 115. Warakaulle DR, Anslow P. Differential diagnosis of intracranial lesions with high signal on T1 or low signal on T2-weighted MRI. Clin Radiol 2003; 58(12): 922–33. Zafar AM, Zuberi L, Khan AH, Ahsan H. Utility of MRI in assessment of pituitary iron overload. J Pak Med Assoc 2007; 57(9): 475–7.

349

CASE

Part V

77

Metabolic Diseases Involving Central Nervous System Prasad B. Hanagandi, Rahul J. Vakharia, Inder Talwar

Clinical Presentation A 24-year-old woman presented to our facility with a 13-year history of juvenile-onset diabetes mellitus and diabetes insipidus. Pubescent-onset diabetes was followed by gradually progressive ataxia and visual and sensory neural hearing loss. Insulin therapy maintained blood sugar levels. A water deprivation and urine osmolality test confirmed the presence of diabetes insipidus. Audiometry results noted marked bilateral sensorineural hearing loss. Funduscopy examination revealed diffuse pallor of optic disks.

Imaging (A) (B)

Fig. 77.1 (A) Coronal T1WI and (B) Coronal fat-saturated T2WI through the orbits.

Advanced Neuroradiology Cases, ed. Amaral et al. Published by Cambridge University Press. © Cambridge University Press 2016.

351

Part V. Metabolic Diseases Involving CNS: Case 77 Fig. 77.2 Midsagittal T1WI through the pituitary gland.

(A)

(B)

Fig. 77.3 (A) Axial T2WI and (B) Axial FLAIR images at the level of lateral ventricles.

352

Part V. Metabolic Diseases Involving CNS: Case 77 Fig. 77.4 Axial T2WI through the pons and middle cerebellar peduncles.

353

Part V. Metabolic Diseases Involving CNS: Case 77

Wolfram Syndrome Primary Diagnosis Wolfram syndrome

Differential Diagnoses Multiple system atrophy Spinocerebellar ataxia type 3 (Machado-Joseph disease) Dentatorubral pallidoluysian atrophy Friedrich ataxia Leber hereditary optic atrophy

Imaging Findings Fig. 77.1: (A) Coronal T1WI image of the orbits showed diffuse thinning of the bilateral optic nerves (black arrows). (B) Corresponding coronal fat-saturated T2WI demonstrated hyperintense signal (white arrows). Fig. 77.2: Sagittal T1WI image shows absence of posterior pituitary bright signal (small arrow) and thinning of optic chiasm (arrow); also noted is brainstem and cerebellar atrophy. Fig. 77.3: (A) Axial T2WI of the brain at the level of lateral ventricles demonstrated symmetric, high signal changes in the bilateral optic radiations (arrows), especially in the peritrigonal white matter. (B) Axial FLAIR demonstrated similar findings. Fig. 77.4: T2WI of the brainstem demonstrated cerebellar and pontine atrophy with T2 hyperintense signal changes in bilateral middle cerebellar peduncles and pons.

Discussion A clinical presentation of diabetes insipidus, diabetes mellitus, optic pallor (suggestive of optic atrophy), and deafness is highly suggestive of Wolfram syndrome (WS). Neuroimaging findings comprising optic atrophy, brainstem and cerebellar atrophy, and absent normal T1 hyperintense signal of the posterior pituitary gland with endocrinology correlation confirm the diagnosis of WS. The brainstem imaging features share a considerable overlap with spinocerebellar ataxia type 3, dentatorubral pallidoluysian atrophy, and Friedrich ataxia. However, these neurodegenerative conditions have a predominant brainstem and cerebellar volume loss, with involvement of the superior cerebellar peduncle. The cruciform hyperintense signal on T2WI involving the middle cerebellar peduncles closely resembles multiple system atrophy, but differs in the age of presentation. Patients with Friedrich ataxia also have diabetes mellitus and visual manifestations; however, cerebellar and brainstem symptoms predominate over other signs. In addition, spinal cord involvement is another differentiating feature. Leber hereditary optic atrophy, a distinct hereditary optic neuropathy with mitochondrial inheritance, needs to be considered. It has predilection for the visual pathway and the spinal cord and is often misdiagnosed as multiple sclerosis. The preserved normal high T1 signal in the posterior pituitary

354

and lack of endocrinology and auditory symptoms are important findings. Wolfram syndrome is a rare genetic disorder first described by Wolfram and Wagener in 1938. It consists of juvenile-onset non-autoimmune diabetes insipidus, diabetes mellitus, optic atrophy, and deafness, and is referred to as DIDMOAD (diabetes insipidus, diabetes mellitus, optic atrophy, and deafness). With an autosomal recessive or mitochondrial pattern of inheritance, the prevalence of WS ranges from 1:100,000 to 1:700,000 and is caused by a WFS1 gene mutation located on chromosome 4p16.1 that encodes the wolframin protein. Wolfram syndrome has a typical clinical presentation consisting of progressive diabetes insipidus, optic atrophy, deafness, urinary tract obstruction, and large atonic bladder with sphincter dyssynergia incontinence. Neuropsychiatric symptoms have also been described with WS in approximately 60% of patients. Diabetes mellitus is often the first disease symptom, usually diagnosed very early in childhood and followed by optic atrophy, around 10–11 years of age. On imaging, brain CT is usually normal but cerebellar and brainstem atrophy changes have been reported. However, MR imaging is more specific and sensitive, with diffuse atrophy of the optic nerves and chiasm on T1 and T2 coronal images. The abnormal high T2 signal in the periventricular locations involving the optic radiation is secondary to significant axonal loss with demyelination and gliosis. Diabetes insipidus is attributable to hypothalamic degeneration with loss of vasopressin-secreting neurons in the supraoptic and paraventricular nuclei resulting in loss of normal high T1 signal in the neurohypophysis. Hypothalamic degeneration and loss of pancreatic islet cells has been hypothesized as the cause of diabetes mellitus. Degeneration of pontine and vestibulocochlear nuclei and inferior colliculi, and ballooning of Purkinje cells has been found on autopsy studies and explains high T2 signal involving the pontocerebellar tracts and cerebellar atrophy, as well as the neurologic symptoms. The imaging features of WS overlap with several neurodegenerative pathologies and hereditary optic neuropathies. Early diagnosis and treatment with hormonal therapy can improve the quality of life. The presence of optic atrophy and signal changes in the visual pathway, loss of normal high T1 signal in the neurohypophysis, and brainstem and cerebellar volume loss, combined with non-autoimmune juvenile diabetes mellitus should help in clinching the diagnosis of this rare neurodegenerative disorder.

Key Points  The diagnosis of WS should be strongly considered in a patient presenting with diabetes insipidus, diabetes mellitus, optic atrophy, and deafness.  Typical imaging findings include unilateral or bilateral optic atrophy, hypothalamic and infundibular atrophy, absence of the normal high T1 signal of posterior pituitary, brainstem and cerebellar atrophy.

Part V. Metabolic Diseases Involving CNS: Case 77

Suggested Reading Barrett TG, Bundey SE. Wolfram (DIDMOAD) syndrome. J Med Genet 1997; 34(10): 838–41. Chaussenot A, Bannwarth S, Rouzier C, et al. Neurologic features and genotype-phenotype correlation in Wolfram syndrome. Ann Neurol 2011; 69(3): 501–8. Ito S, Sakakibara R, Hattori T. Wolfram syndrome presenting marked brain MR imaging abnormalities with few neurologic abnormalities. AJNR Am J Neuroradiol 2007; 28(2): 305–6.

Pakdemirli E, Karabulut N, Bir LS, Sermez Y. Cranial magnetic resonance imaging of Wolfram (DIDMOAD) syndrome. Australas Radiol 2005; 49(2): 189–91. Simsek E, Simsek T, Tekgül S, et al. Wolfram (DIDMOAD) syndrome: a multidisciplinary clinical study in nine Turkish patients and review of the literature. Acta Paediatr 2003; 92(1): 55–61. Waschbisch A, Volbers B, Struffert T, et al. Primary diagnosis of Wolfram syndrome in an adult patient–case report and description of a novel pathogenic mutation. J Neurol Sci 2011; 300(1–2): 191–3.

355

CASE

Part V

78

Metabolic Diseases Involving Central Nervous System Anderson B. Belezia, Lázaro Luís Faria do Amaral

Clinical Presentation A 57-year-old man presented with impaired gait coordination, ataxia, ocular symptoms, nausea, vomiting, and mental confusion. His medical history was notable for previous chronic alcohol abuse.

Imaging Fig. 78.1 Enhancement CT study through the level of the cerebral peduncles.

Advanced Neuroradiology Cases, ed. Amaral et al. Published by Cambridge University Press. © Cambridge University Press 2016.

357

Part V. Metabolic Diseases Involving CNS: Case 78

(B)

(A)

Fig. 78.2 (A–B) Axial FLAIR through the level of the Sylvius aqueduct.

(B)

(A)

Fig. 78.3 (A) Coronal T2 and (B) Axial FLAIR through the level of the third ventricle.

358

Part V. Metabolic Diseases Involving CNS: Case 78

(A)

(B)

Fig. 78.4 (A) Axial T1 postgadolinium and (B) Coronal T1 postgadolinium through the level of the cerebral peduncles and the third ventricle.

(A)

(B)

Fig. 78.5 (A) Axial T2 and (B) Spectroscopy through the level of the Sylvius aqueduct.

359

Part V. Metabolic Diseases Involving CNS: Case 78

Wernicke Encephalopathy Primary Diagnosis Wernicke encephalopathy

Differential Diagnoses Ischemia of the artery of Percheron Deep cerebral vein thrombosis Acute disseminated encephalomyelitis Variant Creutzfeldt-Jakob disease Cytomegalovirus encephalitis

Imaging Findings Fig. 78.1: In the enhanced CT study, there was an evident enhancement of both mammillary bodies. Fig. 78.2: (A) Axial FLAIR sequence at the level of the thalami showed bilateral and symmetric hyperintensity involving the peri-third ventricular medial thalami. (B) Axial FLAIR sequence at the level of the midbrain showed symmetric involvement of the periaqueductal gray matter of the midbrain as well. Fig. 78.3: (A–B) T2-FLAIR sequence demonstrated hyperintensity adjacent to the third ventricle. Fig. 78.4: (A) Axial TI and (B) Coronal T1 postgadolinium MR images demonstrated enhancement of the mammillary bodies. Fig. 78.5: MR spectroscopy of another patient with the same disease showed a lactate peak on the periaqueductal matter.

Discussion In a patient with a history of alcohol abuse presenting with neurologic manifestations including altered mental status and ataxia, imaging findings demonstrating alterations in the median thalami and periventricular regions of the third ventricle are suggestive of Wernicke encephalopathy (WE), although these imaging findings are usually found in association with other typical alterations of WE. However, in rare cases, these alterations represent the only findings of WE and the differential diagnosis should include ischemic events encephalomyelitis, cytomegalovirus encephalitis, primary cerebral lymphoma, variant Creutzfeldt-Jakob disease (vCJD), and several viral infections. Wernicke encephalopathy should be considered for every patient with decreased consciousness that arrives to the emergency room. In this patient, no restricted diffusion was demonstrated on imaging studies that would suggest an ischemic event. In addition, no signs of deep venous sinus thrombosis were noted, excluding a thrombotic event. Patients with acute disseminated encephalomyelitis (ADEM) usually have a prior history of viral infection or recent vaccination and are typically children or adolescents. The gray matter, especially the basal ganglia, is frequently involved; however, unlike typical WE findings, in ADEM there is also involvement of subcortical locations and lesions demonstrate ring-like (usually incomplete) enhancement. Variant Creutzfeldt-Jakob disease is characterized by rapidly progressive dementia, sensory and psychiatric symptoms, with cerebral atrophy, myoclonus followed by death. Basal

360

ganglia, especially the putamen and caudate, are involved. Patients with vCJD demonstrate characteristic hockey stick and pulvinar sign. In cytomegalovirus encephalitis, usually there are T2WI hyperintense lesions in the white matter that are most evident in a periventricular distribution. Enhancement is rare and can be seen in the ependymal surface when ventriculitis is present. Enhancement of the mammillary bodies after gadolinium administration is relatively specific to WE and is not a common finding of the differential diagnoses mentioned above. Wernicke encephalopathy is an acute neurologic emergency due to thiamine (vitamin B1) deficiency. However, the disease may not manifest in all patients with thiamine deficiency because of a genetic susceptibility found in some patients who develop WE. Moreover, blood serum levels of thiamine may not be reduced at the time of clinical onset, and replacement of the vitamin does not always fully reverse the clinical picture, limiting the value of testing thiamine levels under certain circumstances. Wernicke encephalopathy is commonly associated with chronic alcohol abuse, but may be related to hyperemesis gravidarum, anorexia nervosa, prolonged fasting/hunger strike, psychogenic food refusal, use of infant formula that does not contain thiamine, bariatric surgery, gastrectomy, malignant tumors of the gastrointestinal tract, prolonged intravenous feeding, and refeeding (orally or intravenously) after starvation without providing thiamine. The classic triad of WE is ophthalmoplegia, ataxia, and confusion, but is only found in a minority of all patients with WE. Korsakoff syndrome may also present, occurring during the chronic stage of untreated WE. Korsakoff syndrome is characterized by neurologic signs of anterograde amnesia, impaired temporal order recall, confabulation, impaired recognition, loss of insight, and retrograde amnesia. Involvement of the brainstem segments of cranial nerves III, IV, and/or VI causes the ophthalmoplegia. Ataxia is seen secondary to involvement of the superior cerebellar vermis and brainstem segments of the vestibular nerve. Amnestic symptom and confusion are probably associated with involvement of the medial thalami and the mammillary body. The pathologic findings in WE are usually represented by edema, spongy degeneration of the neuropil, neuron sparing, swelling of capillary endothelial cells, and extravasation of red blood cells. Typically, neuropathologic and neuroimaging lesions of WE are located in the peri-midline, in both periventricular and periaqueductal sites, in which there is abundant thiamine-related glucose and oxidative metabolism – most susceptible to thiamine deficiency. Magnetic resonance imaging studies of the most commonly involved WE sites, including the peri-third ventricular medial thalami, the periaqueductal gray matter of the midbrain, the peri-fourth ventricular pons and cerebellar vermis, and the peri-midline mammillary bodies, may permit accurate imaging diagnosis of WE. Signal intensity alterations in the cerebellum, cerebellar vermis, cranial nerve nuclei, red nuclei, dentate nuclei, caudate

Part V. Metabolic Diseases Involving CNS: Case 78

nuclei, splenium, and cerebral cortex represent atypical MRI findings in patients with WE. In the acute stage of WE, involved sites demonstrate hyperintense signal on T2 and FLAIR sequences, variable enhancement after intravenous administration of gadolinium, and variable restricted diffusion. On CT studies, these sites can be hypodense; however, the sensitivity of this finding is very low. Areas with restricted diffusion in the setting of acute WE do not indicate irreversible cytotoxic edema and may be salvageable if treated promptly with thiamine. Brain images of alcoholic patients with WE during the acute phase of the disease can be different from those of non-alcoholic patients. In patients with alcoholism, atrophy of the mammillary bodies, infratentorial regions, supratentorial cortex, and corpus callosum may be found in association with the alterations typical of WE. These abnormalities may be due to previous subclinical episodes. Contrast enhancement, particularly of the mammillary bodies, is more common in alcoholic WE patients than in non-alcoholic WE patients. In follow-up studies, variable imaging findings will be noted, depending on when the diagnosis of WE was established and when thiamine was administered. Patients with complete clinical resolution typically show complete resolution of imaging findings. The other end of the spectrum can also occur, with diffuse atrophy and parenchymal loss in patients with significant clinical abnormalities. In most patients, imaging findings are between these extremities, with variable atrophy, and resolution of abnormal signal, abnormal enhancement, and restricted diffusion. There is very little information about MR spectroscopy in WE. Low levels of NAA/creatine in the thalami during the

acute stage of the disease with complete resolution after treatment can occur. Abnormal lactate peak levels may be found in some patients whose outcomes may range from complete resolution following treatment to chronic parenchyma loss.

Key Points  Wernicke encephalopathy is a potentially reversible condition with high morbidity and mortality rates related to the timing of thiamine treatment.  Since the classic clinical triad is present in only a minority of patients and there is a broad range of MR findings associated with WE, it is important to be familiarized with all aspects of WE.

Suggested Reading Andrade CS, Lucato LT, Martin MGM, et al. Non-alcoholic Wernicke’s encephalopathy: broadening the clinicoradiological spectrum. Br J Radiol 2010; 83: 437–46. Bae SJ, Lee HK, Lee JH, Choi CG, Suh DC. Wernicke’s encephalopathy: atypical manifestation at MR imaging. AJNR Am J Neuroradiol 2001; 22(8): 1480–2. Kornreich L, Bron-Harlev E, Hoffmann C, et al. Thiamine deficiency in infants: MR findings in the brain. AJNR Am J Neuroradiol 2005; 26(7): 1668–74. Zuccoli G, Gallucci M, Capellades J, et al. Wernicke encephalopathy: MR findings at clinical presentation in twenty-six alcoholic and nonalcoholic patients. AJNR Am J Neuroradiol 2007; 28(7): 1328–31. Zuccoli G, Pipitone N. Neuroimaging findings in acute Wernicke’s encephalopathy: review of the literature. AJR Am J Roentgenol 2009; 192(2): 501–8.

361

CASE

Part V

79

Metabolic Diseases Involving Central Nervous System Ricardo Tavares Daher, Asim K. Bag, Lázaro Luís Faria do Amaral

Clinical Presentation A 55-year-old man presented to our facility with progressive spastic paraplegia. Clinical examination revealed tetraparesis (sensitive level at T10), bilateral lower limb proprioceptive decrease, and a reduction in visual acuity. Magnetic resonance imaging of the brain, and the cervical, dorsal, and lumbar spine was performed (see below for imaging findings). Routine hematologic tests and serum B12 levels were submitted.

Imaging (A) (B)

(C)

Fig. 79.1 (A) Axial FLAIR, (B) Axial T2WI, and (C) Postgadolinium axial T1WI through the level of the optic tracts.

Advanced Neuroradiology Cases, ed. Amaral et al. Published by Cambridge University Press. © Cambridge University Press 2016.

363

Part V. Metabolic Diseases Involving CNS: Case 79

(A)

Fig. 79.2 (A–B) Sagittal T2WI of the cervical spine through the midline.

364

(B)

Part V. Metabolic Diseases Involving CNS: Case 79

(A)

(B)

(C)

(D)

Fig. 79.3 (A–D) Axial T2WI through the cervical spinal cord.

365

Part V. Metabolic Diseases Involving CNS: Case 79

Subacute Combined Degeneration with Optic Tract Involvement Primary Diagnosis Subacute combined degeneration with optic tract involvement

Differential Diagnoses Copper deficiency N2O overdose Vacuolar myelopathy in AIDS Vasculitis

Imaging Findings Fig. 79.1: (A) Axial FLAIR, (B) Axial T2WI, and (C) Postgadolinium axial T1WI demonstrated hyperintensity signal of both optic tracts without enhancement after contrast. Fig. 79.2: (A–B) Sagittal T2WI of the cervical spine showed a hyperintense signal in the lateral portion of the cervical spinal cord. Fig. 79.3: (A–D) Axial T2WI of the cervical spine showed hyperintensity signal of the dorsal and lateral columns of the spinal cord.

Discussion Typical clinical presentation combined with typical imaging findings is diagnostic of subacute combined degeneration with optic tract involvement (SCDC). However, copper deficiency and N2O intoxication have identical imaging findings. Copper deficiency is indistinguishable from SCDC, both clinically and on imaging. A high level of suspicion is necessary for accurate diagnosis. History of N2O exposure, when present, is a clue to the right diagnosis. Identical imaging findings are seen in AIDS patients and sometimes in patients with vasculitis, but the clinical profiles are different in these patients. B vitamins act as cofactors in numerous catalytic reactions required for the synthesis and function of neurotransmitters and myelin. Thus, vitamin B12 deficiency may result in injury to the neural tissue. Ludwig Lichtheim described the disease in 1887, recognizing that the pathology of spinal cord disease was associated with pernicious anemia and is different from tabes dorsalis. Subacute combined degeneration (SCD) of the spinal cord is an uncommon cause of myelopathy, but is the most frequent clinical manifestation of vitamin B12 deficiency. Vitamin B12 is contained in essentially all meat and dairy products and is absorbed by the terminal ileum mucosa bounded to intrinsic factor, which is secreted by gastric parietal cells. This condition is usually seen in strict vegetarians, malabsorption syndromes such as bacterial overgrowth of the small bowel, pernicious anemia, regional enteritis, tropical sprue, or surgical procedures such as gastric fundus or ileal resection. Once internalized, intrinsic factor is released and vitamin B12 is transferred to other transport proteins and transported to the portal circulation. The vitamin B12 is mainly taken up by hepatic cells where it is liberated from

366

the transport protein and methylated to methylcobalamin in the cytoplasm. Methylcobalamin plays a key role in the synthesis of methionine, which is further metabolized, to Sadenosylmethionine. S-adenosylmethionine is required in the synthesis of myelin. In vitamin B12 deficiency, this abnormal myelination has predilection for the posterior cord. N2O oxidizes the vitamin B12, makes it inactive, and renders the same effects of vitamin B12 deficiency. Initial symptoms are usually paresthesia in the extremities and usually progress to sensory loss, gait ataxia, and distal weakness. Without treatment, the disease may develop to ataxic paraplegia. Physical examination can demonstrate loss of vibratory and joint position sense, weakness, spasticity, hyperreflexia, and extensor plantar responses. Visual abnormalities are well documented in the setting of vitamin B12 deficiency. Laboratory exams may reveal in the early stage low levels of blood vitamin B12, elevated homocysteine levels, and macrocytic anemia in the evolution, but they are not reliable markers for B12 deficiency. Pathologically, there is a demyelination involving the dorsal and lateral columns (hence, the name combined degeneration) symmetrically, predominantly in the lower cervical and upper thoracic region, with spread in the cranial and caudal directions. This process is seen as hyperintensity on the T2WI and involves the dorsal and lateral columns. On sagittal images, there is a vertically oriented segment of variable length at the posterior aspect of the spinal cord. On cross-section images, bilateral paired areas of T2 hyperintensity are seen as an inverted V or inverted rabbit ears in the expected anatomic location of the dorsal columns. Enhancement can be observed in some cases. Magnetic resonance imaging appearance of the visual pathways in the setting of vitamin B12 deficiency has not been well documented in the literature. Bilateral symmetric T2 hyperintensities involving the optic tracts as shown in this patient is likely due to vitamin B12 deficiency as these abnormalities reverse with treatment. The definitive diagnosis of vitamin B12 deficiency is usually made by a low serum B12 level (< 170 pg/ml by radioimmunoassay), or if the B12 level is borderline, elevated levels of the metabolites homocysteine and methylmalonic acid.

Key Points  Vitamin B12 deficiency should be considered in any patient with progressive sensory symptoms or weakness, mainly in those with elevated mean corpuscular volume with or without anemia.  Characteristic abnormal T2 signal involving the posterior cord, predominantly posterior columns and lateral columns, is the typical imaging manifestation of vitamin B12 deficiency.  Visual problems are well documented in vitamin B12 deficiency that may be associated with abnormal signal in the visual pathway, such as optic tracts in this case.

Part V. Metabolic Diseases Involving CNS: Case 79

Suggested Reading Gupta PK, Gupta RK, Garg RK, et al. DTI correlates of cognition in conventional MRI of normal-appearing brain in patients with clinical features of subacute combined degeneration and biochemically proven vitamin B(12) deficiency. AJNR Am J Neuroradiol 2014; 35(5): 872–7. Madill SA, Riordan-Eva P. Disorders of the anterior visual pathways. J Neurol Neurosurg Psychiatry 2004; 75(Suppl 4): iv12–19. Naidich MJ, Ho SU. Case 87: Subacute combined degeneration. Radiology 2005; 237(1): 101–5.

Pema PJ, Horak HA, Wyatt RH. Myelopathy caused by nitrous oxide toxicity. AJNR Am J Neuroradiol 1998; 19(5): 894–6. Rabhi S, Maaroufi M, Khibri H, et al. Magnetic resonance imaging findings within the posterior and lateral columns of the spinal cord extended from the medulla oblongata to the thoracic spine in a woman with subacute combined degeneration without hematologic disorders: a case report and review of the literature. J Med Case Rep 2011; 5: 166. Ravina B, Loevner LA, Bank W. MR findings in subacute combined degeneration of the spinal cord: a case of reversible cervical myelopathy. AJR Am J Roentgenol 2000; 174(3): 863–5.

367

CASE

Part V

80

Metabolic Diseases Involving Central Nervous System Asim K. Bag

Clinical Presentation A 42-year-old woman, burned on 70% of her body surface, was being treated in the medical intensive care unit of my facility for multi-organ failure. At 2:00 am, her medical condition rapidly became unstable. Blood gas analysis demonstrated multi-electrolyte disturbance. In the process of correcting the electrolyte status, the patient quickly became drowsy followed by rapid-onset seizures. Computed tomography scan of the head was performed and was diagnostically pronounced as normal. An MRI was obtained on the next day (as shown below).

Imaging

Fig. 80.1 Midline sagittal T1WI through the level of the central pons.

Fig. 80.2 Axial FLAIR image through the middle of the pons.

Advanced Neuroradiology Cases, ed. Amaral et al. Published by Cambridge University Press. © Cambridge University Press 2016.

369

Part V. Metabolic Diseases Involving CNS: Case 80

(B)

(A)

Fig. 80.3 (A) Axial DWI through the level of the central pons. (B) ADC map through the same level.

370

Part V. Metabolic Diseases Involving CNS: Case 80

Fig. 80.4 Axial FLAIR image through the basal ganglia.

371

Part V. Metabolic Diseases Involving CNS: Case 80

Osmotic Demyelination Syndrome Primary Diagnosis Osmotic demyelination syndrome

Differential Diagnoses Acute infarction Demyelinating disease Central variant of posterior reversible encephalopathy syndrome (PRES) Brainstem tumor

Imaging Findings Fig. 80.1: Midline sagittal T1WI image demonstrated diffuse area of T1 hypointensity involving the central pons with relatively sparing of the peripheral aspect. Fig. 80.2: Axial FLAIR image through the middle of the pons demonstrated increased area of T2 signal within the central pons with relative sparing of the peripheral pons. Fig. 80.3: (A) Axial DWI and (B) ADC map through the same level demonstrated diffusion restriction at the central pons, in the area of abnormal T2 signal. Fig. 80.4: Axial FLAIR image through the basal ganglia demonstrated abnormally increased FLAIR signal involving bilateral putamen, as well as the head of the caudate nuclei.

Discussion In this patient and clinical setting, the constellation of imaging abnormalities are diagnostic of osmotic demyelination syndrome (ODS). The rapid correction of hyponatremia in an ICU setting, with rapid-onset altered mental status is highly suspicious of ODS. Diffusion restriction and abnormal FLAIR signal at the central pons with sparing of the periphery is a typical imaging appearance associated with ODS. Abnormal FLAIR signal in the basal ganglia further suggests extrapontine involvement and diffusion restriction is conformational of the diagnostic acuity. Although acute demyelinating plaque can have similar signal characteristics, the plaques are more commonly peripheral with variable extension to the central pons and the clinical context is different. Central variant of PRES is due to hypertensive emergency or other predisposing factors (see Part V: Case 62), unlike the predisposing factors causing ODS. The absence of mass effect and the clinical presentation is not suggestive of tumor. Osmotic demyelination syndrome is a rare CNS disorder of unknown prevalence. Although it can involve patients of any age, the most common age of presentation is between 30 and 60 years, with a slight male predominance. The most common predisposing factor for ODS is rapid correction of hyponatremia. The exact mechanism of how the rapid change of natremic status induces demyelination is not known. Other

372

common causes are alcoholism, liver transplantation, anorexia, and following hemodialysis in end-stage renal disease. Patients typically present with rapid-onset confusion and seizures. This may be associated with bulbar or pseudobulbar palsy. Patients may have atypical movement if basal ganglia are involved. They may go through an intermediate improvement of symptoms as the natremic status reaches near to normalcy and then rapidly deteriorates again. With appropriate treatment, all of the symptoms can completely reverse. However, in severe cases, the patient may be in a coma for an extended period, may become quadriparetic, or may even die. If ODS is due to causes other than abnormal hematologic sodium levels, stabilization of the predisposing condition is necessary. In the early phase of ODS, CT scans may appear completely normal. Evaluation of the pons with CT is limited because of artifacts. If there is strong clinical concern, then MRI should always be performed. In the acute stage, diffusion restriction may be the only finding, as other sequences appear normal. Over time, there is gradual development of T1 hypointensity and T2 hyperintensity that typically affects the central pons with sparing of the peripheral pons that may have a bat-wing pattern of involvement due to the sparing of the corticospinal tracts and the transverse pontine fibers. Hemorrhage is not a feature. Extrapontine abnormalities can be seen in up to 50% of cases and typically involves the basal ganglia.

Key Points  Rapid correction of hyponatremia is the most common cause of ODS.  Other predisposing conditions include alcoholism, liver failure, and post-hemodialysis in renal failure patients.  Osmotic demyelination syndrome typically presents with a rapid-onset change in mental status associated with typical T2 signal abnormality involving the central pons with sparing of the periphery. Typically, in the acute stage, the central pons demonstrates diffusion restriction.

Suggested Reading Dervisoglu E, Yegenaga I, Anik Y, Sengul E, Turgut T. Diffusion magnetic resonance imaging may provide prognostic information in osmotic demyelination syndrome: report of a case. Acta Radiol 2006; 47(2): 208–12. Milionis HJ, Liamis GL, Elisaf MS. The hyponatremic patient: a systematic approach to laboratory diagnosis. CMAJ 2002; 166(8): 1056–62. Ruzek KA, Campeau NG, Miller GM. Early diagnosis of central pontine myelinolysis with diffusion-weighted imaging. AJNR Am J Neuroradiol 2004; 25(2): 210–13. Tarhan NC, Agildere AM, Benli US, et al. Osmotic demyelination syndrome in end-stage renal disease after recent hemodialysis: MRI of the brain. AJR Am J Roentgenol 2004; 182(3): 809–16.

CASE

Part V

81

Metabolic Diseases Involving Central Nervous System Kelly Fiorini, Asim K. Bag, Leonardo Furtado Freitas, Lázaro Luís Faria do Amaral

Clinical Presentation A 19-year-old young woman was admitted to our emergency department with signs of gradually progressing dysarthria and difficulty swallowing liquids.

Imaging (A)

(B)

(C)

Fig. 81.1 (A–C) Sagittal FLAIR through the brainstem.

Advanced Neuroradiology Cases, ed. Amaral et al. Published by Cambridge University Press. © Cambridge University Press 2016.

373

Part V. Metabolic Diseases Involving CNS: Case 81

(A) (B)

(C)

Fig. 81.2 (A–C) Axial FLAIR through the brainstem.

374

Part V. Metabolic Diseases Involving CNS: Case 81

(A)

(B)

Fig. 81.3 (A–B) Axial T2WI through the brainstem.

(A)

(B)

Fig. 81.4 (A–B) Axial T1WI postcontrast through the brainstem.

375

Part V. Metabolic Diseases Involving CNS: Case 81

Adult-Onset Alexander Disease Primary Diagnosis Adult-onset Alexander disease

Differential Diagnoses Demyelinating disease Brainstem encephalitis Vasculitis Behçet disease Erdheim-Chester disease

Imaging Findings Fig. 81.1: (A–C) Sagittal FLAIR, and Fig. 81.2: (A–C) Axial FLAIR sequences showed hyperintensity in the medulla and in the cervical spinal cord levels (C1–C2), with extension to the pontine tegument, middle cerebellar peduncles, and cerebellar dentate nucleus. Fig. 81.3: (A–B) Axial T2WI showed signal abnormalities in the cerebellar peduncles and medulla. Fig. 81.4: (A–B) Postcontrast, axial T1WI was characterized by enhancement in the medulla and midbrain.

Discussion Classic clinical presentation, and typical MRI findings including atrophy and abnormal T2 signal of the medulla and upper cervical spinal cord, with patchy brainstem enhancement are suggestive of Alexander disease. Although demyelinating diseases can involve the brainstem, selective medullary atrophy is not a known imaging feature of any form of demyelinating disease. Similarly, selective medullary atrophy has not been described in association with brainstem encephalitis or vasculitis. Central nervous system involvement in Behçet disease typically involves the mesodiencephalic area, not the medulla or upper cervical cord. Enhancement is atypical in Behçet disease. Intense transversely oriented enhancement of the pons is a known imaging manifestation of infratentorial intra-axial involvement in Erdheim-Chester disease. Adult-onset Alexander disease (AOAD) is an autosomal dominant condition due to mutations of the encoding region of glial fibrillary acidic protein (GFAP). An abundance of rosenthal fibers and eosinophilic cytoplasmic astrocytic inclusions are histopathologic hallmarks of AOAD. It is a rare neurologic disorder characterized by a peculiar form of leukodystrophy, also known as fibrinoid leukodystrophy, which usually becomes clinically evident during infancy, although neonatal, juvenile, and adult variants are recognized. Although autosomal dominant transmission is the rule in infantile variant, de novo sporadic mutation can also be seen in AOAD. In AOAD, symptoms typically manifest in affected individuals aged 12 years or older, and unlike the infantile variant, AOAD primarily involves the brainstem. As a result of this predominant brainstem involvement, patients typically present with brainstem symptoms including progressive ataxia, spastic paraparesis, and bladder or bowel dysfunction.

376

Symptomatic severity varies, although intellectual impairment is typically absent. Seizure is also extremely rare, suggesting relative supratentorial brain sparing. The typical radiologic picture of AOAD at presentation is abnormal T2 signal and mild to severe atrophy of the medulla oblongata that extend caudally to the level of C1–C2, in association with abnormal T2 signal in the periventricular white matter. In patients whose imaging changes are confined to the brainstem and whose clinical symptomatology is less specific, then the highest level of suspicion should be exerted to consider AOAD as a diagnosis. The periventricular white matter shows abnormal T2 signal that is less dramatic compared to the infantile variant. Characteristically irregular areas of contrast enhancement are mainly found in the brainstem and cerebellum, as shown in our patient. Patchy areas of enhancement can be seen in the brainstem and in the cerebellum. These abnormalities do not extend cranially to the pons in most cases. However, abnormal T2 signal can be seen in the middle and superior cerebellar peduncles. In the upper part of the spinal cord, abnormal T2 signal is predominantly located in the central gray matter and in the lateral columns. In some patients, abnormal T2 signal can extend down to the level of C6. Minimal atrophy of the entire cervical cord, even without any abnormal T2 signal, is almost a constant finding in AOAD patients. Brain proton spectroscopy is non-specific, although decreased NAA peaks may be found, indicating loss of viability/neuronal density. Increased peak of myoinositol (MI), an astrocytic marker, with reversal of its relations with creatine (Cr) may also be noted.

Key Points  Progressive atrophy of the medulla and upper cervical spinal cord with hyperintense signal on T2WI with associated clinical symtoms is highly suggestive of AOAD.  In patients with suggestive MRI appearances, further molecular testing to examine GFAP gene analysis should be performed to confirm the diagnosis of AOAD.

Suggested Reading Farina L, Pareyson D, Minati L, et al. Can MR imaging diagnose adult-onset Alexander disease? AJNR Am J Neuroradiol 2008; 29(6): 1190–6. Messing A, Brenner M, Feany MB, Nedergaard M, Goldman JE. Alexander disease. J Neurosci 2012; 32(15): 5017–23. Pareyson D, Fancellu R, Mariotti C, et al. Adult-onset Alexander disease: a series of eleven unrelated cases with review of the literature. Brain 2008; 131(Pt 9): 2321–31. Stumpf E, Masson H, Duquette A, et al. Adult Alexander disease with autosomal dominant transmission: a distinct entity caused by mutation in the glial fibrillary acid protein gene. Arch Neurol 2003; 60(9): 1307–12. van der Knaap MS, Naidu S, Breiter SN, et al. Alexander disease: diagnosis with MR imaging. AJNR Am J Neuroradiol 2001; 22(3): 541–52.

CASE

Part V

82

Metabolic Diseases Involving Central Nervous System Bruno Siqueira Campos Lopes, Asim K. Bag, Lázaro Luís Faria do Amaral

Clinical Presentation A 25-year-old man with dwarfism, coarse facial features, and low visual acuity presented with an inability to walk.

Imaging (A)

(B)

(C)

Fig. 82.1 (A) Sagittal T1WI and (B–C) Sagittal 3D-CISS MR images through the midline and parasagittal.

Advanced Neuroradiology Cases, ed. Amaral et al. Published by Cambridge University Press. © Cambridge University Press 2016.

377

Part V. Metabolic Diseases Involving CNS: Case 82

(B) (A)

(C)

378

(D)

Fig. 82.2 (A–D) Axial T2WI through the level of the lateral ventricles and centrum semiovale.

Part V. Metabolic Diseases Involving CNS: Case 82

(A)

(B)

(C)

Fig. 82.3 (A–B) Sagittal T2WI and (C) Axial T2WI of the spine through the midline and at the level of dorsal vertebral body.

379

Part V. Metabolic Diseases Involving CNS: Case 82

Mucopolysaccharidosis – MPS Type II (Hunter Syndrome) Primary Diagnosis Mucopolysaccharidosis – MPS type II (Hunter syndrome)

Differential Diagnoses Dilated perivascular spaces (normal variant) Lowe syndrome Hypomelanosis of Ito Foramen magnum stenosis Down syndrome Rheumatoid arthritis associated with atlantoaxial subluxation

Imaging Findings Fig. 82.1: (A) Sagittal T1-weighted and (B–C) Sagittal 3D-CISS MR images of the brain showed prominent perivascular spaces in white matter and deep gray matter. Fig. 82.2: (A–D) Axial T2weighted MR images showed prominent perivascular spaces in the corpus callosum and deep white and gray matter. Fig. 82.3: (A–B) Sagittal T2-weighted and (C) Axial T2-weighted MR images of the spine showed posterior scalloping and anterior beaking in the vertebral bodies. Foramen magnum and cervical spinal stenosis is evident, due to the thickened dura mater.

Discussion The MR imaging findings demonstrating presence of perivascular spaces (white matter lesions) and spinal stenosis, in conjunction with the clinical features seen in this patient strongly suggest a diagnosis of mucopolysaccharidosis (MPS). Mucopolysaccharidosis is a heterogeneous group of lysosomal storage disorders caused by an inherited enzyme deficiency. It results in an abnormal accumulation of glycosaminoglycans (GAGs) causing widespread cellular dysfunction and physiologic abnormalities in multiple cell types. Depending on the age of onset, clinical disease features, and associated enzyme deficiency, MPS is categorized into seven broad groups: I (which has three subtypes), II, III, IV, VI, VII, and IX. With the exception of MPS type II (Hunter syndrome), an X-linked disorder, all MPS subtypes are autosomal recessive. It is a multi-organ system disease that primarily involves the skeletal, circulatory, digestive, respiratory, and nervous systems. Depending on the subtype and quantity of residual functioning enzyme, the severity of MPS phenotypic expression varies, as does the impact on connective tissue. Central nervous system involvement varies widely among MPS groups and individually within each group and subtype. Magnetic resonance spectroscopy may show an abnormal peak at 3.3 to 4.4 ppm, higher than myoinositol, which may represent accumulated GAGs. The most commonly demonstrated CNS imaging abnormality is the appearance of dilated perivascular spaces and white matter lesions/abnormalities. Other imaging abnormalities include brain and optic nerve atrophy, focal or diffuse gray matter abnormalities, cervical cord compression, and hydrocephalus. Profound dilatation of

380

perivascular spaces is primarily indicative of MPS type I and II, although it may be seen in other MPS groups. Dilated perivascular spaces can be seen anywhere in the brain including the corpus callosum (most profoundly in the periventricular white matter). Hydrocephalus may be due to altered CSF absorption from the dural deposition of MPS or increased venous pressure. The development of hydrocephalus in early childhood may cause macrocephaly. Narrowing of the foramen magnum and upper cervical area is a known manifestation of many MPS groups and is usually due to atlantoaxial instability. Prominent perivascular spaces can be seen in numerous pathologic conditions (such as Lowe syndrome and hypomelanosis of Ito) but may be accidentally found in healthy patients as well. Diagnostic confirmation of MPS can be made in the presence of clinical findings, demonstrated spinal abnormalities, the presence of prominent perivascular spaces, and the thickening of the dura mater, which appears dark on T2weighted images because of the mucopolysaccharide deposits. Since MPS adversely affects connective tissue, a vast number of skeletal deformities (dysostosis multiplex) are common, such as anomalies of vertebral shape (platyspondylia and bulletshaped vertebrae), thoracolumbar kyphosis, odontoid hypoplasia, and atlantoaxial instability, as well as cardiovascular and respiratory symptoms, depending on the group/subtype. Evaluation of spine in the setting of MPS is important to rule out spinal cord compression secondary to spinal involvement that is most common in MPS IV and MPS IS (formerly MPS V). Spinal cord compression in patients with MPS is most commonly at the level of the foramen magnum/upper cervical region. The definitive diagnosis of MPS can be confirmed by analyzing blood to identify specific enzyme deficiencies. Therapeutic interventions in MPS target correction of enzyme activity through bone marrow, hematopoietic cell, and umbilical cord blood transplantation, as well as enzyme-replacement therapy.

Key Points  Hydrocephalus, white matter abnormality, and presence of dilated perivascular spaces are suggestive of MPS.  Narrowing at the foramen magnum and upper cervical region is a known manifestation of MPS.

Suggested Reading Cimaz R, La Torre F. Mucopolysaccharidoses. Curr Rheumatol Rep 2014; 16(1): 389. van der Knaap MS, Valk J, eds. Magnetic Resonance of Myelination and Myelin Disorders, 3rd edn. Berlin, New York: Springer; 2005. Muenzer J. Overview of the mucopolysaccharidoses. Rheumatology (Oxford) 2011; 50(Suppl 5): v4–12. Rasalkar DD, Chu WC, Hui J, et al. Pictorial review of mucopolysaccharidosis with emphasis on MRI features of brain and spine. Br J Radiol 2011; 84(1001): 469–77. Zafeiriou DI, Batzios SP. Brain and spinal MR imaging findings in mucopolysaccharidoses: a review. AJNR Am J Neuroradiol 2013; 34(1): 5–13.

CASE

Part VI

83

Central Nervous System Tumors Fabrício Guimarães Gonçalves, Lázaro Luís Faria do Amaral

Clinical Presentation A 23-year-old woman presented with a history of persistent headache and balance problems. She denied any other symptoms and her medical history was non-contributory. Magnetic resonance studies of the brain were obtained.

Imaging Fig. 83.1 Midline sagittal T1WI.

Fig. 83.2 Coronal T2WI through the cerebellum.

Fig. 83.3 Axial T1WI with contrast through the posterior fossa.

Advanced Neuroradiology Cases, ed. Amaral et al. Published by Cambridge University Press. © Cambridge University Press 2016.

381

Part VI. Central Nervous System Tumors: Case 83

Rosette-Forming Glioneuronal Tumor Primary Diagnosis Rosette-forming glioneuronal tumor

Differential Diagnoses Dysembryoplastic neuroepithelial tumor (DNET) Pilocytic astrocytoma Medulloblastoma Ependymoma

Imaging Findings Fig. 83.1: Sagittal T1WI showed multiple, small, rounded and oval-shaped hypointense lesions in the cerebellar vermis with minimal mass effect. Fig. 83.2: Coronal T2WI showed multiple hyperintense nodules in the cerebellar vermis, with no adjacent vasogenic edema. Fig. 83.3: Axial T1 postcontrast image showed multicystic appearance of the lesion, without any enhancement. It is to be noted that the cystic lesions extend beyond the margin of the fourth ventricle.

Discussion In a young adult patient, the presence of a midline vermian multicystic tumor without enhancement is suggestive of rosette-forming glioneuronal tumor (RGNT). Although RGNT was initially described as DNET of the posterior fossa, true posterior fossa DNET is exceedingly rare. Pilocytic astrocytoma is typically a cystic tumor, with an intensely enhancing solid component. Medulloblastoma typically demonstrates diffusion restriction and is a solid or predominantly solid tumor (with few cysts), rather than a purely cystic tumor. Ependymoma of the fourth ventricle arises from the ependymal lining of the ventricle and is an intraventricular tumor. Extraventricular posterior fossa ependymoma is extremely rare, and thus excluded. Rosette-forming glioneuronal tumor of the fourth ventricle (RGNT) is a recently described mixed tumor that expresses neuronal and glial differentiation with or without a ganglion component. They are rare, benign (WHO grade I), slowgrowing, indolent tumors composed of distinct neurocytic rosettes or perivascular pseudorosettes and an astrocytic component. Typically, they are found in young female patients (30 ± 12.8 years – 2× more common than in males). Recognized as a distinct entity in 2002, as opposed to DNET of the cerebellum, RGNT was thought to occur exclusively in the fourth ventricle and the adjacent parenchyma. It is now known that this tumor can occur in other CNS locations including the pineal region, optic nerve, and spinal cord. In some patients, these tumors are asymptomatic and are discovered incidentally. When symptomatic, patients commonly complain of headache, vomiting, ataxia, vertigo, neck pain, and cranial nerve palsies. Owing to the typical location (fourth ventricle), patients may show obstructive hydrocephalus. On CT imaging, RGNTs are predominantly hypodense tumors, with few areas of calcifications. On MRI imaging,

382

RGNTs are well-defined solid-cystic masses which are T1 hypointense and T2 hyperintense with variable degrees of enhancement (ring or heterogeneous) but little mass effect or surrounding edema. In one-quarter of reported cases, these lesions show calcification. These lesions are typically in the midline, and primarily involve the floor of the fourth ventricle with dorsal invasion of the vermis and ventral invasion of the brainstem either directly or as a satellite lesion. Although satellite lesions are common, the exact mechanism of satellitosis is not understood. However, satellitosis may be a clue to the diagnosis. From the fourth ventricle, RGNTs may extend to the cerebral aqueduct. The tumor can be completely solid or completely cystic, or may be mixed. Focal enhancement may be seen and can be nodular, ring-like, or linear. Typically, RGNT does not involve the lateral recesses of the fourth ventricle. Intratumoral hemorrhage may be seen. On pathology, RGNT specimens are known to demonstrate dual histologic characteristics composed of neurocytic and astrocytic/glial components, hypothetically originating from the periventricular germinal matrix. The neurocytic components are formed by small cells with hyperchromatic nuclei forming rosettes disposed around an eosinophilic core stained with synaptophysin and absent vessels. The organization of the astrocytic/glial component with eosinophilic processes resembles pilocytic astrocytomas, with positive immunostaining for glial fibrillary acid protein (GFAP) and S-100 protein. Generally, surgery is the treatment modality of choice for RGNT. It has been documented that total or subtotal surgical resection results in satisfactory tumor treatment, with excellent outcome and minor morbidity.

Key Points  The differential diagnosis of midline vermian multicystic, solid, or mixed tumor without prominent enhancement in a young adult patient should include RGNT.  Satellite lesions are common findings in RGNT and can be seen in the brainstem as well as in the adjacent cerebellum, and are a clue to the diagnosis.

Suggested Reading Johnson M, Pace J, Burroughs JF. Fourth ventricle rosette-forming glioneuronal tumor. Case report. J Neurosurg 2006; 105(1): 129–31. Komori T, Scheithauer BW, Hirose T. A rosette-forming glioneuronal tumor of the fourth ventricle: infratentorial form of dysembryoplastic neuroepithelial tumor? Am J Surg Pathol 2002; 26(5): 582–91. Marhold F, Preusser M, Dietrich W, Prayer D, Czech T. Clinicoradiological features of rosette-forming glioneuronal tumor (RGNT) of the fourth ventricle: report of four cases and literature review. J Neurooncol 2008; 90(3): 301–8. Vajtai I, Arnold M, Kappeler A, et al. Rosette-forming glioneuronal tumor of the fourth ventricle: report of two cases with a differential diagnostic overview. Pathol Res Pract 2007; 203(8): 613–19.

CASE

Part VI

84

Central Nervous System Tumors Fabrício Guimarães Gonçalves, Asim K. Bag, Lázaro Luís Faria do Amaral

Clinical Presentation A previously healthy four-year-old boy presented with a twoweek history of generalized tonic-clonic seizures. His neurologic tests were normal. He did not have a history of febrile seizures. The presence of neurofibromatous stigmata (café au lait spots or neurofibromas) was not noted during physical exam. Computed tomography imaging and MRI studies of the head were performed.

Imaging Fig. 84.1 Axial CT image.

Advanced Neuroradiology Cases, ed. Amaral et al. Published by Cambridge University Press. © Cambridge University Press 2016.

383

Part VI. Central Nervous System Tumors: Case 84 Fig. 84.2 Axial T1WI.

Fig. 84.3 Axial T2WI.

384

Part VI. Central Nervous System Tumors: Case 84 Fig. 84.4 Axial T2* gradient: Axial T1WI postcontrast image.

385

Part VI. Central Nervous System Tumors: Case 84

Meningioangiomatosis Primary Diagnosis Meningioangiomatosis

Differential Diagnoses Meningioma Cavernous malformation Arteriovenous malformation

Imaging Findings Fig. 84.1: Axial CT image showed a peripheral serpiginous/ gyriform calcified lesion, associated with adjacent moderate vasogenic edema, located in the posterior left insular cortex/ subcortical tissue, subjacent to the left sylvian fissure. Fig. 84.2: Axial T1WI demonstrated an ill-defined, isointense mass, with edematous adjacent white matter in the left sylvian fissure and adjacent cortex. Fig. 84.3: Axial T2WI better demonstrated the edema involving the left temporal and insular regions, and the signal loss from the calcified lesion. Fig. 84.4: Axial T2* gradient: Axial T1WI postcontrast image demonstrated an irregular ring-enhancing lesion adjacent to the cortex in the temporal and insular region. Note that the nodule has irregular contours.

Discussion The clinical presentation of generalized tonic-clonic seizures in a young person, combined with characteristic gyral calcifications, and an enhancing lesion on imaging, is very suggestive of meningioangiomatosis (MA). Meningioma is usually a dural-based tumor that frequently presents with thickening of the adjacent dura (dural tail sign), rather than as a cortical/subcortical lesion, such as MA. However, if the MA lesion is focal and nodular, then meningioma should be considered in the differential diagnosis. Venous vascular malformation or cavernous malformation can affect any areas of the brain, including the cortical/subcortical areas. However, on both T1WI and T2WI MR imaging, they are usually heterogeneous in appearance because of the presence of hemorrhage of different ages and have a characteristic surrounding hypointense rim on T2WI and GRE images due to hemosiderin deposition. None of these features is present in this patient’s images. No prominent vascular flow void or nidus was noted on imaging to suggest arteriovenous malformation. Meningioangiomatosis is a benign and rare hamartomatous lesion that typically involves the leptomeninges and adjacent cortical tissue. Previously, MA was thought to represent either atypical vascular malformations or direct brain infiltration of a leptomeningeal meningioma. More recently, authors have defined MA masses as congenital hamartomatous lesions with meningothelial cell proliferation of the leptomeninges and underlying cortex. Meningioangiomatosis may occur sporadically and in association with neurofibromatosis type 2 (NF2), meningiomas, oligodendroglioma, and/or arteriovenous malformation.

386

Meningioangiomatosis is more frequent in males than in females, with no propensity to grow. It more commonly affects children and young adults, with up to 85% of the patients being less than 30 years of age. Sporadic MA is at least four times more common than if associated with NF2. Headaches and seizures (typically refractory to treatment) are the most common clinical presentation of MA, with seizures alone being the more prevalent clinical problems, occurring in 81% of patients with MA. Lesions are more frequently located in the frontal and temporal regions and more commonly in the right hemisphere. Some MA lesions may not be depicted in non-contrastenhanced CT, as they may not present with calcification. Varying degrees of lesional calcification are identified in up to 90% of cases. Lesional calcification may be focal, extensive, or patchy. Approximately 50% of cases present as hypodense lesions, which enhance after contrast administration. These lesions are typically nodular in appearance (less frequently irregular in contour), firm and well circumscribed, measuring around 3 cm in diameter. Meningioangiomatosis almost always demonstrates abnormal MRI. These lesions are typically hypointense on T1 and hyperintense on T2 with gyriform FLAIR hyperintensities. Cystic changes and T2* susceptibility artifacts have also been described. The most common and typical imaging findings include intense contrast enhancement in up to 80% and variable calcification in up to 90% of cases. Another important imaging characteristic of MA is variable amount of perilesional edema and mass effect, which can occur in up to one-half of the cases. These lesions are not seen on catheter angiogram. Besides its rarity, MA poses a significant challenge in diagnosis, based solely on imaging findings. The main differential diagnosis includes meningiomas, cavernous malformations, and other vascular pathologies. The most typical clinical and imaging scenario is of a young individual with headache and difficult-to-control seizures, who presents with a calcified lesion identified on CT and a puzzling mass on MRI. Microscopic pathology findings include meningovascular proliferation with or without leptomeningeal calcification, which can extend into the perivascular spaces, sparing the white matter. The hallmarks of MA lesions include proliferation of small vessels within the cortex, concentric perivascular proliferation of spindle-shaped cells with fibrosis (perivascular cuffing), and calcification. Meningioangiomatosis is typically treated with en bloc resection, with 100% seizure control. There is no need for an expanded excision of the surrounding brain tissue. No adjuvant therapy is usually needed. Recurrence does not usually occur.

Key Point  Meningioangiomatosis should be considered as a diagnosis (or at least in the differential diagnosis) if the patient has the characteristic triad of young age of onset, epileptic attacks (generalized tonic-clonic seizures), and characteristic imaging findings of a cortical/subcortical

Part VI. Central Nervous System Tumors: Case 84

calcified lesion (with hypointensity on T2WI and hypointensity to signal loss on GRE image), with variable amount of enhancement on contrast imaging.

Suggested Reading Abdulazim A, Samis Zella MA, Rapp M, et al. Meningioangiomatosis in a patient with progressive focal neurological deficit-case report and review of literature. Br J Neurosurg 2013; 27(2): 253–5. Feng R, Hu J, Che X, et al. Diagnosis and surgical treatment of sporadic meningioangiomatosis. Clin Neurol Neurosurg; 115(8): 1407–14.

Jamil O, Ramkissoon S, Folkerth R, Smith E. Multifocal meningioangiomatosis in a 3-year-old patient. J Neurosurg Pediatr 2012; 10(6): 486–9. Kashlan ON, Laborde DV, Davison L, et al. Meningioangiomatosis: a case report and literature review emphasizing diverse appearance on different imaging modalities. Case Rep Neurol Med 2011; 2011: 361203. Shah A, Korya D, Larsen BT, et al. Meningioangiomatosis: a rare presentation with progressive cortical blindness. Neurology 2013; 81(5): 511–12.

387

CASE

Part VI

85

Central Nervous System Tumors Fabrício Guimarães Gonçalves, Asim K. Bag, Lázaro Luís Faria do Amaral

Clinical Presentation A previously healthy 27-year-old woman presented to an outpatient department with long-standing history of low-grade headache, which had recently started to worsen. Her medical history was non-contributory, with no reported use of oral contraception or family history of migraine. She did not complain of any visual problems. Hematologic findings were normal. Imaging studies, including CT scan of the head and contrast-enhanced MRI of the brain were obtained.

Imaging

Fig. 85.2 Axial T1WI through the lesion. Fig. 85.1 Axial non-contrast CT scan through the lesion.

Advanced Neuroradiology Cases, ed. Amaral et al. Published by Cambridge University Press. © Cambridge University Press 2016.

389

Part VI. Central Nervous System Tumors: Case 85 Fig. 85.3 Axial FLAIR image through the lesion.

Fig. 85.4 Axial GRE through the lesion.

390

Part VI. Central Nervous System Tumors: Case 85 Fig. 85.5 Axial T2WI through the lesion.

Fig. 85.6 Axial T1WI with contrast through the lesion.

391

Part VI. Central Nervous System Tumors: Case 85

Astroblastoma Primary Diagnosis Astroblastoma

Differential Diagnoses Supratentorial ependymoma Pleomorphic xanthoastrocytoma Ganglioglioma Supratentorial juvenile pilocytic astrocytoma Metastasis Oligodendroglioma

Imaging Findings Fig. 85.1: Axial non-enhanced CT image of the brain demonstrated a complex, left temporal lobe mass with a centrally located gross calcified component, surrounded by non-calcified tumor tissue and an anterior polar cystic lesion. Fig. 85.2: Axial T1W image showed the complex and heterogeneous solid-cystic mass with a central, marked hypointense nodule, with adjacent cystic and solid components. Note that the calcified area appears hypointense. Fig. 85.3: Axial FLAIR image demonstrated mild peritumoral abnormal signal posterior to the mass. Note that there is signal void at the central calcified nodule. Fig. 85.4: Axial GRE image demonstrated signal void at the centrally located calcified nodule. Fig. 85.5: Axial T2W image demonstrated that the lesion itself has three major components: a centrally located hypointense (due to calcification) component; a surrounding, solid component demonstrating multiple, small cysts; and a larger cystic component at the anterior aspect. Fig. 85.6: Axial T1W image with contrast demonstrated enhancement of the soft tissue component (surrounding the calcified nodule), which presented moderate enhancement, with no cystic parietal enhancement, and no significant contrast uptake in the calcified nodule.

Discussion Astroblastoma should be included in the differential diagnosis of a relatively benign-appearing solid-cystic tumor with calcification, multiple cysts in the solid component (bubbly appearance), and heterogeneous enhancement in a relatively young patient. Many tumors can have similar appearances. Supratentorial ependymomas usually have a cystic component with high signal on T2-weighted images. Ependymomas may also present cystic component, which are usually simple with the solid component lacking a bubbly appearance as seen in cases of astroblastomas. Ependymomas tend to show moderate amount of peripheral T2 hyperintensity for their size due to prominent peritumoral infiltration invading the adjacent parenchyma. Pleomorphic xanthoastrocytoma, ganglioglioma, rare supratentorial juvenile pilocytic astrocytomas, and rare supratentorial hemangioblastomas all generally present with a strongly enhancing mural nodule within a single large cyst, a very distinct pattern seen in cases of astroblastomas. Oligodendrogliomas are also rare lesions that arise in the white matter and invade the cortex. They may be large, have cystic components, which

392

may be seen as bubbly components, and have gross grouped calcification (present in 70–80% of the cases). Calcification is a consistent imaging feature seen in most reported cases of astroblastomas, which tend to be punctate. Astroblastomas are distinctive neuroepithelial tumors of no definitive origin initially described by Bailey and Cushing in 1924 and further characterized by Bailey and Bucy in 1930. This is a rare (only 0.45–2.8% of all neuroglial tumors) tumor. Astroblastomas can occur in any age group, but generally occur in children and young adults, with no gender predilection. Clinical signs and symptoms depend on the localization and size of the neoplasm, with headache and seizures being the most frequent manifestations. The origin of astroblastomas has been debated but a definitive precursor cell has not yet been identified. Recently, it has been postulated that astroblastomas may arise from abnormally persisting groups of embryonal precursor cells during embryogenesis such as tanycytes, a transitional cell type that is between astrocyte and ependymal cells. Astroblastomas share features of astrocytomas, ependymomas, and of non-neuroepithelial tumors on histopathology owing to their astroblastic aspects. Lack of fibrillarity is an essential feature in distinguishing astroblastomas from other glial neoplasms. Immunohistochemically, astroblastomas are immunoreactive for glial fibrillary acidic protein (GFAP), S-100 protein, and vimentin. The majority of lesions display a focal cytoplasmic immunoreactivity for epithelial membrane antigen (EMA). Astroblastomas are defined histologically by the presence of perivascular pseudorosettes and prominent perivascular hyalinization. Two distinct histologic types have been described: a lowgrade type, in which a better-differentiated pattern was apparent and a favorable postoperative prognosis may be expected; and a high-grade type, showing more anaplastic microscopic features, in which postoperative survival is usually short. High-grade lesions show focal or multifocal regions of high cellularity, anaplastic nuclear features, elevated mitotic indices, vascular proliferation, and necrosis with pseudopalisading. Although malignant astroblastomas may show infiltration of brain parenchyma, most of them are non-infiltrating lesions. Total resection is reported to be the method of treating an astroblastoma. Close follow-up of all cases and adjuvant therapy for high-grade and recurrent cases is recommended. The more favorable prognosis is almost invariably associated with circumscription of the tumor that might permit the total resection of tumor in all grades. Imaging findings may also assist in the preoperative diagnosis of astroblastomas. Typically, they present as large, peripherally located supratentorial masses. On MRI, they are cystic and solid tumors with a bubbly appearance to the solid component. Disregarding their large size, astroblastomas have relatively little T2 adjacent hyperintensity, possibly because of their lack of tumor infiltration of the surrounding brain parenchyma. Astroblastomas also tend to demonstrate low signal on T2-weighted images and may demonstrate increased density on CT scans. Even though astroblastomas are classically described

Part VI. Central Nervous System Tumors: Case 85

as non-infiltrating masses in the cerebral hemispheres, tumor may be seen in the corpus callosum, cerebellum, brainstem, and optic nerves. Astroblastomas typically show rim enhancement on CT and T1W MRI. However, heterogeneous gadolinium enhancement may be seen. Less common findings are intratumoral hemorrhage and intraventricular location.

Ganapathy S, Kleiner LI, Mirkin DL, Broxson E. Unusual manifestations of astroblastoma: a radiologic-pathologic analysis. Pediatr Radiol 2009; 39(2): 168–71.

Key Point

Krengel S, Hauschild A, Schafer T. Melanoma risk in congenital melanocytic naevi: a systematic review. Br J Dermatol 2006; 155(1): 1–8.

 Astroblastoma should be considered in the differential diagnosis of a heterogeneous tumor with both calcification and solid-cystic components, particularly if there are multiple cysts in the solid component and the patient presents at a younger age.

Kim KH, Chung SB, Kong DS, Seol HJ, Shin HJ. Neurocutaneous melanosis associated with Dandy-Walker complex and an intracranial cavernous angioma. Childs Nerv Syst 2012; 28(2): 309–14.

Navarro R, Reitman AJ, de Leon GA, et al. Astroblastoma in childhood: pathological and clinical analysis. Childs Nerv Syst 2005; 21(3): 211–20.

Suggested Reading

Port JD, Brat DJ, Burger PC, Pomper MG. Astroblastoma: radiologicpathologic correlation and distinction from ependymoma. AJNR Am J Neuroradiol 2002; 23(2): 243–7.

Bell JW, Osborn AG, Salzman KL, et al. Neuroradiologic characteristics of astroblastoma. Neuroradiology 2007; 49(3): 203–9.

Salvati M, D’Elia A, Brogna C, et al. Cerebral astroblastoma: analysis of six cases and critical review of treatment options. J Neurooncol 2009; 93(3): 369–78.

393

CASE

Part VI

86

Central Nervous System Tumors Fabrício Guimarães Gonçalves, Lázaro Luís Faria do Amaral

Clinical Presentation A 37-year-old woman with one previous episode of seizure and an abnormal CT examination of the brain was referred to the neurology clinic for follow-up. She was otherwise healthy, with no other previous complaints or neurologic symptoms. Physical and neurologic examinations were unremarkable. Hematology studies were normal with no significant abnormalities. Owing to the previous abnormal CT study, which demonstrated an ill-defined lesion in the right frontal lobe (not shown), MRI with contrast was performed.

Imaging Fig. 86.1 Sagittal T1WI.

Advanced Neuroradiology Cases, ed. Amaral et al. Published by Cambridge University Press. © Cambridge University Press 2016.

395

Part VI. Central Nervous System Tumors: Case 86 Fig. 86.2 Axial T2WI.

Fig. 86.3 Axial FLAIR image.

396

Part VI. Central Nervous System Tumors: Case 86 Fig. 86.4 Axial enhanced T1WI.

Fig. 86.5 Axial DWI.

397

Part VI. Central Nervous System Tumors: Case 86

Protoplasmic Astrocytoma Primary Diagnosis Protoplasmic astrocytoma

Differential Diagnoses Dysembryoplastic neuroepithelial tumors Fibrillary astrocytoma Glioblastoma

Imaging Findings Fig. 86.1: Sagittal T1WI showed a hypointense mass in the right frontal lobe, mainly subcortical in location (arrow). Fig. 86.2: Axial T2WI demonstrated that the lesion had marked, increased homogeneous signal with expansion of the right superior frontal gyrus. The mass involved the subcortical white matter and adjacent cortex (arrow), with mild mass effect but no significant adjacent vasogenic edema. Fig. 86.3: Axial FLAIR image showed marked T2 signal attenuation inside the mass, which also showed internal stranding (septations) and thick ring of peripheral hyperintensity, lacking significant edema (arrow). Gradient echo images did not demonstrate any signs of gross calcification or hemorrhage inside the mass (images not shown). Fig. 86.4: Axial enhanced T1WI showed no significant enhancement inside or in the periphery of the lesion. Fig. 86.5: Axial DWI showed an incomplete ring of increased signal (arrow), with mildly decreased ADC values (images not shown).

Discussion The imaging findings described above are quite typical for a protoplasmic astrocytoma (PA). Typically, PAs are masses that demonstrate marked T1 hypointense (Fig. 86.1) and T2 hyperintense (Fig. 86.2) signal with no significant mass effect. These masses have internal FLAIR signal suppression with a variable ring that lacks suppression (increased signal) surrounding the mass (Fig. 86.3). Protoplasmic astrocytomas typically demonstrate minimum or no contrast enhancement. An important feature of PAs is the presence of a complete/incomplete ring of increased diffusion weighted imaging signal (Fig. 86.5). Based on the imaging features, this lesion could be confused with a dysembryoplastic neuroepithelial tumor (DNET), in which suppression of internal FLAIR signal has been also described (see Part VI: Case 111). However, DNETs tend to be smaller than PAs, with a mean diameter of 30 mm compared to a diameter of around 54 mm for PAs. Similarly, DNETs also involve the cortex, whereas subcortical white matter is more commonly involved in cases of PAs. Another distinct feature of DNETs is that they tend to be triangular in shape, tapering towards the ventricle. Both DNETs and PAs may show portions with marked T2-FLAIR signal suppression (FLAIR hypointense signals); however, PAs exhibit more extension. The peripheral, FLAIR hyperintense rim tends to be thicker and more pronounced in cases of PAs than in DNETs.

398

Fibrillary astrocytomas (FAs) share imaging characteristics with PAs, as they both demonstrate iso- to hypodensity with no contrast enhancement on CT. On MRI, they can also demonstrate T1 hypointense and T2 hyperintense signal with little or no significant enhancement. The absence of internal FLAIR suppressed/hypointense signal, however, is what differentiates FAs from PAs. The marked T2 hyperintensity and FLAIR signal suppression/hypointensity may resemble cystic lesions or even necrotic tumors such as glioblastoma. Glioblastoma is the least likely differential diagnosis as these tumors are highly malignant, and typically show necrotic components, contrast enhancement, hemorrhage, neovascularity, high CBV values, and high lactate peaks. Protoplasmic astrocytoma is a rare variant of low-grade astrocytomas (LGAs), comprising 5% of LGAs, with fairly characteristic imaging and histology features. The age of presentation ranges from 2.5 to 41 years of age (mean, 21 years), with a sex predilection for males. The majority of patients present with seizures and, less commonly, headaches. Hydrocephalus, focal neurologic deficit, and even personality changes can be found, depending on the lesion size and location. Symptoms can last from 7 months to 28 years (mean duration, 6.6 years). Few patients succumb to the disease. Classified as WHO grade II tumors, PAs are composed of neoplastic astrocytes with rounded prominent nuclear contour and little cytoplasm. The PA tumoral matrix consists of numerous, prominent microcystic spaces filled with mucinous fluid. Mitoses, microvascular proliferation, and necrosis are absent. Protoplasmic astrocytomas tend to have indolent behavior and occur in younger-aged patients. These tumors have a tendency to be large, well defined, and more commonly located in the cortical and subcortical white matter of the temporal and frontal lobes. On CT, PAs appear as hypodense masses with variable mass effect, typically with no significant enhancement. On MRI, the tumoral areas show hypointense signal, compared to white matter, on T1-weighted images. On T2-weighted images, PAs show marked and intense hyperintense signal, without vasogenic edema, hemorrhage, or calcification. One important feature of PAs is FLAIR suppressed signal of T2 hyperintense portions of the tumor, indicating that portions of the tumor are cystic in nature. The areas of FLAIR signal suppression are varied, representing more than 50% of the global tumoral extent. Areas of FLAIR hypointensity should not be mistaken for cystic degeneration of necrotic areas, commonly seen in higher grades of glioma. Typically, PAs do not enhance or demonstrate mild enhancement. On perfusion studies, these lesions usually show very low CBV in relation to normal white matter. On spectroscopy, PAs typically show high choline peaks, higher cholineto-creatine ratio, attenuation of NAA peaks, and markedly high choline-to-NAA ratios, indicating a high-grade lesion. It is important to note that spectral analysis of the cystic components of PAs do not demonstrate elevated lipid/lactate peaks

Part VI. Central Nervous System Tumors: Case 86

nor show depletion of other metabolites, which are typically seen in true necrotic lesions. Despite treatment, PAs have a similar outcome to other LGAs, although they apparently have a better prognosis than other subtypes of LGAs.

Key Point  Masses that demonstrate marked T1 hypointense and T2 hyperintense signal with no significant mass effect or that demonstrate minimum or no contrast enhancement are suggestive of PAs.

Suggested Reading Manley S, Crooks D, Artingstall L, et al. Diffuse central nervous system protoplasmic astrocytoma. Pediatr Blood Cancer 2010; 54(5): 768–9. Prayson RA, Estes ML. Protoplasmic astrocytoma. A clinicopathologic study of 16 tumors. Am J Clin Pathol 1995; 103(6): 705–9. Tay KL, Tsui A, Phal PM, Drummond KJ, Tress BM. MR imaging characteristics of protoplasmic astrocytomas. Neuroradiology 2011; 53(6): 405–11.

399

CASE

Part VI

87

Central Nervous System Tumors Asim K. Bag, Lázaro Luís Faria do Amaral

Clinical Presentation A previously healthy five-year-old girl presented with a twomonth history of persistent headache that was not responsive to acetaminophen. During the past two weeks, she developed a balance disorder and subsequent walking difficulties. On questioning, her mother reported that the patient had also been vomiting in the morning almost everyday for the last several weeks. A posterior fossa mass was suspected and an MRI was obtained (Figs. 87.1–87.4).

Imaging Fig. 87.1 Axial T2WI through the posterior fossa.

Advanced Neuroradiology Cases, ed. Amaral et al. Published by Cambridge University Press. © Cambridge University Press 2016.

401

Part VI. Central Nervous System Tumors: Case 87 Fig. 87.2 Axial DWI through the same level. Apparent diffusion coefficient map showed low values (not shown here).

402

Part VI. Central Nervous System Tumors: Case 87

Fig. 87.3 Axial T1WI with contrast through the same level.

Fig. 87.4 Coronal T1WI with contrast through the posterior fossa.

403

Part VI. Central Nervous System Tumors: Case 87

Medulloblastoma with Extensive Nodularity Primary Diagnosis Medulloblastoma with extensive nodularity

Differential Diagnoses Atypical teratoid/rhabdoid tumor Lhermitte-Duclos disease Atypical choroid plexus carcinoma

Imaging Findings Fig. 87.1: Axial T2WI image through the posterior fossa demonstrated an extensive nodular T2 hypo- to isointense mass in the right side of the posterior fossa involving both the vermis as well as the right cerebellar hemisphere, with extensive peritumoral abnormal T2 hyperintensity, severe mass effect, and leftward displacement of the fourth ventricle and brainstem. Fig. 87.2: Axial DWI image through the same level demonstrated increased signal within the nodular areas of the tumor. With low ADC values, this feature is suggestive of decreased diffusivity. Fig. 87.3: Axial T1WI with contrast through the same level demonstrates intense enhancement of the nodular components. Extensive nodularity of the lesion is better demonstrated on this image. Fig. 87.4: Coronal T1WI with contrast through the posterior fossa also demonstrated an intensely enhancing extensively nodular lesion involving the midline, as well as the right cerebellar hemisphere.

Discussion Imaging demonstrated an extensively nodular tumor involving the vermis as well as the right cerebellar hemisphere, with extensive peritumoral T2 hyperintensity and significant mass effect to the fourth ventricle and the brainstem. The mass is iso- to hypointense on T2WI and demonstrates diffusion restriction, suggesting a highly cellular tumor. In a five-yearold patient, an aggressive-looking mass in the posterior fossa with high cellularity is almost diagnostic of medulloblastoma (MB). The extensive nodularity suggests medulloblastoma with extensive nodularity (MBEN). Atypical teratoid/rhabdoid tumor can have a similar imaging appearance but it is extremely rare in the posterior fossa and in this age group. Predominantly a hemispheric lesion, Lhermitte-Duclos disease (LDD) does not demonstrate intense contrast enhancement or show decreased diffusivity. In fact, diffusivity is increased in LDD. Choroid plexus carcinoma also can have a similar appearance, but it is an extremely rare tumor. In contrast to MBEN, it predominantly involves the atria of the lateral ventricle, rather than the fourth ventricle.

404

Medulloblastoma is a high-grade (WHO grade IV) embryonal tumor of the cerebellum. It is the most common malignant brain tumor of childhood. It occurs at all ages with the peak incidence between four and seven years of age. Seventy percent of all MBs occur in children less than seven years of age. Overall 65% of all MBs are encountered in males. At least 75% of childhood MBs arise from the vermis, projecting into the fourth ventricle. Increased cerebellar hemisphere involvement is correlative with increasing patient age. Typical MBEN presenting symptoms include cerebellar syndrome (truncal ataxia and disturbed gait) with features of increased intracranial pressure including headache and morning vomiting due to mass effect, as well as obstructive hydrocephalus. Medulloblastoma with extensive nodularity is one of the known histopathologic subtypes of MB. Previously, MBEN was referred to as cerebellar neuroblastoma. This tumor usually occurs in infants and typically has an expanded lobular architecture. The extensive nodularity is thought to be due to the presence of excessive small cells with round nuclei in the background of the reticulin-free zone. With treatment, the tumor may dedifferentiate into a predominantly ganglion cell tumor. Deeper understanding of the molecular/genetic pathways and correlative histopathology of MB has led to the currently accepted classification system that divides MB into four different subgroups. These four subgroups include the WNT group, SHH group, Group 3, and Group 4 (see Table 87.1); MBEN is seen only in the SHH subgroup of MBs. Of all the four MB subgroups, only SHH incorporates all dominant histologic subtypes. In infants, the tumor is at the midline; however, cerebellar hemisphere tumors are more common in adolescent and adult patients. The majority of the SHH subgroup tumors present in early childhood or in adults with very low incidence in the intervening age groups. With molecular profiling it is possible to separate the infant SHH tumors from the adult SHH subgroups. Owing to extensive heterogeneity of the SHH subgroups, it is currently hypothesized that the SHH subgroup constitutes 2–3 distinct subtypes of these tumors. Genetically targeted therapy is being developed to treat SHH subgroup tumors. Inhibitors of the SHH pathway have been shown to have a dramatic response, though temporary, in SHH subgroup tumors. The MBEN tumors have a characteristic imaging appearance. On T2WI, this extensive nodular tumor may have a gyriform appearance. On contrast-enhanced images, the tumor has an extensive nodular appearance (grape-like). Typically, the tumor demonstrates low diffusivity (high signal on DWI, with low ADC). As characteristic of the SHH subgroup, this tumor does not demonstrate metastasis at presentation.

Part VI. Central Nervous System Tumors: Case 87 Table 87.1. Medulloblastoma Subgroups

WNT

SHH

Group 3

Group 4

Incidence

10%

30%

25%

35%

Genetics

CTNNB1 mutation Monosomy 6 WNT signaling MYC expression +

PTCH1/SMO/SUFU mutation 9q deletion SHH signaling MYCN amplification +

MYC amplification +++ Photoreceptor/GABAergic

CDK6 amplification Isochromosome 17q Neuronal/glutaminergic pathway Minimal MYC expression

Demographics

Adults and children

Adults, children, and infants

Children and infants

Adults, children, and infants

Sex

M=F

M=F

M>F

M>F

Histology

Classic, rarely LCA

Desmoplastic/nodular, LCA, MBEN

Classic, LCA

Classic, LCA

Metastasis at Presentation

Rare

Uncommon

Very frequent

Frequent

Prognosis

Very good

Infants good, others intermediate

Very poor

Intermediate

Abbreviations: WNT, wingless; SHH, sonic hedgehog; M, male; F, female; LCA, large cell anaplastic; MBEN, medulloblastoma with extensive nodularity.

Key Points  Currently MBs are stratified into four subgroups: WNT, SHH, Group 3, and Group 4 based on genetic morphology, clinical presentation, and prognosis.  MBEN is a tumor of the SHH subgroup with good prognosis in infants and an intermediate prognosis in adults.  MBEN has an extensive nodular appearance on imaging, can have gyriform or grape-like appearance with

avid enhancement, and reduced diffusivity of the nodular areas.

Suggested Reading Gerber NU, Mynarek M, von Hoff K, et al. Recent developments and current concepts in medulloblastoma. Cancer Treat Rev 2014; 40(3): 356–65. Northcott PA, Korshunov A, Pfister SM, Taylor MD. The clinical implications of medulloblastoma subgroups. Nat Rev Neurol 2012; 8(6): 340–51.

405

CASE

Part VI

88

Central Nervous System Tumors Anderson B. Belezia, Lázaro Luís Faria do Amaral

Clinical Presentation A 52-year-old man presented with a three-week history of ataxia, bilateral dysmetria, and headache.

Imaging (A) (B)

Fig. 88.1 (A–B) Axial T2WI through the level of the cerebellar hemispheres.

Advanced Neuroradiology Cases, ed. Amaral et al. Published by Cambridge University Press. © Cambridge University Press 2016.

407

Part VI. Central Nervous System Tumors: Case 88

(A)

(B)

(C)

Fig. 88.2 (A–C) Axial DWI through the level of the cerebellar hemispheres.

408

Part VI. Central Nervous System Tumors: Case 88

(A) (B)

(C)

Fig. 88.3 (A–C) Axial ADC maps with values through the level of the cerebellar hemispheres.

(B)

(A)

Fig. 88.4 (A) Axial T1WI postgadolinium of the brain. (B) Coronal T1WI postgadolinium with fat saturation through the cerebellar hemispheres.

409

Part VI. Central Nervous System Tumors: Case 88

Multifocal Desmoplastic Medulloblastoma Primary Diagnosis Multifocal desmoplastic medulloblastoma

Differential Diagnoses Other medulloblastoma variants Atypical teratoid/rhabdoid tumor Lhermitte-Duclos disease Ependymoma Lymphoma Pilocytic astrocytoma

Imaging Findings Fig. 88.1: (A), and (B) Axial T2WI showed three hyperintense masses: one in the inferior aspect of the right cerebellar hemisphere, one in the left cerebellar hemisphere, and another small mass in the left aspect of the cerebellar vermis. Fig. 88.2: (A–C) Axial DW images of the brain demonstrated hyperintensity of each mass. Fig. 88.3: (A–C) Axial ADC maps with values confirming diffusion restriction in all three lesions. Fig. 88.4: (A) Axial T1W postgadolinium image of the brain and (B) Coronal T1W postgadolinium image with fat saturation demonstrated moderate enhancement in all three lesions.

Discussion Medulloblastoma is one of the most common pediatric CNS tumors, second only to astrocytoma, the most common tumor of the posterior fossa and the most common CNS malignant neoplasm found in this age group. It is a malignant, invasive embryonal neoplasm with tendency for CSF dissemination; hence, it is classified as a WHO grade IV neoplasm. The patient’s age at presentation, and the number and cerebellar location of the lesions and their distinctive signal and enhancement patterns demonstrated on imaging are suggestive of medulloblastoma. Medulloblastoma may depict classic histologic features or features of one of its variants: desmoplastic medulloblastoma, medulloblastoma with extensive nodularity, or anaplastic medulloblastoma. Typically, the desmoplastic variant of medulloblastoma usually occurs in a slightly older population, namely adolescents or young adults. In addition, although the majority of medulloblastomas arise in the midline (about 75%), this variant tends to involve one of the cerebellar hemispheres and may show multiple peripheral small cysts. Atypical teratoid/rhabdoid tumor of the posterior fossa can be indistinguishable from medulloblastoma on imaging studies alone, requiring additional diagnostic studies. The characteristic striated pattern of Lhermitte-Duclos disease can differentiate it from medulloblastoma. Ependymoma is typically calcified and plastic, and usually extends through the foramen of Luschka into the adjacent cerebellopontine cisterns, not seen in this patient. Lymphoma may demonstrate restriction on imaging as well, but it is usually supratentorial and may have more intense enhancement to gadolinium. Pilocytic astrocytoma should be considered in the differential diagnosis of pediatric patients, but it lacks the typical hyperdensity/diffusion restriction of medulloblastoma.

410

There are four molecular subgroups of medulloblastomas, each one arising from a different cytogenetic pathway: wingless (WNT), sonic hedgehog (SHH), Group 3, and Group 4 (see Table 87.1, Case 87). Desmoplastic medulloblastomas show a strong correlation with the SHH pathway, although there is no direct correlation between every molecular subgroup and each histologic variant. Multifocal desmoplastic medulloblastomas are very rare, and usually occur in patients with Gorlin syndrome (nevoid basal cell carcinoma syndrome). In general, the onset of clinical symptoms in patients with medulloblastomas is rapid, reflecting the aggressive behavior of the tumor. Headache and vomiting due to intracranial hypertension, the last usually related to aqueduct stenosis resulting in hydrocephalus, are the most frequent symptoms. The desmoplastic variant can present with dysmetria related to cerebellar hemispheric involvement. On CT imaging studies, medulloblastomas appear as a hyperattenuated mass, usually well circumscribed, in the posterior fossa. Cyst formations and calcifications are relatively common features of these tumors. On MR imaging studies, these tumors are hypointense on T1-weighted images and isoto hyperintense on T2-weighted images. On diffusion images, they often restrict because of their dense cellularity, which also explains their hyperdensity on CT exams. Contrast enhancement is quite variable, ranging from small to marked, with varied demonstration of heterogeneous enhancement. Surgical resection with adjuvant chemotherapy is the treatment of choice for patients with medulloblastoma. Recent studies show that the identification of the molecular subgroup of the tumor may play a role in patient prognosis and treatment decisions. For example, WNT tumors have a five-year survival of 95% in children and nearly 100% in adults.

Key Points  Medulloblastoma is a malignant, invasive embryonal neoplasm with a tendency for CSF dissemination and it is classified as a WHO grade IV neoplasm.  The desmoplastic variant tends to involve one of the cerebellar hemispheres and may show multiple peripheral small cysts, although it can arise in the midline.  Prognosis and treatment differ for each medulloblastoma molecular-based subtype.

Suggested Reading Ciccarino P, Rotilio A, Rossetto M, et al. Multifocal presentation of medulloblastoma in adulthood. J Neurooncol 2012; 107(2): 233–7. Koeller KK, Rushing EJ. From the archives of the AFIP, medulloblastoma: a comprehensive review with radiologicpathologic correlation. Radiographics 2003; 236: 1613–37. Levy RA, Blaivas M, Muraszko K, Robertson PL. Desmoplastic medulloblastoma: MR findings. AJNR Am J Neuroradiol 1997; 18: 1364–6. Perreault S, Ramaswamy V, Achrol AS, et al. MRI surrogates for molecular subgroups of medulloblastoma. AJNR Am J Neuroradiol 2014; 357: 1263–9. Yeom KW, Mobley BC, Lober RM, et al. Distinctive MRI features of pediatric medulloblastoma subtypes. AJR Am J Roentgenol 2013; 200(4): 895–903.

CASE

Part VI

89

Central Nervous System Tumors Taciana Mara Filomeno Orsini, Leonardo Furtado Freitas, Lázaro Luís Faria do Amaral

Clinical Presentation A 63-year-old woman, with a three-year history of benign tumor in the anterior cranial fossa, presented to our facility with recent onset of progressive headache, anosmia, and altered behavior. Previous diagnostic MR imaging (Fig. 89.1 A–B) demonstrated a T1-weighted hyperintense lesion in the midline of the skull base that was suppressed on T1-weighted fat suppression, suggestive of a dermoid cyst. Three-year follow-up MR imaging studies of the brain (Figs. 89.2–89.5) were performed to evaluate recent neurologic symptoms.

Imaging (A)

(B)

Fig. 89.1 (A) Sagittal T1WI and (B) Sagittal T1WI fat-suppression images through the midline.

Advanced Neuroradiology Cases, ed. Amaral et al. Published by Cambridge University Press. © Cambridge University Press 2016.

411

Part VI. Central Nervous System Tumors: Case 89

(B)

(A)

Fig. 89.2 (A–B) Sagittal T1WI through the level of the frontal basal lobe.

(A) (B)

Fig. 89.3 (A–B) Axial T2 through the level of the third and lateral ventricles.

412

Part VI. Central Nervous System Tumors: Case 89

(A)

(B)

Fig. 89.4 (A–B) Axial T1-weighted postcontrast fat-suppression images through the frontal lobes.

(A) (B)

Fig. 89.5 (A–B) Sagittal T1-weighted postcontrast fat-suppressed images through the midline and parasagittal.

413

Part VI. Central Nervous System Tumors: Case 89

Malignant Transformation of Dermoid Cyst Primary Diagnosis Malignant transformation of dermoid cyst

Differential Diagnoses Lipoblastic meningioma Liposarcoma

Imaging Findings Fig. 89.1 (A) Sagittal T1WI and (B) Sagittal T1WI fatsuppression images showed a T1 hyperintense lesion in the midline of the skull base that suppressed on T1-weighted fat suppression, suggestive of dermoid cyst. Fig. 89.2 (A–B) Sagittal T1WI showed an expansive process centered in the skull base that invaded the basal surface of both frontal lobes, more importantly in the right side, with some foci of fat inside the lesion and vasogenic edema in the adjacent brain parenchyma. Fig. 89.3 (A–B) Axial T2 confirmed the extensive lesion in both frontal lobes, larger on the right side, with central hyperintense area in the center, surrounded by vasogenic edema. Fig. 89.4 (A–B) Axial T1WI postcontrast fat suppression showed an extensive and heterogeneous enhancing lesion in both basal frontal lobes that invaded the rostrum of corpus callosum, with central necrosis in the right side. Fig. 89.5 (A–B) Sagittal T1WI postcontrast fat suppression showed an extensive basal frontal lesion that invaded the frontal lobes, with central necrosis in the right side. Note the suppression of the foci on fat inside the lesion.

Discussion The history of a known dermoid cyst, demonstrated on prior imaging and visible on current images, in the presence of an intratumoral fat signal is suggestive of malignant degeneration of the dermoid cyst. The presence of an abnormal T2 signal in the peritumoral bed in addition to significant lesion enhancement is typical of malignant transformation, as seen in this patient. Intracranial dermoid cysts account for approximately 0.5% of brain tumors. They develop during the fourth or fifth week of fetal development from aberrant ectodermal embryonic tissue in the neural groove that typically contains mature squamous epithelium, keratin, and adnexal elements such as hair follicles and sebaceous glands. Dermoid cysts are usually extra-axial and occur near the midline (parasellar/frontobasal) and third and fourth ventricles. The suprasellar cistern is the most common site: other common locations for dermoid cysts include the posterior fossa and the anterior skull base/frontonasal region. Imaging findings of lipoblastic meningioma, a WHO grade I tumor, can demonstrate intratumoral fat signal. However, intense enhancement and abnormal peritumoral T2 signal is uncommon in this tumor. The presence of fat signal is also seen in liposarcoma, but they are not known to develop in the central skull base-frontal lobe region. Dermoid cysts can be complicated if they rupture (please see Part VI: Case 113), with consequential development of meningitis and hydrocephalus. Another extremely rare

414

complication is malignant transformation of dermoid cyst into a squamous cell cancer. Although this is extremely rare, malignant transformation of a benign cyst is the third most common cause of intracranial squamous cell cancer, after metastasis and direct extension of the tumor. Epidermoid cysts, not dermoid cysts, more commonly degenerate into squamous cell cancer. The underlying etiology of the transformation is unknown; however, it is thought that persistent chronic inflammatory changes lead to the development of squamous cell cancer, which can be further complicated by leptomeningeal spread. On CT images, dermoid cysts present as circumscribed, very-low-density ( 20 to 140 HU) lesions, without any contrast enhancement. Calcification may be seen in up to 20% of cysts. They are associated with vasogenic edema and rarely cause hydrocephalus. As these tumors have adnexal elements including fat, they are hyperintense on T1WI as well as fast spin echo T2WI sequences. Typically, these tumors do not enhance with contrast. Image-based diagnosis of malignant degeneration in dermoid cysts can be challenging. A sudden increase in size, development of nodular intratumoral enhancement, and development of new abnormal T2 signal in the peritumoral areas are some of the documented imaging findings of malignant degeneration. Once malignant degeneration occurs, there is a rapid change of the patient’s clinical course with an extremely poor prognosis. Surgical resection is the treatment of choice whenever possible.

Key Points  Although rare, malignant degeneration to squamous cell cancer of intracranial inclusion cyst is a known phenomenon.  Epidermoid cysts more commonly degenerate to squamous cell cancer as compared to dermoid cysts.

Suggested Reading Brown JY, Morokoff AP, Mitchell PJ, Gonzales MF. Unusual imaging appearance of an intracranial dermoid cyst. AJNR Am J Neuroradiol 2001; 22(10): 1970–2. Hamlat A, Hua ZF, Saikali S, et al. Malignant transformation of intracranial epithelial cysts: systematic article review. J Neurooncol 2005; 74(2): 187–94. Jain R, Gujar S, McKeever P, Robertson P, Mukherji S. Imaging findings associated with childhood primary intracranial squamous cell carcinoma. AJNR Am J Neuroradiol 2003; 24(1): 109–11. Kano T, Ikota H, Kobayashi S, et al. Malignant transformation of an intracranial large epidermoid cyst with leptomeningeal carcinomatosis: case report. Neurol Med Chir (Tokyo) 2010; 50(4): 349–53. Lakhdar F, Hakkou el M, Gana R, Maaqili RM, Bellakhdar F. Malignant transformation six months after removal of intracranial epidermoid cyst: a case report. Case Rep Neurol Med 2011; 2011: 525289. Smirniotopoulos JG, Chiechi MV. Teratomas, dermoids, and epidermoids of the head and neck. Radiographics 1995; 15(6): 1437–55.

CASE

Part VI

90

Central Nervous System Tumors Sonali H. Shah, Prasad B. Hanagandi, Lázaro Luís Faria do Amaral

Clinical Presentation A 30-year-old man presented with a six-month history of headache, vomiting, and blurring of vision. In the past two months, he had developed left-sided hemiparesis, abnormal behavior, and had experienced multiple episodes of generalized tonic-clonic seizures. His routine hematologic evaluation was unremarkable with no obvious risk factors noted. Computed tomography and MR imaging were recommended for further evaluation.

Imaging Fig. 90.1 Axial postcontrast CT image at the level of lateral ventricles.

Advanced Neuroradiology Cases, ed. Amaral et al. Published by Cambridge University Press. © Cambridge University Press 2016.

415

Part VI. Central Nervous System Tumors: Case 90

(A) (B)

(C) (D)

Fig. 90.2 (A) Axial T1W, (B) Axial T2W, (C) Axial FLAIR, (D) Axial GRE, (E) Axial DWI, and (F) Axial postgadolinium T1W images at the level of lateral ventricles.

416

Part VI. Central Nervous System Tumors: Case 90

(E)

(F)

Fig. 90.2 (cont.)

Fig. 90.3 Single voxel MR spectroscopy through the solid component of the right frontal lobe mass lesion.

417

Part VI. Central Nervous System Tumors: Case 90

Extraventricular Neurocytoma Primary Diagnoses Extraventricular neurocytoma

Differential Diagnoses Glioblastoma Ependymoma Oligodendroglioma Ganglioglioma Primitive neuroectodermal tumor Hemorrhagic metastases

Imaging Findings Fig. 90.1: Axial postcontrast CT image at the level of lateral ventricles showed a right frontal, parafalcine heterogeneously enhancing mass with moderate vasogenic edema and mass effect, with a focal cystic component that revealed a bloodfluid level. Fig. 90.2: (A) Axial pre-gadolinium T1W, (B) T2W, and (C) FLAIR images demonstrated a mixed signal intensity, cortical-based, right parafalcine frontal mass lesion with a cystic component that showed mixed hypo-isointense areas and a hyperintense blood-fluid level. (D) Axial GRE sequence depicted multiple foci of blooming/T2 shortening, representing areas of hemorrhage. (E) Axial DWI sequence showed minimal diffusion restriction in the solid component of the mass lesion. (F) Axial postgadolinium T1W image showed heterogeneous enhancement in the solid part of the lesion. Fig. 90.3: Single voxel MR spectroscopy at TE 135 showed elevated choline peak at 3.2 ppm (arrow).

Discussion A predominantly cortical-based, heterogeneously enhancing, right frontal lobe mass/lesion with a cystic component containing blood-fluid levels, demonstrating mild to moderate associated perilesional edema and considerable mass effect is suggestive of an aggressive, high-grade tumor. For example, glioblastoma, ependymoma, or hemorrhagic metastasis may demonstrate similar imaging findings. However, the age of the patient, tumor location, and cystic component, in the presence of mild to moderate edema, is suggestive of an extraventricular neurocytoma. Radical tumor excision and histopathologic evaluation including immunohistochemical staining confirmed the diagnosis of extraventricular neurocytoma. In contrast to neurocytomas, oligodendrogliomas are usually less well demarcated and tend to have larger, coarse calcification (seen in about 20–90%). Focal calvarial remodeling is also often noted. High-grade gliomas also have obscured and infiltrating margins, but calcification is very uncommon. Ependymomas are often seen adjacent to the ventricles with relatively mild edema; the parietal lobes are the favored sites for cystic ependymomas. Hemorrhagic metastatic lesions have similar imaging features. However, the presence of calcification and negative systemic imaging evaluation for a

418

primary malignancy do not favor this diagnosis. The most favoured location for ganglioglioma is the temporal lobes. The cystic component is often central with a peripheral solid component; hemorrhage is an unusual feature of ganglioglioma. Occurrence of calcification in gangliogliomas has been estimated to occur in 25–28%. Primitive neuroectodermal tumor has a predilection for the frontoparietal lobes and is seen in the pediatric population (less than five years of age). Extraventricular neurocytomas are rare parenchymal tumors that seem to exhibit neuronal differentiation, similar to central neurocytoma, occurring remote from the ventricular system but with wide variability. Owing to its variable presentation, the most recent WHO classification of CNS tumors (2007) categorizes extraventricular neurocytoma as a new entity. These tumors show wide inconsistency with regard to their morphologic features, cellularity, and proliferation rate. Thus, they pose a diagnostic challenge because of the significant imaging and histopathologic overlap noted among other brain tumors, especially oligodendroglioma. Extraventricular neurocytomas are tumors of young adults and adolescents with no gender predilection. They can occur at a variety of locations but are most frequently seen in the cerebral hemispheres, especially in the frontoparietal regions, with usual manifestations of headache, seizure, and hemiparesis. Extraventricular neurocytomas are often large, welldemarcated, cortical-based lesions with hemorrhage, calcification, cystic necrosis, and intense enhancement, and should be included in the differential diagnoses for cerebral masses. Though a prospective diagnosis of this lesion is difficult, because of the rare incidence of this entity it should be considered, especially in a young individual. These masses can also be purely cystic, a cyst with a mural nodule, or purely solid. They closely resemble an oligodendroglioma, which are cortical-based tumors with calcification. Histopathologic evaluation of extraventricular neurocytoma reveals uniform round cells with regular nuclei and a diagnostic immunohistochemical staining, which are strongly synatophysin positive (Syn) (+++); glial fibrillary acid protein negative (GFAP) ( ); Vimetin (Vim) ( ), S-100 (+++); and IDH1 gene mutation negative. It also appears that atypical histologic features correlate with aggressiveness and higher recurrence rates. Surgery plays a central role for management of these tumors with favorable response seen in most patients.

Key Points  Extraventricular neurocytoma is a rare neoplasm showing significant imaging and histopathologic overlap with appearances of other primary brain neoplasms.  However, a well-demarcated, large, cortical-based cerebral hemispheric mass with cystic components, calcification, hemorrhage, and intense enhancement in a young patient should suggest the possibility of extraventricular neurocytoma.

Part VI. Central Nervous System Tumors: Case 90

Suggested Reading Huang WY, Zhang BY, Geng DY, Zhang J. Computed tomography and magnetic resonance features of extraventricular neurocytoma: a study of eight cases. Clin Radiol 2013; 68(4): e206–12. Liu K, Wen G, Lv XF, et al. MR imaging of cerebral extraventricular neurocytoma: a report of 9 cases. AJNR Am J Neuroradiol 2013; 34(3): 541–6. Louis DN, Ohgaki DH, Wiestler OD, Cavenee WK. WHO Classification of Tumours of the Central Nervous System. Lyon: IARC Press; 2007. Myung JK, Cho HJ, Park CK, et al. Clinicopathological and genetic characteristics of extraventricular neurocytomas. Neuropathology 2013; 33(2): 111–21.

Niiro T, Tokimura H, Hanaya R, et al. MRI findings in patients with central neurocytomas with special reference to differential diagnosis from other ventricular tumours near the foramen of Monro. J Clin Neurosci 2012; 19(5): 681–6. Yang GF, Wu SY, Zhang LJ, et al. Imaging findings of extraventricular neurocytoma: report of three cases and review of the literature. AJNR Am J Neuroradiol 2009; 30(3): 581–5. Yi KS, Sohn CH, Yun TJ, et al. MR imaging findings of extraventricular neurocytoma: a series of ten patients confirmed by immunohistochemistry of IDH1 gene mutation. Acta Neurochir (Wien) 2012; 154(11): 1973–80.

419

CASE

Part VI

91

Central Nervous System Tumors Leslie Lamb, Prasad B. Hanagandi, Jeffrey Chankowsky, Raquel del Carpio-O’Donovan

Clinical Presentation A 45-year-old woman presented with a one-year history of gradually progressive and insidious onset intermittent headache accompanied by nausea, vomiting, ataxia, and dysarthria. Computed tomography and MRI (see MRI images below) were performed.

Imaging (B) (A)

Fig. 91.1 (A) Axial non-contrast CT image and (B) Axial postcontrast CT image through the posterior fossa at the level of pons and cerebellar hemispheres.

Advanced Neuroradiology Cases, ed. Amaral et al. Published by Cambridge University Press. © Cambridge University Press 2016.

421

Part VI. Central Nervous System Tumors: Case 91 Fig. 91.2 Axial T2WI through the posterior fossa at the level of pons and cerebellar hemispheres.

Fig. 91.3 Right parasagittal T1WI through the right cerebellar hemisphere.

422

Part VI. Central Nervous System Tumors: Case 91 Fig. 91.4 Right parasagittal postcontrast fat-suppressed T1WI through the right cerebellar hemisphere.

Fig. 91.5 Axial postcontrast fat-saturated T1WI through the posterior fossa at the level of pons and cerebellar hemispheres.

423

Part VI. Central Nervous System Tumors: Case 91

Cerebellar Liponeurocytoma Primary Diagnosis Cerebellar liponeurocytoma

Differential Diagnoses Medulloblastoma Ependymoma Cerebellar lipoastrocytoma Metastases

Imaging Findings Fig. 91.1: (A) Axial non-contrast CT image showed heterogeneously hyperdense and (B) Axial postcontrast CT showed enhancing lesion in the right cerebellar hemisphere with focal areas of fat density (arrow). Fig. 91.2: Axial T2WI MRI demonstrated a lesion with heterogeneous signal and focal hyperintense areas (arrow). Note the mass effect with complete absence of vasogenic edema. Fig. 91.3: Sagittal T1WI demonstrated linear streaky hyperintense foci (arrow). Fig. 91.4: Sagittal T1WI postcontrast, fat-suppressed MRI and Fig. 91.5: Axial T1WI postcontrast, fat-saturated image through the same level showing areas of fat suppression (arrow).

Discussion Computed tomography and MR imaging showed a heterogeneously enhancing cerebellar lesion featuring fatty streaks. These imaging features and the lack of perilesional edema in an adult patient combined with typical cerebellar symptoms are the key features suggesting a cerebellar liponeurocytoma. The two main differential diagnoses one has to consider are adult medulloblastoma and ependymoma. Medulloblastoma appears hyperdense on CT, has relatively low T2 signal, demonstrates peritumoral edema, and often shows homogeneous enhancement and diffusion restriction; however, lipid components are noticeably absent. Ependymomas tend to be intraventricular and have low signal on T1-weighted and high signal on T2-weighted images with calcification and hemorrhage. The presence of lipidized cells seen in other neuroectodermal tumors such as cerebellar lipoastrocytoma, pleomorphic xanthoastrocytoma, supratentorial PNET (primitive neuroectodermal tumor), glioblastoma, and low-grade spinal cord astrocytoma can mimic liponeurocytoma. However, the lipidized cells are more commonly seen in the pediatric population. Depending on the primary source of origin, metastatic lesions have a varied appearance on CT and MR imaging and demonstrate vasogenic edema, while lipid component is absent. In 2000, the WHO classified cerebellar liponeurocytoma as a distinct entity. They are well-differentiated tumors of neuroectodermal origin, and classified as WHO grade I–II neoplasms, with a low to moderate mitotic activity. These tumors have glio-neuronal components with interspersed

424

lipidized cells that resemble mature adipocytes. In the past, cerebellar liponeurocytoma has been referred to as lipidized medulloblastoma, lipomatous medulloblastoma, lipomatous glioneurocytoma, and medullocytoma. Cerebellar liponeurocytomas are rare, slow-growing parenchymal neoplasms of adults, with distinctive morphology, typically occurring in the cerebellum. Liponeurocytomas are also known to occur in extracerebellar supratentorial locations and are usually restricted to the lateral ventricles. They often occur between the third and sixth decades of life (mean age 50 years) without gender predilection. However, some instances in the literature suggest a female preponderance. Patients often present with headaches or more focal cerebellar symptoms, depending on the location of tumor. On CT imaging, cerebellar liponeurocytoma has a predominant hypodense appearance, with scattered areas of even lower attenuation representing fat density with heterogeneous postcontrast enhancement. On MR imaging, lesions are iso- to hypointense on T1WI and mildly hyperintense to cortex on T2WI and very often lack vasogenic edema. Scattered streaks or laminated foci of hyperintensity on T1- and T2-weighted sequences correspond to fat density, as seen on CT. The tumors enhance inhomogeneously and the lipomatous component is suppressed on postgadolinium fat-saturated sequence and is a good diagnostic clue to confirm the adipose tissue. Surgical resection and biopsy is recommended to establish a definitive diagnosis. Once resected, two-thirds recur within 1 to 12 years and the average 5-year survival is approximately 48%. A low proliferative index carries better prognosis. Adjuvant radiotherapy may be necessary in tumors with a higher proliferative index.

Key Points  Cerebellar liponeurocytoma is a rare tumor, has been categorized as a distinct entity, separate from medulloblastoma, and carries better prognosis. Supratentorial liponeurocytomas are also known to exist and very few cases have been described in the literature.  The presence of a lipid component, heterogeneous enhancement, and lack of peritumoral edema are typical imaging findings in patients with cerebellar liponeurocytoma.

Suggested Reading Beizig N, Ziadi S, Ladib M, Mokni M. Cerebellar liponeurocytoma: case report. Neurochirurgie 2013; 59(1): 39–42. Chakraborti S, Mahadevan A, Govindan A, et al. Supratentorial and cerebellar liponeurocytomas: report of four cases with review of literature. J Neurooncol 2011; 103(1): 121–7.

Part VI. Central Nervous System Tumors: Case 91 De Aranjo AS, De Anjion PH, Maldaum MV, et al. Cerebellar liponeurocytoma: a literature review and case report. Neurosurg Q 2011; 21: 39–41.

Ikushima I, Korogi Y, Makita O, et al. MRI of arachnoid granulations within the dural sinuses using a FLAIR pulse sequence. Br J Radiol 1999; 72(863): 1046–51.

Guan JT, Geng YQ, Cheng Y, Guo YL, Wu RH. Magnetic resonance imaging of cerebellar liponeurocytoma. A case report and review of the literature. Neuroradiol J, 2012; 25(3): 331–6.

Oudrhiri MY, Raouzi N, El Kacemi I, et al. Understanding cerebellar liponeurocytomas: case report and literature review. Case Rep Neurol Med 2014; 2014: 186826.

425

CASE

Part VI

92

Central Nervous System Tumors Prasad B. Hanagandi

Clinical Presentation A 56-year-old man presented with a 10-month history of gradually progressive cognitive decline. His medical history was notable for radiotherapy and chemotherapy treatment for nasopharyngeal cancer (Fig. 92.1) six years prior to onset of current clinical symptoms. The patient had a follow-up MRI three years after the treatment which demonstrated no obvious abnormality (Fig. 92.2). Physical examination did not reveal signs or symptoms of infection. Previous follow-up MR imaging (Figs. 92.2–92.4) and flexible endoscopic assessment of the nasopharynx was negative for mucosal recurrence. Recent mini-mental state examination to assess cognitive impairment revealed a score of 15. Hematologic assessment and CSF analysis was unremarkable.

Imaging (A) (B)

Fig. 92.1 (A) Axial postcontrast T1WI and (B) Coronal postcontrast T1WI of the nasopharynx at the time of initial diagnosis.

Advanced Neuroradiology Cases, ed. Amaral et al. Published by Cambridge University Press. © Cambridge University Press 2016.

427

Part VI. Central Nervous System Tumors: Case 92 Fig. 92.2 Axial postcontrast T1WI through the nasopharynx and skull base (three years post-radiotherapy follow-up scan).

(B) (A)

Fig. 92.3 (A) Axial T2WI and (B) Axial FLAIR of the brain at the level of temporal lobes ( six years post-radiotherapy follow-up scan).

428

Part VI. Central Nervous System Tumors: Case 92

(A) (B)

Fig. 92.4 (A) Axial postcontrast T1WI and (B) Coronal postcontrast T1WI of the brain at the level of anterior temporal lobes and sphenoid sinus (six years post-radiotherapy follow-up scan).

429

Part VI. Central Nervous System Tumors: Case 92

Delayed Postradiation Temporal Lobe Necrosis Primary Diagnosis Delayed postradiation temporal lobe necrosis

Differential Diagnoses Local tumor recurrence Parenchymal brain metastasis Glioblastoma Herpes encephalitis

Imaging Findings Fig. 92.1: (A) Axial postcontrast T1W and (B) Coronal postcontrast MR images of the nasopharynx, at the time of initial diagnosis, showed an enhancing mass lesion in the right side of the nasopharynx (arrows). Fig. 92.2: Axial postcontrast T1WI (three-year follow-up) showed complete resolution, with no evidence of residual or recurrent disease (arrow). Fig. 92.3: (A) Recent (six years post-radiotherapy scan) follow-up Axial T2W and (B) Axial FLAIR MR brain images depict confluent white matter signal changes in the anterior temporal lobes (arrows) with relatively well-preserved cortex. Fig. 92.4: (A) Axial postcontrast T1W and (B) Coronal postcontrast MR images at the level of anterior temporal lobes showed nodular and asymmetric Swiss cheese pattern of enhancement.

Discussion Gradually progressive cognitive decline in a patient with past history of radiotherapy for treated nasopharyngeal cancer and focal changes on MR imaging are the clues that suggest the diagnosis of postradiation temporal lobe necrosis. The anterior temporal lobes are invariably involved in the radiation port resulting in these changes. The main differential diagnosis is local tumor recurrence, which typically has an extracranial component with leptomeningeal/dural involvement. Alternate differential diagnoses include parenchymal brain metastasis which, as opposed to radiation-induced necrosis, are often multiple and may be cystic or solid with disproportionate vasogenic edema. These entities are not confined to the radiation field and are not commonly seen in head and neck cancers. Glioblastoma may be induced by radiation, but this most often extends beyond the radiation field and lacks the bilateral presentation often seen in radiation necrosis. If present, the Swiss cheese enhancement pattern further helps in differentiating radiation necrosis from metastatic disease or a glial neoplasm. Based on T2weighted and FLAIR images; herpes encephalitis can have similar appearance. The relevant history, lack of acute clinical presentation, and negative CSF analysis excludes the diagnosis. Nasopharyngeal cancer causes 50,000 deaths a year worldwide with an incidence of 80,000 new cases reported annually. It shows marked geographic and racial variability, with increased prevalence in Southern China, where the annual incidence is estimated to be as high as 21 cases per 100,000 residents versus 0.2 to 0.4 in the United States and White

430

population. Currently, radiotherapy is the mainstay of treatment and the anterior temporal lobes are invariably included in the radiation port. Radiotherapy for head and neck cancer is known to cause neurotoxicity with neurologic complications, especially in nasopharyngeal cancers. Radiation destroys the endothelial lining of small blood vessels, resulting in obliterative arteriopathy and leading to ischemic injury and necrosis of the brain parenchyma that predominantly involves the white matter. Four main clinical presentations secondary to radiotherapy for nasopharyngeal cancer have been described 1) temporal lobe epilepsy in 31%, 2) vague symptoms such as dizziness or impaired memory in 39%, 3) non-specific symptoms such as mild headache or symptoms of increased intracranial pressure in 14%, and 4) the remaining had no signs or symptoms. Dementia, as the sole presenting symptom of temporal lobe necrosis, is rare. On CT, the most common findings are ill-defined, focal, or confluent areas of contrast enhancement and cerebral edema predominantly conforming to the radiation field. The majority of patients with radiation-induced temporal lobe necrosis have bilateral involvement. To begin with, the imaging abnormalities can be unilateral but eventually develop bilateral changes. Edema and mass effect can be marked, resulting in brain herniation. Radiation-induced temporal lobe changes are best seen on contrast-enhanced MRI: necrosis occurs anywhere between 3 and 10 years post-treatment. On MRI, these changes are better delineated with predominant white matter involvement. White matter hyperintense signal changes on T2-weighted and FLAIR images represent combination of vasogenic edema and radiation-induced leukoencephalopathy. Cortical involvement is usually focal and less intense. The Swiss cheese enhancement pattern is a typical imaging finding on gadolinium images. If conventional imaging methods yield ambiguous results, perfusion, spectroscopy, and PET imaging studies can aid in differentiating imaging necrosis from tumoral pathologies. Treatment options for postradiation necrosis are limited as although steroids are helpful in the early stages to control edema and mass effect, their long-term use is associated with several complications and is not beneficial. Anticoagulants and antiplatelet medications used to prevent endothelial damage and thrombosis have met with little success in treating postradiation necrosis. These medications have a limited role and carry the potential risk of hemorrhage. Bevacizumab, a monoclonal antibody, which acts against VEGF (vascular endothelial growth factor) helps in normalization of the blood-brain barrier and has been shown to improve neurocognitive symptoms. Large-scale studies are required to prove its efficacy in patients with temporal lobe necrosis.

Key Points  Radiation necrosis of temporal lobes occurs because of inevitable inclusion of the anterior part of middle

Part VI. Central Nervous System Tumors: Case 92

cranial fossa, which contains the anterior temporal lobes, which are invariably included in the radiation port.  Swiss cheese enhancement pattern is often noted in cases with radiation necrosis. Very often, bilateral changes are noted, but asymmetric unilateral changes can be seen.

Chen J, Dassarath M, Yin Z, et al. Radiation induced temporal lobe necrosis in patients with nasopharyngeal carcinoma: a review of new avenues in its management. Radiat Oncol 2011; 6: 128.

Suggested Reading Chan YL, Leung SF, King AD, et al. Late radiation injury to the temporal lobes: morphologic evaluation at MR imaging. Radiology 1999; 213(3): 800–7.

Lee AW, Foo W, Chappell R, et al. Effect of time, dose and fractionation on temporal lobe necrosis following radiotherapy for nasopharyngeal carcinoma. Int J Radiat Oncol Biol Phys 1998; 40(1): 35–42.

Chang ET, Adami HO. The enigmatic epidemiology of nasopharyngeal carcinoma. Cancer Epidemiol Biomarkers Prev 2006; 15: 1765–77.

Lee AW, Ng SG, Ho JHC. Clinical diagnosis of late temporal lobe necrosis following radiation therapy for nasopharyngeal carcinoma. Cancer 1988; 61:1535–42.

Chong VF, Fan YF, Mukherji SK. Radiation-induced temporal lobe changes: CT and MR imaging characteristics. Am J Roentgenol 2000; 175(2): 431–6. Ferlay J, Shin HR, Bray F, et al. Estimates of worldwide burden of cancer in 2008: GLOBOCAN 2008. Int J Cancer 2010; 127(12): 2893–917.

431

CASE

Part VI

93

Central Nervous System Tumors Asim K. Bag

Clinical Presentation A 45-year-old woman presented to the emergency department with a 2-week history of headache and an unsteady gait, both of which had worsened acutely in the last four to five days. A brain MRI revealed a large mass centered in the right posterior parietal lobe, crossing the midline through the splenium of the corpus callosum. A tissue biopsy of the mass histopathologically confirmed glioblastoma (GBM) and the mass was then partially resected, as shown in Fig 93.1. Molecular analysis of the tissue sample revealed that the MGMT

promoter region of the tumor tissue was hypermethylated. Following resection, she was treated with concurrent temozolomide (TMZ) and radiation using the standard Stupp protocol. Forty days after completion of the concurrent chemoradiation, another MRI was obtained (Fig. 93.2) as a part of standard clinical protocol. No change in her neurologic symptoms was noted. As she did not have any worsening of symptoms, no change in therapy was offered. Ninety days after completion of chemoradiation, follow-up MR images were obtained (Fig. 93.3).

Imaging (B) (A)

Fig 93.1 (A) Axial postcontrast T1WI sequence and (B) Axial FLAIR image through the splenium of the corpus callosum (three days post-partial tumor resection).

Advanced Neuroradiology Cases, ed. Amaral et al. Published by Cambridge University Press. © Cambridge University Press 2016.

433

Part VI. Central Nervous System Tumors: Case 93

(A)

(B)

434

Fig 93.2 (A) Axial postcontrast T1WI and (B) Axial FLAIR image through the splenium of the corpus callosum (40 days after completion of concurrent TMZ and radiotherapy).

Part VI. Central Nervous System Tumors: Case 93

(A)

Fig 93.3 (A) Axial postcontrast T1WI sequence and (B) Axial FLAIR image through the splenium of the corpus callosum (90 days after completion of concurrent TMZ and radiotherapy).

(B)

435

Part VI. Central Nervous System Tumors: Case 93

Pseudoprogression due to Chemoradiation Primary Diagnosis Pseudoprogression due to chemoradiation

Differential Diagnosis True tumor progression

Imaging Findings Fig. 93.1: (A) Axial postcontrast T1WI through the splenium of the corpus callosum demonstrated a large surgical cavity involving the right of the posterior parietal lobe and the corpus callosum. There is significant residual tumor in the splenium of the corpus callosum. (B) Axial FLAIR image through the same level demonstrated minimal abnormal FLAIR signal beyond the margin of the enhancing component of the tumor. Fig. 93.2: (A) Axial postcontrast T1WI sequence obtained little more than one month after completion of the concurrent chemoradiation demonstrated significant enlargement of the enhancing component of the tumor. (B) Axial FLAIR demonstrated enlargement of the abnormal, peri-enhancing, hyperintense FLAIR areas as well. Fig. 93.3: (A) Three-month follow-up scan demonstrates significant improvement of both the enhancing component as well as (B) the FLAIR peritumoral abnormal hyperintense areas.

Discussion Significant worsening of the enhancing component and the peri-enhancing FLAIR abnormality of a tumor within three months of completing concurrent chemoradiation in a GBM patient with GBM with hypermethylated MGMT gene promoter region is highly suggestive of pseudoprogression (PP). Follow-up scans demonstrating significant improvement of the enhancing component as well as improvement of the perienhancing FLAIR hyperintense areas, without therapy changes, confirms the diagnosis of PP, and rules out the possibility of true tumor progression. Pseudoprogression can be defined by worsening of the enhancing portion of the previously treated tumor and peritumoral FLAIR abnormality in the absence of increased tumor activity. It can be seen in up to one-third of the GBM patients treated with concurrent TMZ and radiotherapy (Stupp protocol). As the name suggests, the worsening of apparent imaging abnormalities improves over time, as the underlying mechanism originates from extensive brain injury due to chemoradiation, rather than tumor proliferation. Although PP can be seen in patients treated exclusively with radiotherapy, it is more commonly seen in patients treated with combined TMZ and radiotherapy, typically within three months of completing chemoradiation. Pseudoprogression can occur even six months after chemoradiation is completed. In the literature, the incidence of PP is variably reported as low as 9% to as high as 65%. This variation stems from the use of differing criteria to define PP in terms of either the increased degree of abnormal enhancement or

436

the degree of decreased enhancement on follow-up MRI. In a recently published large prospective study, the incidence of PP demonstrated was approximately 10%. Epigenetic silencing of the gene encoding one of the DNA-repair proteins, O6-methyl-guanine methyl transferase (MGMT), has been linked to PP. MGMT is responsible for repairing the DNA once it is damaged by radiation and chemotherapy. If the MGMT promoter region is hypermethylated, the gene is silenced and tumor cells are less efficient at repairing DNA damage, particularly if a concurrent chemoradiation regimen is used. In approximately onethird of patients with GBM, the MGMT promoter is not hypermethylated. However, the MGMT promoter is hypermethylated in up to 66% of patients with PP. MGMT promoter methylation status can predict PP in up to 91% if the MGMT promoter is hypermethylated versus only 59% in wild type. MGMT methylation status has prognostic implications as well. In patients whose tumor has a MGMT promoter that is hypermethylated, the time to progression is 21.9 months versus 9.2 months in patients with an unmethylated, wild type MGMT promoter. Up to 60% of PP patients do not have any worsening of clinical symptoms. However, the remaining 40% of patients may develop new seizures, signs of increased intracranial pressure, or rarely, focal neurologic deficit. On imaging, there is enlargement or worsening of the enhancing component as well as the peritumoral abnormal T2 hyperintense areas of the tumor, as compared to the immediate, baseline postoperative MRI. According to RANO criteria, any abnormal enlargement of the enhancing component of the tumor within three months of completing chemoradiation is considered PP; therefore, no change in management is implemented, in the absence of any new enhancement or enhancement outside the radiation field. However, this three-month criterion has been questioned recently. Diffusion and perfusion imaging may be complementary as PP typically demonstrates increased ADC value on DWI and low rCBV on perfusion imaging in most cases; however, exception to this rule has been reported. A neuroradiologist can comfortably diagnose predominant tumor recurrence verses predominant PP; however, diagnosis is challenging if there is an equal mixture of these two conditions. In a recent study, it was shown that perfusion MRI with ferumoxytol is more sensitive for differentiating between these two.

Key Points  Worsening of the enhancing component of a tumor with or without worsening of the peritumoral FLAIR abnormality within three months of completing concurrent TMZ and radiotherapy in patients with GBM is highly suggestive of pseudotumor.  Pseudotumor is more highly suggested if the promoter region of the MGMT gene of the tumor cell is hypermethylated.

Part VI. Central Nervous System Tumors: Case 93

 Enhancement in a region outside of the original irradiated field is suggestive of tumor progression.

Suggested Reading Nasseri M, Gahramanov S, Netto JP, et al. Evaluation of pseudoprogression in patients with glioblastoma multiforme using

dynamic magnetic resonance imaging with ferumoxytol calls RANO criteria into question. Neuro Oncol 2014; 16(8): 1146–54. Radbruch A, Fladt J, Kickingereder P, et al. Pseudoprogression in patients with glioblastoma: clinical relevance despite low incidence. Neuro Oncol 2014; 17(1): 151–9.

437

CASE

Part VI

94

Central Nervous System Tumors Asim K. Bag

Clinical Presentation A 59-year-old man with a history of glioblastoma (GBM) presented with worsening orientation, seizures, and speech problems. He had previously undergone standard treatment for GBM, including partial resection, concurrent temozolomide and radiotherapy, and subsequent adjuvant temozolomide, nine months prior to onset of recent difficulties. Magnetic

resonance imaging was obtained (Fig. 94.1). Based on the clinicoradiologic profile, tumor recurrence was diagnosed and bevacizumab therapy was started. His symptoms improved significantly with this new therapy and an MRI was repeated (Fig. 94.2) two months after initiation of bevacizumab therapy. Three months after starting bevacizumab, he developed worsening symptoms again, and an MRI was obtained (Fig. 94.3).

Imaging (A)

(B)

Fig. 94.1 (A) Axial postcontrast T1WI and (B) Axial FLAIR image through the frontal lobe (before initiation of bevacizumab therapy).

Advanced Neuroradiology Cases, ed. Amaral et al. Published by Cambridge University Press. © Cambridge University Press 2016.

439

Part VI. Central Nervous System Tumors: Case 94

(A)

(B)

440

Fig. 94.2 (A) Axial postcontrast T1WI and (B) Axial FLAIR image through the frontal lobe (two months after initiation of bevacizumab).

Part VI. Central Nervous System Tumors: Case 94

(A)

Fig. 94.3 (A) Axial postcontrast T1WI and (B) Axial FLAIR image through the frontal lobe (three months after initiation of bevacizumab).

(B)

441

Part VI. Central Nervous System Tumors: Case 94

Pseudoresponse Associated with Antiangiogenic Therapy Primary Diagnosis Pseudoresponse associated with antiangiogenic therapy

Differential Diagnosis True tumor response

Imaging Findings Fig. 94.1: (A) Axial postcontrast T1WI through the frontal lobe (obtained before initiation of bevacizumab) demonstrated a heterogeneously enhancing mass centered in the left frontal lobe. Postsurgical changes are evident in the left frontotemporal regions. (B) Axial FLAIR image through the same level demonstrates FLAIR hyperintensity diffusely involving the left frontal lobe that also extends to the anterior temporal lobe. It can be noted that there is mass effect to the third ventricle, as well as the anterior interhemispheric fissure. Fig. 94.2: (A) Axial postcontrast T1WI through the frontal lobe (two months after initiation of bevacizumab) demonstrates significant improvement of the left frontal lobe heterogeneous enhancement. (B) Axial FLAIR image through the same level demonstrates minimal, if any, improvement of the FLAIR hyperintense area. The mass effect to the third ventricle, as well as the anterior interhemispheric fissure, is not evident on this scan. Fig. 94.3: (A) Axial postcontrast T1WI through the frontal lobe (three months after initiation of bevacizumab) demonstrates further enlargement of the tumor’s enhancing component as compared to both the pretreatment and twomonth follow-up. (B) Axial FLAIR shows there is also interval worsening of the FLAIR hyperintense areas. Mass effect to the third ventricle is noticeable.

Discussion Significant improvement of a tumor’s enhancing component and FLAIR hyperintense areas (after initiation of the bevacizumab therapy) that demonstrate subsequent worsening of enhancement and FLAIR hyperintense areas, on follow-up

442

scans, is consistent with pseudoresponse. As the tumor reappears on the final follow-up image, true tumor response is ruled out as a diagnosis. Bevacizumab is a humanized monoclonal antibody against vascular endothelial growth factor (VEGF). Bevacizumab therapy elicits a rapid decrease of contrast enhancement due to its anti-permeability effect due to repair of the altered blood-brain barrier. Associated subtle improvement of the surrounding FLAIR abnormality, mass effect, and vasogenic edema may also be present. Perfusion images may demonstrate decreased Ktrans and rCBV values. The apparent response on postcontrast images induced by the effect of bevacizumab is known as pseudoresponse. It is typically reversed (worsening of tumor) on follow-up scans. Interpretation of post-treatment GBM imaging is challenging and is evolving in response to the clinical use of multiple drugs targeting different aspects of tumor physiology. Appropriate knowledge of the treatment is required for accurate assessment of the images. For this reason, a RANO criterion for evaluation of GBM treatment response also includes the abnormal FLAIR hyperintense areas within the assessment criteria, in addition to the contrastenhancing component.

Key Point  Any improvement of the contrast-enhancing component of high-grade gliomas with recent treatment with bevacizumab should always be interpreted cautiously as probable pseudoresponse.

Suggested Reading Fatterpekar GM, Galheigo D, Narayana A, Johnson G, Knopp E. Treatment-related change versus tumor recurrence in high-grade gliomas: a diagnostic conundrum–use of dynamic susceptibility contrast-enhanced (DSC) perfusion MRI. AJR Am J Roentgenol 2012; 198(1): 19–26. Hygino da Cruz LC, Jr., Rodriguez I, Domingues RC, Gasparetto EL, Sorensen AG. Pseudoprogression and pseudoresponse: imaging challenges in the assessment of posttreatment glioma. AJNR Am J Neuroradiol 2011; 32(11): 1978–85.

CASE

Part VI

95

Central Nervous System Tumors Asim K. Bag

Clinical Presentation A 72-year-old man presented with recently developed history of disorientation and worsening of headache. His right parietal lobe glioblastoma was treated by maximal safe resection followed by concurrent temozolomide and radiation and adjuvant temozolomide one year prior

to this presentation. Six months after the original tumor diagnosis, he developed recurrence and bevacizumab therapy was initiated. The patient was on bevacizumab therapy. His MRI after four months of bevacizumab therapy (Fig. 95.1) and after this presentation (Fig. 95.2) is shown below.

Imaging (A)

(B)

(B) (A)

Fig. 95.1 (A) Axial postcontrast T1WI through the ventricles and (B) Axial FLAIR image through the ventricles (four months after initiation of bevacizumab therapy).

Fig. 95.2 (A) Axial postcontrast T1WI through the ventricles and (B) Axial FLAIR image through the ventricles (six months after initiation of bevacizumab therapy).

Advanced Neuroradiology Cases, ed. Amaral et al. Published by Cambridge University Press. © Cambridge University Press 2016.

443

Part VI. Central Nervous System Tumors: Case 95

Distal Glioblastoma Recurrence after Antiangiogenic Therapy Primary Diagnosis Distal glioblastoma recurrence after antiangiogenic therapy

Differential Diagnoses Metachronous glioblastoma (GBM) Metastasis

Imaging Findings Fig. 95.1: (A) Axial postcontrast T1WI through the ventricles obtained four months after initiating bevacizumab therapy demonstrated subtle abnormal enhancement at the resection bed in the right parietal lobe without any nodular enhancing focus. No abnormal enhancement is noted in the genu of the corpus callosum. (B) Axial FLAIR image through the same level demonstrated FLAIR hyperintense areas in the right parietal lobe and in the periventricular white matter. There is no mass effect to the ventricle. No obvious FLAIR abnormality was noted in the genu of the corpus callosum. Fig. 95.2: (A) Axial postcontrast T1WI through the ventricles obtained six months after initiating bevacizumab therapy demonstrated intense enhancement of the left side of the genu of the corpus callosum, far away from the primary tumor site. Another subtle enhancement can also be noted at the subcortical left frontal lobe. (B) FLAIR image through the same level demonstrates new expansion and hyperintensity of the genu of the corpus callosum with mass effect to the left frontal horn. It can be noted that there is no significant change in the primary tumor site on both postcontrast as well as FLAIR images. There was also increased rCBV and diffusion restriction at the left side of the genu of the corpus callosum (not shown).

Discussion New area of abnormal tumor-like enhancement distant from the primary tumor site in patients with known diagnosis of GBM and long history of treatment with bevacizumab is worrisome for distal GBM recurrence. Metachronous GBM is theoretically possible but is extremely uncommon outside the setting of bevacizumab therapy. The patient had no known cancer other than GBM, ruling out the possibility of brain metastasis from systemic cancer. Additionally, imaging features are not suggestive of metastasis. In GBM, bevacizumab has been shown to induce dramatic improvement in tumor contrast enhancement as well as improvement of the peritumoral FLAIR abnormality and mass effect. Quality of life is significantly improved and time to progression is increased when bevacizumab is administered either alone or in combination with chemotherapy. While the initial response rate to bevacizumab in GBM patients is quite remarkable, it does not have any survival benefit. Additionally, some tumors do not respond to bevacizumab at all,

444

and some tumors demonstrate rapid development of bevacizumab resistance. Although the underlying mechanism is not clearly understood, there are several explanations for the development of resistance to bevacizumab. The first hypothesis posits that evasive resistance to bevacizumab develops by recruiting alternative proangiogenic signaling, protecting the tumor vasculature with recruitment of proangiogenic inflammatory cells, or by increasing protective pericyte coverage and accentuated invasiveness of the tumor cells towards a more favorable microenvironment – either to co-opt a normal blood vessel or to metastasize to a distant place. The second hypothesis suggests that intrinsic resistance to bevacizumab therapy develops. The receptor tyrosine kinase, MET, a cellular receptor for hepatocyte growth factor, has recently been identified as a key mediator of tumor invasion, following angiogenesis inhibition. Several other molecular mechanisms have been identified that promote resistance to antiangiogenic therapy. In addition, antiangiogenic therapy activates GBM stem cells and can cause an angiogenesis-independent environment of tumor growth accompanied by the upregulation of proinvasive genes, dedifferentiation into endothelial cells, and acquisition of a more mesenchymal phenotype that favors invasive growth. The incidence of invasion following bevacizumab therapy has been debated and its variability is mostly due to the lack of a universal definition of recurrence. Although GBMs treated with standard concurrent temozolomide and radiation therapy followed by adjuvant temozolomide demonstrate an increased incidence of recurrent and progressive disease outside of the defined contrast enhancement in up to 20% of patients, the incidence of distal recurrence outside of the defined contrast enhancement in patients treated with bevacizumab has been shown to be as high as 30–60%. This high incidence of distal recurrence in GBM patients with bevacizumab treatment has great clinical importance. Careful evaluation of the entire brain is essential for early identification of distal recurrence.

Key Point  A mass with tumor-like enhancement, diffusion restriction, and increased perfusion distant from the primary tumor site in a GBM patient on bevacizumab is suggestive of distal tumor recurrence.

Suggested Reading Gilbert MR, Dignam JJ, Armstrong TS, et al. A randomized trial of bevacizumab for newly diagnosed glioblastoma. N Engl J Med 2014; 370(8): 699–708. Lu KV, Bergers G. Mechanisms of evasive resistance to anti-VEGF therapy in glioblastoma. CNS Oncol 2013; 2(1): 49–65. Soda Y, Myskiw C, Rommel A, Verma IM. Mechanisms of neovascularization and resistance to anti-angiogenic therapies in glioblastoma multiforme. J Mol Med (Berl) 2013; 91(4): 439–48.

CASE

Part VI

96

Central Nervous System Tumors Asim K. Bag, Philip R. Chapman

Clinical Presentation A 25-year-old previously healthy man presented with a onemonth history of excessive and inappropriate sleepiness (e.g., sleeping at work and falling asleep during interviews), strange thoughts, hallucinations, loss of short-term memory, and increased anxiety. His mother denied history of congenital infection and noted that her son was extremely lethargic. He

also reported an unintentional 30-pound weight gain. Physical exam revealed no fever or neck rigidity, and vital signs were normal. Routine hematologic studies were normal. Routine CSF analysis, PCR studies, and CSF-PCR for herpes viruses and CSF FTA-ABS tests were also negative. Psychogenic etiology for symptoms was considered and MRI (shown below) was performed to rule out structural lesions.

Imaging Fig. 96.1 Coronal T2WI through the anterior commissure.

Fig. 96.3 Postcontrast axial T1WI sequence through the level of the hypothalamus.

Fig. 96.2 Axial FLAIR image through the hypothalamus.

Fig. 96.4 Postcontrast coronal T1WI sequence through the same level as Fig. 96.1.

Advanced Neuroradiology Cases, ed. Amaral et al. Published by Cambridge University Press. © Cambridge University Press 2016.

445

Part VI. Central Nervous System Tumors: Case 96

Anti-Ma2-Associated Paraneoplastic Encephalitis Primary Diagnosis Anti-Ma2-associated paraneoplastic encephalitis (AMAPE)

Differential Diagnoses Limbic encephalitis due to other causes Herpes encephalitis Metabolic/toxic encephalopathy Neurosyphilis

Imaging Findings Fig. 96.1: Coronal T2WI through the anterior commissure demonstrates diffuse, confluent abnormal T2 signal involving the bilateral hypothalamus and anterior thalamus. Fig. 96.2: Axial FLAIR image through the hypothalamus demonstrates abnormal FLAIR signal and subtle swelling involving the bilateral hypothalamus and anterior thalamus. Fig. 96.3: Postcontrast axial T1WI sequence through the level of the hypothalamus demonstrates abnormal patchy enhancement involving the bilateral hypothalamic regions. Fig. 96.4: Postcontrast coronal T1WI sequence through the same level as Fig. 96.1 demonstrates patchy enhancement involving the bilateral hypothalamus and the adjacent brain tissue at the margin of the third ventricle in the areas of abnormal T2 signal.

Discussion Subacute-onset narcolepsy, memory problems, and weight gain suggest the presence of mixed diencephalitis and limbic encephalitis. Confirmed by MRI, these symptoms are also highly suggestive of paraneoplastic diencephalitis/limbic encephalitis, which in a young male patient is usually due to AMAPE, secondary to testicular germ cell tumor. In this patient, testicular ultrasound demonstrated a 4 cm hard mass in his right testicle and bilateral testicular microlithiasis. Further CSF analysis was positive for anti-Ma2 antibody that confirmed the diagnosis. A common manifestation of many paraneoplastic syndromes, limbic encephalitis has been characterized by the identification of multiple autoantibodies manifested in response to various systemic cancers. These include antibodies against 1) the N-methyl-D-aspartate receptor in ovarian teratoma, 2) the α-amino-3-hydroxy-5-methyl-4-isoxazolepropionic acid receptor in breast cancer and small cell lung cancer (SCLC), 3) the collapsin response mediator protein-5 in SCLC and thymoma, 4) the GABAB receptor in SCLC, 5) the antineuronal nuclear antibody type 1 (Hu) in SCLC, and of course, antibodies against Ma2 in testicular germ cell tumors. However, this patient only presented with testicular germ cell tumors and no other cancers. Limbic encephalitis can be due to viral infection, especially herpes simplex virus in immune competent patients or HHV-6 infections in immunocompromised patients. However, this patient did not have any systemic symptoms of infection or

446

any known immunodeficiency. The nature of subacute symptoms and negative CSF analysis for herpes viruses eliminated an infectious etiology. Neurosyphilis has no specific imaging abnormality and this patient did not have any history of congenital syphilis. Central nervous system manifestation of acquired syphilis within 25 years is extremely rare. Negative CSF FTA-ABS results further rule out the possibility of neurosyphilis. The patient was an otherwise healthy male, was not undergoing treatment for systemic disease, and he did not use recreational drugs, thus eliminating likelihood of metabolic/ toxic causes of limbic encephalitis. AMAPE usually occurs in young men with testicular germ cell tumors, most commonly non-seminomatous germ cell tumor. Other rare causes of AMAPE include breast cancers in women and non-small cell lung cancer in both older men and women. AMAPE most commonly presents with any combination of limbic encephalitis or diencephalitis and brainstem encephalitis in young male patients prior to the primary tumor diagnosis, in as many as 62% of cases. The association of AMAPE and testicular tumor is very strong. In fact, it is possible that young men with AMAPE will have a microscopic tumor below the detection threshold of imaging techniques. For this reason, both ultrasound and MRI should be performed to reveal potential microcalcification or subtle enhancement on MRI. Empiric orchiectomy has also been suggested if there is clinical deterioration, testicular enlargement, previous cryptorchidism, or evidence of testicular microlithiasis, in the absence of other tumors. Neurologic manifestations of AMAPE include isolated limbic encephalitis, diencephalitis, or brainstem encephalitis, or any combination of these diseases. The most common presentation is brainstem encephalitis with either limbic encephalitis or diencephalitis. Seizure is another common manifestation of AMAPE. Common manifestations of limbic encephalitis include short-term memory deficits, confusional states, and decline of cognitive functions. Weight gain or loss, excessive sleepiness, narcolepsy-cataplexy, endocrinopathy, and hyperthermia have been described in cases of diencephalitis. Abnormal eye movements, opsoclonus, oculogyric crisis, Horner’s sign, dysarthria and dysphagia, facial weakness, and sensorineural hearing loss have been described as common presentation of brainstem encephalitis associated with AMAPE. Although brainstem encephalitis with limbic encephalitis or diencephalitis combination is the most common clinical manifestation of AMAPE, the most common imaging abnormality is a combination of limbic encephalitis and diencephalitis. Abnormal T2 signal is seen in the mesial temporal lobe(s) with or without enhancement. This can be unilateral or bilateral. Similar imaging abnormalities are noted in the hypothalamus with varying extension to the basal ganglia and thalamus in diencephalitis. Abnormal T2 signal in the midbrain, pons, anterior medulla, and middle cerebellar peduncles has been described in association with brainstem encephalitis. Anti-Ma2 antibody is seen in the CSF as well as in serum.

Part VI. Central Nervous System Tumors: Case 96

Key Point  Acute/subacute-onset diencephalitis/limbic encephalitis in a previously healthy young male patient is usually due to AMAPE and warrants a search for testicular germ cell tumor.

Suggested Reading Dalmau J, Graus F, Villarejo A, et al. Clinical analysis of anti-Ma2associated encephalitis. Brain 2004; 127(Pt 8): 1831–44.

Dauvilliers Y, Bauer J, Rigau V, et al. Hypothalamic immunopathology in anti-Ma-associated diencephalitis with narcolepsy-cataplexy. JAMA Neurol 2013; 70(10): 1305–10. Saket RR, Geschwind MD, Josephson SA, Douglas VC, Hess CP. Autoimmune-mediated encephalopathy: classification, evaluation, and MR imaging patterns of disease. Neurographics 2011; 1(1): 2– 16(15). Tenner L, Einhorn L. Ma-2 paraneoplastic encephalitis in the presence of bilateral testicular cancer: diagnostic and therapeutic approach. J Clin Oncol 2009; 27(23): e57–8.

447

CASE

Part VI

97

Central Nervous System Tumors Fabrício Guimarães Gonçalves, Asim K. Bag, Lázaro Luís Faria do Amaral

Clinical Presentation A 67-year-old woman with a long history of headache presented to the emergency department with new-onset convulsions. Her husband told the emergency department physician that his wife had also complained about visual problems. No signs or symptoms of systemic cancer were noted. A noncontrast head CT scan (Fig. 97.1) demonstrated a right occipital lobe mass. Based on the CT results, a contrast-enhanced brain MRI was obtained (Figs. 97.2–97.4).

Imaging

Fig. 97.1 Axial CT scan of the head without contrast through the level of occipital lobes.

Fig. 97.2 Axial T1WI MRI without contrast through the level of occipital lobes.

Advanced Neuroradiology Cases, ed. Amaral et al. Published by Cambridge University Press. © Cambridge University Press 2016.

449

Part VI. Central Nervous System Tumors: Case 97 Fig. 97.3 Axial T2WI MRI without contrast through the level of occipital lobes.

Fig. 97.4 Axial T1WI MRI with contrast through the level of occipital lobes.

450

Part VI. Central Nervous System Tumors: Case 97 Fig. 97.5 Lateral projection of mid-arterial phase image of digital subtraction angiogram of left external carotid artery injection.

451

Part VI. Central Nervous System Tumors: Case 97

Microcystic Meningioma Primary Diagnosis Microcystic meningioma

Differential Diagnoses Typical meningiomas/angiomatous meningioma Astrocytoma Metastatic clear cell carcinoma Epidermoid Chordoma/chondrosarcomas

Imaging Findings Fig. 97.1: Axial CT scan of the head, without contrast, through the level of the occipital lobes demonstrated a large, oval, homogeneously hypodense mass in the right posterior temporal and occipital lobes with mass effect to the right lateral ventricle, without any leftward deviation of the posterior falx. The hypodensity is more like CSF than fat. Fig. 97.2: Axial T1WI image, without contrast, through the same level demonstrated a large hypointense mass in the right posterior temporal and occipital lobes, with numerous intratumoral cysts associated with mass effect. Fig. 97.3: Axial T2WI of the mass better demonstrated the cystic nature of the mass. The mass effect to the ventricle is well appreciated. It is to be noted that there is no peritumoral abnormal T2 signal. Fig. 97.4: Axial T1WI with contrast demonstrated heterogeneous, marked enhancement of the septi surrounding the hypointense cysts resulting in a honeycomb appearance of the dural-based mass. Fig. 97.5: Selective digital subtracted angiography image demonstrated tumoral blush from external carotid artery injection.

Discussion Although the lesion is homogeneously hypodense on unenhanced CT scan images, the microcystic appearance on all the MRI sequences is diagnostic of microcystic meningioma (MM). The diffusion image demonstrated T2 shine-through effect (not shown), further suggesting the presence of microcysts in the tumor. On external carotid angiography, there was a prominent tumoral blush in the arterial phase, suggesting intense vascularity and further supporting the diagnosis of MM. The persistent headache with visual abnormality is due to a posteriorly located mass close to the visual pathway. The mass is homogeneously hypodense on unenhanced CT scan. This appearance is non-specific and can be seen in any cystic lesion of the brain, such as astrocytomas, and benign intraparenchymal cysts, such as neuroepithelial or epidermoid cysts. Neuroepithelial cysts are usually incidental. No diffusion restriction (not shown) was present to indicate that the lesion was epidermoid. Although it is hard to delineate intra- versus extra-axial masses on T2-weighted sequence, dural-based enhancement is better appreciated on the postcontrast scan, suggesting that the tumor is extra-axial. Typical meningiomas are not cystic. High signal on T2-weighted sequence with

452

heterogeneous enhancement can be seen in chordoma/chondrosarcomas; however, the location of the mass in this patient is not a known location for these types of tumors. Chondroma can have a similar appearance, but usually the chondroid matrix is appreciated on non-contrast head CT images. Furthermore, chondromas demonstrate minimal peripheral enhancement, rather than the honeycomb-type enhancement noted in this patient’s images. The absence of any known malignancy rules out the possibility of metastasis. The angiomatous variant of meningioma may demonstrate a similar extent of edema of the adjacent brain. However, it should not be confused with the MM, as the angiomatous variant typically demonstrates intense homogeneous tumor enhancement due to presence of an excessive amount of variably sized blood vessels, as compared to tumor cells. Meningiomas account for about 24–30% of all primary intracranial tumors. Meningiomas typically occur in middleaged and older adult patients with a peak in the sixth and seventh decades. However, meningiomas also occur in pediatric and elderly populations. Meningiomas typically affect females more than males. Most meningiomas involve the cerebral convexities, along the falx and venous sinuses. Other common sites include the olfactory groove, sphenoid ridges, sellar/parasellar regions, optic nerve sheath, petrous ridges, tentorium, and posterior fossa. Most meningiomas are benign, WHO grade 1 lesions and grow slowly over time. The presenting symptoms are secondary to mass effect to the adjacent brain. Specific neurologic deficits depend upon the location of the tumor. Headache and seizures are other common presentations. On imaging, most meningiomas are usually iso- to hyperdense on non-contrast CT scan images, and typically demonstrate intense enhancement with contrast on both CT and MRI. Frequently there is thickening and enhancement of the adjacent dura, the dural tail sign, which may not always represent extension of tumor. This can be reactive as well. This typical imaging appearance may not be present in some histologic subtypes of meningiomas. Many histologic subtypes of meningiomas have been described. Microcystic meningioma is a rare variant of meningiomas with microcystic changes. This variant is typically benign, WHO grade 1. These tumors were originally described by Masson as humid meningiomas, based on their gross morphology. This variant is characterized by cells with thin, elongated processes that surround a pale eosinophilic mucinous fluid – the microcysts. Microcystic meningiomas are more commonly found in the convexity, along the falx cerebri. Although any age group can be affected by MM, they more commonly develop in adults, usually after the fourth decade of life, and are rare in children. Presenting symptoms are similar to other subtypes of meningiomas. On CT, MM can show iso- to hypodensity and variable patterns of enhancement. Occasionally these tumors can be confused with brain edema because of its hypodense appearance. Hyperostosis of the adjacent bone is common and can be

Part VI. Central Nervous System Tumors: Case 97

seen in approximately one-half of patients. Edema of the adjacent brain is commonly seen. The variable and heterogeneous patterns of enhancement, disproportionate edema of the adjacent brain, and lack of dural tail often lead physicians to confuse MM with aggressive lesions. On MRI, typical MM demonstrates low signal on T1weighted and high signal on T2-weighted images, with contrast enhancement in the majority of the cases. Owing to high mucinous fluid content, MM can follow CSF signal intensity on precontrast T1- and T2-weighted sequences. The presence of a broad-based cystic tumor in a typical location for a meningioma may be helpful in the diagnosis of a MM. Higher incidence of edema is another characteristic and common MRI finding of MM, compared to typical presenting meningiomas. Significant edema of adjacent brain is seen in up to 87.5% of patients with meningiomas and may be seen even in small lesions. Reasons for the disproportional edema are not fully understood and it has been suggested that high levels of vascular endothelial growth factor (VEGF) expression and cortical penetration with arachnoid disruption may play a role in edema formation. Variable enhancement pattern on MRI has been described in MM. Some lesions may present with only peripheral enhancement, which may have a honeycomb-like enhancement pattern, or may appear as a

large, heterogeneously enhancing mass with both solid and cystic components.

Key Point  A dural-based cystic lesion with heterogeneous enhancement and hyperostosis of the adjacent bones are suggestive of MM. Edema of the adjacent brain is more commonly seen than in most of the other subtypes of benign meningiomas; otherwise, clinical presentation is similar to other meningiomas.

Suggested Reading Manwaring J, Ahmadian A, Stapleton S, et al. Pediatric microcystic meningioma: a clinical, histological, and radiographic case-based review. Childs Nerv Syst 2013; 29(3): 361–5. Matsushima N, Maeda M, Takamura M, et al. MRI findings of atypical meningioma with microcystic changes. J Neurooncol 2007; 82(3): 319–21. Paek SH, Kim SH, Chang KH, et al. Microcystic meningiomas: radiological characteristics of 16 cases. Acta Neurochir (Wien) 2005; 147(9): 965–72. Watts J, Box G, Galvin A, et al. Magnetic resonance imaging of meningiomas: a pictorial review. Insights Imaging 2014; 5(1): 113–22.

453

CASE

Part VI

98

Central Nervous System Tumors Leslie Lamb, Prasad B. Hanagandi

Clinical Presentation A 72-year-old woman presented with a one-year history of gradual onset, and gradually progressive unilateral left arm and leg weakness and incoordination.

Imaging (A)

(B)

Fig. 98.1 (A) Axial non-contrast CT image at the level of frontoparietal convexities and (B) Coronal non-contrast CT image at the level of body of lateral ventricles.

Advanced Neuroradiology Cases, ed. Amaral et al. Published by Cambridge University Press. © Cambridge University Press 2016.

455

Part VI. Central Nervous System Tumors: Case 98

(A)

(B)

Fig. 98.2 (A) Axial T1WI at the level of frontoparietal convexities and (B) Right parasagittal T1WI.

Fig. 98.3 Axial T2WI at the level of frontoparietal convexities.

456

Fig. 98.4 Axial FLAIR at the level of frontoparietal convexities.

Part VI. Central Nervous System Tumors: Case 98

(A)

(B)

Fig. 98.5 (A) Diffusion-weighted image and (B) ADC map at the level of frontoparietal convexities.

(A) (B)

Fig. 98.6 (A) Axial T1WI postgadolinium at the level of frontoparietal convexities and (B) Coronal T1WI fat-suppressed postgadolinium.

457

Part VI. Central Nervous System Tumors: Case 98

Dural Convexity Chondroma Primary Diagnosis Dural convexity chondroma

Differential Diagnoses Meningioma Chondrosarcoma Extra-axial metastasis Calcified hematoma Extra-axial neurinoma

Imaging Findings Fig. 98.1: (A) Axial non-contrast CT and (B) Coronal images showed a heterogeneous, spontaneously hyperdense extra-axial mass lesion arising from the right frontal convexity. Fig. 98.2: (A) Axial T1WI and (B) Sagittal T1WI showed a hypointense lesion. Fig. 98.3: Axial T2WI and Fig. 98.4: Axial FLAIR sequences showed that the lesion is hyperintense with mass effect. Fig. 98.5: (A) DWI and (B) ADC sequences demonstrated facilitated diffusion. Fig. 98.6: (A) Axial T1WI postgadolinium and (B) Coronal T1WI showed moderate and heterogeneous enhancement.

Discussion Differential diagnosis of a well-defined, hyperdense extra-axial tumor that exhibits moderate enhancement, and lacks perilesional edema, dural tail sign, and diffusion restriction should include chondroma, or other extra-axial lesions. Chondromas are rare benign tumors that can arise at sites in the body containing cartilaginous bone. They are predominantly seen in short, tubular bones, especially metacarpals and phalanges; they are known to have an association with Ollier disease, Maffucci syndrome, and Noonan syndrome. In the head and neck region, they are frequently seen at the skull base, arise from the embryonic cartilaginous remnants, and involve the synchondroses of middle cranial fossa. They tend to have a predilection for spheno-ethmoidal, spheno-petrosal, spheno-parietal, and petro-occipital regions, which account for the majority of all cases. Intracranial chondromas are exceedingly rare and constitute approximately 0.2–0.5% of all tumors. Approximately 15–20% of chondromas originate from the dural lining around the convexity. Isolated case reports describe less common locations including the falx, choroid plexus, and brain parenchyma. These tumors tend to occur between the second and sixth decades of life, with a peak incidence in the third decade but no gender preference. Although uncommon in the pediatric age group, they have been known to occur as early as 15 months of age. The histopathogenesis of intracranial chondromas remains controversial; however, four theories have been proposed: 1) metaplasia of meningeal fibroblasts; 2) metaplasia of perivascular mesenchymal tissue; 3) development from heterotopic

458

chondrocytes; and 4) cartilaginous activation of fibroblasts by trauma or inflammation. Clinical presentation is location-dependent and often delayed as they are slow-growing tumors. Non-specific symptoms are due to mass effect and raised intracranial pressure, which cause headache, seizures, cranial nerve palsies, deafness, visual impairment, limb weakness, and unsteady gait. Computed tomography and MRI have key imaging features differentiating chondromas from meningiomas and other extra-axial neoplasms. On CT, the tumor density depends on the extent of calcification. Calcifications are common, occurring in 60% of all intracranial chondromas. Stippled, flocculent, and ring calcifications are often found in convexity chondromas. Falx chondromas, however, typically lack calcifications. Based on their CT imaging features they have been classified into two broad categories. Type I lesions are referred to as classic chondroma, which are isodense and homogeneous; type II tumors have central hypodense areas. Chondromas exhibit mild to moderate degree of enhancement, after intravenous contrast administration, especially on the delayed phase, and often lack the dural tail sign. Peritumoral edema is very rare. These key features are vital in differentiating a chondroma from a meningioma. However, 10–15% of meningiomas have atypical peripheral enhancement with cystic degeneration due to necrosis and lipidization. Cranial chondromas are avascular on angiography, in contrast to meningioma. On MR imaging, the tumor is iso- to hypointense on T1WI and hyperintense on T2WI and FLAIR images. Patchy, honeycomb pattern of enhancement on postgadolinium studies resembling punica granatum seeds sign has been described. Lack of diffusion restriction is another important imaging feature. Chondrosarcomas are typically hypo- to isointense on T1 and hyperintense on T2 with strong heterogeneous enhancement. Extra-axial metastases have similar imaging features to meningioma in terms of enhancement, perilesional edema, and lack the typical calcification seen in chondroma. Calcified hematoma can have heterogeneous or usually peripheral calcification, enhancement, and exhibit T2 shortening on gradient and susceptibility sequence. Extra-axial neurinomas have a heterogeneous T2 hyperintense signal with significant enhancement and often lack calcification. Complete surgical resection of the tumor and dural attachment is the treatment of choice with typically favorable outcome. Postsurgical patients have good long-term prognosis and no recurrence in cases with total lesion removal. Cellular atypia in resected specimens is an indicator of malignant transformation. Chondrosarcomas are known to have almost 20% recurrence rates. Chondromas do not respond to radiation; thus, radiation should not be used to treat patients with incomplete resection or non-resectable tumors. On the contrary, radiotherapy can increase the risk of malignant transformation.

Part VI. Central Nervous System Tumors: Case 98

Key Points  A well-defined, moderately enhancing, extra-axial lesion lacking a dural tail sign, perilesional edema sign, and diffusion restriction are the important imaging features that differentiate a chondroma from meningioma and other extra-axial lesions.  Chondromas characteristically feature a flocculent or stippled ring pattern of calcification on CT scan that further aids in their diagnosis.

Suggested Reading Abeloos L, Maris C, Salmon I, et al. Chondroma of the dural convexity: a case report and literature review. Neuropathology 2012; 32(3): 306–10. Colpan E, Attar A, Erekul S, Arasil E. Convexity dural chondroma: a case report and review of the literature. J Clin Neurosci 2003; 10(1): 106–8.

Duan F, Qiu S, Jiang J, et al. Characteristic CT and MRI findings of intracranial chondroma. Acta Radiol 2012; 53(10): 1146–54. Erdogan S, Zorludemir S, Erman T, et al. Chondromas of the falx cerebri and dural convexity: report of two cases and review of the literature. J Neurooncol 2006; 80(1): 21–5. Fountas KN, Stamatiou S, Barbanis S, Kourtopoulos H. Intracranial falx chondroma: literature review and a case report. Clin Neurol Neurosurg 2008; 110(1): 8–13. Kawabata Y, Miyake H, Horikawa F. A solitary convexity dural chondroma: the proposed role of diffusion-weighted MR imaging in the differential diagnosis of intracranial chondroma and meningioma. A case report. Neuroradiol J 2010; 23(4): 496–500. Nakayama M, Nagayama T, Hirano H, Oyoshi T, Kuratsu J. Giant chondroma arising from the dura mater of the convexity. Case report and review of the literature. J Neurosurg 2001; 94(2): 331–4.

459

CASE

Part VI

99

Central Nervous System Tumors Stephanie Lam, Prasad B. Hanagandi, Jeffrey Chankowsky

Clinical Presentation A 65-year-old woman presented with a six-year history of a long-standing headache and visual field defects that worsened over the last six months with new-onset nausea and vomiting.

Imaging (A)

(B)

Fig. 99.1 (A) Axial non-contrast CT at the level of third and lateral ventricles and (B) midsagittal reformatted CT of the head.

Advanced Neuroradiology Cases, ed. Amaral et al. Published by Cambridge University Press. © Cambridge University Press 2016.

461

Part VI. Central Nervous System Tumors: Case 99 Fig. 99.2 Midsagittal T1WI of the head.

Fig. 99.3 Axial T2WI of the head at the level of third and lateral ventricles.

462

Part VI. Central Nervous System Tumors: Case 99

(B) (A)

Fig. 99.4 (A) DWI and (B) ADC images at the level of third and lateral ventricles.

(A)

(B)

Fig. 99.5 (A) Axial postcontrast image through the level of third and lateral ventricles. (B) Midsagittal postcontrast image.

463

Part VI. Central Nervous System Tumors: Case 99

Third Ventricular Craniopharyngioma Primary Diagnosis Third ventricular craniopharyngioma

Differential Diagnoses Colloid cyst Germ cell tumor Glioma Lymphoma Chordoid glioma Choroid plexus papilloma

Imaging Findings Fig. 99.1: (A) Axial non-contrast and (B) Sagittal reformatted CT images of the head showed a dense, heterogeneous, solidcystic lesion with foci of calcification occupying the third ventricle. Fig. 99.2: Sagittal T1WI of the head. Fig. 99.3: Axial T2WI of the head showed a solid-cystic, mixed signal intensity lesion in the third ventricle. Fig. 99.4: (A) There is no evidence of diffusion restriction or corresponding changes on ADC map (B). Fig. 99.5: (A) Axial postcontrast and (B) Midsagittal postcontrast images showed heterogeneous enhancement. The pituitary gland is seen separate from the lesion. However, the optic chiasm is encased by the mass.

Discussion Findings of a mixed signal intensity lesion with heterogeneous enhancement (solid portion of lesion), in conjunction with history of a slowly progressive, symptomatic headache and visual disturbances, are typical of a craniopharyngioma. Craniopharyngiomas are often mixed solid and cystic lesions. Cystic areas may be iso-, hyper-, or hypointense, relative to brain tissue, on T1-weighted sequences with thin enhancing wall. The solid components of the lesion enhance heterogeneously. Calcifications are characteristic in suprasellar craniopharyngiomas, but are rarely observed in intraventricular craniopharyngiomas. If present, calcifications usually demonstrate susceptibility effects on gradient echo sequence. Common differential diagnoses for a third ventricular craniopharyngioma include colloid cyst, germ cell tumor, glioma, lymphoma, chordoid glioma, and choroid plexus papilloma. Colloid cysts are usually located in the roof of the third ventricle, are often hyperdense on non-contrast CT scan, hyperintense on T1, and iso- to hypointense on T2WI. However, the lack of calcification and enhancement in colloid cysts excludes this entity as the primary diagnosis. Hypothalamic germ cell tumors can arise from the infundibular stalk or from the floor of the third ventricle. They feature intense, homogeneous enhancement but lack cystic and calcified components, in contrast to the heterogeneous enhancement of the solid portion of craniopharyngiomas. Although hypothalamic glial tumors may have solid and cystic components, they feature marked homogeneous enhancement but lack calcification. Lymphomas are solid, avidly enhancing tumors and are

464

hypointense on T2WI with diffusion restriction, uncharacteristic of craniopharyngiomas. In adults, chordoid glioma has a predominantly solid component, and enhances significantly. Although a few cases of choroid glioma report the presence of cystic components and necrosis, the presence of calcification is uncommon. The third ventricle is an unusual location for choroid plexus papilloma, whose lesions feature cauliflower morphology, may contain flow voids, and enhance avidly, not seen in craniopharyngiomas. Craniopharyngiomas are benign, mixed epithelial tumors derived from the squamous epithelial remnants of Rathke’s pouch in the subpial space. They may arise at any point along the pituitary-hypothalamus axis, from the pituitary gland to the floor of the third ventricle. Rare ectopic locations include the third ventricle, nasopharynx, pineal gland, sphenoid sinus, and the clivus. Craniopharyngiomas account for 2.5–4% of all intracranial tumors. They present a bimodal age distribution, with peak incidence between 5 and 14 years and 50 and 74 years of age. Intraventricular craniopharyngiomas occur primarily in the adult age group. No gender preponderance has been identified. Headache, nausea, and vomiting are common presenting complaints in intraventricular craniopharyngiomas, due to obstructive hydrocephalus. Other clinical symptoms include hypothalamic, amnesic, behavioral, and gait disturbances, seen more frequently than in their extraventricular counterparts. Two main histologic types of craniopharyngiomas exist, adamantinomatous and squamous-papillary. Intraventricular craniopharyngiomas are predominantly of the squamouspapillary type, which tend to be smaller, round in shape, and primarily solid. When cysts are present, they are more often hypointense on T1WI. Overall, squamous-papillary craniopharyngiomas have a more heterogeneous appearance and enhancement pattern. In contrast, adamantinomatous craniopharyngiomas typically display multilobulated borders, with prominent cystic components that demonstrate hyperintense contents on T1WI. Suprasellar craniopharyngiomas may involve the third ventricle, to various degrees, without truly being intrinsic third ventricle craniopharyngiomas (ectopic tissue in the third ventricle from which a craniopharyngioma arises). A suprasellar craniopharyngioma may be wholly extraventricular, expand in the chiasmatic cistern, and push the floor of the third ventricle superiorly, mimicking an intraventricular position. Identification of the intact floor of the third ventricle overlying the lesion permits differentiation from a true intraventricular craniopharyngioma. Craniopharyngiomas may initially develop in the suprasellar compartment, but subsequently invade the third ventricle through the floor. Craniopharyngiomas may also arise directly from the floor of the third ventricle and expand into the ventricle. An intrinsic third ventricle craniopharyngioma is located entirely in the ventricle, and an intact floor can be identified on imaging. The hypothalamus is usually below the level of the craniopharyngioma, which is another useful characteristic for identifying the

Part VI. Central Nervous System Tumors: Case 99

intraventricular location of the tumor. Making this distinction is important for surgical planning.

Key Points  Craniopharyngiomas are usually mixed solid and cystic lesions with calcification. Intraventricular craniopharyngiomas have a greater tendency to be more solid and calcification is less common.  The heterogeneous appearance and enhancement of the solid portions of the craniopharyngioma, as well as the possible presence of cystic and calcific components, help distinguish it from other lesions of the third ventricle.  Intrinsic third ventricular craniopharyngiomas are rare and are located entirely in the ventricle; the floor of the third ventricle can be seen intact and separate from the tumor.

Suggested Reading Greiner FG, Takhtani D. Neuroradiology case of the day. Malignant mixed germ cell tumor with yolk sac and teratomatous components. Radiographics 1999; 19(3): 826–9. Pascual JM, Prieto R, Carrasco R, Barrios L. Displacement of mammillary bodies by craniopharyngiomas involving the third ventricle: surgical-MRI correlation and use in topographical diagnosis. J Neurosurg 2013; 119(2): 381–405. Saleem SN, Said AH, Lee DH. Lesions of the hypothalamus: MR imaging diagnostic features. Radiographics 2007; 27(4): 1087–108. Shogan P, Banks KP, Brown S. AJR teaching file: intraventricular mass. AJR Am J Roentgenol 2007; 189(6 Suppl): S55–7. Tayari N, Etemadifar M, Hekmatnia A, et al. Intrinsic third ventricular craniopharyngioma: a case report. Int J Prev Med 2011; 2(3): 178–85.

465

CASE

Part VI

100

Central Nervous System Tumors Stephanie Lam, Prasad B. Hanagandi, Lázaro Luís Faria do Amaral

Clinical Presentation A 63-year-old woman presented with a two-year history of long-standing headache and she reported a single recent episode of generalized tonic-clonic seizures. Routine hematologic laboratory workup was unremarkable.

Imaging Fig. 100.1 Axial CT image at the level of lateral ventricles, sylvian fissure, and insular cortex.

Advanced Neuroradiology Cases, ed. Amaral et al. Published by Cambridge University Press. © Cambridge University Press 2016.

467

Part VI. Central Nervous System Tumors: Case 100

(A) (B)

Fig. 100.2 (A) Axial T2WI and (B) Axial FLAIR images at the level of lateral ventricles, sylvian fissure, and insular cortex. Fig. 100.3 Axial DWI at the level of lateral ventricles, sylvian fissure, and insular cortex.

468

Part VI. Central Nervous System Tumors: Case 100

(B) (A)

Fig. 100.4 (A–B) Sagittal T1WI at the level of sylvian fissure and insular cortex.

Fig. 100.5 Perfusion map image at the level of lateral ventricles, sylvian fissure, and insular cortex.

469

Part VI. Central Nervous System Tumors: Case 100

Intraparenchymal Epidermoid Cyst Primary Diagnosis Intraparenchymal epidermoid cyst

Differential Diagnoses Arachnoid cyst Dermoid cyst Cystic neoplasm or metastasis Abscess Neuroepithelial cyst Hydatid cyst

Imaging Findings Fig. 100.1: Contrast CT axial image showed a lobulated hypodense and non-enhancing cystic lesion involving the left insular cortex and extending into the adjacent sylvian fissure. Few, small foci of peripheral calcification are noted. Fig. 100.2: (A) T2WI demonstrates an intra-axial lesion that appeared heterogeneously hyperintense on T2 and (B) FLAIR images with no evidence of vasogenic edema or significant mass effect. Fig. 100.3: DWI demonstrated diffusion restriction. Fig. 100.4: (A) T1WI sagittal images demonstrated a predominantly isointense lesion with few hyperintense foci that (B) showed peripheral enhancement. Fig. 100.5: Perfusion map shows hypoperfusion around the lesion.

Discussion The extra-axial intracranial epidermoid cyst has classic imaging features: isodense to CSF on CT, isointense to CSF on T1WI and T2WI, mixed signal on FLAIR images and with restricted diffusion. Their much rarer counterparts, intraparenchymal epidermoids, are more difficult to diagnose accurately, as they usually lack this typical appearance. The differential diagnoses include dermoid cyst, cystic neoplasm, neuroepithelial cyst, hydatid cyst, cystic metastasis, and brain abscess. Unlike epidermoid cysts, cystic neoplasms, metastases, and abscesses will often have enhancing rims and surrounding edema. However, neuroepithelial cysts, arachnoid cysts, and hydatid cysts can all share the CSF-like signal intensity of the epidermoid cyst. Nevertheless, unlike an epidermoid, a neuroepithelial cyst will have sharp and irregular borders, an arachnoid cyst will not demonstrate diffusion restriction, and a hydatid cyst will show the characteristic presence of a cyst and a pericyst. Dermoid cysts may cause restricted diffusion, but in contrast to epidermoids, they more commonly contain areas of fat, calcifications, and complex fluid. Intraparenchymal epidermoid cysts are benign lesions arising from neuroectodermal epithelial cells. Their exact pathogenesis is still a subject of debate; the most popular theory suggests that they arise from epithelial rests that become sequestered in the primitive cerebral hemisphere during neural tube closure in the third to fifth week of

470

embryogenesis. Intraparenchymal epidermoid cysts are rare, accounting for only 1.5% of intracranial epidermoid cysts and represent only 0.3–1.8% of primary intracranial tumors, with a female preponderance. Malignant transformation is exceedingly rare, although there have been case reports of transformation into squamous cell carcinoma, and of malignant melanoma arising within the lesion. Of the small number of intraparenchymal epidermoid cysts that have been reported in the literature, the most common locations in descending order of frequency are the frontal lobe, the temporal lobe, the parietal lobe, the occipital lobe, and the thalamus. Depending on the location of the tumor, various symptoms may arise, including headache, seizures, and hemiparesis. On CT, intraparenchymal epidermoids are usually homogeneously hypodense with well-defined and lobulated margins. They can occasionally demonstrate cystic or calcified areas. Other non-calcified hyperdense areas can also be observed and are thought to reflect dense cystic contents or fat saponification with hemosiderin-laden macrophages. Mass effect can be present to a variable degree depending on the size of the lesion but perilesional edema is typically absent. Enhancement is usually absent, though rare cases of peripheral enhancement have been reported. On MRI, intraparenchymal epidermoid cysts often exhibit iso- to slightly hyperintense signal intensity on T1WI and hyperintense signal on T2WI, though with more heterogeneity than typically seen in their extra-axial counterparts. Areas of high T1 and low T2 signal can be present, and are thought to represent a combination of proteinaceous material, debris, keratin, and incomplete calcification. Intralesional hemorrhage has also been reported. Hair-curled or whirlpool-like patterns are thought to represent desquamation of squamous epithelium. Diffusion-weighted imaging usually demonstrates heterogeneously hyperintense signal, with corresponding iso- or slightly hyperintense signal on ADC map. Minimal peripheral enhancement can be seen in some cases, making diagnosis difficult. Very rarely, restricted diffusion is absent, and in these circumstances, difficulties in establishing the diagnosis may arise.

Key Points  Intraparenchymal epidermoid cysts have slightly different imaging characteristics than extra-axial intracranial epidermoid cysts.  They generally exhibit CSF-like density/signal intensity, but have a more heterogeneous appearance. They can occasionally present areas of calcification, dense or proteinaceous fluid, hemorrhage, and minimal peripheral enhancement.  Lack of perilesional edema and the presence of diffusion restriction are important characteristics that differentiate intraparenchymal cysts from other differential diagnoses.

Part VI. Central Nervous System Tumors: Case 100

Suggested Reading Hu XY, Hu CH, Fang XM, Cui L, Zhang QH. Intraparenchymal epidermoid cysts in the brain: diagnostic value of MR diffusionweighted imaging. Clin Radiol; 63(7): 813–18. Iaconetta G, Carvalho GA, Vorkapic P, Samii M. Intracerebral epidermoid tumor: a case report and review of the literature. Surg Neurol 2001; 55(4): 218–22.

Kaido T, Okazaki A, Kurokawa S, Tsukamoto M. Pathogenesis of intraparenchymal epidermoid cyst in the brain: a case report and review of the literature. Surg Neurol 2003; 59(3): 211–16. Lian K, Schwartz ML, Bilbao J, et al. Rare frontal lobe intraparenchymal epidermoid cyst with atypical imaging. J Clin Neurosci 2012; 19(8): 1185–7.

471

CASE

Part VI

101

Central Nervous System Tumors Prasad B. Hanagandi, Jeffrey Chankowsky, Raquel del Carpio-O’Donovan

Clinical Presentation A 39-year-old woman presented with a two-year history of non-specific headache. Neurologic examination was unremarkable. Past medical history was otherwise uneventful.

Imaging Fig. 101.1 Axial non-contrast CT image through the anterior part of middle cranial fossa and at the level of suprasellar cistern.

Advanced Neuroradiology Cases, ed. Amaral et al. Published by Cambridge University Press. © Cambridge University Press 2016.

473

Part VI. Central Nervous System Tumors: Case 101 Fig. 101.2 Axial T1WI MRI at the level of cerebral peduncles.

Fig. 101.3 Axial T2WI MRI at the level of cerebral peduncles.

474

Part VI. Central Nervous System Tumors: Case 101

(A)

(B)

Fig. 101.4 (A) Axial DWI and (B) ADC map at the level of cerebral peduncles.

(A)

(B)

Fig. 101.5 (A) Axial postgadolinium at the level of anterior part of middle cranial fossa. (B) Coronal image through the frontal horns.

475

Part VI. Central Nervous System Tumors: Case 101

White Epidermoid Cyst Primary Diagnosis White epidermoid cyst

Differential Diagnoses Supratentorial neuroenteric cyst Dermoid cyst Hemorrhagic arachnoid cyst

Imaging Findings Fig. 101.1: Non-contrast CT scan of the brain showed a welldefined, spontaneously hyperdense extra-axial lesion in the right anterior middle cranial fossa temporal convexity with a small speck of calcification along the posterior aspect of the lesion (arrow). Fig. 101.2: Axial fat-saturated T1WI showed a homogeneously hyperintense lesion. Fig. 101.3: Axial T2WI demonstrated a profoundly hypointense lesion. Fig. 101.4: (A) DWI showed that the lesion lacks diffusion restriction. (B) ADC map demonstrated similar changes. Fig. 101.5: (A) Axial fatsaturated, postgadolinium and (B) Coronal fat-saturated, postgadolinium images demonstrated lack of enhancement.

Discussion Hyperdense lesion on CT exhibiting T1 hyperintense signal and profoundly hypointense on T2-weighted sequence with T2-dark through effect on DWI are the characteristics of a high proteinaceous content epidermoid cyst known as a white epidermoid. Neuroenteric cysts can have similar signal characteristics to white epidermoid cysts; however, they are usually found closer to the midline and are more commonly seen in the posterior fossa, anterior to the medulla, cerebellopontine angle cistern, and spine. Supratentorial neuroenteric cysts are rare. However, they can have similar imaging features on CT and T1WI images, but are often hyperintense on T2WI images. Dermoid cysts are another common differential option for white epidermoids; however, they lack the characteristic fat density on CT scans, and do not suppress on T1WI fatsaturated sequence because of their lack of lipid components. Although the convexity location may favor an arachnoid cyst, they typically follow CSF signal on all pulse sequences. However, they can be complicated by hemorrhage or presence of high proteinaceous contents and are associated with subdural hemorrhage. However, the lack of scalloping, a commonly seen feature of epidermoid cysts arising in the temporal convexity, makes the diagnosis of arachnoid cyst less likely. Epidermoid cysts are commonly located in the cerebellopontine angle cistern, fourth ventricle, and in suprasellarparasellar locations. The classic epidermoid cyst typically insinuates into the cisterns, has iso- to hypointense signal on T1WI sequence, and has hyperintense signal on T2WI, which is incompletely suppressed on FLAIR sequence. Epidermoid

476

cysts typically show diffusion restriction, which confirms their diagnosis. White epidermoid cyst is a rare variant of a congenital inclusion cyst. A typical epidermoid cyst contains desquamated keratin debris and is rich in cholesterol crystals, giving it a pearly white appearance. The underlying cause of the hyperdense appearance on CT or the T1 hyperintense signal in white epidermoid cysts is unknown. However, its characteristic color and appearance has been attributed to paramagnetic effects and hemorrhage, as well as the presence of high proteinaceous contents, calcium soaps with saponification of debris, polymorphonuclear leukocytes, and iron-containing pigment or ferro calcium complexes. The low T2 signal varies with the degree of protein concentration and fluid content. The diverse chemical composition and physical state of the contents can alter diffusion and ADC signal characteristics. According to one case report in the literature, the lack of diffusion restriction has been explained by T2-dark through effect.

Key Points  Intracranial cystic lesions can be diagnostic dilemmas. They are frequently seen in uncommon locations and are constituted by a combination of altered chemical components.  The characteristic findings: 1) hyperdensity on CT, 2) T1 hyperintensity, 3) profound T2 hypointense signal and changes, and 4) lack of diffusion restriction have been described in the imaging literature and explained by the T2-dark through effect.  The peripheral calcification as in our patient is seen in 10–20% of epidermoid cysts and further strengthens the diagnosis.

Suggested Reading Ben Hamouda M, Drissi C, Sebai R, et al. Atypical CT and MRI aspects of an epidermoid cyst. J Neuroradiol 2007; 34(2): 129–32. Little MW, Guilfoyle MR, Bulters DO, et al. Neurenteric cyst of the anterior cranial fossa: case report and literature review. Acta Neurochir (Wien) 2011; 153(7): 1519–25. Ochi M, Hayashi K, Hayashi T, et al. Unusual CT and MR appearance of an epidermoid tumor of the cerebellopontine angle. AJNR Am J Neuroradiol 1998; 19(6): 1113–15. Osborn AG, Preece MT. Intracranial cysts: radiologic-pathologic correlation and imaging approach. Radiology 2006; 239(3): 650–64. Preece MT, Osborn AG, Chin SS, Smirniotopoulos JG. Intracranial neurenteric cysts: imaging and pathology spectrum. AJNR Am J Neuroradiol 2006; 27(6): 1211–16. Timmer FA, Sluzewski M, Treskes M, et al. Chemical analysis of an epidermoid cyst with unusual CT and MR characteristics. AJNR Am J Neuroradiol 1998; 19(6): 1111–12.

CASE

Part VI

102

Central Nervous System Tumors Harry S. Hardin, Asim K. Bag

Clinical Presentation A 56-year-old man, with no significant past medical history, presented to the emergency department with a 3-month history of weight loss and dark stools. His wife stated that his behavior had changed and that he had been acting peculiarly. Neurologic exam revealed altered mental status, with orthostatic vital signs. Hematologic studies demonstrated hemoglobin level of 8 mg/dl and a low mean corpuscular volume; however, the total and differential white cell counts were normal. An emergency MRI was performed.

Imaging Fig. 102.1 Axial precontrast, non-fat-suppressed T1WI through the cerebellum.

Advanced Neuroradiology Cases, ed. Amaral et al. Published by Cambridge University Press. © Cambridge University Press 2016.

477

Part VI. Central Nervous System Tumors: Case 102 Fig. 102.2 Axial fat-suppressed T2WI through the same level.

Fig. 102.3 Axial postcontrast, fat-suppressed T1WI through the same level.

478

Part VI. Central Nervous System Tumors: Case 102 Fig. 102.4 Coronal GRE image through the mass.

Fig. 102.5 Axial DWI through the same level.

479

Part VI. Central Nervous System Tumors: Case 102

Brain Metastases from Mucin-Producing Primary Primary Diagnosis Brain metastases from mucin-producing primary

Differential Diagnoses Metastasis from other primary malignancies Brain abscess Multifocal glioblastoma (GBM)

Imaging Findings Fig. 102.1: Axial precontrast, non-fat-suppressed T1WI through the cerebellum demonstrated a large, right cerebellar mass that is heterogeneously hyperintense on T1WI sequence, predominantly at the posterior aspect of the mass. Fig. 102.2: Axial fat-suppressed T2WI through the same level demonstrated that the mass has an extremely heterogeneous T2 signal appearance. In particular, the posterior aspect of the mass is heterogeneously hypointense. Fig. 102.3: Axial postcontrast, fat-suppressed T1WI through the same level demonstrates non-suppression of the high T1 signal, as evidenced on the precontrast images. In addition, there is heterogeneous enhancement of the remainder of the mass and another tiny, nodular, enhancing lesion immediately adjacent to the larger mass. Fig. 102.4: Coronal GRE image through the mass does not demonstrate any area of hemorrhage within the T1 hyperintense or T2 hypointense area of the tumor. Fig. 102.5: Axial DWI through the same level does not demonstrate increased signal within the mass. Apparent diffusion coefficient map (not shown) did not demonstrate low value either to suggest diffusion restriction.

Discussion This patient has multifocal and bilateral intra-axial brain masses present. His age, weight loss, dark stools, and iron deficiency anemia strongly suggest a primary colonic carcinoma. His lack of substance abuse or other significant medical history makes infection an unlikely alternative. The central, solid component of the mass demonstrates T2 hypointensity and T1 hyperintensity. This is not due to hemorrhage, as evidenced on GRE sequence, or fat, as suggested by the fatsuppressed postcontrast T1WI. The cumulative imaging findings in this patient are suggestive of mucin production by the dominant lesion, indicating metastasis from a primary adenocarcinoma. Subsequently, the patient underwent colonoscopy with biopsy of a large mass, revealing colonic adenocarcinoma of the mucinous subtype. The clinical profile does not suggest infection or abscess. The absence of diffusion restriction eliminates abscess as a potential diagnosis. The presence of abnormal T1 hyperintensity that is not due to hemorrhage or fat is difficult to explain by multifocal GBM. In addition, the patient’s clinical profile does not fit with GBM.

480

The most common tumor of the posterior fossa in an adult patient is due to metastasis from another primary. Brain parenchymal metastasis can be from any primary, although metastasis from breast and lung are the two most common. The most common location of brain metastasis is the supratentorial brain parenchyma. Approximately 15% of metastases are found in the cerebellum and approximately 50% of all brain metastases are solitary. The imaging findings of a brain metastasis are largely dependent upon the biology of the primary tumor. Multiple lesions in the brain always raise the concern for a metastasis. Usually, metastases appear iso- to hypointense on T1WI sequence and hyperintense on T2WI sequence. Highly cellular primary tumors with high nuclear-to-cytoplasmic ratios frequently appear hypointense on T2WI sequence. Unlike infiltrating high-grade primary astrocytic tumors, metastases have a well-defined pathologic margin, surrounded by varying degrees of peritumoral vasogenic edema, without any infiltrating component. This pathologic feature can be used to differentiate a solitary brain metastasis from an infiltrating astrocytic tumor on imaging, particularly with advanced imaging techniques. All advanced imaging techniques such as MR spectroscopy, diffusion and perfusion MRI, reveal tumor signature in the peritumoral area – beyond the enhancing component. Conventional MRI may be helpful in identifying a primary lesion, in some cases. The mucin-producing tumors may have high T1 signal and low T2 signal due to mucin deposition. However, this is not a rule, as the T1 and T2 signal characteristics largely depend upon the physical state of the mucin, particularly the protein and water content. High protein content may cause both T1 and T2 shortening, thereby causing T1 hyperintensity and T2 hypointensity. In other words, mucinous metastasis may have T1 iso- to hypointense and T2 hyperintense appearance as well. Careful exclusion of other causes of T1 hyperintensity (such as hemorrhage, melanin, fat, or free radicals) and T2 hypointensity (hemorrhage, high-grade tumor, dense calcification) should be excluded before entertaining the diagnosis of mucinous metastasis.

Key Points  Metastasis should be considered as the first diagnosis in a patient with multifocal brain masses.  Prominent T2 hypointensity with or without T1 hyperintensity within a lesion that is potentially considered a metastasis is strongly suggestive of a mucin-producing primary lesion.

Suggested Reading Lassman AB, DeAngelis LM. Brain metastases. Neurol Clin 2003; 21(1): 1–23, vii. Walker MT, Kappor KV. Brain metastases. In: Raizer JJ, Abrey LE, eds. Brain Metastases. New York: Springer; 2007: 31–52.

CASE

Part VI

103

Central Nervous System Tumors Asim K. Bag, Lázaro Luís Faria do Amaral

Clinical Presentation A 35-year-old woman presented with headache. A CT scan demonstrated a left temporal lobe lesion with few calcifications at the center. An MRI was performed. She did not have any systemic complaints.

Imaging Fig. 103.1 Sagittal precontrast T1WI sequence.

Advanced Neuroradiology Cases, ed. Amaral et al. Published by Cambridge University Press. © Cambridge University Press 2016.

481

Part VI. Central Nervous System Tumors: Case 103 Fig. 103.2 Axial T2WI through the lesion.

Fig. 103.3 Axial GRE image through the lesion.

482

Part VI. Central Nervous System Tumors: Case 103 Fig. 103.4 ADC map through the lesion.

Fig. 103.5 Axial postcontrast T1WI sequence through the lesion.

483

Part VI. Central Nervous System Tumors: Case 103

Intra-axial CNS Dermoid Cyst Primary Diagnosis Intra-axial CNS dermoid cyst

Differential Diagnoses Epidermoid cyst Oligodendroglioma Lipoma

Imaging Findings Fig. 103.1: Sagittal precontrast T1WI demonstrated a T1 heterogeneous mass involving the supramarginal gyrus and posterior aspect of the superior temporal gyrus. There were a few areas of T1 hyperintensity, matching the intensity of the subcutaneous fat. Fig. 103.2: Axial T2WI through the lesion demonstrated a thick-walled, unilateral, intra-axial lesion that demonstrated heterogeneous T2 hyperintensity. There was no mass effect to the adjacent brain tissue or abnormal T2 signal in the peritumoral area. Fig. 103.3 Axial GRE image through the lesion demonstrated a few linear areas of susceptibilityrelated signal drop within the core of the lesion corresponding to calcifications seen on the CT scan (not shown). Fig. 103.4: ADC map through the lesion demonstrated that most of the lesion has low ADC values, even lower than the adjacent white matter. Fig. 103.5: Postcontrast axial T1WI sequence through the lesion demonstrated heterogeneous signal within the lesion with no enhancement. Peripheral foci of T1 hyperintensity were noted in the anterolateral aspect.

Discussion A benign-appearing intracranial mass containing both lipid and calcification is suggestive of a dermal inclusion cyst. Although diffusion restriction is characteristic in epidermoid cysts, diffusion restriction due to decreased water content has been described in intraparenchymal dermoid cysts (DCs). Epidermoid cysts typically demonstrate CSF intensity on T1weighted sequences and do not demonstrate any fat or calcification. Although speckled calcification can be seen in lipomas, the entire tumor usually shows a homogeneously bright signal on the T1-weighted sequence, rather than a heterogeneously low signal, as seen in this tumor. Oligodendrogliomas may be low attenuating, but never demonstrate fat attenuation. In addition, oligodendrogliomas may demonstrate enhancement with contrast. Dermoid cysts arise from the inclusion of ectodermal cells during neural tube closure, around the fifth week of embryogenesis. They are rare lesions and account for approximately 0.5% of all intracranial tumors. As the dermal cell-line produces hair and secretions from glandular components, DCs enlarge slowly over time. Most of the intracranial DCs are extra-axial and most commonly located in the suprasellar cistern, followed by the posterior fossa, frontonasal area, and occipital lobes. Posterior fossa lesions are frequently associated

484

with the dermal sinus and may be complicated with recurrent meningitis. A thick capsule of stratified squamous epithelium demarcates DC from normal brain tissue. Sebaceous material, hair follicles, keratin debris, and hair can be seen at the core of the cystic lesion. The presence of keratin debris may explain diffusion restriction seen in some DCs. Lipid and cholesterol can be seen and produce T1 hyperintensity. As the outer DC layer is composed of squamous epithelium, rarely DCs may be complicated by presence of squamous cell carcinomas. Although DCs are usually asymptomatic, large lesions can present with headache. Rupture of the DC is common because of the relentless production of material from the different dermal elements. Rupture of the DCs induces chemical meningitis, a neurologic emergency. In the case of rupture, patients present with severe headache, seizures, and vasospasm. In severe cases, there may be infarction and even death. The fat-mixed contents of DCs typically demonstrate a low signal on CT scans. Calcification can be seen in up to 50% of cases and is usually peripheral, but inter-tumor calcification can be seen as well. If ruptured, fat droplets are usually seen in the subarachnoid space and non-dependent portion of the ventricles, mainly in the frontal horns, and in the anterior temporal horns. Spin echo T2-weighted sequence demonstrates chemical shift artifact at the fat-non-fat interface in the frequency-encoding direction. On the fast spin echo sequence, DCs are usually hyperintense and calcific foci may appear as blooming artifacts in GRE sequence. Typically, DCs do not enhance with contrast. However, ruptured DCs may demonstrate intense peripheral enhancement due to severe inflammatory response. Spectroscopy may demonstrate a lipid peak.

Key Points  Intra-axial DCs are rare and careful evaluation of the mass is required for appropriate diagnosis.  Dermoid cysts are benign-appearing masses that are low attenuation on CT, contain calcification and lipids, and do not demonstrate contrast enhancement.  A ruptured DC is a neurologic emergency and should be promptly identified and treated.

Suggested Reading Caldarelli M, Colosimo C, Di Rocco C. Intra-axial dermoid/ epidermoid tumors of the brainstem in children. Surg Neurol 2001; 56(2): 97–105. Dias MS, Walker ML. The embryogenesis of complex dysraphic malformations: a disorder of gastrulation? Pediatr Neurosurg 1992; 18(5–6): 229–53. Orakcioglu B, Halatsch ME, Fortunati M, Unterberg A, Yonekawa Y. Intracranial dermoid cysts: variations of radiological and clinical features. Acta Neurochir (Wien) 2008; 150(12): 1227–34; discussion 1234. Velho VL, Khan SW, Agarwal V, Sharma M. Intra-axial CNS dermoid cyst. Asian J Neurosurg 2012; 7(1): 42–4.

CASE

Part VI

104

Central Nervous System Tumors Asim K. Bag

Clinical Presentation A 62-year-old woman with a long history of migraine presented to the emergency department after experiencing a seizure. She also stated that for the previous two to three weeks prior, she had felt dizzy, tired, and intermittently unbalanced. She also stated that acquaintances noted that her left upper lip was droopy. She denied any focal weakness, visual problem, worsening headache, or vomiting. Computed tomography was performed and revealed a brain mass; MRI was performed for further evaluation.

Imaging (A)

(B)

Fig. 104.1 (A) Axial FLAIR image through the level of the midbrain. (B) Axial FLAIR image through a slightly lower level of the midbrain.

Advanced Neuroradiology Cases, ed. Amaral et al. Published by Cambridge University Press. © Cambridge University Press 2016.

485

Part VI. Central Nervous System Tumors: Case 104

(A)

(B)

486

Fig. 104.2 (A) Axial DWI image through the level of the midbrain. (B) ADC map through the level of the midbrain.

Part VI. Central Nervous System Tumors: Case 104 Fig. 104.3 Axial postcontrast TWI through the level of the midbrain.

487

Part VI. Central Nervous System Tumors: Case 104

Diffuse Astrocytoma/Gliomatosis Cerebri Primary Diagnosis Diffuse astrocytoma/Gliomatosis cerebri

Differential Diagnoses Infiltrating low-grade astrocytoma Small vessel ischemic disease Lymphomatosis cerebri Viral encephalitis

Imaging Findings Fig. 104.1: (A) Axial FLAIR image through the level of the midbrain demonstrates abnormal FLAIR signal that has a mass-like appearance centered in the right frontal lobe. Note there is extensive FLAIR signal abnormality in the right frontal lobe, anterior to the mass, with blurring of the gray-white differentiation, as well as along the periventricular white matter posteriorly as far as the occipital lobe. There is also ill-defined abnormal FLAIR signal in the left frontal lobe, left insula/subinsular region, and along the periventricular white matter posteriorly as far as the occipital lobe. (B) Axial FLAIR image through a slightly lower level demonstrates ill-defined abnormal FLAIR signal in bilateral inferior frontal lobes and bilateral temporal lobes. Fig. 104.2: (A) Axial DWI image and (B) ADC map through the level of the midbrain, same as Fig. 104.1A, demonstrate focal area of diffusion restriction at the right frontal lobe. Note high ADC value in bilateral frontal lobes, left subinsular regions. Fig. 104.3: Axial postcontrast TWI through the level of the midbrain, same as Fig. 104.1 A, demonstrates multifocal areas of enhancement in the right frontal lobe. The larger area of enhancement demonstrates a more heterogeneous pattern with central area of necrosis.

Discussion The difference between an infiltrating low-grade astrocytoma (ILGA) and a gliomatosis cerebri (GC), from a practical standpoint, is the extent of tissue involvement. When an ILGA involves more than three cerebral lobes, by definition, it becomes a GC. Such an extensive infiltrative pattern has an underlying aggressive pattern, as compared to a typical ILGA. Although small vessel ischemic disease (SVID) in the setting of hypertension and chronic renal disease can have extensive signal abnormality, the geographic pattern of involvement is different. In SVID, the involvement is predominantly limited to white matter in the deep cerebral and periventricular regions. Subcortical involvement may be seen in SVID, but this involvement in the subcortical region is usually patchy and the gray-white differentiation is maintained. Infiltrating lymphoma (lymphomatosis cerebri) may have similar growth patterns, but the tumor is usually hypointense on T2WI and demonstrates enhancement in the involved areas of brain. Diffuse non-specific white and gray matter abnormality can

488

be seen in viral encephalitis as well but the clinic setting is different in viral encephalitis (viral prodrome, fever, and headache and neck rigidity). Masses consisting of a diffuse, glioma pattern infiltrating an extensive CNS region of three or more cerebral lobes meet WHO criteria for a gliomatosis cerebri. Typically, there is bilateral cerebral and/or deep gray matter involvement that may extend to the cerebellum, brainstem, and in some cases, the spinal cord. This extensively infiltrative aggressive tumor is usually of astrocytic lineage. Gliomatosis cerebri behave like WHO grade III tumors and are either primary or secondary. In primary GC, there is extensive infiltration at the time of presentation, whereas in secondary GC, progressive infiltration occurs over time. Primary GC can be further subclassified as type 1, in which there is extensive infiltrative tumor at clinical presentation with no tumor mass, or type 2, in which there is a tumor mass, in addition to an extensively infiltrative tumor. In this patient, the tumor demonstrates behavior of a type 2 GC. Although GC can present in any age group, the peak incidence is between 40 and 50 years of age, it has no gender predilection, and it can involve any area of brain. However, GC does have a predilection for the right hemisphere. Clinical presentation is dependent upon the areas of brain infiltrated by the tumor. Headache, memory problems, lethargy, seizures, and focal neurologic deficits are common clinical presentations. In primary type 1 GC, there is diffuse abnormal FLAIR signal in the areas of infiltrative tumor with any identifiable focal mass. Although the tumor may cause mass effect to the ventricles, there is usually no demonstrable enhancement with contrast. Magnetic resonance spectroscopy demonstrates tumor signature (low NAA, high choline-to-creatine ratio) along the entire region of abnormal FLAIR signal. A mass can be seen either in type 2 primary GC or in type 1 primary GC patients, with dedifferentiation to a higher-grade tumor. The mass usually demonstrates all imaging features of a highgrade tumor (such as heterogeneous enhancement, increased rCBV, and subtle diffusion restriction). Because of extensive involvement, surgery is not a treatment option. This tumor is treated by whole brain radiation plus temozolomide and nitrosoureas.

Key Points  Typical imaging findings of GC include extensive, multicerebral lobe involvement, predominantly white matter FLAIR abnormalities with blurring of the gray-white differentiation and varying extension to the basal ganglia and brainstem.  Although GC is a high-grade tumor, the infiltrating tumor component shows signs of a low-grade tumor with no enhancement or mass effect.  Secondary GC presents with a mass with typically highgrade tumor features, in an infiltrating background.

Part VI. Central Nervous System Tumors: Case 104

Suggested Reading Chen S, Tanaka S, Giannini C, et al. Gliomatosis cerebri: clinical characteristics, management, and outcomes. J Neurooncol 2013; 112(2): 267–75. Fuller GN, Kros JM. Glioblastoma. In: Louis DN, Ohgaki H, Wiestler OD, Cavenee WK, eds. WHO Classification of Tumours of the Central Nervous System. Lyon: IARC Press; 2007: 50–2.

Kleihues P, Burger PC, Aldape KD, et al. Astrocytic tumors. In: Louis DN, Ohgaki H, Wiestler OD, Cavenee WK, eds. WHO Classification of Tumours of the Central Nervous System. Lyon: IARC Press; 2007: 14–80. Ruda R, Bertero L, Sanson M. Gliomatosis cerebri: a review. Curr Treat Options Neurol 2014; 16(2): 273.

489

CASE

Part VI

105

Central Nervous System Tumors Asim K. Bag

Clinical Presentation A 54-year-old man with newly diagnosed glioblastoma (GBM) was initially treated with maximal safe resection followed by concurrent chemoradiation and subsequent maintenance chemotherapy. Nine months after initiation of treatment, he developed radiographic and clinical progression according to the RANO criteria. Bevacizumab therapy was started and neurologic symptoms improved significantly. Follow-up included bimonthly MRI scans. Diffusion-weighted MRI and ADC maps shown below were obtained two months, four months, and six months after initiation of the bevacizumab therapy. During this six-month course of bevacizumab therapy, neurologic symptoms of the patient were stable.

Imaging (A) (B)

Fig. 105.1 (A) Axial MR DWI and (B) ADC map through the level of the third ventricle (two months post-bevacizumab treatment initiation).

Advanced Neuroradiology Cases, ed. Amaral et al. Published by Cambridge University Press. © Cambridge University Press 2016.

491

Part VI. Central Nervous System Tumors: Case 105

(A) (B)

Fig. 105.2 (A) Axial MR DWI and (B) ADC map through the level of the third ventricle (four months post-bevacizumab treatment initiation).

492

Part VI. Central Nervous System Tumors: Case 105

(A) (B)

Fig. 105.3 (A) Axial MR DWI and (B) ADC map through the level of the third ventricle (six months post-bevacizumab treatment initiation).

493

Part VI. Central Nervous System Tumors: Case 105

Persistent Diffusion Restriction after Bevacizumab Therapy Primary Diagnosis Persistent diffusion restriction after bevacizumab therapy (also known as bevacizumab-related imaging abnormality)

Differential Diagnoses Recurrence of glioblastoma (GBM) Acute infarction

Imaging Findings Fig. 105.1: (A) Axial MR DWI and (B) ADC map through the level of the third ventricle two months post-bevacizumab treatment initiation demonstrates a tiny focal area of high signal in the periventricular white matter and in the corpus callosum, without any low ADC value, likely postsurgical changes. Note there is no significant abnormality at the right side of the splenium of the corpus callosum. Also, note the mass effect to the right lateral ventricle. Fig. 105.2: (A) Axial MR DWI and (B) ADC map through the level of the third ventricle four months after initiating bevacizumab treatment demonstrates curved area of diffusion restriction in the right paraventricular white matter reaching the midline through the right side of the splenium of the corpus callosum. Note the very low ADC value in these areas, much lower than expected from recurrence of tumor. Also, note that the mass effect to the right lateral ventricle has significantly improved. Fig. 105.3: (A) Axial MR DWI and (B) ADC map through the level of the third ventricle six months after initiating bevacizumab treatment demonstrates interval worsening of curved area of diffusion restriction in the right periventricular white matter reaching the midline through the right side of the splenium of the corpus callosum. During these six months, however, there was gradual improvement of the peritumoral FLAIR abnormality and no worsening of enhancement (not shown). Relative cerebral blood volume in areas of diffusion restriction showed low value, even lower than normal-appearing white matter. Note a new area of diffusion restriction in the right frontal lobe, an area of distal recurrence.

Discussion This patient’s recurrent GBM is being treated with antiangiogenic therapy, bevacizumab. During this treatment, the patient developed intense diffusion restriction in the relatively uninvolved peritumoral area. This new diffusion abnormality is not associated with any mass effect, abnormal enhancement, or increased perfusion. In addition, the patient was stable in terms of neurologic symptoms. The clinical and imaging findings are suggestive of persistent diffusion restriction in GBM patients during Avastin (bevacizumab) therapy, likely due to atypical necrosis. The lack of mass effect or enhancement and the presence of increased perfusion eliminate recurrence as the source of the diffusion restriction. Moreover, this patient was clinically

494

stable, and did not meet RANO criteria for tumor progression. Although the pathophysiologic mechanism of diffusion restriction may be related to abnormal tissue perfusion, in this occurrence it does not follow any known arterial distribution. Clinical context does not suggest venous infarct. The neuroradiologic interpretation of post-treatment GBM images is extremely difficult, particularly because of the increased availability and use of biologically active treatments targeting genetic and molecular defects. Complete knowledge of previous and ongoing treatment is key to accurate image interpretation. Avastin (bevacizumab) is the brand name of the antiangiogenic, humanized monoclonal antibody against vascular endothelial growth factor A, one of the strongest proangiogenic molecules in GBM. It is now routinely used as first-line treatment for recurrent GBM. Bevacizumab therapy rapidly improves contrast enhancement and even FLAIR abnormalities owing to the drug’s rapid action on tumor vasculature. In a subgroup of patients, prolonged diffusion retraction that is often remote from the tumor bed may be present. Although the pathogenesis is not clear, the toxic-ischemic effect of bevacizumab has been suggested as the cause of this. Prior radiationinduced vascular injury is thought to play a role as well. Bevacizumab further injures already damaged blood vessels, causing critical ischemia that is not severe enough to cause frank infarction but is sufficient to induce a chronic state of ischemia and cytotoxic edema – resulting in diffusion restriction. Histopathologic correlation studies have demonstrated areas of coagulative necrosis in the areas of diffusion restriction, and abundance of hypoxia inducible factor 1-alpha accumulation in the nucleus, rather than tumor growth. Accurate diagnosis of this condition is extremely crucial, as treatment for conditions that mimic the bevacizumab-induced diffusion restriction is entirely different.

Key Point  Persistent intense diffusion restriction in the peritumoral area, often involving the corpus callosum, during treatment with bevacizumab in patients with recurrent GBM is due to ischemic necrosis, rather than tumor.

Suggested Reading Farid N, Almeida-Freitas DB, White NS, et al. Restriction-spectrum imaging of bevacizumab-related necrosis in a patient with GBM. Front Oncol 2013; 3: 258. Futterer SF, Nemeth AJ, Grimm SA, et al. Diffusion abnormalities of the corpus callosum in patients receiving bevacizumab for malignant brain tumors: suspected treatment toxicity. J Neurooncol 2014; 118(1): 147–53. Mong S, Ellingson BM, Nghiemphu PL, et al. Persistent diffusionrestricted lesions in bevacizumab-treated malignant gliomas are associated with improved survival compared with matched controls. AJNR Am J Neuroradiol 2012; 33(9): 1763–70. Rieger J, Bahr O, Ronellenfitsch MW, Steinbach J, Hattingen E. Bevacizumab-induced diffusion restriction in patients with glioma: tumor progression or surrogate marker of hypoxia? J Clin Oncol 2010; 28(27): e477; author reply e8.

CASE

Part VI

106

Central Nervous System Tumors Victor Hugo Rocha Marussi, Lázaro Luís Faria do Amaral

Clinical Presentation A 30-year-old man presented to the outpatient clinic of our hospital with a four-week history of gradually worsening pain in his lower neck, and tingling and numbness in his upper extremities, and weakness of his lower extremities. He reported

Parinaud syndrome, intracranial hypertension, and a two-day history of bladder and bowel function loss. On questioning, he revealed he had experienced a moderate headache for the past two to three months. Routine hematologic studies were normal and CSF studies were unrevealing.

Imaging (A)

(B)

(A)

(B)

Fig. 106.1 (A) Axial DWI of the brain and (B) ADC map through the level of the pineal region.

Fig. 106.2 (A) Axial T1WI postgadolinium and (B) Sagittal T1WI postgadolinium through the level of the pineal region.

Advanced Neuroradiology Cases, ed. Amaral et al. Published by Cambridge University Press. © Cambridge University Press 2016.

495

Part VI. Central Nervous System Tumors: Case 106

(A)

(B)

Fig. 106.3 (A) Sagittal T2WI, (B) Sagittal T1WI postgadolinium with fat suppression, and (C) Sagittal T1WI postgadolinium images of the spinal canal with fat suppression through the midline.

496

Part VI. Central Nervous System Tumors: Case 106

(C)

Fig. 106.3 (cont.)

497

Part VI. Central Nervous System Tumors: Case 106

Pineoblastoma with Drop Metastasis Primary Diagnosis Pineoblastoma with drop metastasis

Differential Diagnoses Germ cell tumors Pineal parenchymal tumor Astrocytoma Meningioma Metastasis

Imaging Findings Fig. 106.1: (A) Axial DWI and (B) Axial ADC map of the brain showed a mass-effect lesion in the pineal region, with restricted diffusion. Fig. 106.2: (A) Axial and (B) Sagittal T1WI postcontrast showed marked lesion enhancement, compression in the aqueduct of Sylvius, and hydrocephalus. Fig. 106.3: (A) Sagittal T2-weighted image of the cervicothoracic region and (B–C) postcontrast sagittal T1WI images with fat suppression of the cervicothoracic region and lumbar regions showed CSF drop metastasis in the spinal canal.

Discussion Pineoblastomas (PBs) are the most primitive and malignant neoplasm (WHO grade IV) of all pineal parenchymal tumors (PPTs), accounting for approximately 40% of PPTs. Decidedly more prevalent in children, most PBs present in the first two decades of life with no gender predilection. Symptoms include signs of elevated intracranial pressure, Parinaud syndrome, and obstructive hydrocephalus. Pineoblastomas are generally large, heterogeneous tumors that frequently demonstrate necrosis and intratumoral hemorrhage, as well as infiltration into adjacent structures with CSF dissemination. Typically, PBs are hyperattenuating masses on CT, have low ADCs and restricted diffusion on DWI. On MR images, their appearance reflects their highly cellular histologic features, in addition to heterogeneous, avid postcontrast enhancement. Imaging findings including demonstrated compression of the aqueduct of Sylvius, hydrocephalus, and intradural extramedullary spinal metastases in a patient with history of headache and extremity numbness/weakness are strongly suggestive of PB with drop metastasis. Multiple lesion types should be excluded when confirming the diagnosis of a pineal region mass. Although the literature suggests that pineal region tumors lack pathognomonic imaging patterns, an extensive knowledge of the patient’s age, gender, and imaging findings combined with a comprehensive familiarity of pineal lesions and their clinical features can narrow the differential diagnosis for more accurate and rational therapeutic planning. Germ cell tumors (GCTs) are the most frequently encountered type of tumor in the pineal region. Their

498

diagnosis is usually made by the second decade of life and they are more common in males than females. They tend to disseminate along CSF pathways with drop metastases to the spine. Metastases are often found in the suprasellar cistern, although synchronous tumors may also independently arise in this region. The most common subtype of GCT is a germinoma. It usually presents as a homogeneous, hyperdense mass with typical, but not always present, engulfed central calcifications on CT, a signal intensity similar to that of gray matter on both T1- and T2-weighted MR sequences, and with postcontrast enhancement. In contrast, teratomas are multilocular heterogeneous masses with mixed signal, that include areas of high signal intensity on T1-weighted sequence due to the presence of fat or lipid components and areas of low signal from calcification that can show enhancement of the soft tissue component in postcontrast MR images. It is extremely important that individuals with a pineal region mass should have levels of serum or CSF alpha-fetoprotein and beta-HCG (human chorionic gonadotropin), and placental alkaline phosphatase measured. If these markers are elevated, the presence of a GCT can be presumed. Pineal parenchymal tumors are less common than GCT lesions and usually are either pineocytomas or PBs, since the other two entities of this group, pineal parenchymal tumor of intermediate differentiation (PPTID) and papillary tumor of the pineal region (PTPR), are very rare. These more common entities classically result in preexisting pineal calcifications dispersing to the periphery of the lesion (explosion) – differentiating them from other cell type tumors, such as germinomas. Pineocytoma is the most differentiated tumor of the group and is usually diagnosed during adulthood rather than childhood, like PB. Characteristic imaging findings of a pineocytoma demonstrate a well-defined, round, homogeneous mass with uniform homogeneous contrast enhancement. Astrocytomas found in the pineal region, most commonly pilocytic subtypes, typically arise from the neighboring tectal plate or thalamus (rarely from the pineal gland itself) and have a similar appearance on CT and MRI as when encountered elsewhere in the brain. Typically affecting females between the fifth and seventh decades of life, meningomas have a similar appearance in the pineal region as elsewhere in the brain, and usually arise from the tentorium cerebelli and falx. The dural tail attached to the tentorium is a distinctive feature, but not always present. Pineal metastasis is very uncommon. Although a variety of tumors can metastasize to the pineal gland, the most frequent pineal metastasis is lung cancer. Leptomeningeal seeding is a common finding with pineal metastases, occurring in 67% of patients. Patients with pineal lesions who present with a known malignancy should raise the suspicion of metastatic involvement.

Part VI. Central Nervous System Tumors: Case 106

Key Point  Imaging findings, including a lesion with restricted diffusion in the pineal region, and hydrocephaly, due to compression of the aqueduct, are suggestive of pineoblastoma.

Dumrongpisutikul N, Intrapiromkul J, Yousem DM. Distinguishing between germinomas and pineal cell tumors on MR imaging. AJNR Am J Neuroradiol 2012; 33(3): 550–5. Fang AS, Meyers SP. Magnetic resonance imaging of pineal region tumours. Insights Imaging 2013; 4(3): 369–82.

Suggested Reading

Gasparetto EL, Cruz Jr LC, Doring TM, et al. Diffusion-weighted MR images and pineoblastoma: diagnosis and follow-up. Arq Neuropsiquiatr 2008; 66(1): 64–8.

Banks KP, Brown SJ. AJR teaching file: solid masses of the pineal region. AJR Am J Roentgenol 2006; 186(3): S233–5.

Smith AB, Rushing EJ, Smirniotopoulos JG. From the archives of the AFIP: lesions of the pineal region: radiologic-pathologic correlation. Radiographics 2010; 30(7): 2001–20.

499

CASE

Part VI

107

Central Nervous System Tumors Afonso C. P. Liberato, Lázaro Luís Faria do Amaral

Clinical Presentation A 49-year-old man presented with recent onset cerebellar ataxia and intense headache. Cerebrospinal fluid was normal. There was no fever or other symptoms. In the physical examination, the patient had abdominal pain.

Imaging (A)

(B)

(C)

Fig. 107.1 (A) Axial FLAIR, (B) Axial T2WI, and (C) Axial DWI through the fourth ventricle.

Advanced Neuroradiology Cases, ed. Amaral et al. Published by Cambridge University Press. © Cambridge University Press 2016.

501

Part VI. Central Nervous System Tumors: Case 107

(A) (B)

(C)

Fig. 107.2 (A) Axial T1WI and (B–C) Axial T1WI postgadolinium through the fourth ventricle.

502

Part VI. Central Nervous System Tumors: Case 107

(A)

(B)

(C)

Fig. 107.3 (A–C) Sagittal contrast-enhanced T1WI with fat suppression through the spinal canal. Fig. 107.4 Coronal reconstruction contrastenhanced CT scan through the abdomen.

503

Part VI. Central Nervous System Tumors: Case 107

Renal Cell Carcinoma Metastasis to Posterior Fossa Primary Diagnosis Renal cell carcinoma metastasis to posterior fossa

Differential Diagnoses Hemangioblastoma Arteriovenous malformation Highly vascular glioblastoma

Imaging Findings Fig. 107.1: (A) Axial FLAIR and (B) Axial T2W images showed a mixed signal intensity, intra-axial, and lobulated lesion with a predominantly hyperintense signal, relative to the cerebellar white matter. Prominent flow voids in and around the lesion were noted with no significant peritumoral edema. (C) Axial DWI showed no restricted diffusion. Fig. 107.2: (A) Axial T1WI showed mixed signal intensity lesion with associated flow voids. (B) Axial T1WI postgadolinium showed intense contrast enhancement of the lesion with a central nonenhancing area, suggestive of necrosis. (C) Axial T1WI also showing a small enhancement nodule inside the right internal auditory canal. Fig. 107.3: (A–C) Sagittal contrast-enhanced T1WI of the cervical, thoracic, and lumbar spines with fat suppression showed multiple intradural extramedullary enhancing nodular lesions, compatible with drop metastases. Fig. 107.4: Coronal reconstruction, contrast-enhanced CT scan of the abdomen showed a large heterogeneously enhancing mass at the upper pole of the left kidney, compatible with renal cell carcinoma.

Discussion Imaging findings demonstrating a vascular, posterior fossa mass with flow voids, which lacks a cystic component, in the absence of diffusion restriction, in a patient with known renal cell carcinoma are suggestive of renal cell cancer metastases. Hemangioblastomas (HGBLs) are the second most common infratentorial parenchymal mass in adults (after metastases) and 25–40% of cases of HGBL occur in association with von Hippel-Lindau syndrome. Although HGBLs often demonstrate presence of multiple lesions on imaging, they are more common in a younger age group. A vascular tumor may be purely cystic, or a purely solid, large cyst with a mural nodule (most common pattern, seen in 60% of cases), or solid mass with internal cysts. Symptoms are non-specific and the most common presenting complaint is headache. The typical imaging finding of a HGBL is an intra-axial cystic mass with an intensely enhancing mural nodule (with the cystic component larger, compared to the solid nodule) that demonstrates flow voids from vascular structure and abuts the pia. Although presence of a nodule abutting the pia is considered a classic imaging finding of HGBL, this finding is also present in other tumors as well. The cystic component of an HGBL lesion does not usually enhance, owing to its high

504

protein fluid component, and may demonstrate associated presence of hemorrhage. Generally, if hemorrhage is associated with the cystic component, the lesion appears hyperintense in comparison to CSF on T1-weighted MR sequences. The solid component is isodense on CT, isointense on T1weighted sequence, and hyperintense on T2-weighted sequence, with or without associated internal flow voids on MR images. Glioblastoma subtypes can occur in adults and generally appear as an irregular, ring-enhancing mass; however, they rarely develop in the posterior fossa. Arteriovenous malformations (AVMs) are much more common supratentorial lesions than infratentorial. They can appear as iso- to hyperdense serpentine vessels with or without associated calcification on CT and typically present with a bag of worms (tangle of serpentine flow voids) that corresponds to the nidus, associated with arterial feeders and draining veins, better demonstrated in postcontrast, angiographic studies. Hemorrhage and gliosis in the brain parenchyma around the lesion is also found in AVM images. In summary, they can have similar imaging findings as seen in our case, especially because of the marked contrast enhancement of the lesion and the prominent flow voids previously described. However, unlike the imaging seen in our case, AVMs typically show minimal or no mass effect and do not show necrosis. Although 85% of metastatic lesions are supratentorial, metastases are still the most common intra-axial neoplasm of the adult posterior fossa. Up to 30% of all intracranial metastases occur in the posterior fossa. Usually they present as multiple lesions, but in the posterior fossa, there is a high incidence of solitary lesions (25–50%), which typically occur near the brain surface at the corticomedullary junction. The primary site of origin for metastases most commonly includes the lungs, breast, gastrointestinal sites, melanoma, and rarely genitourinary, as in our case. At imaging, metastatic intra-axial lesions usually present as round and well-circumscribed lesions with a large area of peritumoral cerebral edema, although edema may be absent in small cortical lesions. On MR, most metastases are iso- to hypointense on T1-weighted images. However, they can demonstrate T1 hyperintensity if hemorrhage has occurred, or if the lesion has high melanin content. Frequently, metastatic lesions appear hyperintense on T2-weighted images, but may appear iso- to hypointense as well. Metastases can be solitary and very large with a necrotic central portion, as in our case, and can be very small and numerous. The pattern of contrast enhancement can be homogeneous, nodular, inhomogeneous, or ring-like.

Key Points  Hemangioblastoma is typically considered a potential diagnosis when a highly vascular posterior fossa mass is seen in adults.

Part VI. Central Nervous System Tumors: Case 107

 In the absence of renal cell carcinoma, other hypervascular metastasis should also be considered, particularly if the tumor morphology is predominantly solid rather than the typically solid-cystic mass of HGBL.  Hypervascular metastasis should also be considered, depending on imaging findings.

Ho VB, Smirniotopoulos JG, Murphy FM, Rushing EJ. Radiologicpathologic correlation: hemangioblastoma. AJNR Am J Neuroradiol 1992; 13(5): 1343–52.

Suggested Reading

Sundaram C, Rammurti S, Reddy JJ, Prasad SS, Purohit AK. Hemangioblastoma: a study of radiopathologic correlation. Neurol India 2003; 51(3): 373–5.

Ghods AJ, Munoz L, Byrne R. Surgical treatment of cerebellar metastases. Surg Neurol Int 2011; 2: 159.

Moin H, Mohagheghzadeh P. A rare case of metastatic renal cell carcinoma in posterior fossa. J Res Med Sci 2005; 10(5): 314–15. Prvulovic, M. Posterior fossa metastases: are there pathognomonic MRI features? Arch Oncol 2000; 8(1): 5.

505

CASE

Part VI

108

Central Nervous System Tumors Afonso C. P. Liberato, Lázaro Luís Faria do Amaral

Clinical Presentation A 49-year-old woman presented to our facility with signs and symptoms of intracranial hypertension, including headache, nausea, vomiting, and visual disturbances. Magnetic resonance studies were performed on her brain (see results below) and body (no masses or indications of primary malignancy noted).

Imaging (B)

(A)

Fig. 108.1 (A) Sagittal T1 non-contrast and (B) Axial T2 images through the level of the atrium of the lateral ventricle.

Advanced Neuroradiology Cases, ed. Amaral et al. Published by Cambridge University Press. © Cambridge University Press 2016.

507

Part VI. Central Nervous System Tumors: Case 108

(A)

(B)

(C)

Fig. 108.2 (A) Axial MPGR-T2* and (B) Axial DWI and (C) ADC images through the level of the atrium of the lateral ventricle.

508

Part VI. Central Nervous System Tumors: Case 108

(A)

(B)

Fig. 108.3 (A) Axial non-contrast T1-weighted and (B) Axial contrast-enhanced T1-weighted images through the level of the atrium of the lateral ventricle.

(A)

(C)

(B)

Fig. 108.4 (A–C) Axial contrast-enhanced T1-weighted images through the level of the lateral ventricles.

509

Part VI. Central Nervous System Tumors: Case 108

Intraventricular Hemangiopericytoma/Solitary Fibrous Tumor Primary Diagnosis Intraventricular hemangiopericytoma/solitary fibrous tumor

Differential Diagnoses Meningioma Choroid plexus metastasis Choroid plexus carcinoma

Imaging Findings Fig. 108.1 (A) Sagittal T1 non-contrast and (B) Axial T2 images showed a bulky, intraventricular, lobulated mass with dilatation of the temporal horn of the lateral ventricle and midline shift, surrounded by vasogenic edema in the adjacent brain parenchyma. Fig. 108.2 (A) Axial MPGR-T2* MR images showed small foci of hypointensities within the lesion that could be due to hemorrhage or calcification. (B) Axial DWI and (C) ADC images showed restricted diffusion at the periphery of the mass and a central area of facilitated diffusion. Fig. 108.3 (A) Axial non-contrast T1-weighted MR image again showed heterogeneous signal intensity in the lesion. (B) Axial contrast-enhanced T1-weighted MR image showed marked heterogeneous contrast enhancement of the mass with non-enhancement central area, suggestive of necrosis. Fig. 108.4 (A–C) Axial contrast-enhanced T1-weighted MR images, three years after total surgical resection of the mass, demonstrated multiple nodular intraventricular enhancing lesions, compatible with recurrence of the primary diagnosis.

Discussion Imaging findings demonstrating an extremely heterogeneous mass with heterogeneous enhancement, low T2-weighted signal, and diffusion restriction of its solid component are consistent with a malignant tumor. These imaging findings are indicative of a tumor whose features include a high cell density and/or a high nuclear-to-cytoplasmic ratio, particularly with the presence of an abnormal T2 signal in the extraventricular brain parenchyma. The combined tumor location and aggressive nature based on imaging findings, in a middle-aged patient, are suggestive of a hemangiopericytoma (HPC). Although psammomatous meningiomas may have a similar imaging appearance, intraventricular meningiomas are typically homogeneously enhancing masses located in the atrial region, without any abnormal signal in the extraventricular brain parenchyma. Metastasis is excluded as a diagnosis, as there were no primary malignancies noted elsewhere. Choroid plexus carcinoma is usually a tumor of children aged three to five years, not middle-aged adults. Intracranial HPCs are very rare tumors accounting for less than 1% of all CNS tumors with aggressive behavior, including

510

local recurrence and distant metastasis. They can occur in all age groups; however, they are more common between the fourth and sixth decades (mean age at diagnosis is 43 years) and have a slight male predominance. Hemangiopericytomas/solitary fibrous tumors are very cellular and highly vascular neoplasms, frequently found as large supratentorial, extra-axial, lobulated masses with dural attachment (falx, tentorium, and dural sinuses). Adjacent bone erosion, as well as dural tail sign (seen in 50% of cases) can be associated imaging findings. On perfusion imaging, HPC tumors typically demonstrate high rCBV. It is not possible to differentiate HPC tumors from more benign meningiomas based on intratumoral CBV because some meningioma variants (angiomatous) are extremely vascular, although they are WHO grade I tumors. On perfusion studies, benign meningiomas show elevated rCBV levels, a potential means of differentiating HPCs by comparing rCBV levels at the adjacent brain parenchyma. Rare locations including intraparenchymal, skull base, or/ and intraventricular, as our case, are reported sites of origin for HPCs. Headache is the most common presenting symptom, followed by vomiting, focal neurologic deficit, and seizures. Computed tomography images of HPC commonly demonstrates hyperdense, extra-axial masses with low-density cystic or necrotic areas that can invade and destroy bone. Magnetic resonance images show mixed signal intensity in all sequences and prominent flow voids; both CT and MR images show marked and heterogeneous contrast.

Key Points  Intraventricular hemangiopericytoma/solitary fibrous tumor should be considered if a tumor with aggressive features on imaging arises from the choroid plexus in an adult patient in the absence of a known malignancy in other body parts.  Local recurrence and distant metastasis is very common.

Suggested Reading Chiechi MV, Smirniotopoulos JG, Mena H. Intracranial hemangiopericytomas: MR and CT features. AJNR Am J Neuroradiol 1996; 17(7): 1365–71. Kumar N, Kumar R, Kapoor R, et al. Intracranial meningeal hemangiopericytoma: 10 years experience of a tertiary care institute. Acta Neurochir (Wien) 2012; 154(9): 1647–51. Liu G, Chen ZY, Ma L, et al. Intracranial hemangiopericytoma: MR imaging findings and diagnostic usefulness of minimum ADC values. J Magn Reson Imaging 2013; 38(5): 1146–51. Suzuki S, Wanifuchi H, Shimizu T, Kubo O. Hemangiopericytoma in the lateral ventricle. Neurol Med Chir (Tokyo) 2009; 49(11): 520–3. Yeh Y-S, Tseng Y-Y, Lin J-W, Lee W-H, Yang S-T. Intraventricular hemangiopericytoma in the trigone of the right lateral ventricle. J Exp Clin Med 2011; 3(6): 318–22.

CASE

Part VI

109

Central Nervous System Tumors Prasad B. Hanagandi, Leslie Lamb, Lázaro Luís Faria do Amaral

Clinical Presentation A 31-year-old female with known, long-standing history of seizure disorder since the age of 8 years. Her past medical history did not include childhood exanthematous illness or mental retardation. Her history and her neurologic examination were otherwise unremarkable. Hematologic investigations revealed normal WBC count, SED, and C-reactive protein level. Cerebrospinal fluid analysis was negative.

Imaging

Fig. 109.1 Midsagittal non-contrast T1WI of the brain through the corpus callosum and pituitary gland. Fig. 109.2 Parasagittal non-contrast T1WI of the brain through region of left frontal operculum.

Advanced Neuroradiology Cases, ed. Amaral et al. Published by Cambridge University Press. © Cambridge University Press 2016.

511

Part VI. Central Nervous System Tumors: Case 109

512

Fig. 109.3 Axial T1WI at the level of cerebral peduncles and sylvian fissures.

Fig. 109.4 Axial non-contrast CT at the level of ambient cistern and sylvian fissures.

Fig. 109.5 Axial non-contrast T1WI of the brain at the level of lateral ventricles and inferior frontal gyrus.

Fig. 109.6 Axial T2WI of the brain at the level of lateral ventricles and inferior frontal gyrus.

Part VI. Central Nervous System Tumors: Case 109 Fig. 109.7 Axial CTA of circle of Willis at the level of sylvian fissure.

Fig. 109.8 Parasagittal reformatted CTA of circle of Willis through region of left frontal operculum.

513

Part VI. Central Nervous System Tumors: Case 109

Interhemispheric and Pericallosal Lipoma with Cortical Dysplasia Primary Diagnosis Interhemispheric and pericallosal lipoma with cortical dysplasia

Differential Diagnoses Dermoid Osteolipoma Encephalocraniocutaneous lipomatosis

Imaging Findings Fig. 109.1: Sagittal T1WI demonstrated a curvilinear pericallosal-interhemispheric lipoma, but otherwise normalappearing morphology of corpus callosum. Fig. 109.2: Parasagittal T1WI showed extension of the lipoma into the left inferior frontal sulcus and pars opercularis (arrow). Fig. 109.3: Axial T1WI through the level of the cerebral peduncles demonstrated lesion extension into the proximal sylvian fissure (arrow). Fig. 109.4: Axial CT demonstrated fat density in the left inferior frontal sulcus and interhemispheric fissure (arrows). Fig. 109.5: Axial T1WI and Fig. 109.6: Axial T2WI demonstrated a dysplastic left inferior frontal gyrus (arrows). Fig. 109.7: Axial CT angiography and Fig. 109.8: Oblique sagittal images demonstrated dilated, tortuous arteries.

Discussion The T1 hyperintense signal extending along the interhemispheric fissure and pericallosal location, inferior frontal sulcus, frontal operculum, and proximal sylvian fissure with fat attenuation density on CT confirms the diagnosis of lipoma. The major differential diagnosis to consider is a dermoid cyst, which also shows evidence of fat density and is usually located in the midline. They are non-enhancing, lobulated, and well defined, unless ruptured, when they present with fat globules in the subarachnoid space. Dermoids tend to have variable solid-cystic components. However, lipomas often have near homogeneous fat density artifact. Osteolipomas are lesions that are peripherally calcified with central fatty component and can be differentiated from lipoma. Encephalocraniocutaneous lipomatosis (ECCL) is a complex syndrome with leptomeningeal lipomatosis, lipomas involving the skull, heart, and eyes, brain malformations, and mental retardation. Absence of multiple lipomas and mental retardation in this patient case excludes ECCL. Several intracranial tumors can have lipomatous differentiation/transformation such as neuroectodermal tumors, glial neoplasms of brain and spinal cord, meningiomas, and cerebellar liponeurocytomas. Intracranial lipomas (also known as lipomatous hamartomas) are rare congenital brain malformations, with an estimated incidence of 0.1–0.4% of all brain tumors. Clinically, they can be difficult to diagnose, as they are often

514

asymptomatic. Occasionally patients with intracranial lipomas can present with seizures (up to 50% of patients) or mental disorders. They have also been described in association with a rare inherited disorder, familial multiple lipomatosis (FML). Lipomas typically occur in the midline and over 80% are found supratentorially. Approximately 20% occur infratentorially, typically in the cerebellopontine angle, but can also be seen in the jugular foramen and foramen magnum. They are predominantly found above the corpus callosum (~30–40%) and termed interhemispheric lipomas. These may extend into the lateral ventricles and choroid plexus. Frequently, they are seen in the pericallosal location. They can also be seen in the pineal region (20–25%), attached to the tectum suprasellar region (15–20%), or attached to the tuber cinereum. Small lipomas involving the sylvian fissures, middle cranial fossa, and insular regions are rare and account for 3% of the intracranial lipomas. The pathogenesis of intracranial lipomas is not clearly established; however, the leading theory suggests they result from abnormal persistence and abnormal differentiation of the meninx primitiva, the undifferentiated mesenchymal precursor of the leptomeninges, during the development of subarachnoid cisterns. Other theories suggest the lesions represent hypertrophy of preexisting meningeal adipose tissue, metaplasia of meningeal connective tissues, or a defective junction of the cutaneous ectoderm recovering the mesoderm occurs during neural tube development. More than 50% of lipomas occur in conjunction with brain malformations of varying degrees. They can contribute to the development of focal cortical dysplasia as demonstrated in our case, because of the physical cortical interruption and focal perfusion insufficiency. Lipomas are associated with various CNS anomalies such as agenesis/ dysgenesis of the corpus callosum, absence of septum pellucidum, cranium bifidum, spina bifida, encephaloceles, and myelomeningoceles. Pachygyria, polymicrogyria, and gray matter heterotopia have all been described in lipomas. Hypervascularization surrounding the lipoma can occur with adjacent venous angiomas, saccular aneurysms, dilated and tortuous feeding arteries, and abnormal arterial branches. On imaging, lipomas are well-delineated lobulated extra-axial/pial-based masses with fat attenuation. Interhemispheric lesions can present as two types – curvilinear (thin lipoma which curves around the corpus callosum) or tubulonodular (bulky mass often seen in association with calcifications and usually associated with corpus callosum agenesis). On MR imaging they are hyperintense on T1WI and appear hypointense on T1- and T2-weighted fatsuppression techniques. Adipose tissue lesions also feature chemical shift artifact. Patients with hemispheric lipomas often respond to medical management. Given the paucity of cases, their response to surgical management cannot be accurately evaluated.

Part VI. Central Nervous System Tumors: Case 109

Key Points  Lipomas are well-delineated, lobulated, extra-axial masses with fat attenuation, most commonly seen in the pericallosal region.  Lipomas can be associated with parenchymal and vascular anomalies such as cortical dysplasia, polymicrogyria, and hypogenesis or agenesis of the corpus callosum among others.

Suggested Reading Devasia B, Samuel AS, Kesavadas C. Lipomatous cortical dysplasia with callosal lipoma: a rare association. Pediatr Radiol 2006; 36 (1): 83.

Guye M, Gastaut JL, Bartolomei F. Epilepsy and perisylvian lipoma/ cortical dysplasia complex. Epileptic Disord 1999; 1(1): 69–73. Kakita A, Inenaga C, Kameyama S. Cerebral lipoma and the underlying cortex of the temporal lobe: pathological features associated with the malformation. Acta Neuropathol 2005; 109: 339–45. Mishra A, Chandrasekharan K. Right frontal surface lipoma associated with cortical dysplasia: an unusual location and unusual association. Eur J Radiol Extra 2011;78: e73–5. Nunes JC, Martins RF, Bastos A, et al. Brain lipoma, corpus callosum hypoplasia and polymicrogyria in familial multiple lipomatosis. Clin Neurol Neurosurg 2013; 115: 1157–9. Yildz H, Hakyemez B, Korglu M, Yesildag A, Baykal B. Intracranial lipomas: importance of localization. Neuroradiology 2006; 48(1):1–7.

515

CASE

Part VI

110

Central Nervous System Tumors Bruno Augusto Telles, Lázaro Luís Faria do Amaral

Clinical Presentation A five-year-old previously healthy boy presented with a severe headache and acute onset of vomiting, without other signs or symptoms.

Imaging (B)

(C)

(A)

Fig. 110.1 (A) Sagittal T1WI and (B–C) Axial DWI through the frontal lobe.

(A)

(B)

Fig. 110.2 (A–B) Axial T2WI through the frontal lobe.

Advanced Neuroradiology Cases, ed. Amaral et al. Published by Cambridge University Press. © Cambridge University Press 2016.

517

Part VI. Central Nervous System Tumors: Case 110

(B) (A)

Fig. 110.3 (A–B) Axial FLAIR through the frontal lobe. Fig. 110.4 Axial GRE-T2* WI sequence through the lateral ventricles.

518

Part VI. Central Nervous System Tumors: Case 110

(A) (B)

Fig. 110.5 (A) Axial T1WI and (B) Sagittal T1WI postgadolinium through the lesion in the left frontal lobe.

519

Part VI. Central Nervous System Tumors: Case 110

Supratentorial Ependymoma Primary Diagnosis Supratentorial ependymoma

Differential Diagnoses Atypical teratoid/rhabdoid tumor Primitive neuroectodermal tumor Teratoma Choroid plexus carcinoma

Imaging Findings Fig. 110.1: (A) Sagittal T1WI showed a large, subcortical, solid-cystic mass in the left frontal lobe that is iso- to hypointense. (B–C) Axial DW MR images showed restricted diffusion in the solid component of the mass. Fig. 110.2: (A–B) Axial T2W images demonstrated a large frontal mass, with low signal on T2. Fig. 110.3: (A–B) Axial FLAIR images showed a moderately hyperintense tumor with central areas of high signal intensity due to necrosis. Fig. 110.4: Axial GRE-T2* WI sequence showed hypointense areas in the solid component representing hemorrhage or calcifications. Fig. 110.5: (A) Axial T1W and (B) Sagittal T1W postgadolinium images demonstrated a moderate enhancement of the tumor with nonenhancing areas of necrosis.

Discussion In children between the age of 1 and 10 years, intracranial hypertension and a heterogeneous subcortical solid-cystic mass is suggestive of an ectopic ependymoma. Atypical teratoid/rhabdoid tumor (AT/RT) and primitive neuroectodermal tumor (PNET) should be considered in infants (< 1 year of age), particularly if the tumor demonstrates diffusion restriction on DWI. Choroid plexus carcinoma should be considered if the tumor is intraventricular and arises from the atrial choroid plexus with invasion of the adjacent brain and demonstrates diffusion restriction. Teratoma should be considered in the differential diagnosis if the tumor is at midline and presents in infants less than six months of age. Ependymomas are glial tumors derived from differentiated ependymal cells lining the ventricles of the brain and the central canal of the spinal cord. They are common neoplasms, constituting 3–9% of all neuroepithelial neoplasms, 6–12% of all pediatric brain tumors, and almost one-third of all brain tumors in patients younger than three years of age. Forty percent of ependymomas are supratentorial, while 60% are infratentorial in location. The supratentorial ependymomas

520

generally manifest in an older age group (mean age, 18–24 years). Radiologically, supratentorial ependymomas more commonly arise from brain parenchyma, unlike infratentorial ependymomas, which are more often located in the fourth ventricle or its recesses. Location of the ependymoma has a variable appearance on imaging. They are usually welldemarcated, iso- to hyperdense masses, as compared to the adjacent gray matter. These tumors are usually large at presentation, with calcification and/or cystic degeneration present in up to 40%. Signal characteristics of the tumor become very complex in the presence of intratumoral hemorrhage. The solid component enhances variably with contrast administration and can demonstrate intense homogeneous enhancement or rim-like enhancement. The peritrigonal region is more commonly involved, although subcortical and intraventricular involvement is possible. Radical resection of the tumor is the treatment of choice. Postoperative radiation therapy must be administered in every case of partially resected ependymomas, as well as for those cystic extraventricular ependymomas or lesions located near eloquent brain areas, even after apparent total resection.

Key Points  Supratentorial ependymoma should be considered as a diagnosis with a heterogeneous off-midline extraventricular parenchymal tumor with both solid and cystic components.  Variable contrast enhancement of the solid component is a typical imaging finding of ependymomas, in addition to occasional calcification and intratumoral hemorrhage.  The typical age at presentation is between one and five years.

Suggested Reading Allen JC, Siffert J, Hukin J. Clinical manifestations of childhood ependymoma: a multitude of syndromes. Pediatr Neurosurg 1998; 28(1): 49–55. Lefton DR, Pinto RS, Martin SW. MRI features of intracranial and spinal ependymomas. Pediatr Neurosurg 1998; 28(2): 97–105. Mermuys K, Jeuris W, Vanhoenacker PK, Van Hoe L, D’Haenens P. Best cases from the AFIP: supratentorial ependymoma. Radiographics 2005; 25(2): 486–90. Raybaud C, Barkovich AJ. Intracranial, orbital, and neck masses of childhood. In: Barkovich AJ, Raybaud C, eds. Pediatric Neuroimaging, 5th edn. Philadelphia, PA: Lippincott Williams & Wilkins; 2012: 756. Yuh EL, Barkovich AJ, Gupta N. Imaging of ependymomas: MRI and CT. Childs Nerv Syst 2009; 25(10): 1203–13.

CASE

Part VI

111

Central Nervous System Tumors Afonso C. P. Liberato, Lázaro Luís Faria do Amaral

Clinical Presentation A 28-year-old man with a long history of intractable epilepsy, with noticeable absence of neurologic deficit, presented to our facility for evaluation.

Imaging (A)

(B)

(C)

Fig. 111.1 (A) Axial FLAIR through the lateral ventricles, (B) through the centrum semiovale, and (C) through the frontal lobes.

Advanced Neuroradiology Cases, ed. Amaral et al. Published by Cambridge University Press. © Cambridge University Press 2016.

521

Part VI. Central Nervous System Tumors: Case 111

(B)

(A)

(C)

Fig. 111.2 (A) Axial 3D-CISS through the right frontal operculum, (B) through the left middle frontal gyrus, and (C) through the left superior frontal gyrus.

522

Part VI. Central Nervous System Tumors: Case 111

(A)

(B)

(C)

Fig. 111.3 (A) Axial contrast-enhanced T1WI through the right frontal operculum, (B) through the left middle frontal gyrus, and (C) through the left superior frontal gyrus.

Fig. 111.4 T2 Perfusion-DSC through the large lesion in the right frontal operculum.

523

Part VI. Central Nervous System Tumors: Case 111

Multifocal Dysembryoplastic Neuroepithelial Tumor Primary Diagnosis Multifocal dysembryoplastic neuroepithelial tumor

Differential Diagnoses Low-grade astrocytoma Ganglioglioma Angiocentric glioma Oligodendroglioma Pleomorphic xanthoastrocytoma

Imaging Findings Fig. 111.1: (A–C) Axial FLAIR showed a multifocal hyperintense lesion in the right frontal operculum, left superior frontal gyrus, and left middle frontal gyrus. Fig. 111.2: (A–C) Axial 3D-CISS showed a solid multifocal hyperintense lesion in the right frontal operculum, left superior frontal gyrus, and left middle frontal gyrus. Fig. 111.3: (A–C) Axial contrastenhanced T1WI showed a solid, multifocal non-enhancement lesion in the right frontal operculum, left superior frontal gyrus, and left middle frontal gyrus. Fig. 111.4: T2 perfusionDSC showed a low CBF in the large lesion on the right frontal operculum (cold perfusion).

Discussion In the brain of a young patient with refractory epilepsy, intraaxial, cortical-based masses, with a cyst-like appearance, that lack perilesional edema or contrast enhancement demonstrate the typical imaging appearance suggestive of a multifocal dysembryoplastic neuroepithelial tumor (DNET). Low-grade astrocytoma is also a T2-hyperintense, intraaxial mass that does not enhance; nevertheless, cortical involvement, the presence of a FLAIR hyperintense ring around the lesion, and scalloping of the overlying bone are not features of this entity. The most common appearance of gangliogliomas on MR imaging is that of a cyst, with a strongly enhancing mural nodule that frequently demonstrates some calcification in the lesion, which was absent in this patient. Oligodendrogliomas typically manifest as a round or oval sharply marginated mass involving the cortex or subcortical white matter. Occasionally, the tumor margins are not well defined, as the tumor appears to blend imperceptibly into the normal adjacent brain parenchyma. Calcification, usually coarse in morphology, is noted in 20–91% of cases. Occasionally, cystic degeneration and hemorrhage may be seen. Subtle ill-defined enhancement following intravenous contrast material administration is seen in 15–20% of oligodendrogliomas and is associated with higher-grade tumors. Angiocentric glioma typically presents as a cortical tumor with a hyperintense appearance on T1-weighted imaging and has a stalk-like extension of the lesion projecting to the

524

ventricular surface, both of which were absent in this patient. Although usually superficial tumors, pleomorphic xanthoastrocytomas (PXAs) typically manifest as cystic tumors with a strongly enhancing mural nodule, often with an adjacent dural tail of enhancement, features not present in our case. A benign and rare primary brain neoplasm, DNET is included in the neuronal and mixed neuronal-glial tumor category of WHO grade I. Although any lobe in the brain may be involved, DNET presents as a supratentorial and cortical intra-axial lesion that is characterized by a multinodular architecture and is usually located in the temporal lobe. Differentiating DNET from other brain tumors is important because patients with DNET benefit from complete resection, as the presence of residual tumor is a risk factor for relapse of seizures. Localized DNET is widely reported, but multifocal DNET remains an extremely rare entity, with only a handful of reported cases in the literature. Clinicoradiologic criteria for the diagnosis of DNET are as follows: 1) a history of refractory partial seizures, with or without generalization, beginning before 20 years of age; 2) the absence of neurologic deficit; 3) the cortical location of the lesion; 4) the presence of remodeling of the inner table; and 5) the absence of mass effect and peritumoral edema. A well-demarcated, lobulated, or multilobulated-appearing mass, DNET is typically hypodense on CT scan images, hypointense on T1, and hyperintense on T2-weighted MR images. It may demonstrate intratumoral cystic or microcystic changes in up to 40% of cases. Calcification on CT images can be seen in up to one-third of DNET tumors. Contrast enhancement can be seen in up to 40% of patients. On spectroscopy, it demonstrates increased myoinositol (mI) peak and is metabolically inactive on FDG and amino acid PET imaging. Our patient’s clinicoradiologic presentation demonstrated all of the DNET diagnostic criteria. In addition, his imaging studies demonstrated other typical radiologic features of DNET, namely a wedge-like shape, the presence of internal septations, scalloping of the overlying cortical bone, a FLAIR hyperintense ring sign, high ADC values, and the absence of calcification and contrast enhancement, within his tumor.

Key Points  Intra-axial cortical-based masses with a cyst-like appearance in the brain in a young patient with refractory epilepsy, without perilesional edema or contrast enhancement, demonstrate the typical imaging appearance of a DNET.  Under rare conditions, DNET can be multifocal and have similar imaging abnormalities.

Suggested Reading Fernandez C, Girard N, Paz Paredes A, et al. The usefulness of MR imaging in the diagnosis of dysembryoplastic neuroepithelial tumor in children: a study of 14 cases. AJNR Am J Neuroradiol 2003; 24 (5): 829–34.

Part VI. Central Nervous System Tumors: Case 111 Krossnes BK, Wester K, Moen G, Mørk SJ. Multifocal dysembryoplastic neuroepithelial tumour in a male with the XYY syndrome. Neuropathol Appl Neurobiol 2005; 31(5): 556–60. Ostertun B, Wolf HK, Campos MG, et al. Dysembryoplastic neuroepithelial tumors: MR and CT evaluation. AJNR Am J Neuroradiol 1996; 17(3): 419–30.

Stanescu Cosson R, Varlet P, Beuvon F, et al. Dysembryoplastic neuroepithelial tumors: CT, MR findings and imaging follow-up: a study of 53 cases. J Neuroradiol 2001; 28(4): 230–40. White RD, Kanodia AK, Sammler EM, Brunton JN, Heath CA. Multifocal dysembryoplastic neuroepithelial tumour with intradural spinal cord lipomas: report of a case. Case Rep Radiol 2011; 2011: 734171.

525

CASE

Part VI

112

Central Nervous System Tumors Lucído Portella Nunes Neto, Christiane Monterio Siqueira Campos, Lázaro Luís Faria do Amaral

Clinical Presentation A 60-year-old man presented to our facility with headache and diplopia due to right oculomotor nerve palsy and right trigeminal nerve sensory loss (cavernous sinus syndrome). No other focal neurologic deficits or systemic symptoms were noted.

Imaging (A)

(B)

Fig. 112.1 (A) Sagittal and (B) Coronal T1WI through the level of the right temporal horn of the lateral ventricles.

(A)

(B)

Fig. 112.2 (A) Axial T2WI and (B) MPGR-T2* through the temporal lobes.

Advanced Neuroradiology Cases, ed. Amaral et al. Published by Cambridge University Press. © Cambridge University Press 2016.

527

Part VI. Central Nervous System Tumors: Case 112

(A) (B)

(C)

Fig. 112.3 (A–C) Coronal T1 postcontrast dynamic images through the level of the right temporal lobe.

528

Part VI. Central Nervous System Tumors: Case 112

(B)

(A)

Fig. 112.4 (A) Coronal and (B) Axial delayed T1WI postcontrast through the temporal lobes (after Figs. 112.1–112.3).

Fig. 112.5 Sagittal further delayed T1WI postcontrast (after Figs. 112.1–112.4) through the right temporal lobe.

529

Part VI. Central Nervous System Tumors: Case 112

Cavernous Sinus Hemangioma Primary Diagnosis Cavernous sinus hemangioma

Differential Diagnoses Schwannoma Meningioma Chordoma Chondrosarcoma

Imaging Findings Fig. 112.1 (A) Sagittal and (B) Coronal T1WI demonstrated a large mass involving the right parasellar region, exhibiting T1WI hypointense signal. Fig. 112.2 (A) Axial T2WI demonstrated marked hyperintensity. (B) T2* sequence demonstrated multiple prominent flow voids. The mass dislocates the right internal carotid artery medially and anteriorly. Fig. 112.3 (A–C) Coronal dynamic contrast-enhanced images demonstrated progressive filling-in enhancement. Fig. 112.4 (A) Coronal and (B) Axial delayed T1WI (obtained after Figs. 112.1–112.3) demonstrated greater enhancement. Fig. 112.5 Sagittal further delayed T1-weighted image (obtained after Figs. 112.1–112.4) revealed nearly homogeneous enhancement.

Discussion Cavernous sinus syndrome, secondary to a parasellar mass that demonstrates a low signal on T1WI, marked high signal on T2WI, and progressive postcontrast filling-in enhancement (over time) is highly suggestive of a cavernous sinus hemangioma (CSH). The presence of multiple flow voids also suggests the presence of a hypervascular lesion. The two most common primary tumors of the cavernous sinus are meningiomas and schwannomas. Schwannomas are isointense to hypointense on T1WI and hyperintense on T2WI; however, their enhancement pattern is often heterogeneous, and they follow the course of the nerve from which they arise, exhibiting a dumb-bell shape. Meningiomas show intense and homogeneous enhancement by contrast medium, but their signal is most commonly similar to gray matter on T1 and T2. Angioblastic meningiomas and tumors of cartilaginous origin may present with high signal intensity on T2 images and intense, homogeneous enhancement after contrast administration – making differential diagnosis challenging. However, unlike a CSH, the dynamic contrast-enhanced imaging does not demonstrate progressive contrast filling-in. In addition, chordomas and chondrosarcomas may show calcification, osseous erosion, or destruction – not seen in our patient. Radiolabeled red cell blood pool scintigraphy may be useful to distinguish CSH from other entities, since meningiomas and tumors of cartilaginous origin demonstrate a photopenic cavernous sinus region, in contrast to the progressive increase in tracer accumulation exhibited by cavernous hemangioma.

530

The third most common primary tumor of the parasellar region, CSHs represent less than 1% of all parasellar masses and occur more frequently in women during their fifth decade of life. They usually present as cavernous sinus syndrome, with paresis of one or more cranial nerves (III to VI) and may initially present during pregnancy. Formed by sinusoidal spaces with an endothelial lining containing slow-flowing or stagnant blood, CSHs are hamartomas or vascular malformations. Histopathologically, they are classified into three subtypes. Type A has a large number of sinusoids, with a large lumen, thin walls, and little connective tissue in between. Type B has a solid parenchyma with welldeveloped vascularization and connective tissue. In type C, the two patterns coexist. Cavernous sinus hemangioma has a similar histologic appearance to intra-axial cavernous hemangioma; however, they have a different clinical presentation and different imaging appearance. Intra-axial cavernous hemangiomas typically present as cavernous sinus mass without any appreciable blooming artifact or hypointense hemosiderin ring. The characteristic image appearance of cavernous sinus mass is low signal on T1-weighted images, marked high signal on T2-weighted images, and progressive filling-in enhancement after contrast administration. Vascular blush is seen in up to 80% of CSHs. Labeled red cell blood pool scintigraphy demonstrates a progressive increase in tracer accumulation. Gamma Knife radiosurgery and surgical resection are the treatments of choice, the latter preferred for larger lesions. Cavernous sinus lesions are common and have a broad differential diagnosis. Cavernous hemangiomas may be less common, but their preoperative diagnosis is crucial, since they tend to bleed more during surgery and thus require a different surgical approach and technique. The rate of cavernous hemangioma misdiagnosis remains high and radiologists have a crucial role in making an accurate diagnosis – either in determining an appropriate diagnostic protocol, with dynamic contrast-enhanced imaging, or suggesting further investigation with scintigraphy.

Key Points  Classical imaging findings of CSH include a T1 hypointense, T2 hyperintense mass, and progressive fillingin enhancement.  Unlike intra-axial cavernous malformation, seizure is not a clinical manifestation and blood products are typically absent.

Suggested Reading Jinhu Y, Jianping D, Xin L, Yuanli Z. Dynamic enhancement features of cavernous sinus cavernous hemangiomas on conventional contrast-enhanced MR imaging. AJNR Am J Neuroradiol 2008; 29(3): 577–81. Lee JH, Lee HK, Park JK, Choi CG, Suh DC. Cavernous sinus syndrome: clinical features and differential diagnosis with MR imaging. AJR Am J Roentgenol 2003; 181(2): 583–90.

Part VI. Central Nervous System Tumors: Case 112 Razek AA, Castillo M. Imaging lesions of the cavernous sinus. AJNR Am J Neuroradiol 2009; 30(3): 444–52. Salanitri GC, Stuckey SL, Murphy M. Extracerebral cavernous hemangioma of the cavernous sinus: diagnosis with MR imaging and labeled red cell

blood pool scintigraphy. AJNR Am J Neuroradiol 2004; 25(2): 280–4. Yin YH, Yu XG, Xu BN, et al. Surgical management of large and giant cavernous sinus hemangiomas. J Clin Neurosci 2013; 20(1): 128–33.

531

CASE

Part VI

113

Central Nervous System Tumors Anderson B. Belezia, Lázaro Luís Faria do Amaral

Clinical Presentation A 47-year-old man presented to our facility with an acute onset of intense headache. Computed tomography and MRI studies of the brain were performed (Figs. 113.1 and 113.2). Four years after initial presentation, he returned to our facility after an ictus of aphasia and right hemiplegia. Follow-up MR studies were performed (Figs. 113.3 and 113.4).

Imaging (A)

(B)

Fig. 113.1 (A–B) Non-contrast CT through the middle fossa and lateral ventricles.

Advanced Neuroradiology Cases, ed. Amaral et al. Published by Cambridge University Press. © Cambridge University Press 2016.

533

Part VI. Central Nervous System Tumors: Case 113

(B) (A)

Fig. 113.2 (A–B) Non-contrast T1WI through the middle fossa and lateral ventricles.

(A) (B)

Fig. 113.3 (A–B) MRI – PDWI through the lateral ventricles (follow-up four years after onset).

534

Part VI. Central Nervous System Tumors: Case 113

(A) (B)

Fig. 113.4 (A–B) MRI – T2WI through the lateral ventricles (follow-up four years after onset).

535

Part VI. Central Nervous System Tumors: Case 113

Ruptured Dermoid Cyst with Cerebral Ischemia Primary Diagnosis Ruptured dermoid cyst with cerebral ischemia

Differential Diagnoses Epidermoid cyst Craniopharyngioma Teratoma Lipoma

Imaging Findings Fig. 113.1: (A–B) Axial non-contrast CT images demonstrated a round-oval mass in the right medial fossa, with fat attenuation and fat in the subarachnoid space. Fig. 113.2: (A–B) Non-contrast T1WI showed a mass with hyperintense contents (similar to fat), and demonstrated the presence of fat-like droplets in the subarachnoid cisterns and cerebral sulci, most evident on the left sylvian fissure. Fig. 113.3: (A–B) Axial, proton density MR images (follow-up four years after onset) showed an ischemic area in the left frontal operculum and thalamus ipsilateral. Fig. 113.4: (A–B) T2WI images (followup four years after onset) confirm the hyperintense area on the left thalamus and left frontal lobe due to stroke, secondary to chemical meningitis.

Discussion The presence of fat droplets in the CSF spaces of both the sulci and the ventricles are classical imaging findings of a ruptured dermoid cyst, as is the typical clinical presentation with acuteonset severe headache seen in this patient. Unlike dermoid cysts, epidermoid cysts typically resemble CSF (not fat) on imaging studies, do not have dermal appendages, and are usually located off midline. Craniopharyngiomas are usually strikingly hyperintense on T2-weighted images and enhance strongly. Teratomas may also have a similar location but usually occur in the pineal region. Lipomas demonstrate homogeneous fat attenuation and/or signal intensity and show a chemical shift artifact on imaging. Dermoid cysts are rare congenital lesions of ectodermal origin that arise from inclusion of ectodermal elements in the neural groove at its time of closure and account for less than 0.5% of all intracranial tumors. They usually present in the third decade of life and are most commonly located in the midline posterior fossa, but may also occur in the cisterns, sella turcica, and intraventricular region. Dermoids located in the CSF cisterns lack communication between the cyst and the subarachnoid space. However, spontaneous (most common), iatrogenic, or traumatic rupture results in the dissemination of lipid material from the dermoid tumors into the CSF. Dermoid cyst rupture disperses the fatty content into the subarachnoid space cisterns and

536

ventricles – causing chemical meningitis, hydrocephalus, vasospasm, and cerebral ischemia. Clinical symptoms of acute rupture are diverse and include headache, nausea, vomiting, vertigo, vision problems, hemiplegia, mental changes, and coma. Chemical meningitis may lead to transient cerebral ischemia, secondary to vasospasm with complicating infarction and patient death. Post-cystic rupture symptoms are usually time delayed over a period of months to years, as the irritative effects of the spilled contents is time sensitive. Infarction may occur anywhere in the affected area, depending on the involved vessels. Imaging findings vary, depending on whether or not the cyst has ruptured. Unruptured cysts are homogeneous with low attenuation values on CT scans. On MRI, these lesions have the same imaging characteristics as fat. They are hyperintense on T1-weighted MR images and do not enhance. Dermoid cyst masses have heterogeneous signal intensity on T2-weighted MR images and vary from hypo- to hyperintense depending on the cystic components such as bones, cartilage, and calcifications, throughout or within one or more location of the lesion. The best diagnostic imaging clue of a ruptured dermoid cyst is the demonstration of fat-like droplets in the subarachnoid cisterns and sulci. Within the ventricles, a flat fluid level may be observed that is characterized by a high-intensity fluid level anterior to the hypointensity of CSF in T1-weighted sequences. Extensive pial enhancement can be seen from chemical meningitis caused by ruptured cysts. In addition, there may be subarachnoid spread, as well as sulcal widening. A chemical shift artifact is frequently projected into the lesion on long TR sequences. In patients with large dermoid cysts with previously reported neurologic symptoms, surgical resection is recommended, although the risk of injury to nearby neurovascular structures must be considered. In ruptured dermoid cysts, corticoids may be administered to alleviate the symptoms of chemical meningitis. Although recurrence after subtotal resection is extremely rare, close follow-up is recommended. In most cases, the disseminated fat lesions accumulated from rupture will remain stable in position, without consequences.

Key Points  Intracranial dermoid cysts are benign, rare, congenital lesions.  When intact, dermoid cysts demonstrate very low density on CT and hyperintense signal on T1 MR images, with little to no contrast enhancement.  Upon rupture of a dermoid cyst, the widespread presence of T1 hyperintense droplets in the CSF spaces is typically seen.  Imaging studies should be evaluated carefully for complications related to cystic rupture, such as ischemia, infarction, or chemical meningitis.

Part VI. Central Nervous System Tumors: Case 113

Suggested Reading Carberry GA, Medhkour A, Elsamaloty H. Intraventricular rupture of an intracranial dermoid cyst in a woman with pseudotumour cerebri. Pan Arab J Neurosurg 2011; 15 (2): 64–6. Čatić, AJ, Nikšić M, Kadenić Z. Ruptured intracranial dermoid cyst: a case report. J Health Sci 2012; S.1.2(3): 232–5.

Esquenazi Y, Kerr K, Bhattacharjee MB, Tandon N. Traumatic rupture of an intracranial dermoid cyst: case report and literature review. Surg Neurol Int 2013; 4: 80. Kang MG, Kim KJ, Seok JI, Lee DK. Intracranial dermoid cyst rupture with midbrain and thalamic infarction. Neurology 2009; 72(8): 769. Liu JK, Gottfried ON, Salzman KL, Schmidt RH, Couldwell WT. Ruptured intracranial dermoid cysts: clinical, radiographic, and surgical features. Neurosurgery 2008; 62(2): 377–84; discussion 384.

537

CASE

Part VI

114

Central Nervous System Tumors Bruno Augusto Telles, Asim K. Bag, Lázaro Luís Faria do Amaral

Clinical Presentation A previously normal, immunocompetent, 59-year-old woman presented with a history of facial paresis and leftsided paresthesia that progressed over a two-month period. She denied any history of fever. Except for the left-sided hemiparesis, neurologic examination was otherwise normal,

and no sign of meningism was noted. Image findings were normal (with exception of featured studies) and did not reveal any primary malignancies. Values from all hematologic tests including WBC count, SED, and C-reactive protein were normal. Cerebrospinal fluid was also negative for clinical findings.

Imaging (B) (A)

Fig. 114.1 (A) Axial FLAIR and (B) Axial DWI through the level of the lateral ventricles.

Advanced Neuroradiology Cases, ed. Amaral et al. Published by Cambridge University Press. © Cambridge University Press 2016.

539

Part VI. Central Nervous System Tumors: Case 114

(A)

(B)

Fig. 114.2 (A) Axial precontrast T1WI and (B) Axial postcontrast T1WI through the level of the lateral ventricles.

(A)

(B)

Fig. 114.3 (A) Non-contrast perfusion, arterial spin labeling (ASL), and (B) Perfusion T2-DSC through the level of the lateral ventricles.

540

Part VI. Central Nervous System Tumors: Case 114

(B)

(A)

Fig. 114.4 (A–B) 3D spectroscopy with TE = 144 ms in the central part of the lesion.

(B)

(A)

Fig. 114.5 (A–B) 3D spectroscopy with TE = 144 ms in the peritumoral area of the FLAIR abnormalities.

541

Part VI. Central Nervous System Tumors: Case 114

Cystic Glioblastoma Primary Diagnosis Cystic glioblastoma

Differential Diagnoses Brain abscess Solitary metastasis Tumefactive demyelination Juvenile pilocytic astrocytoma Pleomorphic xanthoastrocytoma Neuroglial cyst

Imaging Findings Fig. 114.1: (A) Axial FLAIR and (B) Axial DWI demonstrated a hypointense lesion with significant perilesional edema and no restricted diffusion. Fig. 114.2: (A) Axial precontrast T1WI and (B) Axial postcontrast T1WI demonstrated a large irregular and thick ring-enhancing lesion in the right frontal lobe. Fig. 114.3: (A) Perfusion without contrast, arterial spin labeling (ASL), showed increased rCBF in the periphery of the lesion. (B) Contrast-enhanced dynamic susceptibility-weighted perfusion MRI showed significantly elevated CBV at the periphery of the lesion. Fig. 114.4: (A) Axial FLAIR localizer and (B) 3D multivoxel spectroscopy images with TE of 144 ms (not shown) demonstrated increased choline-to-creatine ratio, decreased NAA peak over the enhancement component of the tumor, and a large inverted lactate peak at the nonenhancing central part. Fig. 114.5: (A) Axial FLAIR localizer and (B) 3D multivoxel spectroscopy images with TE of 144 ms (not shown) demonstrated increased choline-to-creatine ratio and decreased NAA peak, and an inverted lactate peak is also noted in the peritumoral area of the FLAIR abnormality beyond the enhancing component.

Discussion Significantly increased CBF (evidenced by ASL images), CBV (evidenced by DSC perfusion images), and Ktrans (evidenced by DCE images) are suggestive of significantly increased neoangiogenesis with leakiness of the blood vessels that is typically seen in glioblastoma (GBM), the most common primary CNS malignancy in adults. Presence of a large area of central necrosis and suggestive spectroscopy findings confirmed the diagnosis. Two major differential diagnoses that should be excluded are abscess and metastasis. The absence of systemic and neurologic signs of infection, and the presence of diffusion restriction at the tumor core rule out pyogenic abscess. Significantly, increased perfusion parameters at the lesion margin are suggestive of an angiogenic tumor, rather than an abscess. However, subtle increased perfusion parameters in association with abscess have been described in the literature. Abscesses originating from fungal infection (blastomycosis or

542

aspergillosis) or amoebiasis do not demonstrate any significantly increased perfusion. Brain metastases are often multiple and tend to occur peripherally at gray-white matter junctions, although solitary brain metastases are common. Almost 100% signal recovery and tumoral spectral pattern beyond the enhancing component of the tumor is suggestive of an infiltrative tumor, rather than a metastasis. Tumefactive demyelination is excluded by the presence of complete rather than incomplete ring enhancement, significantly elevated perfusion parameters, and the infiltrative nature of the lesion. Increased perfusion parameters and infiltrative growth pattern are also not features of juvenile pilocytic astrocytoma (WHO grade 1) or pleomorphic xanthoastrocytoma (WHO grade 2) tumors. Neuroglial cysts, which lack enhancement and increased perfusion on imaging, can also be excluded. Glioblastoma is the most prevalent primary brain tumor, representing between 12% and 15% of all intracranial neoplasms. Primary tumors account for 95% of all GBMs. The median age for GBM diagnosis is 64 years, but it can occur at any age. The male-to-female incidence ratio is almost 1. The clinical symptoms vary according to location, but seizures, focal neurologic deficits, and mental status changes are the most common symptoms. Glioblastoma has extremely variable imaging appearance. A thick and irregular ring enhancement surrounding a necrotic core with an infiltrative pattern of growth is the most common imaging pattern. Completely necrotic GBM, as seen in this patient, is a rare presentation of GBM and should always be considered in the differential diagnosis of a cystic lesion. The histopathologic hallmark of GBM is endothelial proliferation and necrosis (pseudopalisading). Significantly increased perfusion parameters and central necrosis on imaging correlate with the histopathologic findings. Increased choline-to-creatine ratio over the enhancing component is suggestive of active cell growth. Elevated choline-to-creatine ratios beyond the enhancing margin are key imaging findings and can be used to differentiate GBM from other lesions such as abscess, demyelination, and metastasis. Dual rim sign on SWI has recently been described in abscesses and can be used to differentiate abscess from GBM.

Key Points  A symptomatic solitary cystic lesion should be evaluated with advanced MRI modalities.  Increased perfusion parameters and almost complete signal recovery in the time-to-signal intensity curve on DSC perfusion MRI are characteristic features of a GBM.  Diffusion restriction at the center of the lesion is a key imaging finding in abscess, and is not present in this patient.  Infiltrating growth pattern beyond the enhancing margin is a feature of GBM.

Part VI. Central Nervous System Tumors: Case 114

Suggested Reading Hakyemez B, Erdogan C, Yildirim N, Parlak M. Glioblastoma multiforme with atypical diffusion-weighted MR findings. Br J Radiol 2005; 78(935): 989–92. Muccio CF, Leonini S, Esposito G, Cerase A. Pyogenic abscess from Providencia stuartii mimicking necrotic tumour at perfusion-weighted imaging. Neurol Sci 2011; 32(5): 919–23. Rees JH, Smirniotopoulos JG, Jones RV, Wong K. Glioblastoma multiforme: radiologic-pathologic correlation. Radiographics 1996; 16(6): 1413–38; quiz 1462–3.

Toh CH, Wei KC, Chang CN, et al. Differentiation of pyogenic brain abscesses from necrotic glioblastomas with use of susceptibility-weighted imaging. AJNR Am J Neuroradiol 2012; 33(8): 1534–8. Toh CH, Wei KC, Ng SH, et al. Differentiation of brain abscesses from necrotic glioblastomas and cystic metastatic brain tumors with diffusion tensor imaging. AJNR Am J Neuroradiol 2011; 32(9): 1646–51.

543

CASE

Part VI

115

Central Nervous System Tumors Bruno Augusto Telles, Asim K. Bag, Lázaro Luís Faria do Amaral

Clinical Presentation A previously healthy three-year-old boy presented with acuteonset headache, vomiting, and visual disturbances.

Imaging (A)

(B)

Fig. 115.1 (A–B) Axial T2WI through the sellar and suprasellar region.

Advanced Neuroradiology Cases, ed. Amaral et al. Published by Cambridge University Press. © Cambridge University Press 2016.

545

Part VI. Central Nervous System Tumors: Case 115

(A)

(B)

Fig. 115.2 (A–B) Axial DWI through the suprasellar region.

(A) (B)

Fig. 115.3 (A–B) Sagittal T2WI through the midline.

546

Part VI. Central Nervous System Tumors: Case 115

(A)

(B)

Fig. 115.4 (A–B) Coronal T2WI at the level of the third ventricle.

(A)

(B)

Fig. 115.5 (A–B) Axial T1WI postgadolinium MR image through the suprasellar region.

547

Part VI. Central Nervous System Tumors: Case 115

Pilomyxoid Astrocytoma Primary Diagnosis Pilomyxoid astrocytoma

Differential Diagnoses Juvenile pilocytic astrocytoma Chordoid glioma of the third ventricle Ependymoma metastases Glioblastoma

Imaging Findings Fig. 115.1: (A–B) Axial T2WI showed a large, lobulated, heterogeneous, solid, cystic mass that predominantly involved the hypothalamus and optic chiasm region, in addition to extending into the right medial temporal lobe. The solid components demonstrated intense contrast enhancement. Fig. 115.2: (A–B) Axial DWI did not demonstrate diffusion restriction. Fig. 115.3: (A–B) Sagittal T2WI demonstrated extension of the lesion’s suprasellar component into the third ventricle. Fig. 115.4: (A–B) Coronal T2WI clearly demonstrated the parasellar component of the lesion, which is larger on the right side. Fig. 115.5: (A–B) Axial T1WI postcontrast showed enhancement of the lesion’s solid component and peripheral enhancement in the cystic component.

Discussion Chordoid glioma is a rare, slow-growing, non-invasive neoplasm (WHO grade II) containing both glial and chordoid histologic elements, more common in adult patients. It arises from the anterior third ventricle and is frequently adherent to the hypothalamus, ovoid in shape, well circumscribed, located in the anterior third ventricle, and separated from the pituitary gland. They invade the hypothalamus and displace the infundibulum posteriorly. At MR imaging, chordoid glioma appears as a well-defined ovoid mass that is isointense on T1-weighted images and enhances intensely with gadolinium. Most tumors are solid, but in 25% of patients, a small central cystic area is present, but calcification is rare. Ependymoma metastases are exluded in the absence of previous patient history of primary ependymoma. Glioblastoma is a more infiltrative lesion, with necrosis, and is rare in pediatric patients. In a pediatric patient, a large, predominantly midline, multilobulated, solid-cystic mass centered in the hypothalamus/optic chiasm region, which extends to the medial temporal lobe and demonstrates intense enhancement of the solid component on T2-weighted images, is a classic presentation of a pilomyxoid astrocytoma (PMA). A recently described neoplasm, PMA was originally classified as an infantile variant of pilocytic astrocytoma (PA). Now recognized as a distinct entity, PMA has a unique histologic appearance and differs from PA in its presentation, as well as its clinical course. A rare childhood tumor, whose incidence is unknown, PMA has been described in patients with a wide age range of presentation from early childhood to late adulthood.

548

However, the most common age of presentation is during early childhood with a mean presentation age of 18 months, lower than that of PA, which is 58 months. There is no known sex predilection for PMA. Like many pediatric tumors, the most common presenting clinical features of PMA are the signs and symptoms related to increased intracranial pressure. Similar to PA, PMA lesions can occur anywhere along the neuraxis; unlike PA, PMA lesions have a predilection for the hypothalamic/chiasmatic region. Almost 80% of all PMAs are located in this area. Atypical sites of origin such as the cerebellum, thalamus, brainstem, and spinal cord have been described; however, these locations are rare and are usually seen in older children or adult patients. In addition to an older diagnosispresentation age, PA lesions favor the posterior fossa. Although PMA and PA lesions have many imaging features in common, PMA lesions typically demonstrate a cystic tumor component that is smaller than the solid component, excluding PA as a diagnosis in this patient. Glioblastoma is usually a tumor of adulthood and is extremely rare in the diencephalon region, thus an unlikely diagnosis for this patient. Typically, PMA is a well-circumscribed tumor with both solid and cystic components. Unlike PA, the cystic component is smaller in comparison to the solid component and a small cyst is present in up to 85% of all PMA tumors. The solid component always demonstrates intense enhancement. Unlike high-grade tumors, the solid component does not demonstrate diffusion restriction or peritumoral edema. Infiltration is more common in PMA, as compared to PA, as spectroscopic abnormalities have been demonstrated beyond the margin of the tumor. Intratumoral hemorrhage can be seen in up to 20% of PMAs, and if present, can be useful to differentiate PMA from PA, as hemorrhage in PA is rare. Unlike PA, CSF dissemination is more common in PMAs and can be seen on imaging at presentation. Similar to PA, PMA demonstrates a high Cho/Cr ratio, a high lipid peak, and low NAA and peak on MR spectroscopy, a feature of aggressive tumors. Pilomyxoid astrocytoma is a WHO grade II neoplasm. The dominant histopathologic finding is a myxoid tumor matrix with monomorphic bipolar tumor cells arranged in an angiocentric pattern. Although Rosenthal fibers are typically absent, mitotic figures may be present. Pilomyxoid astrocytoma is associated with shorter progression-free and overall survival rates, with higher recurrence and higher rate of CNS dissemination than other childhood brain tumors. As it is an aggressive tumor, early diagnosis and intervention has the potential to affect short-term patient outcomes.

Key Points  Pilomyxoid astrocytoma most commonly occurs in the hypothalamic-chiasmatic area with frequent encroachment towards the medial temporal lobe either unilaterally or bilaterally in patients less than four years of age.  Pilomyxoid astrocytoma lesions have an H-shaped appearance on coronal plane MRI.

Part VI. Central Nervous System Tumors: Case 115

 It is solid-cystic tumor with a smaller cystic component, compared to the solid component.  Intratumoral hemorrhage is present in up to 20% of patients.

Suggested Reading Arslanoglu A, Cirak B, Horska A, et al. MR imaging characteristics of pilomyxoid astrocytomas. AJNR Am J Neuroradiol 2003; 24(9): 1906–8. Linscott LL, Osborn AG, Blaser S, et al. Pilomyxoid astrocytoma: expanding the imaging spectrum. AJNR Am J Neuroradiol 2008; 29(10): 1861–6.

Osborn AG, Salzman KL, Thurnher MM, Rees JH, Castillo M. The new World Health Organization Classification of Central Nervous System Tumors: what can the neuroradiologist really say? AJNR Am J Neuroradiol 2012; 33(5): 795–802. Pomper MG, Passe TJ, Burger PC, Scheithauer BW, Brat DJ. Chordoid glioma: a neoplasm unique to the hypothalamus and anterior third ventricle. AJNR Am J Neuroradiol 2001; 22(3): 464–9. Raybaud C, Barkovich AJ. Intracranial, orbital, and neck masses of childhood. In: Barkovich AJ, Raybaud C, eds. Pediatric Neuroimaging, 5th edn. Philadelphia, PA: Lippincott Williams & Wilkins; 2012: 658.

549

CASE

Part VI

116

Central Nervous System Tumors Satya Patro, Rahul J. Vakharia, Prasad B. Hanagandi

Clinical Presentation A 58-year-old male presented with an approximately 8- to 9-month history of gradual onset headache with left-sided weakness. He reported experiencing several episodes of generalized tonic-clonic seizures. He later developed neuropsychiatric symptoms with memory disturbances and loss of social

inhibitions. There was no history of fever or weight loss. Hematologic studies were non-contributory. Cerebrospinal fluid analysis revealed presence of an increased number of lymphocytes and an elevated protein level. Postsurgical resection histopathology revealed a caseating centrally necrotic lesion surrounded by lymphocytes, epithelioid, and giant cells.

Imaging Fig. 116.1 Axial T1WI of the brain at the level of lateral ventricles.

(A)

(B)

Fig. 116.2 (A) Axial T2WI of the brain at the level of lateral ventricles and (B) Coronal T2WI at the level of frontal lobes.

Advanced Neuroradiology Cases, ed. Amaral et al. Published by Cambridge University Press. © Cambridge University Press 2016.

551

Part VI. Central Nervous System Tumors: Case 116 Fig. 116.3 Axial GRE image of the brain at the level of lateral ventricles.

(B)

(A)

Fig. 116.4 (A) Coronal T1-weighted, postgadolinium-enhanced image of the brain at the level of frontal lobes. (B) Axial T1-weighted, postgadolinium-enhanced image of the brain at the level of lateral ventricles.

552

Part VI. Central Nervous System Tumors: Case 116 Fig. 116.5 Perfusion MR image of the brain at the level of lateral ventricles.

Fig. 116.6 Single voxel spectroscopy of the brain at 135 TE through the mass lesion.

553

Part VI. Central Nervous System Tumors: Case 116

Central Nervous System Tuberculoma Primary Diagnosis Central nervous system tuberculoma

Differential Diagnoses Glioblastoma Metastasis Lymphoma Pyogenic abscess Fungal abscess

Imaging Findings Fig. 116.1: Axial T1WI of the brain demonstrated a heterogeneous mass lesion in the right frontal lobe, with a predominantly hypointense signal. Fig. 116.2: (A) Axial and (B) Coronal T2-weighted images also demonstrated a predominantly hypointense signal. Fig. 116.3: Axial GRE image of the brain showed no presence of hemorrhage. Fig. 116.4: (A) Coronal and (B) Axial postgadolinium images showed irregular, thickwalled, and nodular peripheral enhancement. Fig. 116.5: Perfusion MR image noted absence of intralesional or perilesional elevated rCBV. Fig. 116.6: Single voxel spectroscopy at 135 TE showed elevated lipid/lactate peak at 1.3 ppm.

Discussion A constellation of imaging findings demonstrating a T1- and T2-weighted hypointense lesion with thick and irregular rim enhancement, extensive perilesional oedema with low rCBV on perfusion imaging, and lipid peak at 1.3 ppm with a decrease in NAA/choline ratio on MR spectroscopy strongly favors the diagnosis of a giant tuberculoma. Unlike tuberculomas, glioblastoma (GBM) lesions are usually hypointense on T1-weighted and hyperintense on T2-weighted sequences. Depending on the presence of intratumoral hemorrhage and necrosis, the imaging features of GBM can vary. Magnetic resonance spectroscopy in a GBM reveals elevated choline/creatine and choline/NAA ratios; MR perfusion images typically show significantly elevated rCBV, from the solid component. Metastatic lesions can have similar imaging features to GBM on conventional imaging sequences, with elevated rCBV. The lack of NAA peak on spectroscopy helps in differentiating non-neuronal tumors from GBM. Depending on immune status, primary CNS lymphomas have varied imaging patterns and range from T2 hypointense with diffusion restriction and homogeneous enhancement to heterogeneous signal changes and enhancement. They have elevated choline/creatine ratios, lipid peaks, and elevated rCBV, which is low in comparison to GBM and metastases. Fungal abscesses are commonly seen in immunocompromised patients and often show a T2 hypointense center with peripheral enhancement and demonstrate enhancing intracavitary projections, which can have diffusion restriction. Multiple peaks of trehalose sugars at 3.6 and 3.8 ppm can aid in differentiating fungal abscesses from a tuberculoma. Pyogenic

554

abscesses are T2 hyperintense, with peripheral enhancement and homogeneous diffusion restriction. Succinate and acetate peaks are additional peaks noted on MR spectroscopy. Advanced techniques such as magnetization transfer (MT) imaging can further aid the diagnosis. Neurotuberculosis results from hematogenous dissemination and constitutes 1% of all tuberculosis and 10–15% of extrapulmonary tuberculosis cases. The common spectrum of CNS involvement includes tuberculous meningitis and its complications, tuberculoma(s), focal cerebritis, and tubercular abscess. Tuberculoma begins as a non-caseating granuloma consisting of a necrotic center surrounded by lymphocytes and epithelioid and Langerhans giant cells encircled by a richly vascular zone. These lesions can further evolve into caseating abscesses and may continue to grow – forming a giant tuberculoma. The central caseation is initially solid and comprises a cheesy material with a high lipid content, macrophage infiltration, fibrosis/gliosis, free radicals, and may contain few bacilli. The center is surrounded by a thick collagenous capsule, epithelioid cells, multinucleated giant cells, and macrophages. Tuberculomas are commonly accompanied by extensive perilesional edema. Magnetic resonance imaging features vary, according to the stage of the lesion. Non-caseating granulomas are usually isointense to hypointense on T1-weighted images, hyperintense on T2-weighted images, and hyperintense on FLAIR. Nodular or ring enhancement may be seen with diffusion restriction. Caseating tuberculomas with a solid center appear relatively isointense to hypointense on both T2- and T1-weighted images. These lesions are surrounded by a rim of variable thickness that may appear hyperintense on T1- and T2weighted images with ring enhancement; evidence of diffusion restriction is noticeably absent. Tuberculomas are characterized by the presence of a target sign consisting of a ringenhancing lesion with an additional central area of enhancement or calcification. The target sign should not be confused with the eccentric target sign seen in toxoplasmosis cases. When liquefaction of the central caseation occurs within a tuberculoma, it appears hypointense on T1-weighted images. T2-weighted images demonstrate a central T2 hyperintense signal with a peripheral T2 hypointense rim. Diffusion restriction of the central liquefying caseous material can be seen with rim enhancement. The varied appearances of tuberculoma are stage dependent, mimicking cancerous lesions and/or abscesses. Advanced neuroimaging techniques such as magnetization transfer (MT) imaging, SWI, MR spectroscopy, and dynamic contrast-enhanced (DCE) perfusion studies are helpful in differentiating tuberculoma from other intracranial spaceoccupying lesions. A decrease in NAA/Cr and in NAA/Cho ratios and a prominent lipid resonance at 1.3 ppm, 2.02 ppm, and 3.7 ppm are seen during in vivo MR spectroscopy. The tuberculoma wall is rich in lipid content, secondary to inflammatory cellular infiltrate and tuberculous breakdown products from degrading bacilli. The tuberculoma wall typically

Part VI. Central Nervous System Tumors: Case 116

demonstrates hyperintensity on MT T1-weighted images and lower MT ratios ranging from 18% to 22%. Susceptiblyweighted images may help differentiate tuberculomas from other lesions by demonstrating a lack of blooming artifact from calcification or blood products. According to recent neuroradiology literature, giant tuberculomas may show varying degrees of vascularity on MR perfusion studies. In this patient, the brain imaging shows a large lesion that appears hypointense on both T1- and T2-weighted images, with thick and irregular rim enhancement. There is associated, extensive perilesional vasogenic oedema. There is low rCBV in the lesion on perfusion imaging and a large lipid peak at 1.3 ppm with a decrease in NAA/Cho ratio on MR spectroscopy. All these features are suggestive of a giant caseating tuberculoma. Polymerase chain reaction analysis aids in diagnosing CNS tuberculosis. A combination of MR images and PCR results can confirm suspected diagnosis of CNS tuberculosis. Surgical excision of the lesion is helpful in establishing the histopathologic diagnosis; however, bacilli may not always be evident.

Key Points  Giant tuberculoma can often be a diagnostic imaging dilemma and can mimic tumoral pathologies.  The imaging features vary according to the caseating or non-caseating contents.

 T2 hypointense signal, spectroscopy, and perfusion imaging are pertinent imaging sequences needed to analyze a mass lesion suspected of being a tuberculoma.  Newer imaging techniques, such as MT imaging, can be problem-solving modalities and used to confirm questionable diagnoses.

Suggested Reading DeLance AR, Safaee M, Oh MC, et al. Tuberculoma of the central nervous system. J Clin Neurosci 2013; 20(10): 1333–41. Gupta RK, Kumar S. Central nervous system tuberculosis. Neuroimaging Clin N Am 2011; 21(4): 795–814, vii-viii. Kumar D, Sheoran RK, Bansal SK, Arora OP. Revisiting the CNS tuberculosis with emphasis on giant tuberculomas and introducing the “outer rim excrescence sign.” Neuroradiol J 2011; 24(3): 357–66. Naphade PS, Raut AA, Pai BU. Magnetic resonance perfusion and spectroscopy in a giant tuberculoma. Neurol India 2011; 59(6): 913–14. Patkar D, Narang J, Yanamandala R, Lawande M, Shah GV. Central nervous system tuberculosis: pathophysiology and imaging findings. Neuroimaging Clin N Am 2012; 22(4): 677–705. Raheja A, Sinha S, Sable MN, Sharma MC, Sharma BS. A case of giant intracranial tuberculoma in an infant: clinical and radiologic pitfalls. J Child Neurol 2015; 30(3): 364–7.

555

CASE

Part VI

117

Central Nervous System Tumors Bruno Augusto Telles, Asim K. Bag, Lázaro Luís Faria do Amaral

Clinical Presentation A previously healthy 16-year-old adolescent boy presented with gradual onset refractory seizures. No previous history of epilepsy or seizure activity was noted. Routine hematologic studies were normal.

Imaging (B)

(A)

Fig. 117.1 (A) Sagittal T1WI through the temporal horn of the lateral ventricle. (B) Coronal T2WI through the third ventricle and the temporal horn of the lateral ventricles.

Advanced Neuroradiology Cases, ed. Amaral et al. Published by Cambridge University Press. © Cambridge University Press 2016.

557

Part VI. Central Nervous System Tumors: Case 117

(A) (B)

Fig. 117.2 (A–B) Coronal 3D-CISS images through the third ventricle and the temporal horn of the lateral ventricles.

(A)

(B)

Fig. 117.3 (A–B) Axial T1-weighted postgadolinium MR images through the temporal horn of the lateral ventricles.

558

Part VI. Central Nervous System Tumors: Case 117

(B)

(A)

Fig. 117.4 (A–B) Coronal T1-weighted postgadolinium MR images through the third ventricle and the temporal horn of the lateral ventricles.

559

Part VI. Central Nervous System Tumors: Case 117

Temporal Horn Choroid Plexus Papilloma Primary Diagnosis Temporal horn choroid plexus papilloma

Differential Diagnoses Subependymoma Choroid plexus carcinoma Choroid plexus metastasis Villous hypertrophy of the choroid plexus

Imaging Findings Fig. 117.1: (A) Sagittal T1-weighted MR image showed a mildly heterogeneous mass in the temporal horn of the left lateral ventricle that was slightly hypointense, as compared to the cerebellum. (B) Coronal T2-weighted MR image demonstrated a hyperintense lesion (as compared to Fig. 117.1 A). Fig. 117.2: (A–B) Coronal 3D-CISS images demonstrated a heterogeneously hyperintense lesion inside of the left temporal horn of the lateral ventricle. Fig. 117.3: (A–B) Axial T1weighted postgadolinium images showed an intense, almost homogeneous enhancement of the lesion. Fig. 117.4: (A–B) Coronal T1-weighted postgadolinium MR images demonstrated significant lesion enhancement and prominence of the temporal horn of the left lateral ventricle.

Discussion The presence of a well-defined, lobulated, intensely enhancing intraventricular tumor with frond-like appearance is typical and suggestive of a choroid plexus papilloma (CPP). Although more commonly found in the atrial choroid plexus, CPP involvement of the temporal horn choroid plexus is possible. However, even if located in the atrial choroid plexus, the typical morphologic pattern of a CPP is maintained. Subependymomas are well-demarcated, nodular intraventricular masses that may cause ventricular dilatation. Unlike CPP, subependymomas have varied enhancement patterns or can be completely non-enhancing. They also lack the typical frond-like appearance of a CPP. Choroid plexus carcinoma (CPCa) is a malignant tumor of the choroid plexus and typically involves the atrial choroid plexus. Usually CPCa is found in young children, demonstrates invasion of the adjacent periventricular regions, and may demonstrate diffusion restriction. Cerebrospinal fluid dissemination occurs with both CPP and CPCa; therefore, CSF dissemination is neither a distinguishing feature nor a reliable predictor of malignancy. Choroid plexus papilloma is the most common choroid plexus neoplasm and may develop when the differentiating fetal choroid plexus is transiently ciliated. Choroid plexus papillomas arise from the choroid plexus, and can involve any location where the choroid plexus is normally found. The atrial choroid plexus is the most common location, followed by the fourth ventricle. Development of CPP outside

560

of the lateral ventricle occurs more commonly in older children and adults. It can also present primarily as a cerebellopontine angle cistern mass, particularly if it arises from the choroid plexus of the lateral recesses of the fourth ventricle. A tumor of young children, the mean presentation age for CPP originating from a supratentorial location is 1.5 years. The infratentorial and atypical sites of origin are more commonly found in adults. The exact etiology of CPP is not clear, although several genetic mutations have been associated with this tumor. In addition, there are syndromic association of CPPs including Aicardi syndrome and Li-Fraumeni syndrome. Usually solitary tumors, CPPs vary in size. Typically, they are iso- to hyperdense on CT scans, hypointense on T1weighted sequences; and hyperintense on T2-weighted sequences. Up to one-fourth of CPPs can demonstrate calcification on CT images and may be associated with blooming artifacts on gradient echo sequences. Intense enhancement is a typical finding, both on CT and on postcontrast T1-weighted sequences. Choroid plexus papilloma typically has a frond-like appearance, which may be considered a characteristic imaging finding of this lesion. Hydrocephalus is frequently associated with these tumors, either due to obstruction or due to overproduction of CSF by the tumor cells. Although a benign tumor, CPP has been associated with leptomeningeal spread. Imaging of the entire neuraxis should be performed at the baseline presentation to rule out leptomeningeal spread. Temporal horn CPP is extremely rare and can present as temporal lobe epilepsy, as seen in our patient. The exact cause of the epilepsy is unknown but it is presumably due to compression of mesial temporal lobe structures. In the reported cases of temporal horn CPPs, hydrocephalus or ventricular entrapment is usually absent. The primary treatment for CPP is gross total resection and usually cures the epilepsy. Recurrence after primary tumor excision is low.

Key Points  A sharply marginated, intensely enhancing mass with a frond-like appearance in the expected locations of choroid plexus is highly suggestive of CPP.  The atrial choroid plexus is the most common location for CPP; other common locations are the fourth and/or third ventricle, and the cerebellopontine cisterns.  Temporal horn CPP is extremely rare.  Temporal horn CPP can present with temporal lobe seizure that can be cured with resection of the tumor.

Suggested Reading Iannelli A, Pieracci N. Tumors in the temporal horn of the lateral ventricle as a cause of epilepsy. J Child Neurol 2008; 23(3): 315–20. Koeller KK, Sandberg GD. From the archives of the AFIP. Cerebral intraventricular neoplasms: radiologic-pathologic correlation. Radiographics 2002; 22(6): 1473–505.

Part VI. Central Nervous System Tumors: Case 117 Naeini RM, Yoo JH, Hunter JV. Spectrum of choroid plexus lesions in children. AJR Am J Roentgenol 2009; 192(1): 32–40.

Phi JH, Chung CK, Lee YK, Chi JG, Kim HJ. Temporal lobe epilepsy caused by choroid plexus papilloma in the temporal horn. Clin Neuropathol 2004; 23(3): 95–8.

Osborn AG. Osborn’s Brain: Imaging, Pathology, and Anatomy. Salt Lake City, Utah; London: Amirsys; Lippincott Williams & Wilkins Europe; 2012.

Raybaud C, Barkovich AJ. Intracranial, orbital, and neck masses of childhood. In: Barkovich AJ, Raybaud C, Pediatric Neuroimaging, 5th edn. Philadelphia, PA: Lippincott Williams & Wilkins; 2012: 756.

561

CASE

Part VII

118

Congenital Diseases Manifesting in Adults Auro Augusto Junqueira Côrtes, Leonardo Furtado Freitas, Asim K. Bag, Lázaro Luís Faria do Amaral

Clinical Presentation A 23-year-old man presented with a five-year history of slowly progressive lower limb spasticity and proximal paraparesis. His medical history noted normal motor and intellectual development until he reached 15 years of age, at which time his development plateaued. Neurologic exam revealed bilateral horizontal nystagmus, bilateral hyperreflexia, mild cognitive impairment, and gait disturbance. Hematologic laboratory studies were normal.

Imaging (B) (A)

Fig. 118.1 (A) Sagittal T1WI through the midline. (B) Coronal T2WI through the level of the cerebellum.

Advanced Neuroradiology Cases, ed. Amaral et al. Published by Cambridge University Press. © Cambridge University Press 2016.

563

Part VII. Congenital Diseases Manifesting in Adults: Case 118

(A)

(B)

564

Fig. 118.2 (A–B) Axial FLAIR MR images through the level of the lateral ventricles.

Part VII. Congenital Diseases Manifesting in Adults: Case 118

(B)

(A)

Fig. 118.3 (A–B) Sagittal T2WI of the cervical and thoracic spine through the midline.

(A)

(B)

Fig. 118.4 (A) Sagittal tractography and (B) Sagittal fiber-tracking images through the level of the corpus callosum.

565

Part VII. Congenital Diseases Manifesting in Adults: Case 118

Hereditary Spastic Paraplegia with Hypoplastic Corpus Callosum Primary Diagnosis Hereditary spastic paraplegia with hypoplastic corpus callosum

Differential Diagnoses Other types of hereditary spastic paraplegia Amyotrophic lateral sclerosis Primary lateral sclerosis Levodopa-responsive dystonia Adrenomyeloneuropathy

Imaging Findings Fig. 118.1: (A) Sagittal T1WI through the midline showed significant thinning of the corpus callosum. (B) Coronal T2WI through the level of the cerebellar hemispheres revealed absence of cerebellar atrophy. Fig. 118.2: (A–B) Axial FLAIR images through the level of the lateral ventricles revealed mild ventricular enlargement and linear hyperintensity in the white matter adjacent to the frontal horns of the lateral ventricles (ear of the lynx sign). Fig. 118.3: (A–B) Sagittal T2WI through the midline of the cervical and thoracic spine confirmed absence of any lesions in the spinal cord. Fig. 118.4: (A) Sagittal tractography through the midline demonstrated significant changes including mean diffusion increase and fractional anisotropy reduction in the corpus callosum. (B) Diffusion tensor imaging fiber tractography through the level of the corpus callosum demonstrated marked reduction of the white matter fibers (tracts) in the central part of the corpus callosum.

Discussion In our patient, adult symptom onset eliminates all pediatric onset spastic paraplegias. Prominent imaging findings demonstrating corpus callosum atrophy are the key to diagnosing this rare form of autosomal recessive hereditary spastic paraplegia (HSP). Motor neuron diseases such as amyotrophic lateral sclerosis (ALS) and primary lateral sclerosis (PLS) can have similar presentations and may be extremely difficult to differentiate from HSP. In fact, motor neuron disease and HSP may be considered as components of a single disease spectrum. Imaging studies from patients with ALS demonstrate specific, typical imaging features that are similar to levodoparesponsive dystonia, which may present with similar clinical features. However, although these pathologies share imaging features, none of these demonstrates atrophy of the corpus callosum. Adrenomyeloneuropathy typically demonstrates spinal cord atrophy on imaging, which was not present in this case.

566

Hereditary spastic paraplegias comprise a genetically and clinically heterogeneous group of neurodegenerative disorders. Characterized by progressive spasticity and lower limb hyperreflexia, this family of disorders is classified as pure/uncomplicated (non-syndromic) or complicated (syndromic) based on their clinical variability. Fifty-six different genetic mutations have been described in the SPG gene (spastic paraplegia), the genetic loci responsible for HSP, that can be transmitted as autosomal dominant, autosomal recessive, or X-linked patterns of inheritance. Hereditary spastic paraplegia with hypoplastic corpus callosum (HSP-HCC) is a rare, complicated form of HSP. It is transmitted in an autosomal recessive manner and mainly described in the Japanese population. In this form, a patient who exhibited normal motor and mental development as a young child usually presents in the second decade of life with cognitive impairment, sensory disturbances, and ataxia. It has been genetically linked to mutation of the spastin protein encoded by the SPG11 locus 15q13–15, although there is heterogeneity. However, HSP-HCC has also been described in patients without this locus mutation. Thus, the exact mechanism of mutation-based development of HSP-HCC is not known. Magnetic resonance imaging findings in patients diagnosed with HSP-HCC include presence of a thin corpus callosum that predominantly involves the anterior aspect of the body and the knee of the corpus callosum. Although not a specific finding, it is distinctive. Other associated imaging findings include frontoparietal atrophy and enlargement of lateral ventricles, reduced size of thalamus, and symmetric white matter lesions. A thin corpus callosum is not specific to this syndrome and whether it represents a congenital hypoplasia or a progressive atrophy remains unknown. The symmetric periventricular white matter lesions adjacent to the frontal horn of the lateral ventricles give a flame or ear of the lynx appearance on axial FLAIR images. The diagnostic criteria of autosomal recessive HSP with a thin corpus callosum include onset of a slowly progressive spastic paraparesis, autosomal recessive inheritance pattern, cognitive impairment, thinning of the corpus callosum revealed by CT/MR imaging, and the exclusion of other disorders by laboratory tests and MR images of the spine and brain.

Key Points  In patients presenting as young adults, with a history of slowly progressive spastic paraplegia, accompanied by gradual cognitive decline, a diagnosis of HSP-HCC should strongly be considered, particularly in the setting of a positive family history.  Presence of a thin corpus callosum on imaging is strongly suggestive of HSP-HCC diagnosis.  The ears of the lynx sign on axial FLAIR images is an early imaging finding, indicative of HSP-HCC.

Part VII. Congenital Diseases Manifesting in Adults: Case 118

Suggested Reading Gucuyener K, Hirfanoglu T, Ok I, Cansu A, Serdaroglu A. Hereditary spastic paraplegia with hypoplastic corpus callosum in a Turkish family. J Child Neurol 2007; 2: 214–17. Hourani R, El-Hajj T, Barada WH, Hourani M, Yamout BI. MR imaging findings in autosomal recessive hereditary spastic paraplegia. AJNR Am J Neuroradiol 2009; 30(5): 936–40. Lossos A, Stevanin G, Meiner V, et al. Hereditary spastic paraplegia with thin corpus callosum: reduction of the SPG11 interval and evidence for further genetic heterogeneity. Arch Neurol 2006; 63 (5): 756–60.

Salinas S, Proukakis C, Crosby A, Warner TT. Hereditary spastic paraplegia: clinical features and pathogenetic mechanisms. Lancet Neurol 2008; 7: 1127–38. Stevanin G, Montagna G, Azzedine H, et al. Spastic paraplegia with thin corpus callosum: description of 20 new families, refinement of the SPG11 locus, candidate gene analysis and evidence of genetic heterogeneity. Neurogenetics 2006; 7: 149–156. Strong MJ, Gordon PH. Primary lateral sclerosis, hereditary spastic paraplegia and amyotrophic lateral sclerosis: discrete entities or spectrum? Amyotroph Lateral Scler Other Motor Neuron Disord 2005; 6(1): 8–16.

567

CASE

Part VII

119

Congenital Diseases Manifesting in Adults Prasad B. Hanagandi, Lázaro Luís Faria do Amaral

Clinical Presentation A four-year-old girl presented with a history of gelastic seizures since she was one month old. Clinical examination revealed imperforate anus, cleft palate, acromesomelic upper limb shortening (Fig. 119.1A) and postaxial polysyndactyly (Fig. 119.1B).

Imaging (A)

(B)

Fig. 119.1 (A) Image of the left upper limb depicting acromesomelic shortening. (B) Image of the hand showing polysyndactyly.

Advanced Neuroradiology Cases, ed. Amaral et al. Published by Cambridge University Press. © Cambridge University Press 2016.

569

Part VII. Congenital Diseases Manifesting in Adults: Case 119 Fig. 119.2 Midsagittal T1W MR image of the brain through corpus callosum and hypothalamo-pituitary axis.

(A)

(B)

Fig. 119.3 (A) Midsagittal T2WI of the brain through corpus callosum and hypothalamo-pituitary axis. (B) Coronal T2WI of the brain through the pituitary and suprasellar region at the level of anterior body of lateral ventricles.

570

Part VII. Congenital Diseases Manifesting in Adults: Case 119 Fig. 119.4 Axial T1W postgadolinium MR image through the hypothalamic mass lesion.

Fig. 119.5 Coronal T2WI of the abdomen through the kidneys.

571

Part VII. Congenital Diseases Manifesting in Adults: Case 119

Pallister-Hall Syndrome Primary Diagnosis Pallister-Hall syndrome

Differential Diagnoses Isolated hypothalamic hamartoma Craniopharyngioma Hypothalamic-opticochiasmatic glioma Complex syndromic associations McKusick-Kaufman syndrome Smith-Lemli-Opitz syndrome Oral-facial-digital syndrome type VI Greig cephalopolysyndactyly syndrome

Imaging Findings Fig. 119.2: Midsagittal T1WI and Fig. 119.3: (A) Midsagittal T2WI through the corpus callosum and hypothalamopituitary axis, and Fig 119.3: (B) Coronal T2WI through the pituitary and suprasellar region at the level of anterior body of lateral ventricles showed a large hypothalamic mass lesion following gray matter signal characteristics. Fig. 119.4: Axial T1W postgadolinium image through the mass lesion did not show any enhancement. Fig. 119.5: Coronal T2WI of the abdomen shows a hypoplastic right kidney with a small cyst (arrow).

Discussion The constellation of clinicoradiologic findings comprising polydactyly, anomalies involving the upper aerodigestive tract and genitourinary system, accompanied by hypothalamic hamartoma confirms the diagnosis of Pallister-Hall syndrome. On MR imaging, hamartomas have signal characteristics resembling gray matter with no evidence of calcification or enhancement. Although very rare, cystic degeneration and liquefactive necrosis changes have been described in the literature. Craniopharyngiomas have a more heterogeneous solidcystic pattern and tend to have areas of calcifications, while hypothalamic-opticochiasmatic gliomas have predominantly T2 hyperintense signal and variable enhancement. There is a considerable degree of multisystem involvement among the above-mentioned list of syndromic differential diagnoses, but the presence of hypothalamic hamartoma is the key feature of Pallister-Hall syndrome. Hypothalamic hamartoma can also be associated with oral-facial-digital syndrome type VI; however, it has other features resembling Joubert syndromerelated disorders with posterior fossa malformations that are not seen with Pallister-Hall syndrome. Mutations in GLI3 gene can cause Greig cephalopolysyndactyly syndrome. Macrocephaly, peculiar skull shape due to craniosynostosis, callosal agenesis, and mental retardation are some of the imaging and clinical features apart from the other overlapping multisystem findings. McKusick-Kaufman and Smith-Lemli-Opitz syndromes share common non-CNS multisystem involvement but hypothalamic hamartoma association is rare.

572

Judith Hall first described Pallister-Hall syndrome in 1980. It is a rare autosomal dominant pleiotropic disorder caused by GLI3 gene mutation on chromosome 7p13. As only a few cases have been described its exact prevalence is unknown. The Pallister-Hall spectrum consists of polydactyly, renal and pulmonary segmentation anomalies, imperforate anus, dysplastic nails, bifid epiglottis, panhypopituitarism, and hypothalamic hamartoma. Other craniofacial associations include cleft palate, cleft uvula, cleft larynx, buccal frenula, flat nasal bridge with short nose, and low set, posteriorly angulated ears. Although the pituitary manifestations can present with acute adrenal insufficiency or hypothyroidism, they can also be asymptomatic. On histologic examination, a hamartoma consists of a disordered arrangement of neurons mixed with astrocytes and white matter, originally described as hypothalamic hamartoblastomas. It is important to diagnose isolated hypothalamic hamartomas, as affected patients are young and present with earlyonset seizures that are resistant to antiepileptic drugs. In addition, these individuals may present with precocious puberty and cognitive impairment. Surgical intervention for hamartoma is associated with poor outcome. In contrast, patients with Pallister-Hall syndrome-associated hamartoma presenting with gelastic seizures respond well to antiepileptic medications. The extent of behavioral and cognitive disabilities is also less severe than isolated hypothalamic hamartoma. Recognizing this entity is extremely important in order to offer patients and their families genetic counseling and screening, and to correlate any associated endocrinologic symptoms.

Key Points  Clinical and endocrinology correlation along with multiorgan imaging is necessary to distinguish Pallister-Hall syndrome from isolated hypothalamic hamartoma.  Pallister-Hall syndrome has considerable multisystem overlapping features with other complex syndromic associations.  Pallister-Hall syndrome-associated hamartoma responds well to antiepileptic drugs and rarely requires surgical intervention.

Suggested Reading Alves C, Barbosa V, Machado M. Giant hypothalamic hamartoma: case report and literature review. Childs Nerv Syst 2013; 29(3): 513–16. Boudreau EA, Liow K, Frattali CM, et al. Hypothalamic hamartomas and seizures: distinct natural history of isolated and Pallister-Hall syndrome cases. Epilepsia 2005; 46(1): 42–7. Celedin S, Kau T, Kasser J, Kraschl R, Sinzig M. Fetal MRI of a hypothalamic hamartoma in Pallister-Hall syndrome. Pediatr Neurol 2010; 42(1): 59–60. Kuo JS, Casey SO, Thompson L, Truwit CL. Pallister-Hall syndrome: clinical and MR features. AJNR Am J Neuroradiol 1999; 20(10): 1839–41. McCann E, Fryer AE, Craigie R, et al. Genitourinary malformations as a feature of the Pallister-Hall syndrome. Clin Dysmorphol 2006; 15(2): 75–9.

CASE

Part VII

120

Congenital Diseases Manifesting in Adults Prasad B. Hanagandi, Lázaro Luís Faria do Amaral

Clinical Presentation A 10-year-old girl presented with a long-standing history of seizures since birth. Her birth history was uneventful, with no noted evidence of hypoxic ischemic injury. However, she developed vesicular skin eruptions towards the end of her first week of life that eventually evolved into hyperpigmented skin lesions. Laboratory analysis of CSF taken during this episode of vesicular eruption was negative for exanthematous viral infections, including herpes infection. Neurologic examination revealed profound mental retardation and spasticity. Dermatologic examination revealed swirling hyperpigmented skin lesions along the lines of Blaschko involving the anterior and posterior abdominal walls, groin, and lower extremities. General physical examination revealed poor and malformed dentition.

Imaging (B) (A)

Fig. 120.1 (A–B) Axial T2W images at the level of body of lateral ventricles.

Advanced Neuroradiology Cases, ed. Amaral et al. Published by Cambridge University Press. © Cambridge University Press 2016.

573

Part VII. Congenital Diseases Manifesting in Adults: Case 120 Fig. 120.2 Axial FLAIR image through the body of lateral ventricles.

Fig. 120.3 Midsagittal T1WI through the corpus callosum.

574

Part VII. Congenital Diseases Manifesting in Adults: Case 120

Fig. 120.4 Photograph of the abdominal wall showing hyperpigmented skin lesions.

Fig. 120.5 Photograph of the back showing hyperpigmented skin lesions.

Fig. 120.6 Photograph of the lower extremities showing hyperpigmented skin lesions.

575

Part VII. Congenital Diseases Manifesting in Adults: Case 120

Incontinentia Pigmenti Primary Diagnosis Incontinentia pigmenti

Differential Diagnoses Sequelae of neonatal HSV encephalitis Hypoxic ischemic injury

Imaging Findings Fig. 120.1: (A–B) Axial T2WI and Fig. 120.2: Axial FLAIR through the body of lateral ventricles showed extensive asymmetric encephalomalacic changes, predominantly involving the left periventricular region with CSF-filled cavity changes in the subcortical white matter. Fig. 120.3: Sagittal T1WI of the corpus callosum showed hypoplasia. Fig. 120.4: Swirling hyperpigmented lesions following the lines of Blaschko are noted along the anterior abdominal wall and groin (arrows). Fig. 120.5: Picture of the back and Fig. 120.6: Picture of the lower extremities showing similar hyperpigmented lesions.

Discussion In a female with a history of neonatal onset of seizures, negative CSF analysis for HSV infection, and lack of hypoxic injury, the presence of hyperpigmented skin lesions along the lines of Blaschko and MR imaging showing features of periventricular leukomalacia and white matter changes suggests a diagnosis of incontinentia pigmenti. Sequelae of neonatal fulminant HSV encephalitis and hypoxic ischemic injury can have similar imaging appearances but negative CSF analysis and lack of hypoxic ischemic injury exclude these entities. Incontinentia pigmenti, also known as Bloch-Siemens syndrome, Bloch-Sulzberger disease, or melanoblastosis cutis, is a rare X-linked neurocutaneous syndrome, usually affecting females, while often lethal in utero to males. Ectodermal tissues including the skin, eyes, teeth, and central nervous system are primarily affected. The diagnosis is clinical, with pathognomonic skin manifestations, namely cutaneous vesicular eruptions, along the lines of Blaschko, usually evident by the first to second week of life. Skin lesions evolve through four stages: 1) blistering (birth to four months of age); 2) wart-like rash (for several months); 3) swirling macular hyperpigmentation (six months of age to adulthood); 4) linear hypopigmentation. Central nervous system findings including seizures, psychomotor retardation, spasticity, or paralysis have been reported in 30–50% of cases. The disease is X-linked dominant, with mutations of the IKBKG gene, which regulates nuclear factor kappa B, a group of proteins essential for preventing apoptosis. Similar changes have been attributed to the NEMO gene (NF-kappa-B essential

576

modulator) located on Xq28. The mutation can be sporadic (type I) or transmitted (type II). The typical phenotype results from functional mosaicism, a consequence of lyonization. Neuroimaging findings are variable, but usually support the clinical evolution of the disease. In the neonatal period, acute cortical necrosis and subcortical hemorrhages are seen. In older children, manifestations include periventricular leukomalacia, hypoplasia of the corpus callosum, encephalomalacia, multiple cerebral infarction, and retinal vascular abnormalities. Intracranial vascular abnormalities ranging from occlusion and narrowing and impaired filling, to branching have been described in the literature. Sequelae of neonatal fulminant HSV encephalitis and hypoxic ischemic injury can have similar imaging appearances but negative CSF analysis and lack of hypoxic ischemic injury help exclude these entities. Diagnosis of incontinentia pigmenti is ultimately distinguished from other diagnostic considerations by a physical examination, revealing the characteristic skin rash. Thus, consider incontinentia pigmenti in patients presenting with a skin rash accompanied by non-specific neurologic symptoms. Acutely, neuroimaging reveals cortical necrosis and subcortical hemorrhages in the infantile stage, with a wide range of nonspecific chronic findings in later childhood and early adulthood.

Key Points  Incontinentia pigmenti is a rare neuroectodermal disorder with varied cutaneous and CNS manifestations.  The presence of skin lesions that evolve into different stages over time is an important clue that is suggestive of incontinentia pigmenti.  Sequelae of neonatal HSV encephalitis and hypoxic ischemic injury can have similar brain MR imaging appearances.

Suggested Reading Abe S, Okumura A, Hamano SI, et al. Early infantile manifestations of incontinentia pigmenti mimicking acute encephalopathy. Brain Dev 2010; 33(1): 28–34. Ardelean D, Pope E. Incontinentia pigmenti in boys: a series and review of the literature. Pediatr Dermatol 2006; 23: 523–7. Hennel SJ, Ekert PG, Volpe JJ, Inder TE. Insights into the pathogenesis of cerebral lesions in incontinentia pigmenti. Pediatr Neurol 2003; 29: 148–50. Holmstrom G, Thoren K. Ocular manifestations of incontinentia pigmenti. Acta Ophthalmol Scand 2000; 78: 348–53. Wolf NI, Kramer N, Harting I, et al. Diffuse cortical necrosis in a neonate with incontinentia pigmenti and an encephalitis-like presentation. AJNR Am J Neuroradiol 2005; 26: 1580–2.

CASE

Part VII

121

Congenital Diseases Manifesting in Adults Bruno Siqueira Campos Lopes, Lázaro Luís Faria do Amaral

Clinical Presentation A six-year-old boy presented with a long history of seizures and mental retardation. On physical examination, he had more than six hypopigmented macules on his skin (café-au-lait spots) and multiple facial angiofibromas. His mother has neurofibromatosis type 1 (NF-1).

Imaging (A)

(B)

(C)

Fig. 121.1 (A–C) Axial FLAIR images through the level of the brainstem and basal ganglia.

Advanced Neuroradiology Cases, ed. Amaral et al. Published by Cambridge University Press. © Cambridge University Press 2016.

577

Part VII. Congenital Diseases Manifesting in Adults: Case 121

(A) (B)

(C)

Fig. 121.2 (A–C) Axial T1-MTC images through the level of the basal ganglia and lateral ventricles.

578

Part VII. Congenital Diseases Manifesting in Adults: Case 121

(B) (A)

Fig. 121.3 (A–B) Axial T1 postcontrast MR images through the level of the basal ganglia and lateral ventricles.

(A)

(B)

Fig. 121.4 (A–B) Axial MPGR-T2* images through the level of the lateral ventricles.

579

Part VII. Congenital Diseases Manifesting in Adults: Case 121

Hybrid Phakomatosis Primary Diagnosis Hybrid phakomatosis

Differential Diagnoses Acute disseminated encephalomyelitis (ADEM) Gliomatosis cerebri

Imaging Findings Fig. 121.1: (A–C) Axial FLAIR showed some enlarged gyri with T2 high signal in subcortical white matter, extending to lateral ventricles. Some rounded spots with T2 high signal can also be seen in cerebellar white matter, pons, basal ganglia, and both thalami. Fig.121.2: (A–C) Axial T1-MTC showed hyperintense signal in the basal ganglia and radial bands from the subcortical white matter, extending to the lateral ventricles. Fig. 121.3: (A–B) Axial T1WI postgadolinium did not show enhancement. Fig. 121.4: (A–B) Axial MPGR-T2* showed some small calcifications in the subependymal region.

Discussion T1 hyperintense signal in the basal ganglia (vacuolar myelin degeneration) of a patient with a known family history of NF-1 is suggestive of NF-1: the presence of cortical tubers, and subependymal calcified nodules are classical indications of tuberous sclerosis complex (TSC). The patient did not have any history of viral prodrome or acute disease onset thus excluding potential ADEM diagnosis. Although gliomatosis cerebri is a disease of adults, it can present at an early age. However, T1 hyperintensity of the basal ganglia and subependymal nodules are not known findings of gliomatosis cerebri. The term hybrid (double) phakomatosis refers to the concomitance of two or more different phakomatoses in the same patient. Of all the phakomatoses, NF-1 and tuberous sclerosis (TS) most commonly occur together. This is an extremely rare condition, and only a few cases have been described in the literature. Inherited in an autosomal dominant fashion with distinct and well-delineated genetic, biochemical, and physical findings, NF-1 and TS are two neurocutaneous syndromes in

580

which affected individuals develop tumors with increased frequency. Rare cases of patients with NF-1 and TS stigmata have been reported in the literature. In our case, brain MRI showed multiple cortical tubers, radial bands, and subependymal nodules – findings frequently seen in TSC patients. Rounded spots with T2 high signal are usually seen in the cerebellum, brainstem, basal ganglia, thalamus, internal capsule, corpus callosum, and corona radiata of NF-1 patients. Typically, this spares the subcortical U fibers and the centrum semiovale. These lesions are usually multiple without any mass effect or enhancement with contrast. Histopathologically, these areas demonstrate vacuolation of myelin. Unlike the lesions elsewhere, lesions in the globus pallidus demonstrate T1 hyperintensity that develops after the T2 abnormality, due to reactive myelin formation.

Key Points  Hybrid phakomatosis is rare.  Neurofibromatosis type 1 and TSC may occur together.  They demonstrate typical imaging findings of individual phakomatosis.

Suggested Reading Barbier C, Chabernaud C, Barantin L, et al. Proton MR spectroscopic imaging of basal ganglia and thalamus in neurofibromatosis type 1: correlation with T2 hyperintensities. Neuroradiology 2011; 53(2): 141–8. Erbay SH, Oljeski SA, Bhadelia R. Rapid development of optic glioma in a patient with hybrid phakomatosis: neurofibromatosis type 1 and tuberous sclerosis. AJNR Am J Neuroradiol 2004; 25(1): 36–8. Gutmann DH. Parallels between tuberous sclerosis complex and neurofibromatosis 1: common threads in the same tapestry. Semin Pediatr Neurol 1998; 5(4): 276–86. Osborn AG. Osborn’s Brain: Imaging, Pathology, and Anatomy. Salt Lake City, Utah; London: Amirsys; Lippincott Williams & Wilkins Europe; 2012. Wheeler PG, Sadeghi-Nejad A. Simultaneous occurrence of neurofibromatosis type 1 and tuberous sclerosis in a young girl. Am J Med Genet A 2005; 133a(1): 78–81.

CASE

Part VII

122

Congenital Diseases Manifesting in Adults Prasad B. Hanagandi, Satya Patro, Santanu Chakraborty

Clinical Presentation A 65-year-old man presented with an insidious onset and gradually progressive 6–7-year history of intentional tremors and ataxia that failed to respond to anti-Parkinsonism medications. Neurocognitive symptoms were associated with initial mood disturbances that later progressed to dementia. He also had bowel and bladder dysfunction. Dysarthria and spasticity were noted on neurologic examination. There was no history of diabetes, hypertension, chronic alcoholism, or substance abuse. There was no family history of dementia or tremors. Routine hematologic studies, including assays for HIV and VDRL were negative. Serum ceruloplasmin and liver function tests were unremarkable. Cerebrospinal fluid analysis was negative for infectious and inflammatory processes. Genetic and molecular testing was positive for FMR1 gene mutation.

Imaging (B) (A)

Fig. 122.1 (A) Axial FLAIR and (B) Axial T2W images at the level of middle cerebellar peduncles.

Advanced Neuroradiology Cases, ed. Amaral et al. Published by Cambridge University Press. © Cambridge University Press 2016.

581

Part VII. Congenital Diseases Manifesting in Adults: Case 122

(A) (B)

Fig. 122.2 (A) Axial DWI and (B) ADC map at the level of middle cerebellar peduncles.

(B) (A)

Fig. 122.3 (A–B) Axial FLAIR images at the level of corona radiata, centrum semiovale, and lateral ventricles.

582

Part VII. Congenital Diseases Manifesting in Adults: Case 122 Fig. 122.4 Midsagittal T1W image.

583

Part VII. Congenital Diseases Manifesting in Adults: Case 122

Fragile X Tremor Ataxia Syndrome Primary Diagnosis Fragile X tremor ataxia syndrome

Differential Diagnoses Multiple system atrophy Dentatorubral-pallidoluysian atrophy Wilson disease Progressive multifocal leukoencephalopathy (PML) Post-ischemic changes

Imaging Features Fig. 122.1: (A) Axial FLAIR and (B) Axial T2-weighted images showed nearly symmetric hyperintense signal changes in the bilateral middle cerebellar peduncles. Generalized cerebellar volume loss was also noted. Fig. 122.2: (A) Axial DWI and (B) Corresponding ADC map did not show diffusion restriction. Fig. 122.3: (A–B) Axial FLAIR images at the level of the corona radiata, centrum semiovale, and lateral ventricles demonstrated generalized volume loss with deep white matter changes. Fig. 122.4: Midsagittal T1WI demonstrated thinning of the corpus callosum and cerebellar volume loss. The overall morphology and volume of the brainstem, especially the pons, is relatively preserved.

Discussion Progressive intentional tremors and gait ataxia (nonresponsive to anti-Parkinsonism medications), cognitive decline, autonomic disturbances, and nearly symmetric T2W FLAIR hyperintense signal changes in the middle cerebellar peduncles, with diffuse cerebral and cerebellar volume loss on neuroimaging, are the key clinicoradiologic features suggestive of fragile X tremor ataxia syndrome (FXTAS). The list of differentials for symmetric middle cerebellar peduncle hyperintense signal changes is extensive. Multiple system atrophy exhibits similar findings; however, FXTAS demonstrates less severe pontine volume loss and can be confirmed by genetic testing. Dentatorubral-pallidoluysian atrophy (DRPLA), a CAG trinucleotide repeat neurodegenerative disorder, manifests with dementia, seizures, and choreoathetosis. The age of DRPLA onset is variable and based on the

juvenile, early adult, or late-onset types. On imaging it has cerebral, cerebellar, and brainstem volume loss, with white matter changes. The central pons often shows signal changes on T2W and FLAIR sequences with sparing of the middle cerebellar peduncles. The age of onset is more than 50 years in FXTAS and, unlike DRPLA, is associated with CGG trinucleotide repeat sequences. Wilson disease can have similar clinical features; however, the age of presentation, lack of key imaging findings involving the basal ganglia and midbrain, and presence of normal ceruloplasmin levels are some of the salient findings that negate Wilson disease as a diagnosis in this case. Middle cerebellar peduncle hyperintensities can be noted in infectious demyelinating pathologies such as PML; however, negative HIV status and CSF analysis exclude these pathologies. Bilateral middle cerebellar peduncle infarcts are a rare but well-known entity and have similar T2W and FLAIR signal changes; however, the lack of diffusion restriction and duration of clinical presentation do not favor a vascular pathology. FXTAS is a late-onset X-linked dominant neurodegenerative disorder with variable penetrance. It occurs as a result of a mutation in the FMR1 gene (fragile X mental retardation) that affects the CGG trinucleotide repeat sequence and ultimately causes a deficiency of the FMR1 protein (FMRP). FXTAS, commonly seen in males between 50 and 80 years of age, leads to mental retardation and autism. Although it primarily affects males, females with premutation carriers can manifest with features of ovarian failure in 20% of cases. In the general population, the estimated carrier rate of FMR1 premutation is approximately 1/259 for females and 1/800 for males. Literature describes the overall penetrance, which increases with age, in males to be around 40% at 50 years of age. However, the degree of penetrance in females is estimated at approximately 10%. The clinical presentation includes intentional tremor, ataxia, neuropathy, and personality changes in association with memory loss. A combination of clinical presentation, genetic testing, and neuroimaging findings has been put forth as the criteria for diagnosis of FXTAS (see Tables 122.1, 122.2). Generalized cerebral, cerebellar, and brainstem volume loss are the pertinent MR imaging features of FXTAS. Although not pathognomonic, symmetric T2W/FLAIR hyperintense

Table 122.1. FXTAS diagnostic criteria

Criteria

Molecular

Clinical

Radiologic

Major

Intentional tremor Cerebellar gait ataxia

MRI white matter lesions involving middle cerebellar peduncles

Minor

Moderate to severe working memory deficit

MRI lesions involving cerebral white matter

Executive function deficit

Moderate to severe generalized brain atrophy

55 to 200 CGG repeats

584

Part VII. Congenital Diseases Manifesting in Adults: Case 122

Key Points

Table 122.2. FXTAS diagnostic criteria

Definite FXTAS Number/ Clinical 1 type of criteria Radiologic 1

Probable Possible FXTAS FXTAS 1major or 1 2 minor 1

1

Inclusions Ubiquitin-positive intranuclear inclusion

signal changes involving the middle cerebellar peduncles (MCP sign) have been described as a distinct neuroimaging finding and are seen in only 60% of cases. In addition, symmetric patchy and/or confluent hyperintensities can be seen in periventricular and deep white matter of cerebral hemispheres and corpus callosum with sparing of subcortical U fibers. Volumetric studies have also demonstrated changes in the hippocampi and thalami and the overall severity on imaging findings correlates with the CGG trinucleotide repeat length in premutation carriers. Neuroimaging literature describes a wide range of life expectancies ranging from 5 to 25 years with debilitating illness in the terminal stages. Treatment in FXTAS is supportive with physical and occupational therapy. Medications for controlling neuropsychiatric symptoms, tremors, neuropathic pain, and hypertension are implemented to improve the quality of life.

 FXTAS is a rare, X-linked trinucleotide repeat sequence neurodegenerative disease.  Intentional tremors, ataxia, and symmetric hyperintense signal on T2W and FLAIR sequences in the middle cerebellar peduncles are the pertinent features to suspect the possibility of FXTAS.  Definitive diagnosis is achieved by genetic testing. Assessment of other siblings and children is important to detect premutations and carriers.

Suggested Reading Berry-Kravis E, Abrams L, Coffey SM, et al. Fragile X-associated tremor/ataxia syndrome: clinical features, genetics, and testing guidelines. Mov Disord 2007; 22(14): 2018–30, quiz 2140. Brunberg JA, Jacquemont S, Hagerman RJ, et al. Fragile X premutation carriers: characteristic MR imaging findings of adult male patients with progressive cerebellar and cognitive dysfunction. AJNR Am J Neuroradiol 2002; 23(10): 1757–66. Capelli LP, Goncalves MR, Leite CC, et al. The fragile x-associated tremor and ataxia syndrome (FXTAS). Arq Neuropsiquiatr 2010; 68(5): 791–8. Eye PG, Hawley JS. Pearls & Oy-sters: fragile X tremor/ataxia syndrome: a diagnostic dilemma. Neurology 2015; 84(7): e43–5. Kamm C, Healy DG, Quinn NP, et al. The fragile X tremor ataxia syndrome in the differential diagnosis of multiple system atrophy: data from the EMSA Study Group. Brain 2005; 128(Pt 8): 1855–60.

585

CASE

Part VIII

123

Miscellaneous Bruno Siqueira Campos Lopes, Lázaro Luís Faria do Amaral

Clinical Presentation A 70-year-old man presented to our facility with a nine-month history of acute-onset, anterograde, and retrograde amnesic syndrome. He had no prior incidence of neurologic deficit or memory problems.

Imaging (B)

(A)

(C)

Fig. 123.1 (A) Axial FLAIR and (B–C) T2WI images through the level of the third ventricle.

(A)

(B)

Fig. 123.2 (A) Axial and (B) Coronal T2WI through the level of the mammillary bodies.

Advanced Neuroradiology Cases, ed. Amaral et al. Published by Cambridge University Press. © Cambridge University Press 2016.

587

Part VIII. Miscellaneous: Case 123

Anterior Thalamic Infarct with Transneuronal Degeneration of Mammillothalamic Tract (MTT)

Fig. 123.1: (A) Axial FLAIR and (B–C) T2WI showed an old ischemic infarct involving the left anterior thalamus. Fig. 123.2: (A) Axial and (B) Coronal T2W images more clearly demonstrated left mammillothalamic tract involvement and the left mammillary body atrophy, in comparison to the normal, contralateral right-sided structures.

is controversial. There are many reports of acute-onset Korsakoff syndromes and acute-onset amnesias developing after anterior thalamic infarction, suggesting that the key role of the anterior thalamus and the MTT points toward memory. However, the extent, significance, and prominence of the MTT’s exclusive role in memory and amnesic syndromes are contradictory. Some investigators strongly support the concept that the memory impairment following MTT infarctions could be attributed to extension of the lesions into the anterior thalamic nucleus. Acute onset of anomia, anterograde and retrograde amnesia after a MTT infarction, without thalamic involvement, has also been described – emphasizing the importance of the MTT’s role in memory disturbance symptomatology, and excluding the role of the MB and the thalamus. Verbal memory is more commonly affected than non-verbal memory following MTT injury.

Discussion

Key Point

Focal cystic encephalomalacia in the left anterior thalamic nucleus/mammillothalamic tract (MTT) is suggestive of chronic lacunar infarction. The small left mammillary body (MB) (Fig. 123.2) may be secondary to transneuronal degeneration of the MTT. Clinically, the most common cause of amnesic syndrome is Alzheimer disease (AD), which causes atrophy of the mesial temporal lobe, believed to begin at the entorhinal cortex and hippocampus. However, the acute onset of symptoms in this patient rules out AD as a potential diagnosis. In addition, the absence of significant hippocampal atrophy, a common finding in AD, excludes AD as a diagnosis in this patient. First described by the French physician Felix Vicq d’Azyr, the MTT connects the MB with the anterior thalamic nucleus. As part of the Papez circuit (circuit connecting the hypothalamus with the limbus system), disruption of the MTT could cause a memory disturbance. The presence of tumors or damage to the Papez circuit is a known indicator of amnesia and AD. However, the role of the MTT in the memory circuit

 Anterior thalamic or MTT infarction may produce acuteonset amnesias.

Primary Diagnosis Anterior thalamic infarct with transneuronal degeneration of mammillothalamic tract (MTT)

Differential Diagnoses Alzheimer disease Other causes of dementia

Imaging Findings

588

Suggested Reading Kwon HG, Hong JH, Jang SH. Mammillothalamic tract in human brain: diffusion tensor tractography study. Neurosci Lett 2010; 481(1): 51–3. Park KC, Yoon S-S, Chang D-I, et al. Amnesic syndrome in a mammillothalamic tract infarction. J Korean Med Sci 2007; 22: 1094–7. Parkin AJ, Rees JE, Hunkin NM, Rose PE. Impairment of memory following discrete thalamic infarction. Neuropsychologia 1994; 32(1): 39–51. Rousseaux M, Kassiotis P, Signoret JL, Cabaret M, Petit H. [Amnesic syndrome caused by limited infarction in the right anterior thalamus]. Rev Neurol (Paris) 1991; 147(12): 809–18. Yoneoka Y, Takeda N, Inoue A, et al. Acute Korsakoff syndrome following mammillothalamic tract infarction. AJNR Am J Neuroradiol 2004; 25(6): 964–8.

CASE

Part VIII

124

Miscellaneous Aparna Singhal, Asim K. Bag

Clinical Presentation A previously healthy 45-year old man presented with a diffuse, dull headache that worsened after sitting or standing that was frequently associated with nausea. He noted that the frequency and severity of his headache progressively worsened with time. Routine hematologic laboratory studies were normal.

Imaging Fig. 124.1 Sagittal T1WI MR at the level of the midbrain.

Fig. 124.2 Axial T2WI MR at the level of the midbrain.

Fig. 124.3 Axial T1WI MR at the level of the midbrain.

Advanced Neuroradiology Cases, ed. Amaral et al. Published by Cambridge University Press. © Cambridge University Press 2016.

589

Part VIII. Miscellaneous: Case 124

Intracranial Hypotension with Midbrain Swelling Primary Diagnosis Intracranial hypotension with midbrain swelling

Differential Diagnoses Midbrain tumor Abnormality of the diencephalic-mesencephalic junction

Imaging Findings Fig. 124.1: Sagittal T1WI through the midline demonstrated apparent swelling of the mesodiencephalic junction with complete obscuration of the pontomesencephalic junction, flattening of the anterior surface of the pons, and decreased mammillopontine distance. Transforaminal herniation of the cerebellar tonsils was also noted. Fig. 124.2: Axial T2WI through the diencephalon-mesocephalon junction demonstrated apparent swelling of the mesodiencephalic junction, without any obvious signal abnormality. Note that the third ventricle was compressed severely, with no identifiable CSF signal. Fig. 124.3: Axial T1WI, from a slightly lower level, demonstrated swollen appearance of the mesodiencephalic junction. Note that obscuration of the interpeduncular cistern prevents identification of the cerebral peduncles.

Discussion Postural headache is a typical manifestation of spontaneous intracranial hypotension (SIH). The imaging features suggestive of intracranial hypotension with midbrain swelling (IHMS) in this patient include severe brain sagging with transtentorial descent of the third ventricle and the diencephalon, swollen appearance of the diencephalon, flattening of the anterior pons, and a narrow pontomesencephalic angle. Although a tumor-like appearance of the mesencephalicdiencephalic junction is noted, no abnormal T2 signal, enhancement, or diffusion restriction suggestive of tumor was seen. Similar patient imaging features have been described in the literature as abnormalities of the diencephalic-mesencephalic junction or congenital malformations. However, doubts about the existence of such entities persist. Spontaneous intracranial hypotension is a headache disorder defined by presence of a postural headache associated with any of the following: photophobia, nausea, hyperacusis, tinnitus, neck stiffness, and the presence of low CSF pressure on MRI with or without imaging evidence of an active CSF leak that responds to blood patching. Spontaneous intracranial hypotension typically presents around 40–60 years of age as a severe orthostatic headache (worsens in an upright position and improves in a supine position). It is a relatively rare disorder with an estimated annual incidence of 5 cases per 100,000. There is a slight female-to-male predominance with a 2:1 ratio. The etiology of SIH is ascribed to a decrease in CSF pressure, which current evidence suggests is most commonly a result of a spontaneous spinal CSF leak. The cause of the spontaneous leaks is unclear, but heritable connective tissue

590

disorders that may cause dural weakness, such as Marfan syndrome and Ehlers-Danlos syndrome type II, have been implicated. Significant CSF leaks are usually located in the spine, most often at the thoracic level. Reduced CSF pressure may also be a result of trauma, CSF overshunting, exercise, violent coughing, lumbar puncture, severe dehydration, or spontaneous dural tear from a ruptured arachnoid diverticulum. In one-third of cases, a history of minor trauma precedes symptomatic onset. Rarely, spinal pathology, such as degenerative osteophytes, can lead to piercing of the dura. The pathophysiology is explained by the Monroe-Kellie hypothesis, which indicates that intracranial volume is constant, being a sum of brain parenchyma, CSF volume, and intravascular volume. Thus, CSF hypovolemia is compensated by increased intravascular volume and compliant vessel dilatation. Headache could result from stretching of pain-sensitive intracranial structures in a sagging brain, or from vascular congestion. Typical imaging features of SIH include pachymeningeal thickening and enhancement in both supra- and infratentorial compartments, due to dilated subdural blood vessels. Subdural hygromas due to compensatory fluid accumulation from spinal loss of CSF and subdural hematomas due to ruptured bridging veins are common. These may be seen in 17–60% of patients and are a late finding, usually not occurring without dural enhancement. Magnetic resonance imaging of the brain may be normal in up to 20% of patients. The optic chiasm and hypothalamus drape over the sella as the suprasellar cistern gets crowded or effaced. Pituitary enlargement and congestion may be present. Brain descent is seen in 40–50% of cases, with caudal displacement of tonsils in 25–75%. Angiography may demonstrate diffuse enlargement of cortical and medullary veins. Radionuclide cisternography may detect focal accumulation of radiotracer at the leakage site and poor migration of tracer over convexities, with early detection in the bladder and kidneys. Lumbar puncture, performed if neuroimaging is inconclusive, typically has an opening CSF pressure below 6 cm H2O. Cerebrospinal fluid analysis may show reactive mildly elevated protein levels, and pleocytosis (up to 50 cells/ mm3). Computed tomography or MR myelography with contrast may identify the site of the leak, which may be especially useful when planning intervention. Intracranial hypotension with midbrain swelling is a special variant of SIH in which there is severe brain sagging with transtentorial descent of the third ventricle and the diencephalon, and swollen appearance of the diencephalon: flattening of the anterior pons narrows the pontomesencephalic angle. Unlike other cases of SIH, there is also narrowing of the angle between the vein of Galen and straight sinus. In IHMS, dural congestion, enhancement, and subdural fluid accumulation are absent. The third ventricle is usually extremely thin (slit-like) and is located at the level of the midbrain, compared to its normal position. This swelling is likely due to mild vasogenic edema secondary to functional stenosis of the vein of Galen–straight sinus confluence.

Part VIII. Miscellaneous: Case 124

Management of SIH is empirical and no treatment options have been evaluated by randomized controlled trials. Spontaneous remission is common. Conservative management includes initial strict bed rest, analgesics, and adequate oral hydration with caffeine intake. Theophylline and steroids may be considered. If conservative management fails, an epidural blood patch is recommended which involves injecting a volume of 10–20 ml of autologous blood into the epidural space, thought to plug the dural tear, forming a scar in two to three weeks.

the diencephalon, flattening of the anterior pons, and a narrow pontomesencephalic angle.

Suggested Reading Barkovich AJ, Millen KJ, Dobyns WB. A developmental and genetic classification for midbrain-hindbrain malformations. Brain 2009; 132(Pt 12): 3199–230. Hoffmann J, Goadsby PJ. Update on intracranial hypertension and hypotension. Curr Opin Neurol 2013; 26(3): 240–7.

Key Point

Lahoria R, Allport L, Glenn D, et al. Spontaneous low-pressure headache - a review and illustrative patient. J Clin Neurosci 2012; 19(8): 1076–9.

 IHMS is a special variant of SIH in which there is severe brain sagging with transtentorial descent of the third ventricle and the diencephalon, swollen appearance of

Savoiardo M, Minati L, Farina L, et al. Spontaneous intracranial hypotension with deep brain swelling. Brain 2007; 130(Pt 7): 1884–93.

591

CASE

Part VIII

125

Miscellaneous Aparna Singhal, Asim K. Bag

Clinical Presentation A previously healthy 35-year-old woman presented to a neuroophthalmology clinic at our facility with photopsia and monocular transient visual obscuration. On questioning, she revealed that she had been suffering from moderate holocranial headache for the last several months that worsened after awakening and with Valsalva maneuver. She also revealed that she has gained approximately 25 pounds in the last few months. Other than oral contraceptives, she was not on any medication. Her current body mass index was 27.56. Hematologic studies, including serum chemistries, and complete and differential blood counts were normal. There was no evidence of anti-autoantibodies in the serum. A lumbar puncture was planned. However, MRI of the brain was performed before proceeding with the lumbar puncture.

Imaging

Fig. 125.1 Axial T2WI through the orbit.

Fig. 125.2 Axial fat-suppressed T1WI through the orbit.

Advanced Neuroradiology Cases, ed. Amaral et al. Published by Cambridge University Press. © Cambridge University Press 2016.

593

Part VIII. Miscellaneous: Case 125 Fig. 125.3 Coronal STIR image through the orbits.

Fig. 125.4 Midline sagittal T1WI.

594

Part VIII. Miscellaneous: Case 125 Fig. 125.5 Maximum intensity projection (MIP) image of a phase contrast MR venogram.

595

Part VIII. Miscellaneous: Case 125

Idiopathic Intracranial Hypertension Primary Diagnosis Idiopathic intracranial hypertension

Differential Diagnoses Secondary intracranial hypertension Treatment with vitamin A, tetracycline, or lithium Steroid withdrawal Systemic lupus erythematosus

Imaging Findings Fig. 125.1: Axial T2WI through the orbit demonstrated flattening of the posterior sclera (arrow). Fig. 125.2: Axial fat-suppressed T1WI through the orbit demonstrated flattening of the posterior sclera and inversion of the left optic nerve head (arrow). Fig. 125.3: Coronal STIR image through the orbits, posterior to the globe, demonstrated optic nerve sheath dilation with prominent CSF around the optic nerves and hydrops (arrows). Fig. 125.4: Midline sagittal T1WI demonstrated empty sella (arrow). Fig. 125.5: Maximum intensity projection (MIP) image of the phase contrast MR venogram demonstrates gradual tapering of the left transverse sinus (arrows).

Discussion The presence of a long-standing headache, with visual symptoms, in an obese female patient of childbearing age is highly suspicious of idiopathic intracranial hypertension (IIH). A constellation of imaging findings including a flattening of the posterior sclera, inversion of the optic nerve head, dilation of the intraorbital optic nerve sheath, and an empty sella are diagnostic of IIH. These findings are also frequently associated with gradual tapering of one of the transverse sinuses (the left, in our patient). The lack of intracranial mass, or other causes of secondary increased intracranial pressure (ICP) seen on imaging, rules out secondary IIH. The absence of prior drug treatment rules out the possibility of drug-related causes for the increased ICP. Also known as pseudotumor cerebri, IIH is a rare condition (reported annual incidence of 1–2 per 100,000) characterized by presence of an increased ICP in the absence of underlying brain pathology. It is a disease of both pediatric and adult populations, but a female predominance of 9:1 is seen only in adults. Familial occurrences have been reported without clear genetic relationships. Patients present with headache, papilledema, and normal CSF composition, but with elevated CSF pressure. No focal neurologic deficits are seen. Several inconclusively proven causal mechanisms related to altered CSF dynamics have been proposed. The important etiologies include obesity, delayed CSF absorption, and venous outflow abnormality with increased cerebral venous pressure. The proposed pathophysiologic rationales for association with obesity include increased intrathoracic and abdominal

596

pressure, increased central venous pressure and increased ICP, or a hormonal mechanism. There is some convincing evidence of increased resistance to CSF flow, but it is unclear if it is due to decreased CSF absorption or decreased intracranial compliance. Stenotic transverse sinuses have been observed in a large number of IIH patients. Venous stenosis is likely a result of raised ICP. Alternatively, venous sinus stenosis may be primary and increases venous pressure with resultant intracranial hypertension. Typically, an IIH patient is an obese female of childbearing age who presents with headache, diplopia, visual loss, and papilledema without focal deficits, except VI nerve palsy. The headache may be throbbing or pressure-like, unilateral or holocephalic, and exaggerated by activities increasing ICP, e.g., Valsalva, coughing, or bending over. Long-standing papilledema may produce optic atrophy wherein the disk is viewed as pale, gliotic, and flat. Visual field deficits occur in up to 90% with blind spot enlargement and even transient visual loss. Risk of blindness is 4–10%. There may be occasional pituitary dysfunction. Unilateral or bilateral tinnitus has been reported, which could be due to increased CSF pulsations and venous sinus turbulence. Imaging may be completely normal with elevated ICP. Typical imaging findings of IIH include flattening of the posterior sclera, intraocular protrusion of the optic nerve head (reversal of the optic nerve head), distension of the optic nerve sheath (more than 2 mm), vertical tortuosity of the orbital optic nerve, enhancement of the optic nerve head, empty sella turcica, and deformity of the pituitary gland (attributed to chronic pituitary gland compression, and bone remodeling by CSF pulsations). Less commonly, slit-like ventricles are also seen. These changes are reversible on reduction of the ICP. Posterior globe flattening, optic nerve protrusion, and slit-like ventricles have been shown to have high specificity, but none of these signs is highly sensitive. Other findings of IIH include altered flow velocity in the venous sinuses and narrowing of the sinuses. It is not established if the stenosis is the cause of IIH or the effect. If a pressure gradient is present across the dural venous sinus stenosis, stenting of the sinus may be offered as treatment with varying results. The diagnosis is confirmed in presence of elevated ICP of > 25 cm H2O with lumbar puncture in lateral decubitus position without sedation. Normal adult ICP is defined as 7.5–20 cm H2O, with values of 20–30 cm H2O representing mild intracranial hypertension. Intracranial pressure values greater than 20–25 cm H2O mostly require treatment and sustained ICP values of greater than 40 cm H2O represent severe, life-threatening intracranial hypertension. Management depends on the signs and symptoms, and treatment strategies vary. Conservative management includes weight loss, acetazolamide, and diuretics such as furosemide. Lumbar puncture and CSF drainage may provide symptom relief lasting several weeks. Surgical intervention with CSF shunting is indicated in patients with visual loss or impending

Part VIII. Miscellaneous: Case 125

visual loss in patients not responding to medication therapy. Optic nerve sheath fenestration is used to relieve papilledema transiently.

Key Points  Long-standing headache with visual symptoms in an obese female patient of childbearing age with or without history of oral contraceptive pills is highly suggestive of IIH.  Typical imaging findings include: flattening of the posterior sclera, intraocular protrusion of the optic nerve, distension of the optic nerve sheath, vertical tortuosity of the orbital optic nerve, enhancement of the optic nerve head, and empty sella turcica.

Suggested Reading Biousse V, Bruce BB, Newman NJ. Update on the pathophysiology and management of idiopathic intracranial hypertension. J Neurol Neurosurg Psychiatry 2012; 83(5): 488–94. Graff-Radford SB, Schievink WI. High-pressure headaches, lowpressure syndromes, and CSF leaks: diagnosis and management. Headache 2014; 54(2): 394–401. Hoffmann J, Goadsby PJ. Update on intracranial hypertension and hypotension. Curr Opin Neurol 2013; 26(3): 240–7. Yuh EL, Dillon WP. Intracranial hypotension and intracranial hypertension. Neuroimaging Clin N Am 2010; 20(4): 597–617.

597

CASE

Part VIII

126

Miscellaneous Asim K. Bag

Clinical Presentation A 52-year-old man presented to the neurology clinic with a several-week history of problematic short-term memory loss. He denied loss of consciousness, seizures, psychosis, acute cerebellar ataxia, and focal neurologic signs (i.e., cranial nerve deficits or hemiparesis). His past medical history was significant for sarcoidosis of the myocardium that necessitated cardiac transplantation one year prior to onset of current presentation. Post-transplant he was maintained on a tacrolimus (TL)-based regime. From a transplant standpoint, he was doing well. Magnetic resonance imaging was ordered to evaluate his memory problem (images shown below). Following the MRI, lumbar puncture was performed. Cerebrospinal fluid studies were negative; specifically, there was no evidence of viral infection including HHV-6. Hematologic studies were also negative.

Imaging (A)

(B)

Fig. 126.1 (A) Axial FLAIR image and (B) T2WI through the third ventricle.

Advanced Neuroradiology Cases, ed. Amaral et al. Published by Cambridge University Press. © Cambridge University Press 2016.

599

Part VIII. Miscellaneous: Case 126 Fig. 126.2 Postcontrast T1WI through the same level.

600

Part VIII. Miscellaneous: Case 126 Fig. 126.3 Axial DWI through the third ventricle.

601

Part VIII. Miscellaneous: Case 126

Calcineurin Inhibitor-Mediated Limbic Injury Primary Diagnosis Calcineurin inhibitor-mediated limbic injury

Differential Diagnoses Herpes virus type 6 (HHV-6) encephalopathy Hippocampal sclerosis Posterior reversible encephalopathy syndrome (PRES) Paraneoplastic limbic encephalitis

Imaging Findings Fig. 126.1: (A) Axial FLAIR image and (B) T2WI through the third ventricle demonstrated bilateral, almost symmetric, hyperintensity involving the posterior aspects of bilateral mesial temporal lobes (posterior hippocampus and dentate gyrus), predominantly on the left side. Fig. 126.2: Postcontrast T1WI through the same level did not demonstrate any enhancement. Fig. 126.3: Axial DWI through the same level demonstrated subtle increased signal in the same area that is not associated with any low ADC value (not shown).

Discussion After cardiac transplantation, this patient was treated with TL-based immunosuppression to prevent rejection. Following treatment with TL, he developed relatively rapid-onset shortterm memory disturbances. As neurotoxicity is common with TL therapy, evaluation with MRI is warranted. Unlike posterior predominant cortical/subcortical FLAIR abnormalities as seen in PRES, this patient demonstrated bilateral symmetry, dentate gyral FLAIR signal abnormality, and T2 shine-through effects without enhancement, and no FLAIR abnormality anywhere else (not shown). Tacrolimus-induced neurotoxicity was suspected and the drug was temporarily withheld. The patient’s symptoms improved and a repeat MRI showed reversal of abnormal hippocampal signal intensity on FLAIR and on DWI. In patients with immunosuppression following organ transplantation, HHV-6 encephalopathy tends to develop with a predilection to the mesial temporal lobe. However, reversal of MRI abnormality with withdrawal of TL and absence of HHV-6 in blood and CSF rules out HHV-6 encephalopathy. Reversibility with drug withdrawal also rules out hippocampal sclerosis. The absence of a known tumor rules out the possibility of paraneoplastic limbic encephalitis. PRES (see Part V: Case 62 for detailed imaging findings of PRES), a well-known CNS complication of TL, typically manifests abnormal FLAIR signal at the cortical/subcortical white matter of the posterior parietal and occipital lobes. In severe cases, the abnormality can extend to involve the frontal and even lateral temporal lobes with or without associated diffusion restriction, hemorrhage, and enhancement. In atypical PRES, there may be isolated basal ganglia and/or brainstem involvement. However, isolated mesial temporal lobe involvement is not a known manifestation of PRES.

602

Tacrolimus and cyclosporine A are the two commonly used immunosuppressive drugs after solid organ transplantation, as well as after bone marrow transplantation. Both of these drugs act by inhibiting calcineurin, a calcium/calmodulin-dependent phosphatase that induces immunosuppression by reducing transcriptional activation of many inflammatory cytokines including tumor necrosis factor-alpha, interleukin-2, and gamma-interferon. Neurotoxicity is one of the major side effects of calcineurin inhibitors (CNIs) including both TL and cyclosporine A, and can be seen in up to 32% of patients. Common neurologic side effects include stroke-like symptoms, visual disturbances, hallucinations, and seizures. Most of these complications are related to some form of PRES and due to endothelial injury and/or vasospasm. Akinetic mutism and memory loss are relatively less common but well-known neurologic side effects. Selective limbic injury is likely the cause of these conditions and may be due to the selective injury of the dentate gyrus and hippocampus stemming from the abundance of calcineurin in these regions. Selective hippocampal injury is more common with TL than cyclosporine A because TL is approximately 20 times more potent than cyclosporine A. Patients with hippocampal injury demonstrate abnormal T2 signal involving bilateral hippocampi. Bilateral symmetric imaging findings are a useful key to distinguish PRES from CNI-mediated limbic injury. In limbic injury, the imaging abnormality can be more prominently seen on DWI, which could be due to T2 shine-through effect or actual diffusion restriction. Limbic injury is not associated with any abnormal enhancement. Use of offending CNIs should be discontinued to prevent further injury. In conjunction with withdrawal of the offending drug, the signal abnormality improves, as do the patient’s symptoms.

Key Points  The presence of bilateral and symmetric, abnormal medial temporal lobe T2 hyperintensity in a patient presenting with relatively rapid-onset short-term memory problems and a history of prior treatment with CNIs can be due to HHV-6 infection or due to CNI-induced bilateral limbic injury.  Careful workup should be performed to exclude HHV-6 infection, as HHV-6 encephalopathy has a very poor prognosis.

Suggested Reading Lee SH, Kim BC, Yang DH, et al. Calcineurin inhibitor-mediated bilateral hippocampal injury after bone marrow transplantation. J Neurol 2008; 255(6): 929–31. Yoshida Y, Shimada H, Mori T, et al. FK506-associated limbic injury following umbilical cord blood transplantation. Bone Marrow Transplant 2003; 32(5): 523–5.

CASE

Part VIII

127

Miscellaneous Victor Hugo Rocha Marussi, Lázaro Luís Faria do Amaral

Clinical Presentation A 42-year-old woman presented to our facility with persistent headache and dysphasia. She had a history of a large craniectomy for excision of a meningioma in the recent past.

Imaging (A)

(B)

(C)

Fig. 127.1 (A–C) Axial CT (non-enhanced) images through the level of the craniectomy.

Advanced Neuroradiology Cases, ed. Amaral et al. Published by Cambridge University Press. © Cambridge University Press 2016.

603

Part VIII. Miscellaneous: Case 127

(A)

(B)

Fig. 127.2 (A–B) Coronal CT (non-enhanced) images through the level of the craniectomy.

(B) (A)

Fig. 127.3 (A–B) Coronal 3D reconstruction bone and soft tissue images through the cranial vault.

604

Part VIII. Miscellaneous: Case 127

(A)

(B)

p

(C)

Fig. 127.4 (A–C) Axial FLAIR through the craniectomy and the large ischemic area.

605

Part VIII. Miscellaneous: Case 127

Sinking Skin Flap Syndrome (Trephine Syndrome) Primary Diagnosis Sinking skin flap syndrome (trephine syndrome)

Differential Diagnosis Paradoxical herniation

Imaging Findings Fig. 127.1: (A–C) Axial CT images showed sinking skin flap on the left side of the cranium, characterized by the depressed meningocele complex at the craniectomy site. Fig. 127.2: (A–B) Coronal CT images confirmed the sinking skin flap on the left side of the cranium and showed concave deformity of the underlying brain parenchyma without associated signs of subfalcine or transtentorial herniations. Fig. 127.3: (A–B) 3D soft tissue and bone CT reconstruction images showed the sunken appearance of the skin flap that can be observed during physical examination. Fig. 127.4: (A–C) Axial FLAIR showed extensive cortico-subcortical hyperintensities compromising the ipsilateral temporal, frontal, parietal, and insular lobes, as well as the same side basal ganglia, corresponding to areas of gliosis/encephalomalacia, with lateral ventricle dilatation associated.

Discussion In general, there is no differential diagnosis for this syndrome. If you already know this entity, the diagnosis is promptly made. With regard to imaging findings in the postoperative cranium setting, especially after craniectomy, there is one main differential diagnosis to be considered – the paradoxical herniation. An unusual complication of a decompressive craniectomy and a neurosurgical emergency, it occurs in patients with large craniectomy defects who then undergo some CSF drainage (not present in our case). This constellation of postsurgical events leads to a pressure imbalance between the intracranial and extracranial compartments, causing a subfalcine and/or transtentorial herniation away from the craniectomy defect and mesodiencephalic dysfunction. Patients could present with depressed level of consciousness, autonomic instability, signs of brainstem release, and focal neurologic deficits. Sinking skin flap (SSF) syndrome, also known as motor trephine syndrome, is an uncommon late postoperative complication of craniectomy with an estimated prevalence of 13%. It has a tendency to occur between 28 and 188 days after surgery, usually one month postoperatively.

606

The most accepted underlying pathophysiologic mechanism for SSF syndrome is that the exposure of the intracranial contents to external atmospheric pressure alters CSF hydrodynamics, deforms the brain, and reduces cerebral perfusion. Patients present with new-onset symptoms including headaches, seizures, dizziness, easy fatigability, mood changes, and eventually, dysesthesias and paresis. On physical examination, often a sunken appearance of the skin flap is noted, hence the name sinking skin flap syndrome. On CT/MR imaging, a depressed meningocele complex at the craniectomy site and concave deformity of the underlying brain parenchyma is seen. Frequently, there is paradoxical herniation of the brain away from the craniectomy site. Cranioplasty remains the treatment of choice for most patients with SSF syndrome, since it is well documented that some patients show clinical improvement after this procedure. It is believed that postcranioplasty improvement is a consequence of improved postsurgical CBF.

Key Points  Concave deformity of the underlying brain parenchyma without associated signs of subfalcine or transtentorial herniations after a large craniectomy is diagnostic of SSF syndrome.  Recognition of this entity is important, as cranioplasty can reverse the clinical symptomatology, improving patient outcome.

Suggested Reading del Mar Carmona Abellan M, Murie Fernandez M, Esteve Belloch P. Cranioplasty sinking should affect normal brain function mimicking other neurologic illness. AJNR Am J Neuroradiol 2012; 33(4): E65. Kemmling A, Duning T, Lemcke L, et al. Case report of MR perfusion imaging in sinking skin flap syndrome: growing evidence for hemodynamic impairment. BMC Neurol 2010; 10: 80. Sarov M, Guichard JP, Chibarro S, et al. Sinking skin flap syndrome and paradoxical herniation after hemicraniectomy for malignant hemispheric infarction. Stroke 2010; 41(3): 560–2. Sinclair AG, Scoffings DJ. Imaging of the post-operative cranium. Radiographics 2010; 30(2): 461–82. Yang XF, Wen L, Shen F, et al. Surgical complications secondary to decompressive craniectomy in patients with a head injury: a series of 108 consecutive cases. Acta Neurochir (Wien) 2008; 150(12): 1241–7; discussion 1248.

CASE

Part VIII

128

Miscellaneous Prasad B. Hanagandi, Lázaro Luís Faria do Amaral

Clinical Presentation A 24-year-old man presented to the emergency department with a post-traumatic headache following a motor vehicle accident. Neurologic examination was unremarkable with no signs of increased intracranial pressure. Hematologic studies were normal.

Imaging (A)

(B)

Fig. 128.1 (A–B) Axial contrast CT images at the level of frontoparietal convexities and dorsal superior sagittal sinus.

Advanced Neuroradiology Cases, ed. Amaral et al. Published by Cambridge University Press. © Cambridge University Press 2016.

607

Part VIII. Miscellaneous: Case 128

(A)

(B)

608

Fig. 128.2 (A–B) T2WI and 3D-CISS images in midsagittal plane through the sagittal sinus.

Part VIII. Miscellaneous: Case 128

Fig. 128.3 Axial source image of CE-MRV through the dorsal superior sagittal sinus.

Fig. 128.4 CE-MRV MIP image.

609

Part VIII. Miscellaneous: Case 128

Giant Arachnoid Granulation Primary Diagnosis Giant arachnoid granulation

Differential Diagnoses Epidermoid cyst Arachnoid cyst Dermoid cyst Calvarial hemangioma Meningioma Calvarial neoplastic lytic lesions Venous sinus thrombosis

Imaging Findings Fig. 128.1: (A) Contrast CT showed a hypodense filling defect (arrow) in the dorsal superior sagittal sinus with (B) scalloping of the inner table. Fig. 128.2: (A) T2 sagittal and (B) Heavily T2WI high-resolution images depict a linear flow void (arrow) which represents a vein coursing through the arachnoid granulation. Fig. 128.3: Axial source image of CE-MRV showed the enhancing veins traversing through the flow void. Fig. 128.4: CE-MRV MIP image showed the focal filling defect in the dorsal superior sagittal sinus.

Discussion Although few entities can mimic arachnoid granulations, several lesion types should be considered in the list of differential diagnoses for arachnoid granulation. Intraosseous epidermoid cysts have similar signal intensity to CSF and can closely resemble arachnoid granulation. However, the absence of diffusion restriction and lack of complete suppression on FLAIR images confirm the diagnosis of arachnoid granulation. Arachnoid cysts also follow CSF signal intensity on all pulse sequences but heterogeneous enhancement as noted in arachnoid granulation aids in exclusion. The characteristic fat signal often noted in dermoid cysts also aids in ruling out differential diagnoses. Calvarial hemangioma, meningioma, and most neoplastic lesions show intense, solid patterns of enhancement, which are an uncommon trait of arachnoid granulations. Finally, venous sinus thrombosis can closely simulate arachnoid granulations and demonstrate varied density on CT and MRI images based on the stage of blood product degradation. Venous sinus thrombi are often described as having a sausage shape and can contiguously extend into the adjacent sinus. On the contrary, arachnoid granulations are round to oval and do not cross over into the adjacent sinuses. Sharp delineation and involvement of the inner table of the calvarium and a linear traversing vein characterize arachnoid granulations rather than a thrombus. Arachnoid granulations, also termed pacchionian depressions, are expanded arachnoid villi that project into the dural venous sinus lumens, through which CSF is filtered into the central venous circulation. These are incidental findings, are

610

found in two-thirds of the general population, are most often seen in patients greater than 40 years of age, and increase in frequency with age. They are most commonly found in the transverse and posterior superior sagittal sinuses, but they are also frequently seen in the sigmoid, sagittal, and straight sinuses. They often are located in close association to the dural venous sinus, along penetrating veins, and are postulated to be weak areas within the dura through which these villi can project into the sinus lumen. The arachnoid granulation meningothelial lining is thin at the base and absent at its apex. The internal network is supported by collagenous soft tissue filled with CSF flowing from the subarachnoid space. Cerebrospinal fluid migrates to the granulation apex, which is lined by arachnoid cells, and is thought to be actively transported into the venous circulation. Larger granulations are more likely to contain fibo-collagenous tissue and internal veins. Normally, arachnoid granulations range from 5 to 15 mm. There is no common consensus in the literature regarding the size criteria; however, arachnoid granulations greater than 10–15 mm are termed giant arachnoid granulations. Some authors refer to arachnoid granulations as giant when they are large enough to occupy the entire lumen of the sinus and result in focal dilatation. Giant granulations as large as 2.4 cm have been reported in the literature. They may grow to occupy and dilate the dural sinuses or expand the outer table, mimicking osteolytic lesions. They are typically asymptomatic, with rare exceptions, when they can present with headache and features of pseudotumor cerebri if there is a pressure gradient across the giant arachnoid granulation resulting in venous hypertension. Rarely, they can cause partial sinus occlusion. On imaging, arachnoid granulations are single or multiple ovoid lesions and typically appear as focal osseous pits in the inner table of the calvarium. Computed tomography images show a well-circumscribed, discrete filling defect in the dural venous sinus with CSF density. Diagnosis is often established on MRI, where conventionally arachnoid granulations follow CSF/clear fluid signal intensity on T1 and T2WI and typically do not enhance. Often, linear enhancement is noted in large granulations, which is usually due to a traversing vein. Few exceptions to this rule have been demonstrated in the literature where on FLAIR, granulations can be hyperintense, possibly due to pulsation artifact from the adjacent sinus or altered CSF flow in approximately 9–10% of cases. Diffusion restriction changes can be seen in large arachnoid granulations and are presumed to be due to stromal collagenous tissue.

Key Points  Giant arachnoid granulations are usually larger than 10 mm in size and can occupy the entire circumference of the dural venous sinuses.  Unlike the smaller arachnoid granulations, they can be hyperintense on FLAIR sequence and might show

Part VIII. Miscellaneous: Case 128

heterogeneous enhancement with linear draining veins traversing the arachnoid granulation.  Giant arachnoid granulations can mimic several entities, and one has to distinguish arachnoid granulations from other lesions and pathologies.

Suggested Reading Arjona A, Delgado F, Fernandez-Romero E. Intracranial hypertension secondary to giant arachnoid granulations. J Neurol Neurosurg Psychiatry 2003; 74: 418. Chin SC, Chen CY, Lee CC, et al. Giant arachnoid granulation mimicking dural sinus thrombosis in a boy with headache: MRI. Neuroradiology 1998; 40: 181–3.

Ikushima I, Korogi Y, Makita O, et-al. MRI of arachnoid granulations within the dural sinuses using a FLAIR pulse sequence. Br J Radiol 1999; 72(863): 1046–51. Kan P, Stevens EA, Couldwell WT. Incidental giant arachnoid granulations. AJNR Am J Neuroradiol 2006; 27: 1491–2. Mamourian AC, Towfighi J. MR of giant arachnoid granulation: a normal variant presenting as a mass within the dural venous sinus. AJNR Am J Neuroradiol 1995; 16: 901–4. Rodallec MH, Krainik A, Feydy A, et al. Cerebral venous thrombosis and multidetector CT angiography: tips and tricks. Radiographics 2006; 26(Suppl 1): S5–18. Trimble CR, Harnsberger HR, Castillo M, et al. “Giant” arachnoid granulations just like CSF? NOT!! AJNR Am J Neuroradiol 2010; 31: 1724–8.

611

CASE

Part VIII

129

Miscellaneous Asim K. Bag

Clinical Presentation A 17-year-old young man presented to our emergency department with a closed head injury. He had complex maxillofacial injuries involving multiple paranasal sinuses. No evidence of intracranial injury was noted on CT scans. Hematologic

studies were normal. He had no evidence of renal failure or any endocrinopathy. His parents revealed that their son’s early medical history included an unknown blood cancer that required chemotherapy and radiation therapy when he was three years of age. There is no family history of CNS disease.

Imaging Fig. 129.1 Axial noncontrast CT scan of head through the level of centrum semiovale.

Fig. 129.2 Axial noncontrast CT scan of head through the level of basal ganglia.

Fig. 129.3 Axial noncontrast CT scan of head through the posterior fossa.

Advanced Neuroradiology Cases, ed. Amaral et al. Published by Cambridge University Press. © Cambridge University Press 2016.

613

Part VIII. Miscellaneous: Case 129

Mineralizing Microangiopathy Primary Diagnosis Mineralizing microangiopathy

Differential Diagnoses Familial idiopathic basal ganglia calcification (FIBGC) Abnormal calcium metabolism TORCH infection

Imaging Findings Fig. 129.1: Axial non-contrast CT scan of head through the level of centrum semiovale demonstrated bilateral, chunky calcification at the gray-white junction. Fig. 129.2: Axial noncontrast CT scan of head through the level of basal ganglia demonstrated similar chunky calcification at the gray-white junction, predominantly in the posterior aspect of the right temporal lobe. Faint calcification is also noted in bilateral basal ganglia. Note tiny right frontal pneumocephalus from the maxillofacial trauma. Fig. 129.3: Axial non-contrast CT scan of head through the posterior fossa demonstrated no identifiable calcification in the dentate nuclei.

Discussion Calcification at the gray-white junction, with relative sparing of the basal ganglia, in a patient with a known history of prior brain radiation and chemotherapy is highly suggestive of mineralizing microangiopathy. FIBGC, formerly known as Fahr disease, is a congenital disease characterized by abnormal calcium deposition, predominantly in the basal ganglia and dentate nucleus, as well as in other parts of the brain. Unlike this patient, in patients with FIBGC, the calcium deposition always predominantly occurs in the basal ganglia. No known endocrinopathy rules out the possibility of abnormal calcium

614

metabolism, such as hypoparathyroidism, hyperparathyroidism, or renal failure. The patient did not have a history of TORCH infection. Mineralizing microangiopathy is a distinctive histopathologic process in which extensive microvascular injury occurs because of combined chemotherapy and brain radiation, usually delivered early in childhood. It is pathologically characterized by hyalinization of the arterial wall with exuberant calcium deposition. Usually, this abnormal calcification is not symptomatic. Mineralizing microangiopathy is a radiologic diagnosis. Typical imaging findings include calcification at the gray-white junction. The calcification may be arc-like with the open end facing towards the gray matter. Although there can be calcification in the basal ganglia and in the dentate nucleus, these are not the dominant sites of calcification. This geographic distribution of calcification differentiates this entity from other causes of brain calcification. On MRI, the foci of calcification are hyperintense on T1WI sequence because of surface relaxation effect of the calcium. They are typically hypointense on T2WI sequence.

Key Point  Diagnosis of mineralizing microangiopathy should be considered in a relatively young patient with calcification at the gray-white junction with prior history of cranial radiation and chemotherapy.

Suggested Reading Lewis E, Lee YY. Computed tomography findings of severe mineralizing microangiopathy in the brain. J Comput Tomogr 1986; 10(4): 357–64. Shanley DJ. Mineralizing microangiopathy: CT and MRI. Neuroradiology 1995; 37(4): 331–3.

CASE

Part VIII

130

Miscellaneous Afonso C. P. Liberato, Lázaro Luís Faria do Amaral

Clinical Presentation A 28-year-old man presented with a headache. He did not have any additional neurologic signs or clinical symptoms. Multiple diagnostic imaging studies were performed.

Imaging (A) (B)

Fig. 130.1 (A) Axial and (B) Sagittal unenhanced CT scans through the level of the lateral ventricles.

Advanced Neuroradiology Cases, ed. Amaral et al. Published by Cambridge University Press. © Cambridge University Press 2016.

615

Part VIII. Miscellaneous: Case 130

(B)

(A)

Fig. 130.2 (A) Sagittal volumetric FLAIR and (B) Contrast-enhanced T1WI through the level of the lateral ventricles.

(A) (B)

Fig. 130.3 (A) Axial and (B) Sagittal 3D-volumetric, FLAIR reconstructions through the level of the lateral ventricles.

616

Part VIII. Miscellaneous: Case 130

(B)

(A)

Fig. 130.4 (A) Sagittal and (B) Axial 3D-CISS-FIESTA MR images through the level of the lateral ventricles.

(A)

(B)

Fig. 130.5 (A–B) Intraventricular, virtual endoscopic-3D reconstructed MR images through the level of the left lateral ventricle.

617

Part VIII. Miscellaneous: Case 130

Ring-Shaped Lateral Ventricular Nodules Primary Diagnosis Ring-shaped lateral ventricular nodules

Differential Diagnoses Subependymal heterotopia Subependymal nodules in tuberous sclerosis complex Subependymoma

Imaging Findings Fig. 130.1: (A) Axial and (B) Sagittal unenhanced CT images through the level of the lateral ventricles showed small, nodular lesions in the left lateral ventricle, isodense relative to gray matter. One of the lesions demonstrated a central portion that was isodense to CSF. Fig. 130.2: (A) Sagittal volumetric FLAIR MR image through the level of the lateral ventricles showed slight hypersignal, relative to the parenchyma of the lesions’ ring portion. An isointense signal of the lesions’ core portion, relative to CSF, was noted. (B) T1W contrast-enhanced image did not show contrast enhancement of the lesions. Fig. 130.3: (A) Axial and (B) Sagittal 3D-volumetric FLAIR reconstructions better demonstrated the hypersignal of the lesions’ ring portion and the isointense signal of their core portion, relative to CSF. Fig. 130.4: (A) Sagittal and (B) Axial 3D-CISS-FIESTA MR images showed ring-shaped, nodular lesions in both lateral ventricles and their core portion was isointense relative to CSF. Fig. 130.5: (A) and (B) Intraventricular, virtual endoscopic-3D reconstructed MR images through the level of the left lateral ventricle showed the nodular lesions and their round, oval, lobulated configurations and their preferential location in the roof of the body of the lateral ventricle.

Discussion A ring-like appearance, on MR imaging, without any associated neurologic manifestation in an otherwise asymptomatic patient is suggestive of ring-shaped lateral ventricular nodules (RSLVNs). Subependymal heterotopia, heterotopic gray matter situated adjacent to the wall of the lateral ventricles, which always appears isointense to normal gray matter on all sequences, was not noted in our patient. Contrast enhancement was also absent on MR imaging of this patient. Heterotopic tissue appears as broad bands lying along the superolateral border of either lateral ventricle or as multiple nodules, protruding into the lateral ventricle. Clinically, patients with this pathology present with seizures, an incidental finding, and notably absent in our patient. Subependymal nodules are found in 90–100% of patients with tuberous sclerosis, not present in our case. The classic clinical triad for tuberous sclerosis consists of facial lesions, seizures, and mental retardation. Typically, the majority of subependymal nodules are located near the caudate nucleus (adjacent to the foramen of Monro), show variable signal characteristics, often calcify, and frequently demonstrate some enhancement – unlike those seen in our patient. However,

618

subependymal heterotopias and subependymal nodules never show a ring-shaped appearance, in contrast to the nodules demonstrated on our patient’s imaging studies. Of the potential differential diagnoses, subependymoma is the most similar in nature and imaging characteristics to the nodules demonstrated by this patient’s imaging findings. Subependymoma, the most frequent non-enhancing neoplasm found in the lateral ventricle, should be considered when a well-defined, non-enhancing tumor is found in the lateral ventricle of middle-aged or elderly adults, especially if found incidentally. However, to the best of our knowledge, subependymoma has not been described with a ring-shaped appearance. A recently described entity of uncertain origin, RSLVNs are devoid of any correlative pathologic association. They are believed to be a benign, incidental finding on CT and MR imaging studies, although some patients showed, at diagnosis, concomitant variable symptoms such as headaches, attributable to the lesions. One recent study reported a prevalence of RSLVNs in 0.45% of routine patient MR exams, higher than previously reported in the literature. These lesions have a variable configuration (round, oval, and lobulated) and tend to be located in the roof of the body, or frontal horn of the lateral ventricles. On imaging, they usually show isointense signal on T1weighted and T2-weighted sequences, hyperintense signal of the ring portion on FLAIR sequences, and isointense signal of the core portion, relative to CSF, on T1-weighted and T2-weighted sequences. They do not demonstrate contrast enhancement or morphologic changes on follow-up exams in the studies available in the current literature.

Key Points  A ring-like intraventricular nodule in an otherwise asymptomatic patient is the key to diagnosing ring-shaped lateral ventricular nodules.  It is important to diagnose these incidental lesions to avoid further diagnostic workup.

Suggested Reading Braffman BH, Bilaniuk LT, Naidich TP, et al. MR imaging of tuberous sclerosis: pathogenesis of this phakomatosis, use of gadopentetate dimeglumine, and literature review. Radiology 1992; 183(1): 227–38. Koeller KK, Sandberg GD. From the archives of the AFIP. Cerebral intraventricular neoplasms: radiologic-pathologic correlation. Radiographics 2002; 22(6): 1473–505. Nakajima R, Uchino A, Saito N, Nishikawa R. Ring-shaped lateral ventricular nodules detected with brain MR imaging. Magn Reson Med Sci 2013; 12(2): 105–10. Raymond AA, Fish DR, Stevens JM, et al. Subependymal heterotopia: a distinct neuronal migration disorder associated with epilepsy. J Neurol Neurosurg Psychiatry 1994; 57(10): 1195–202. Shimono T, Hosono M, Ashikaga R, et al. Ring-shaped lateral ventricular nodules: an incidental finding on brain magnetic resonance imaging. Neuroradiology 2009; 51(3): 145–50.

CASE

Part VIII

131

Miscellaneous Satya Patro, Prasad B. Hanagandi, Lázaro Luís Faria do Amaral

Clinical Presentation A 48-year-old man presented with a four-year history of gait unsteadiness ataxia, frequent falls, and slurred speech. His past medical history was uneventful and did not include diabetes, hypertension, hypercholesterolemia, or substance abuse. Neurologic examination findings for cranial nerve and cognitive function assessment were normal. On examination, he had marked cerebellar signs, viz: speech dysarthria, dysdiadochokinesia, truncal ataxia, and dysmetria. Hyperreflexia in the lower limbs with plantar responses were flexor. His muscle tone was preserved, with no sensory deficits or extrapyramidal signs. Hematologic studies showed mildly elevated SED and Creactive protein levels. Hematologic screening for infectious agents including HIV, CJD, CMV, hepatitis, and syphilis was negative. Laboratory tests for the presence of celiac or thyroid

autoantibodies, tumor markers, very long chain fatty acids, abnormal levels of alpha-fetoprotein, serum copper, ceruloplasmin, serum vitamin E, and ACE were negative or normal. Antinuclear antibodies (ANAs) were positive with negative double-stranded DNA and ANCA. Antineuronal antibodies (anti-Yo, anti-Hu, anti-Ri, anti-CRMP5, anti-amphiphysin, and anti-GAD) were not detected. Cerebrospinal fluid cytology, biochemistry, and molecular analysis were normal (negative for oligoclonal bands, mycobacterium, JC virus and HHV-6 viruses, and fungi). Genetic tests for fragile X syndrome, Friedreich and spinocerebellar ataxia types 1, 2, 3, 6, 7, and 17 were negative. The values from visual evoked potential were within the upper limit of normal. Brainstem auditory evoked potentials and electronystagmography were not performed.

Imaging

Fig. 131.1 Axial T2WI at the level of centrum semiovale/corona radiata.

Advanced Neuroradiology Cases, ed. Amaral et al. Published by Cambridge University Press. © Cambridge University Press 2016.

619

Part VIII. Miscellaneous: Case 131

Fig. 131.2 Axial DWI image at the level of centrum semiovale/corona radiata. Fig. 131.3 Axial postcontrast T1WI at the level of centrum semiovale/corona radiata.

(B) (A)

Fig. 131.4 (A–B) Axial T2W images at the level of pons, cerebellar hemispheres, middle cerebellar peduncles, and dentate nuclei.

620

Part VIII. Miscellaneous: Case 131

Fig. 131.5 Axial postcontrast T1WI at the level of pons, dentate nuclei, and middle cerebellar peduncles.

Fig. 131.6 Coronal T1WI of bilateral femora.

621

Part VIII. Miscellaneous: Case 131

Erdheim-Chester Disease Primary Diagnosis Erdheim-Chester disease

Differential Diagnoses Acquired and inherited causes of adult-onset ataxia Glioma Multiple sclerosis Lymphoma Sarcoidosis

Imaging Findings Fig. 131.1: T2W hyperintense lesions were noted in the central deep white matter (arrows). Fig. 131.2: Few of these lesions show diffusion restriction on DWI sequence. Fig. 131.3: Postcontrast T1WI shows nodular enhancement (arrow) in the right centrum semiovale. Fig. 131.4: (A–B) Similar confluent T2-hyperintense lesions are seen in the pons, bilateral superior and middle cerebellar peduncles, and dentate nuclei (arrows). Fig. 131.5: Nodular enhancement was seen in the bilateral cerebellar dentate nuclei (arrows). Fig. 131.6: Coronal T1WI demonstrates geographic pattern hypo- to isointense lesions predominantly involving the diametaphyseal regions of bilateral femora (arrows). Biopsies of bone and skin tissue were positive for lymphocytes, macrophages, and lipid-laden histiocytes, thus confirming the diagnosis of ECD.

Discussion The differential diagnoses of Erdheim-Chester disease (ECD)related progressive cerebellar syndrome include alcoholic cerebellar degeneration, paraneoplastic cerebellar degeneration, immune-mediated ataxias, acquired vitamin deficiency, superficial siderosis, chronic CNS infections, spinocerebellar ataxia, fragile X-associated tremor/ataxia syndrome (FXTAS), and multiple system atrophy-cerebellar variant (MSA-C). Cerebellar degeneration related to alcoholism and toxic causes were excluded because of lack of history and absence of cerebellar volume loss. Autoantibody screening and negative imaging results for primary or metastatic tumors further exclude a neoplastic etiology. Lack of abnormal signal changes in the posterior columns of the spinal cord and negative lab evaluation excludes vitamin B12 deficiency. No obvious clinical features, laboratory or imaging findings suggestive of chronic CNS infections such as HIV, CJD, Whipple disease, or neurosyphilis were present. Genetic screening for Friedreich ataxia and spinocerebellar ataxia was negative. FXTAS and MSA-C were excluded because of the absence of specific neuroimaging findings. The neuroimaging findings of ECD can mimic multiple conditions, depending on the pattern of involvement, either meningeal or parenchymal. The meningeal pattern commonly presents with nodular meningeal thickening or mass lesion. Pathologies such as meningioma, lymphoma, tuberculosis, fungal infections, and sarcoidosis can have a similar

622

appearance; however, these conditions can be excluded based on clinical, imaging, laboratory, and histopathologic analysis. In our patient, neuroimaging features were suggestive of parenchymal CNS involvement. The differential diagnoses may include parenchymal tumors such as glioma and lymphoma; however, lack of mass effect, edema, or rapid progression excludes these conditions. Middle cerebellar peduncle lesions can be seen in multiple sclerosis and other infectiousinflammatory demyelinating pathologies. However, the absence of corpus callosum or periventricular lesions and negative oligoclonal band excludes multiple sclerosis. The lack of leptomeningeal nodular enhancement and cranial neuropathies, and sparing of the hypothalamus excludes sarcoidosis. Erdheim-Chester disease is an extremely uncommon, nonLangerhans form of histiocytosis of unknown etiology that can involve various organ systems including the musculoskeletal, cardiac, pulmonary, gastrointestinal, and central nervous systems. It equally affects both males and females in their fifth to seventh decades of life. Clinically, it can present as a focal asymptomatic process or multisystem fatal condition. Characteristic histiocytic organ infiltration often leads to non-specific manifestation of fever, malaise, and weight loss. The most frequent clinical systemic manifestation is skeletal involvement with bone pain. In ECD, the long bones of bilateral upper and lower extremities are commonly involved in a symmetric fashion. Plain film radiography of patients with ECD commonly demonstrates diffuse or patchy increased density, coarsened trabecular pattern, medullary sclerosis, and cortical thickening frequently affecting the meta-diaphysis of the long bones. Magnetic resonance imaging may reveal a geographic pattern of abnormal signal intensity in long bones, with relative sparing of epiphyses. Strong labeling of the distal ends of the long bones has been observed on 99mTc bone scintigraphs. Extraskeletal manifestation is seen in approximately half of the patients with mass formation at sites such as the retroperitoneum, neck, orbit, or breast. Laboratory findings of ECD are non-specific and may include elevated SED, increased levels of alkaline phosphatase, and increased C-reactive protein levels. More specific laboratory findings may be demonstrated, depending on the physiologic functions affected by multisystemic ECD, and could help in understanding the patient’s distribution of the disease. Biopsy of long bones and other systemic sites involved in ECD typically demonstrates lymphocytes, macrophages, and lipid-laden histiocytes with immunohistochemistry positive for CD68 and negative for CD1a. Central nervous system involvement is considered a poor prognostic factor and found in approximately one-third of patients with ECD, and can usually be associated with systemic disease. There are three patterns of CNS involvement in ECD: 1) a parenchymal infiltrative pattern with widespread lesions, nodules, or masses of the brainstem, cerebellum, and cerebral hemispheres (44% of patients); 2) a meningeal pattern with nodular thickening and meningioma-like lesions in approximately 37% of cases; and 3) a mixed pattern of parenchymal and meningeal involvement, which is seen in 19% of patients.

Part VIII. Miscellaneous: Case 131

Diabetes insipidus is the most common neurologic manifestation in ECD. Progressive cerebellar syndrome is the second most frequent CNS manifestation of ECD and can develop over several years. A variety of symptoms can be associated with brainstem and cerebellar involvement of ECD. These patients commonly present with slowly progressive cerebellar dysfunction displaying cerebellar dysmetria, hypermetric saccades, ataxia, dysarthria, nystagmus, dysdiadochokinesia, and pyramidal syndrome. Additionally, unlike most intracranial lesions, cerebellar lesions do not exhibit mass effect. The underlying pathogenesis is again similar to other systemic involvement with infiltration of intracranial tissue by foamy histiocytes with gliosis, extensive loss of myelin sheath, and marked sparing of axons. Magnetic resonance imaging is the modality of choice when evaluating the different CNS manifestations. On MRI, these lesions are characterized by high signal intensity on T2WI, low signal intensity on T1WI, and may show enhancement on contrast study. Cerebellar atrophy is rare and only visible in fewer percentages of patients with ECD. Early diagnosis and recognition of ECD is important, as there is some evidence that treating ECD with interferon alpha may improve survival, as shown in a recent multicenter observational study. Erdheim-Chester disease should be regarded as a rare cause of adult-onset sporadic ataxia, especially when accompanied by enhancing pontine and cerebellar lesions with pathognomonic radiologic findings of sclerotic lesions and 99mTc uptake in the long bones. Extra-neurologic manifestations such as retroperitoneal fibrosis or orbital involvement are an additional clue for diagnosis. Biopsy of the amenable lesions is helpful for definitive diagnosis.

Key Points  Erdheim-Chester disease is a rare cause of adult-onset cerebellar ataxia with a wide list of differential diagnoses presenting with similar clinical presentations.  The presence of geographic pattern bone lesions with CNS imaging features should make one consider ECD in the probable list of differentials.  Skin or bone biopsy aids in the final diagnosis.

Suggested Reading Klockgether T. Sporadic ataxia with adult onset: classification and diagnostic criteria. Lancet Neurol 2010; 9(1): 94–104. Lachenal F, Cotton F, Desmurs-Clavel H, et al. Neurological manifestations and neuroradiological presentation of ErdheimChester disease: report of 6 cases and systematic review of the literature. J Neurol 2006; 253(10): 1267–77. Mazor RD, Manevich-Mazor M, Shoenfeld Y. Erdheim-Chester disease: a comprehensive review of the literature. Orphanet J Rare Dis 2013; 8: 137. Na SJ, Lee KO, Kim JE, Kim YD. A case of cerebral Erdheim-Chester disease with progressive cerebellar syndrome. J Clin Neurol 2008; 4(1): 45–50. Salsano E, Savoiardo M, Nappini S, et al. Late-onset sporadic ataxia, pontine lesion, and retroperitoneal fibrosis: a case of ErdheimChester disease. Neurol Sci 2008; 29(4): 263–7. Sedrak P, Ketonen L, Hou P, et al. Erdheim-Chester disease of the central nervous system: new manifestations of a rare disease. AJNR Am J Neuroradiol 2011; 32(11): 2126–31. Shanmugam SV, Kolappan M, Garg M, et al. A rare cause of lateonset cerebellar ataxia: Erdheim-Chester disease. Cerebellum 2013; 12(4): 593–5.

623

CASE

Part VIII

132

Miscellaneous Asim K. Bag, Fabrício Guimarães Gonçalves, Lázaro Luís Faria do Amaral

Clinical Presentation A previously healthy 35-year-old man presented with slowonset 1–3 Hz rhythmic involuntary movement of the oropharynx. He reported a six-month history of previous onset of visual disturbances and diplopia that subsequently subsided. He does not have any other neurologic or systemic problems. An MRI of the brain was performed (Figs. 132.1–132.4).

Imaging Fig. 132.1 Midline sagittal T1WI.

Advanced Neuroradiology Cases, ed. Amaral et al. Published by Cambridge University Press. © Cambridge University Press 2016.

625

Part VIII. Miscellaneous: Case 132 Fig. 132.2 Axial T2WI through the pons.

Fig. 132.3 Axial FLAIR through inferior medulla.

626

Part VIII. Miscellaneous: Case 132 Fig. 132.4 Axial T2WI through inferior medulla.

Fig. 132.5 Schematic diagram showing the dentato-rubroolivary pathway (Guillain-Mollaret triangle).

Red nucleus cle

n

du

e rp

lla

be

ior

c

Central tegmental tract

r pe

e er

Su

Contralateral dentate nucleus

Inf eri

or

ce

reb

ell

ar

pe

du

nc

le

Inferior olivary nucleus

627

Part VIII. Miscellaneous: Case 132

Hypertrophic Olivary Degeneration Primary Diagnosis Hypertrophic olivary degeneration

Differential Diagnoses Medullary infarction Medullary tumor Demyelinating plaque POLG or SURF1 mutation

Imaging Findings Fig. 132.1: Midline sagittal precontrast T1WI demonstrated a heterogeneous lesion with mixed signal intensity in the dorsal aspect of the pons. Fig. 132.2: Axial T2WI through the pons demonstrated a predominantly hyperintense lesion, surrounded by a thin hypointense rim, secondary to hemosiderin deposition in the posterior aspect of the pons involving the expected area of central tegmental tract and the right superior cerebellar peduncle. Fig. 132.3: Axial FLAIR and Fig. 132.4: T2WI through the level of lower medulla demonstrated hyperintensity and mild enlargement of bilateral olives, with sparing of the adjacent other medullary structures.

Discussion Presentation with palatal myoclonus is highly suggestive of pathology at the medulla, particularly with involvement of the inferior olivary nucleus (ION) at the olive. Bilateral olivary enlargement and hyperintensity on T2WI with sparing of the adjacent medullary structures is diagnostic of hypertrophic olivary degeneration (HOD), due to a venous vascular malformation at the dorsal pons, disrupting the superior cerebellar peduncle (SCP) decussation and the central tegmental tract. Acute onset of visual disturbances six months prior to presentation was likely due to hemorrhage in the venous vascular malformation. Medullary demyelination and infarction typically accompany T2 hyperintensity that lacks ION enlargement. Tumor (astrocytoma or metastasis) may involve the medulla; however, the exclusive involvement of the olives, without involvement of surrounding brain tissue, is not suggestive of tumor. Bilateral HOD has been described with POLG and SURF1 mutation. In these variations of HOD, there are no structural lesions seen at the brainstem. Additional cystic degeneration of the bilateral cerebellar hemisphere is an additional imaging feature of POLG mutation, which is not seen in this patient. Hypertrophic olivary degeneration, first reported in 1887 by Oppenheim, is a distinct and rare form of transsynaptic degeneration (neuronal loss due to loss of input) that occurs as a result of lesional growth in the dentato-rubro-olivary pathway (Guillain-Mollaret triangle). This extrapyramidal system is formed by the ipsilateral ION in the medulla, the red nucleus (RN) in the midbrain, and the contralateral dentate nucleus (DN) in the cerebellum.

628

Dentatorubral fibers connect the RN with the contralateral DN via the SCP that crosses the midline at the decussation of the SCP at the inferior midbrain/superior pons. The RN and the ipsilateral ION are connected via the central tegmental tract. Fibers from the ION project to the contralateral cerebellar cortex through the olivocerebellar tract via the inferior cerebellar peduncle (ICP) and then to the DN (see Fig. 132.5). Lesions disrupting the central tegmental tract cause ipsilateral HOD, whereas lesion growth at the DN or SCP causes contralateral HOD. Bilateral HOD is seen when a lesion involves both the central tegmental tract and the SCP. As there is no direct white matter connection between the ION and the contralateral DN, disruption of this limb of the GuillainMollaret triangle does not cause HOD. Common lesions that are involved in HOD include venous vascular malformation (as in this patient), cavernoma, demyelination, infarction, tumor, or iatrogenic-based lesions. Deafferentation of ION causes supersensitivity denervation and HOD. Histologically, this entity is characterized by neuronal degeneration, vacuolation and swelling, gliosis, and tissue demyelination in the ION. In some cases, no cause could be found and these are termed idiopathic HOD. Pathologic changes in HOD have been described in six stages. In the first stage, no significant changes are noticed immediately after the procursive insult. The second stage begins approximately three weeks after the initial insult and is distinguished by amiculum degeneration, and consequently, notable olivary neuronal hypertrophy. During the third stage, glial hypertrophy occurs that ushers in the fourth stage, subsequent olivary enlargement. In the fifth stage, pseudohypertrophy develops as neurons degenerate; however, large gemistocytic astrocytes persist, denoting the fifth stage of HOD. Finally, more than a year after the initial injury, during the sixth stage, the ION undergoes atrophy. Although hypertrophy resolves, T2 hyperintensity rarely normalizes. Hypertrophic olivary degeneration affects children and adults; the majority of patients present with unilateral HOD. Clinical findings associated with HOD include palatal tremor (PT; 1–3 Hz rhythmic involuntary movement of the oropharynx), ocular myoclonus, and dentatorubral tremor of the upper extremity. Palatal tremor can be an essential diagnostic clue when no etiologies can be identified. Occurrence of symptomatic PT tends to correlate with onset of changes in the ION, reaching its maximum soon after ION dysmyelination peaks. Deafferentation of the ION causes disinhibition of the nucleus, triggering spontaneous rhythmic firing, and stimulating the contralateral dentate to lower motor neuron projections; therefore, ION degeneration causes symptomatic PT. Alternatively, the disrupted dentato-rubro-olivary pathway may become unable to inhibit cranial nerve motor nuclei firing. On MR, T2-weighted images are best in depicting the abnormal and affected ION, which is typically increased in size and shows a hyperintense signal. Imaging findings correlate well with the pathologic features. Increased T2 signal is

Part VIII. Miscellaneous: Case 132

usually seen one month after a cerebellar and/or brainstem infarct. Olivary enlargement is noted as early as six months after injury. Inferior olivary nucleus expansion may cease in 10–16 months; however, T2 signal abnormalities may persist for years or even indefinitely. With diffusion tensor imaging, it has been shown that there is decreased fractional anisotropy of the involved fibers and that the disrupted tracts are smaller compared to the uninvolved side. The myoclonus associated with HOD has been successfully treated with benzodiazepines, carbamazepine, or 5-hydroxytryptophan.

Key Points  Deafferentation of the ION due to the disruption of the dentatorubral, as well as central tegmental tracts, by vascular lesions, infarctions, demyelination, tumors, or iatrogenic causes results in HOD. Genetic causes of HOD have also been identified.  The typical clinical presentation is PT, ocular myoclonus, and dentatorubral tremor of the upper extremities.

 Typical imaging features include enlargements of the involved olive(s) with T2 hyperintensity. Olive changes are dynamic; olivary enlargement occurs 4–6 months after the ictus, and hypertrophy resolves after 10–16 months.

Suggested Reading Hornyak M, Osborn AG, Couldwell WT. Hypertrophic olivary degeneration after surgical removal of cavernous malformations of the brain stem: report of four cases and review of the literature. Acta Neurochir (Wien) 2008; 150(2): 149–56; discussion 156. Kinghorn KJ, Kaliakatsos M, Blakely EL, et al. Hypertrophic olivary degeneration on magnetic resonance imaging in mitochondrial syndromes associated with POLG and SURF1 mutations. J Neurol 2013; 260(1): 3–9. Salamon-Murayama N, Russell EJ, Rabin BM. Diagnosis please. Case 17: hypertrophic olivary degeneration secondary to pontine hemorrhage. Radiology 1999; 213(3): 814–17. Shah R, Markert J, Bag AK, Cure JK. Diffusion tensor imaging in hypertrophic olivary degeneration. AJNR Am J Neuroradiol 2010; 31(9): 1729–31.

629

CASE

Part VIII

133

Miscellaneous Fabrício Guimarães Gonçalves, Lázaro Luís Faria do Amaral

Clinical Presentation A 32-year-old man presented with a history of progressive headache and vertigo. Computed tomography scans of the head were obtained with abnormal findings that did not correlate with clinical presentation (see below). Magnetic resonance imaging was obtained to better depict the incidental CT findings.

Imaging Fig. 133.1 Axial CT image of the brain.

Advanced Neuroradiology Cases, ed. Amaral et al. Published by Cambridge University Press. © Cambridge University Press 2016.

631

Part VIII. Miscellaneous: Case 133 Fig. 133.2 Sagittal T1WI of the brain.

Fig. 133.3 Axial T2WI of the brain.

632

Part VIII. Miscellaneous: Case 133 Fig. 133.4 Coronal T1WI postcontrast image of the brain.

Fig. 133.5 Still photograph of patient.

633

Part VIII. Miscellaneous: Case 133

Neurocutaneous Melanosis Primary Diagnosis Neurocutaneous melanosis

Differential Diagnosis Primary leptomeningeal melanoma

Imaging Findings Fig. 133.1: Axial CT scan of the brain showed a tiny, hyperdense, left-sided, periventricular nodule (circle) with no mass or surrounding edema. Fig. 133.2: Sagittal T1WI showed an oval-shaped, spontaneous hyperintense nodule (arrow) with no mass effect. Fig. 133.3: Axial T2WI demonstrating a slightly hyperintense nodule (arrow) with no associated edema. Fig. 133.4: Coronal T1WI postcontrast image showed mild enhancement of the lesion (arrow), which demonstrated spontaneous T1 hyperintensity. Fig. 133.5: Image of patient showing multiple satellite melanotic nevi.

Discussion Neurocutaneous melanosis (NCM) is a rare congenital disorder characterized by the presence of large or multiple congenital melanocytic nevi (CMN) associated with CNS melanocytic lesions. In 1861, Rokitansky originally described NCM in a 14-year-old girl with a congenital nevus, mental retardation, and hydrocephalus. Demonstration of a T1-hyperintense signal in the absence of a mass or edema, in a symptomatic patient with multiple satellite nevi, is suggestive of NCM. Congenital melanocytic nevi are nevi usually present at birth or within the first few weeks of life. These lesions are classified as neural crest-derived hamartomas and are derived from pigmented cell malformations, formed during ontogenesis. The disease has been associated with deregulation of hepatocyte growth factor/scatter factor, a cytokine that stimulates the proliferation, migration, and morphogenesis of cultured epithelial cells and is also involved in the distribution and proliferation of melanocytes. The original diagnostic criteria for NCM was originally proposed by Fox in 1972 and further revised by Kadonaga and Frieden. The criteria are as follows: 1) large (currently or estimated to become no less than 20 cm in diameter in adults or 6–9 cm in infants); or 2) multiple ( 3) CMN associated with meningeal melanosis or CNS melanoma; no evidence of cutaneous melanoma; and no evidence of meningeal melanoma except in patients whose skin examination reveals no malignant lesions. However, a definitive NCM diagnosis can only be made based on histologic confirmation of the CNS lesions. Patients with NCM usually have more than three CMN on their head, neck, or dorsal spine; and the majority (two-thirds) of patients have a giant bathing trunk melanocytic nevus in the lumbosacral region. Giant CMN occur in approximately 1/ 20,000 newborns and are defined as those that are expected

634

to grow to a diameter of at least 20 cm in adulthood. Patients with giant CMN also have a higher risk for developing meningeal and cutaneous melanoma. Primary leptomeningeal melanoma, which typically results in meningeal pigmented lesions, differs from NCM. In patients with NCM, the meninges are not typically involved and melanin deposits typically occur in the brain parenchyma, choroid plexus, and along the ependymal lining. Melanin parenchymal deposition most likely represents melanocytes tracking along the perivascular spaces. The most frequent location of melanin accumulation in the CNS is the anterior temporal lobes, especially in the region of the amygdala. Other less common locations include the cerebellar hemispheres, thalami, the basifrontal region, midbrain, and spinal cord. Most cases of NCM are sporadic, commonly occurring in Whites with no gender predilection. Complications of patients with NCM can occur in two peaks, the first representing the majority of patients, usually occurring before three years of age and the second, during the second to third decades of life. In the first two years of life, patients with NCM typically develop neurologic manifestations related to increased intracranial pressure or seizures. Other symptoms include irritability, lethargy, emesis, seizures, photophobia, papillary edema, cranial nerve palsies, external ocular motor and facial nerve involvement, pruritus, hypertrichosis, and ulceration of the skin lesions. However, the most serious complication is development of a malignant melanoma. In rare cases, symptoms begin in adulthood, and some patients remain asymptomatic. Hydrocephalus occurs in the majority of patients and is related to either meningeal thickening, CSF outflow obstruction, or decreased CSF reabsorption. Other reported complications include subdural or parenchymal hemorrhage, and neuropsychiatric symptoms including depression and psychosis. Patients with spinal involvement may develop symptoms of myelopathy, syringomyelia, spinal arachnoiditis radiculopathy, and bowel or bladder dysfunction. Other diseases have been described to be associated with NCM such as Dandy-Walker malformation (with an even more worrisome prognosis), lissencephaly, encephalocraniocutaneous lipomatosis, hemimegalencephaly, and corpus callosum agenesis. Malignant melanoma may occur in 40–60% of the patients with NCM; the majority of patients die within three years from benign melanocytic cell overgrowth or malignant melanoma development. On gross examination, the lesions have dense black-brown material overlying the affected surfaces and areas of focal nodularity. On histologic examination, NCM resembles CMN with polygonal cells with prominent nucleoli and cytoplasmic melanin with signs of Virchow-Robin space invasion. On CT, melanin deposits may appear as subtle areas of high attenuation, which are difficult to appreciate unless there is progression to melanoma. In cases of extensive melanosis,

Part VIII. Miscellaneous: Case 133

patients may present with hydrocephalus and leptomeningeal enhancement may be seen. Magnetic resonance imaging is far more sensitive than CT for demonstrating NCM changes and should ideally be performed within the first four months of age, before normal brain myelination, which may obscure melanin deposits. Magnetic resonance imaging findings include T1 hyperintense foci (typically 3 cm or less in size) within the brain parenchyma and meninges. These foci are most commonly found in the anterior temporal lobe (typically in the amygdala), cerebellar white matter, cerebellar nuclei, and brainstem. Leptomeningeal enhancement has been reported in cases of NCM and if associated with hydrocephalus may suggest diffuse leptomeningeal spread. Magnetic resonance imaging is also important to rule out tethered spinal cord, lipomas, and vascular malformations. Hypoplasia of the cerebellum and pons may be seen and is usually associated with melanosis in these locations. Spinal and intracranial arachnoid cysts and spinal lipomas have also been described in cases of NCM. Degeneration into malignant melanoma is indicated by progressive growth, surrounding vasogenic edema or mass effect, or development of central necrosis. Signs of malignant degeneration include the presence of leptomeningeal or intraparenchymal lesion enhancement or focal nodular or thick plaque-like enhancement. Symptomatic and asymptomatic patients present distinct natural history and it is important to note that not every NCM patient with imaging findings will develop symptoms

(10–68%). More than half of symptomatic patients die within three years of diagnosis.

Key Points  Diagnosis of NCM is based on clinical presentation and neuroimaging findings.  Magnetic resonance imaging, preferred over CT, may show increased T1 signal suggestive of melanocyte aggregation in the anterior temporal lobes, amygdala, or the cerebellum.  Thorough neurologic examination, including MRI studies, should be performed on children with congenital nevi, and asymptomatic adults presenting with neurologic symptoms.

Suggested Reading Kim KH, Chung SB, Kong DS, Seol HJ, Shin HJ. Neurocutaneous melanosis associated with Dandy-Walker complex and an intracranial cavernous angioma. Childs Nerv Syst 2012; 28(2): 309–14. Krengel S, Hauschild A, Schafer T. Melanoma risk in congenital melanocytic naevi: a systematic review. Br J Dermatol 2006; 155(1): 1–8. Schreml S, Gruendobler B, Schreml J, et al. Neurocutaneous melanosis in association with Dandy-Walker malformation: case report and literature review. Clin Exp Dermatol 2008; 33(5): 611–14. Smith AB, Rushing EJ, Smirniotopoulos JG. Pigmented lesions of the central nervous system: radiologic-pathologic correlation. Radiographics 2009; 29(5): 1503–24.

635

CASE

Part VIII

134

Miscellaneous Prasad B. Hanagandi, Stephanie Lam, Lázaro Luís Faria do Amaral

Clinical Presentation A 35-year-old woman presented with a one-year history of episodic seizures, and history of progressive, left hemicranial headache that began when she was 7–8 years of age. She reported the onset of facial asymmetry with gradual scarring at the age of 20 years. Clinical examination revealed left-sided facial scarring, asymmetry, and focal alopecia along the left frontal scalp. She was clinically diagnosed to have scleroderma.

Imaging (A)

(B)

Fig. 134.1 (A) Frontal view of the patient at presentation. (B) Superior view of patient’s scalp.

Advanced Neuroradiology Cases, ed. Amaral et al. Published by Cambridge University Press. © Cambridge University Press 2016.

637

Part VIII. Miscellaneous: Case 134

638

Fig. 134.2 Axial FLAIR image of brain at the level of basal ganglia.

Fig. 134.3 Axial T2WI at the level of corona radiata.

Fig. 134.4 Axial SWI of the brain at the level of lateral ventricles and basal ganglia.

Fig. 134.5 Sagittal GRE image.

Part VIII. Miscellaneous: Case 134 Fig. 134.6 Frontal 3D facial surface rendering.

Fig. 134.7 Time of flight MRA of the circle of Willis.

639

Part VIII. Miscellaneous: Case 134

Progressive Hemifacial Atrophy: Parry-Romberg Syndrome with Localized Scleroderma Primary Diagnosis Progressive hemifacial atrophy: Parry-Romberg syndrome with localized scleroderma

Differential Diagnoses Hemifacial microsomia (Goldenhar syndrome) Rasmussen encephalitis Partial lipodystrophy (Barraquer-Simons syndrome) Post-traumatic facial atrophy Postradiation changes Unilateral salivary gland and unilateral facial and masticatory muscle atrophy

Imaging Findings Fig. 134.1: Photographs show (A) left-sided facial scarring and asymmetry (arrows) and (B) focal alopecia. Fig. 134.2: Axial FLAIR MRI demonstrated white matter changes in the left frontal periventricular region (arrow) and focal thinning of the left frontal scalp tissues (arrow). Fig. 134.3: Axial T2WI at the same level also showed white matter changes (arrow). Fig. 134.4: Axial SWI and Fig. 134.5: Parasagittal MRI demonstrated multiple foci of blooming/T2 shortening in the left frontal lobe (arrows). Fig. 134.6: 3D surface rendering image depicts left facial atrophy as en coup de sabre. Fig. 134.7: MRA of the circle of Willis shows normal appearance.

Discussion Unilateral hemifacial atrophy with involvement of skin, subcutaneous fat, muscle, bone, cartilage, and associated CNS findings on imaging as seen in this patient are classic findings of Parry-Romberg syndrome. Although congenital hemifacial microsomia is a nonprogressive developmental disorder involving the first and second branchial arches, it does not present with cutaneous pigmentation changes or thinning of the dermis. Parry-Romberg syndrome has been reported to have an association with Rasmussen encephalitis. Based only on imaging, the unilateral cerebral atrophy can cause difficulty in differentiating it from Rasmussen encephalitis. However, the clinical presentation along with predominant involvement of the subcutaneous fat, muscle, and other hemifacial structures is helpful in distinguishing between the two pathologies. Clinical history excludes post-traumatic atrophy and lack of craniofacial bone deformities on imaging is evident. Postradiation changes are asymmetric, but are usually bilateral, and correlation with lack of history of head and neck malignancy excludes them from the list of differential diagnoses. Partial lipodystrophy, also a bilateral condition, is limited to adipose tissues. Often, unilateral atrophy of the salivary glands and masticator muscles can mimic Parry-Romberg syndrome. However, the atrophy is

640

localized to the gland or muscles with no obvious changes in the subcutaneous fat or underlying bones. Parry-Romberg syndrome is a rare form of self-limiting but progressive hemifacial atrophy beginning in the first and second decades of life. It was first described by Parry in 1825 and Romberg in 1846. Parry-Romberg syndrome is characterized by slow but progressive unilateral facial atrophy resulting in facial asymmetry. The process involves the skin, subcutaneous tissues, muscles, cartilage, and bones. Prevalence is at least 1 in 700,000 in the general population, with a slight female preponderance. The average age of onset is around 10 years of age. The skin can be tense, dry, and hyper- or hypopigmented. A demarcation line, or cleft near the facial midline, may occasionally be identified between the normal and the abnormal skin, giving a coup de sabre appearance. A variety of ocular changes has also been reported, the most common being enophthalmos due to loss of intraorbital fat. Other symptoms include heterochromia, globe retraction, and uveitis. Hair depigmentation or alopecia; smaller ipsilateral ear; lip, tongue, and salivary gland atrophy; dental abnormalities; and osseus defects can also be present. Neurologic symptoms are diverse and include seizures, trigeminal neuritis, and severe headache confined to the side of atrophy. There have even been isolated reports of associated hemibody atrophy. Symptoms typically progress slowly for up to 20 years and eventually stabilize. The pathophysiology of Parry-Romberg syndrome remains unclear, although several hypotheses have been proposed. The most popular suppositions include autoimmunity and disturbance of fat metabolism related to a trophic malformation of the cervical sympathetic nervous system. Trauma, viral infection, hereditary, or endocrine disturbances have also been suggested. Although there have been conflicting theories about a correlation between scleroderma and Parry-Romberg syndrome, the literature mentions that localized scleroderma may be the preceding lesion of progressive hemifacial atrophy. Hence, scleroderma and Parry-Romberg syndrome have overlapping features and are thus a part of the same disease spectrum. Findings of hemiatrophy are well seen with either CT or MR imaging of the face. Atrophy involves the skin, subcutaneous tissues, cartilage, and bones to varying degrees, but is limited by the midline, which is characteristic of the disease. While imaging of the brain can be normal, a wide spectrum of findings has also been described. They are usually ipsilateral, but can rarely be bilateral or contralateral. These findings include cerebral hemispheric atrophy with ventricular enlargement, intracranial calcifications, microhemorrhages, and focal infarcts of the corpus callosum. Non-specific focal or diffuse signal abnormalities in the gray or white matter can also be observed, as can meningocortical dysmorphism, characterized by thickening of the cortex and leptomeninges with leptomeningeal enhancement. Aneurysms and vascular malformations may also be present. Parry-Romberg syndrome is self-limited,

Part VIII. Miscellaneous: Case 134

and has no definitive curative therapy. Treatment aims at providing symptomatic relief and once the disease has stabilized, may include reconstructive surgery. Steroids, antiepileptic medications seizure control, and immunosuppressive therapies have been attempted with varying degrees of success.

Key Points  Hemifacial atrophy with involvement of skin, subcutaneous fat, muscle, cartilage, and bone are important features distinguishing Parry-Romberg syndrome from other conditions.  Parry-Romberg syndrome and scleroderma belong to the same spectrum of disease and can have varied CNS manifestations.  The disease is progressive and self-limiting with no definitive treatment.

Suggested Reading Blitstein MK, Tung GA. MRI of cerebral microhemorrhages. AJR Am J Roentgenol 2007; 189(3): 720–5. Kumar AA, Kumar RA, Shantha GP, Aloogopinathan G. Progressive hemi facial atrophy - Parry Romberg syndrome presenting as

severe facial pain in a young man: a case report. Cases J 2009; 2: 6776. Madasamy R, Jayanandan M, Adhavan UR, Gopalakrishnan S, Mahendra L. Parry Romberg syndrome: a case report and discussion. J Oral Maxillofac Pathol 2012; 16(3): 406–10. Maletic J, Tsirka V, Ioannides P, Karacostas D, Taskos N. ParryRomberg syndrome associated with localized scleroderma. Case Rep Neurol 2010; 2(2): 57–62. Okumura A, Ikuta T, Tsuji T, et al. Parry-Romberg syndrome with a clinically silent white matter lesion. AJNR Am J Neuroradiol 2006; 27(8): 1729–31. Paprocka J, Jamroz E, Adamek D, Marszal E, Mandera M. Difficulties in differentiation of Parry-Romberg syndrome, unilateral facial sclerodermia, and Rasmussen syndrome. Childs Nerv Syst 2006; 22 (4): 409–15. Rangare AL, Babu SG, Thomas PS, Shetty SR. Parry-Romberg syndrome: a rare case report. J Oral Maxillofac Res 2011; 2(2): e5. Sharma M, Bharatha A, Antonyshyn OM, Aviv RI, Symons SP. Case 178: Parry-Romberg syndrome. Radiology 2012; 262(2): 721–5. Stone J. Parry-Romberg syndrome: a global survey of 205 patients using the Internet. Neurology 2003; 61(5): 674–6. Stone J. Parry-Romberg syndrome. Pract Neurol 2006; 6(3): 185–8.

641

CASE

Part VIII

135

Miscellaneous Taleb Al Mansoori, Prasad B. Hanagandi, Lázaro Luís Faria do Amaral

Clinical Presentation A 48-year-old man presented with excessive soft tissue scalp thickening and abnormal odor. He stated that he noticed the changes almost 12 years ago and that the condition had gradually progressed. Physical examination of the head showed bilateral symmetric ridges and furrows extending anteroposteriorly and non-compressible on application of pressure. Neurologic examination and evaluation of the orbits in particular was normal. Laboratory investigations for hematologic studies, liver function, renal function, and endocrinologic studies were normal. Biopsy of the scalp revealed normal histology with thickening of the connective tissue and hypertrophy of the adnexal structures.

Imaging Fig. 135.1 Sagittal T1WI of the brain

Advanced Neuroradiology Cases, ed. Amaral et al. Published by Cambridge University Press. © Cambridge University Press 2016.

643

Part VIII. Miscellaneous: Case 135

(A) (B)

Fig. 135.2 (A) Axial T2WI and (B) Postcontrast T1WI at the level of frontoparietal convexities.

Fig. 135.3 Axial DWI of the brain at the level of frontoparietal convexities.

644

Part VIII. Miscellaneous: Case 135

(A)

(B)

(C)

Fig. 135.4 (A–C) Three-dimensional surface rendering MRI of the scalp and face.

645

Part VIII. Miscellaneous: Case 135

Cutis Verticis Gyrata Primary Diagnosis Cutis verticis gyrata

Differential Diagnoses Associated cutis verticis gyrata entities include: myxedema, leukemia, acanthosis nigricans, amyloidosis diabetes mellitus, syphilis, tuberous sclerosis, and acromegaly Additional pathologic entities associated with cutis verticis gyrata include: Ehlers-Danlos, Noonan, Turner, and fragile X syndromes

Imaging Findings Fig. 135.1: Sagittal T1WI showed diffuse thickening of the scalp soft tissues. Fig. 135.2: (A) Axial T2WI and (B) Postcontrast T1WI also showed thickening of the scalp. Thickening of the underlying subcutaneous fat was noted. Fig. 135.3: Diffusion weighted imaging appeared normal. Fig. 135.4: (A–C) Three-dimensional surface rendering MRI demonstrates anteroposterior orientation of the furrows and ridges.

Discussion Cutis verticis gyrata (CVG) is a term used to describe a rare scalp disorder identified by excessive thickening of the soft tissues of the scalp and characterized by ridges and furrows, which give the scalp a cerebriform appearance. Clinically, the ridges are hard and cannot be flattened on applying pressure. First described in 1837 by Jean-Louis-Marc Alibert, CVG has been called paquidermia verticis gyrata, cutis verticis plicata, and bulldog scalp syndrome. The term cutis verticis gyrata was proposed by Unna in 1907. It was later categorized into primary and secondary forms by Polan E. Butterworth in 1953. The primary type of this condition often occurs after puberty and is demonstrated by the appearance of symmetric ridges in the scalp. The primary type is further classified to essential and non-essential forms. The essential form of primary CVG is usually solitary and is not associated with any additional pathology. On the other hand, the non-essential form is usually associated with neurologic abnormalities, which include microcephaly, mental retardation, strabismus, retinitis pigmentosa, and blindness in some cases. The primary type of CVG is often seen in males. The secondary type of CVG can occur at any age, and unlike the primary type, features an asymmetric ridge pattern. The secondary type is associated with dermatologic diseases such as eczema, psoriasis, and impetigo, as well as systemic diseases such as amyloidosis, acromegaly, leukemia, type II diabetes mellitus, and tuberous sclerosis. Anabolic steroid use is a common cause of the secondary form. The secondary form has been associated with acromegaly, pachydermoperiostosis, pituitary tumors, amyloidosis, myxedema, intracerebral aneurysm, dermatofibroma, cerebriform intradermal nevus, acne conglobata, cutaneous focal mucinosis, acanthosis nigricans, and syphilis. Several syndromic associations are noted with the secondary form of CVG especially Ehlers-Danlos syndrome, Beare-Stevenson syndrome, Noonan

646

syndrome, Michelin tire baby syndrome, fragile X syndrome, and Turner syndrome. The diagnosis of CVG is primarily based on clinical examination and can be incidentally diagnosed when patients are undergoing head CT or MR imaging. Multiplanar imaging depicts the symmetric or asymmetric ridges and the furrows of the scalp depending on the type of the CVG. The ridges and furrows will be oriented in an anterior-posterior pattern and will extend to involve the occipital regions obliquely. The coronal plane is best to evaluate the longitudinal ridges, while the sagittal plane is best to evaluate the transverse ridges. Magnetic resonance imaging also aids in the better assessment of the scalp layers compared to CT. High-resolution MR imaging can help in evaluating the scalp layers, especially the dermis and epidermis, subcutaneous fat, and the galeal/subgaleal/periosteum complex. The scalp thickening is attributed to hyperplasia of fat and connective tissue in the subcutaneous tissue layer, with or without dermal layer abnormalities. Three-dimensional MRI with surface rendering technique is an excellent tool to demonstrate and assess the symmetry and orientation of the thickened scalp folds. Treatment in most cases is aimed at maintaining hygiene to avoid associative unpleasant odor. Plastic surgery is usually performed for cosmetic purposes with excision and healthy skin grafting.

Key Points  Cutis verticis gyrata is a rare clinical condition and often an incidental imaging finding.  It is a common cutaneous manifestation of several disease entities.  The non-essential form of CVG can have neurologic manifestations and evaluation of the orbits is recommended to exclude any associated orbital pathologies.  The secondary form of CVG can be associated with various endocrinologic and dermatologic malignancies.

Suggested Reading Al-Bedaia M, Al-Khenaizan AS. Acromegaly presenting as cutis verticis gyrata. Int J Dermatol 2008; 47(2): 164. Alorainy IA, Magnetic resonance imaging of cutis verticis gyrata. J Comput Assist Tomogr 2008; 32(1): 119–23. Farah S, Farag T, Sabry MA, et al. Cutis verticis gyrata-mental deficiency syndrome: report of a case with unusual neuroradiological findings. Clin Dysmorphol 1998; 7(2): 131–4. Filosto M, Tonin P, Vattemi G, et al. Cutis verticis gyrata, mental retardation and Lennox-Gastaut syndrome: a case report. Neurol Sci 2001; 22(3): 253–6. Oh DJ, Park JH, Kang SH, Hwang SW, Park SW. Primary nonessential cutis verticis gyrata revealed with 3-D magnetic resonance imaging. Acta Derm Venereol 2006; 86(5): 458–9. Okamoto K, Ito J, Tokiguchi S, et al. MRI in essential primary cutis verticis gyrata. Neuroradiology 2001; 43(10): 841–4. Sen F, Cagatay A, Ozsut H, et al. An unusual association of cutis verticis gyrata with empty sella. Eur J Intern Med 2008; 19(6): e23–5.

CASE

Part VIII

136

Miscellaneous Prasad B. Hanagandi, Marc Dilauro

Clinical Presentation A 71-year-old man presented with a one-year history of progressive right-sided facial numbness and paresthesia. Non-tender, crescent-shaped, focal ulceration involving the right nasal ala, sparing the nasal tip and nasal septum developed over a duration of five months. On neurologic examination, facial paresthesia involving the V1–V3 distribution of the right trigeminal nerve was noted. Facial asymmetry and weakness of right-sided mastication muscles was also noted. Two years prior to current illness,

his medical history included surgical intervention for resection of right cerebellopontine angle meningioma followed by an uneventful postoperative recovery for 12 months. He did not have any history of diabetes. Hematologic assays for HIV, VDRL, herpes, c-ANCA, and p-ANCA were negative. Punch biopsy of the right nasolabial fold and nasal ala showed acute ulceration with underlying inflamed fibrovascular tissue that was negative for malignancy. Staining of biopsy tissue for acid-fast bacilli and spirochetal organisms was negative.

Imaging

Fig. 136.1 Axial postcontrast T1WI through the posterior fossa at the level of cerebellopontine angle cisterns.

Fig. 136.2 Axial T2WI through the posterior fossa at the level of cerebellopontine angle cisterns (follow-up).

Advanced Neuroradiology Cases, ed. Amaral et al. Published by Cambridge University Press. © Cambridge University Press 2016.

647

Part VIII. Miscellaneous: Case 136

Fig. 136.3 Axial postcontrast T1WI at the level of nasopharynx and maxillary sinuses (follow-up). Fig. 136.4 Axial postcontrast T1WI at the level of nasopharynx and maxillary sinuses (follow-up).

Fig. 136.5 Axial T1WI at the level of nasopharynx and maxillary sinuses (follow-up).

648

Fig. 136.6 Coronal T2WI at the level of sphenoid sinus and anterior temporal lobes.

Part VIII. Miscellaneous: Case 136

Fig. 136.7 Facial profile image of the patient.

Fig. 136.8 Frontal image of the patient.

Fig. 136.9 Histopathology of the right nasal ala biopsy.

649

Part VIII. Miscellaneous: Case 136

Trigeminal Trophic Syndrome Primary Diagnosis Trigeminal trophic syndrome

Differential Diagnoses Infectious etiologies: herpes, syphilis, or Mycobacterium leprae Neoplasms: basal cell carcinoma, squamous cell carcinoma, or lymphoma Autoimmune disorders: granulomatosis with polyangiitis

Imaging Findings Fig. 136.1: Axial T1W postgadolinium image (preoperative) showed a homogeneously enhancing, extra-axial, right cerebellopontine angle cistern that was consistent with meningioma. Fig. 136.2: Axial T2WI at the level of cerebellopontine angle cistern (postoperative follow-up) showed significant encephalomalacic changes in the right hemipons, especially along the trajectory of the right trigeminal nucleus. Fig. 136.3: Axial postcontrast T1WI at the level of nasopharynx and maxillary sinuses demonstrated abnormal thickening and enhancement along the right nasal ala (arrow). Fig. 136.4: Axial postcontrast T1WI at the level of nasopharynx and maxillary sinuses showed subsequent ulceration and focal defect (arrow) (follow-up) and sparing of the nasal tip. Fig. 136.5: Axial T1WI at the level of nasopharynx and maxillary sinuses demonstrated diffuse atrophy of the muscles of mastication (temporalis, masseter, and pterygoid muscles) (arrows). Fig. 136.6: Coronal T2WI at the level of sphenoid sinus and anterior temporal lobes also showed diffuse muscle atrophy (arrows). Fig. 136.7: Facial profile image of the patient at presentation showed nasal ulceration. Fig. 136.8: Frontal facial image demonstrated facial asymmetry. Fig. 136.9: Punch biopsy of nasolabial fold demonstrated ulcerated squamous epithelium with underlying inflamed granulation tissue and reactive fibrosis. No evidence of invasive carcinoma or vasculitis was noted.

Discussion Trigeminal trophic syndrome (TTS), also referred to as trigeminal trophic ulceration, is a rare entity associated with damage to the central or peripheral trigeminal nerve pathway. The etiologies of trigeminal nerve damage leading to TTS include treatment for trigeminal neuralgia (such as alcohol injection of the Gasserian ganglion), tumors (meningioma, schwannoma, and astrocytoma), infectious processes (herpes, syphilis, and leprosy), cerebrovascular accidents, syringobulbia, and trauma. The characteristic triad of anesthesia, paresthesia, and nasal ala ulceration with sparing of the nasal tip in this patient confirms the diagnosis of TTS. The onset of disease in TTS varies, ranging from weeks to decades following damage to the trigeminal nerve pathway. The incidence of TTS is unknown; however, there is an increased predominance among females (female-to-male ratio of 2.2:1) in the sixth decade of life. The mechanism of ulceration is thought to be due to repetitive self-mutilation in an

650

attempt to relieve the paresthesias experienced by the patient following trigeminal nerve injury. The ulcers are frequently unilateral (the right side is affected two times more commonly than the left side), crescent-shaped, and involve the nasal ala. Ulceration can also involve the forehead, cheek, jaw, and ear and is referred to as ulceration en arc because it distinctly spares the nose tip, which can be involved in malignancies and other granulomatous infections. On MRI, TTS manifests as soft tissue defects involving the nasal ala. Importantly, the nasal tip is spared owing to innervation of the nose tip by the medial nasal branch of the anterior ethmoidal nerve. Furthermore, there is no associated mass lesion to suggest neoplasm. Diffuse volume loss of the ipsilateral muscles of mastication in the distribution of the trigeminal nerve is noted. Remote skull base or intracranial surgeries can cause damage to the trigeminal nerve pathway and can be visualized along the trajectory of the trigeminal nucleus in the brainstem or along its course in the prepontine cistern and Meckel cave. The ulceration associated with TTS demonstrates nonspecific histology with signs of chronic trauma including lichenification, scarring, and pseudoepitheliomatous hyperplasia. The ulcerative lesion has a non-specific appearance; however, microbiology and histopathology serve as a valuable tool in excluding infectious-inflammatory and neoplastic entities. The diagnosis of TTS is often challenging and requires close clinical and radio-pathologic correlation. A comprehensive laboratory workup is essential in order to exclude neoplastic, autoimmune, or infectious etiologies. Treatment of TTS is often challenging. Medications are used to reduce paresthesias (carbamazepine, vitamin B, diazepam, and amitriptyline). Dressings, occlusion masks, and cotton gloves can also be implemented to reduce repeated trauma to the affected area. Antibiotics are also frequently required to prevent against infection. Psychologic management is critical to educating the patient that the ulceration is self-induced via manipulation and digital picking. Transcutaneous electrical nerve stimulation has been described to improved blood supply and wound healing. Reconstructive surgery with regional flaps has been successfully utilized to correct cosmetic defects. Unfortunately, despite the varied treatment modalities the ulceration typically recurs.

Key Points  A triad of nasal ulceration, paresthesia, and anesthesia with sparing of the nasal tip are characteristic features of TTS.  Trigeminal trophic syndrome is a rare condition that occurs following damage to the trigeminal nerve pathway, with a varied spectrum of etiologies.  Non-specific findings on microbiology and histopathology evaluation and negative biopsy for infectious-inflammatory and neoplastic pathologies are important for diagnosing this entity.  An intriguing diagnosis, TTS requires close clinical and radio-pathologic correlation.

Part VIII. Miscellaneous: Case 136

Suggested Reading Bhatti AF, Soggiu D, Orlando A. Trigeminal trophic syndrome: diagnosis and management difficulties. Plast Reconstr Surg 2008; 121(1): 1e–3e. Cardoso JC, Cokelaere K, Maertens M, et al. When nonspecific histology can be a clue to the diagnosis: three cases of trigeminal trophic syndrome. Clin Exp Dermatol 2014; 39(5): 596–9. Dicken CH. Trigeminal trophic syndrome. Mayo ClinProc 1997; 72(6): 543–5. Golden E, Robertson CE, Moossy JJ, Sandroni P, Garza I. Trigeminal trophic syndrome: a rare cause of chronic facial pain and skin ulcers. Cephalalgia 2015; 35(7): 636. Luksić I, Sestan-Crnek S, Virag M, Macan D. Trigeminal trophic syndrome of all three nerve branches: an

underrecognized complication after brain surgery. J Neurosurg 2008; 108(1) 170–3. Nagel MA, Gilden D. The trigeminal trophic syndrome. Neurology 2011; 77(15): 1499. Osaki Y, Kubo T, Minami K, Maeda D. Trigeminal trophic syndrome: report of 2 cases. Eplasty 2013; 13: e60. Tollefson TT, Kriet JD, Wang TD, Cook TA. Self-induced nasal ulceration. Arch Facial Plast Surg 2004; 6(3): 162–6. Weintraub E, Soltani K, Hekmatpanah J, Lorincz AL. Trigeminal trophic syndrome. A case and review. J Am Acad Dermatol 1982; 6: 52–7. Willis M, Shockley WW, Mobley SR. Treatment options in trigeminal trophic syndrome: a multi-institutional case series. Laryngoscope 2011; 121: 712–16.

651

CASE

Part VIII

137

Miscellaneous Prasad B. Hanagandi, Santanu Chakraborty

Clinical Presentation An 82-year-old woman presented with a two-week history of sudden-onset vertigo, dysarthria, and gait imbalance. She reported no history of fever, night sweats, facial weakness, or loss of consciousness. Her medical history includes hypertension with dyslipidemia, bilateral carotid stenosis, and previous treatment

for hypothyroidism. At the time of admission, her blood pressure and glucose level were within normal range. Neurologic examination revealed dysmetria, dysdiadochokinesia. Romberg test was positive. Hematologic studies were unremarkable and serologies for HIV and VDRL were negative. Cerebrospinal fluid analysis was negative for infectious and inflammatory processes.

Imaging Fig. 137.1 Axial noncontrast CT image at the level of middle cerebellar peduncles and pons.

(A)

(B)

Fig. 137.2 (A) Axial T2WI and (B) FLAIR at the level of middle cerebellar peduncles and pons.

Advanced Neuroradiology Cases, ed. Amaral et al. Published by Cambridge University Press. © Cambridge University Press 2016.

653

Part VIII. Miscellaneous: Case 137 Fig. 137.3 Axial T1WI postcontrast image at the level of middle cerebellar peduncles and pons.

(A) (B)

Fig. 137.4 (A) Axial DWI and (B) ADC image at the level of middle cerebellar peduncles and pons.

654

Part VIII. Miscellaneous: Case 137 Fig. 137.5 Axial GRE image at the level of middle cerebellar peduncles and pons.

(A)

(B)

Fig. 137.6 (A) Time of flight MRA of circle of Willis and (B) carotids.

655

Part VIII. Miscellaneous: Case 137

Bilateral Middle Cerebellar Peduncle Infarcts Primary Diagnosis Bilateral middle cerebellar peduncle infarcts

Differential Diagnoses Demyelinating, infectious-inflammatory pathologies: multiple sclerosis (MS), acute disseminated encephalomyelitis (ADEM), progressive multifocal leukoencephalopathy (PML), or HIV Metabolic conditions: Wilson disease, adrenoleukodystrophy, or adrenomyeloneuropathy Neurodegenerative diseases: MSA-C, SCA-3, SCA-6, or DRPLA Wallerian degeneration Hypertensive encephalopathy

Imaging Findings Fig. 137.1: Axial non-contrast CT demonstrates focal hypodensities involving the bilateral middle cerebellar peduncles. Fig. 137.2: (A) Axial T2WI and (B) FLAIR through the same level confirms the presence of hyperintense lesions. Fig. 137.3: Axial T1WI postgadolinium does not demonstrate enhancement. Fig. 137.4: (A) Axial DWI showed diffusion restriction with corresponding changes on (B) Corresponding ADC map. Fig. 137.5: Axial GRE image does not indicate presence of hemorrhage. Fig. 137.6: (A) MRA of the circle of Willis and neck vessels shows diffuse irregularity in the intracranial vertebral arteries. (B) MRA demonstrates atherosclerotic narrowing that impairs visualization of the extracranial segments. Significant atherosclerotic narrowing of both common carotid bifurcations is also noted.

Discussion Demonstration of T2-FLAIR hyperintense signal and diffusion restriction in the bilateral middle cerebellar peduncles (MCPs) of a patient with sudden-onset vertigo and cerebellar neurologic deficits are classic features of acute ischemic changes involving the bilateral AICA (anterior inferior cerebellar artery) territories. Demyelinating pathologies such as MS, ADEM, and infectious etiologies such as HIV and PML are known to present with bilateral MCP T2 and FLAIR hyperintense signal changes. Demyelinating lesions also tend to have supratentorial white matter and spinal cord changes. HIV and PML can have similar imaging features and are associated with volume loss and white matter lesions in the rest of the brain parenchyma. In PML, the pattern of diffusion restriction is along the periphery and represents the zone of demyelination. The sudden onset of symptoms, lack of white matter changes, and pattern of diffusion restriction in a patient with a negative HIV status make demyelinating or infectious etiologies unlikely diagnoses. Several metabolic conditions such as Wilson disease, adrenomyeloneuropathy-adrenoleukodystrophy, and chronic liver disease can present with similar T2-FLAIR signal changes.

656

Given the clinical presentation, these metabolic conditions are remote diagnostic options. Patients with acute hypoglycemia can present with similar T2-FLAIR and diffusion restriction signal; however, hematologic studies did not indicate hypoglycemia. Reversing the hypoglycemic status results in an improvement in clinical symptoms and MRI findings. In our patient’s case, her glycemic status was normal and symptoms persisted. Hypertensive encephalopathy due to uncontrolled blood pressure can present with headache, visual symptoms, and altered mental status. The white matter changes are identical to PRES (posterior reversible encephalopathy syndrome) and represent areas of vasogenic edema. The brainstem, thalami, basal ganglia, and cerebral white matter are also involved. Abnormal T2-FLAIR hyperintense signal and DWI changes can also be seen in cases of paramedian pontomesencephalic infarctions, secondary to Wallerian degeneration, and often develop as early as 10–12 days following an ischemic event. A normal brainstem on imaging, as in our case, makes this entity less likely. Neurodegenerative conditions such as MSAC (multiple system atrophy), various types of spinocerebellar ataxias (types 3 and 6), and dentatorubropallidoluysian atrophy (DRPLA) can have similar T2 and FLAIR signal changes but with associated cerebellar and brachium pontis volume loss. The clinical presentation is long-standing with progressive symptoms. The acute presentation also makes the possibility of tumors such as glioma and lymphoma very unlikely. Tumoral pathologies are often asymmetric, associated with mass effect, and progressive symptoms. The MCPs derive their blood supply predominantly from the AICA with a smaller contribution from the superior cerebellar artery (SCA). The AICA supplies the anterolateral parts of the cerebellum including most of the MCPs, middle and lower lateral pontine region, flocculus, and anterior aspect of the cerebellar lobules. Infarcts of the MCPs are very rare and overall, unilateral infarcts comprise about 0.9–0.12% of acute strokes. Bilateral MCP infarcts are extremely rare and very few case reports of isolated infarcts have been documented in the literature. Most infarcts are known to occur involving the watershed zone of AICA and SCA territories. Patients often present with vertigo, horizontal nystagmus, auditory and speech disturbances, and ataxia in all four limbs. Hypoperfusion of the watershed zone has been proposed as the cause of infarcts. Trauma and dissection are other mechanisms that can cause bilateral MCP infarcts in cases with unusual anatomic vascular variants. Atherosclerotic occlusion of the bilateral extracranial-intracranial vertebral arteries has been described in most of the case reports causing hypoperfusion.

Key Points  Bilateral MCP infarcts are extremely rare and only a handful of cases have been described in the literature.  Hypoperfusion involving the watershed zones of AICA and SCA vascular territories has been proposed as the

Part VIII. Miscellaneous: Case 137

mechanism of infarction leading to ischemic changes. Atherosclerotic disease and trauma have been suggested as other common causes.  The differential diagnoses list for bilateral MCP lesions is extensive. The imaging findings have to be correlated with the duration of onset and severity of clinical presentation with relevant laboratory correlation.

Suggested Reading Akiyama K, Takizawa S, Tokuoka K, et al. Bilateral middle cerebellar peduncle infarction caused by traumatic vertebral artery dissection. Neurology 2001; 56(5): 693–4. Fitzek C, Fitzek S, Stoeter P. Bilateral Wallerian degeneration of the medial cerebellar peduncles after ponto-mesencephalic infarction. Eur J Radiol 2004; 49(3): 198–203.

John S, Hegazy M, Cheng Ching E, Katzan I. Isolated bilateral middle cerebellar peduncle infarcts. J Stroke Cerebrovasc Dis 2013; 22(8): e645–6. Kalla R, Mayer T, Hamann GF. Bilateral anterior inferior cerebellar artery territory brachium pontis infarcts of probable hemodynamic cause. Eur Neurol 2004; 51(4): 233–5. Kataoka H, Izumi T, Kinoshita S, et al. Infarction limited to both middle cerebellar peduncles. J Neuroimaging 2011; 21(2): e171–2. Okamoto K, Tokiguchi S, Furusawa T, et al. MR features of diseases involving bilateral middle cerebellar peduncles. AJNR Am J Neuroradiol 2003; 24(10): 1946–54. Tsukamoto T, Seki H, Saitoh J, Watanabe K. A case of bilateral cerebellar peduncle infarction. Jpn J Med 1991; 30(4): 376–8.

657

Index

abscess, brain, 186, 542, 554 aceruloplasminemia (ACP), 15–16 neuroferritinopathy vs., 8, 16 acromesomelic upper limb shortening, 569 acute cerebellitis, 153–6 acute confusional state, 116 acute disseminated encephalomyelitis (ADEM) acute cerebellitis vs., 156 acute necrotizing encephalopathy vs., 252 hemorrhagic, vs. malaria, 208 Susac syndrome vs., 104 Wernicke encephalopathy vs., 360 acute necrotizing encephalopathy, 249–53 acute hyperammonemic encephalopathy, 301–5 adrenoleukodystrophy, 40 X-linked (XLA), 264 AIDS. See HIV infection/AIDS alcohol abuse Marchiafava-Bignami disease, 308 Wernicke encephalopathy, 357, 361 Alexander disease, adult-onset, 46, 373–6 Alzheimer disease (AD) corticobasal syndrome, 26 hippocampal sclerosis dementia vs., 36 mammillothalamic tract degeneration vs., 588 ammonia toxicity, 304 amnesia acute-onset, after thalamic infarct, 587–8 psychogenic, 116 transient global (TGA), 113–16 amyloid β-related angiitis (ABRA), 94–5 amyloidoma, cerebral (CA), 87–91 amyloidosis, 90–1

658

amyotrophic lateral sclerosis (ALS), 40–1, 212 anterior cerebral artery infarction, 308 anterior inferior cerebellar artery (AICA), 656 antiangiogenic therapy. See bevacizumab antiepileptic drug withdrawal, 327–30 anti-Ma2-associated paraneoplastic encephalitis (AMAPE), 445–7 anti-NMDA receptor encephalitis, 223–6 aquaporin-4 (AQP4) autoantibodies, 258 arachnoid cysts, 476 arachnoid granulations, giant, 607–11 arborized pattern, schistosomiasis, 192 arteriovenous malformations (AVMs) differential diagnosis, 78, 386, 504 radiation-induced tumefactive cyst, 131–4 aspergillosis, 183–8 astroblastoma, 389–93 astrocytoma fibrillary, 398 infiltrating low-grade (ILGA), 488 low-grade, 524 pilocytic (PA), 548 pilomyxoid (PMA), 18, 548–9 pineal region, 498 protoplasmic, 395–9 ATP7A gene, 298 ATP7B gene, 342 ATXN3 gene, 72 atypical teratoid/rhabdoid tumor, 404, 410 autonomic failure, multiple system atrophy, 21–2 autosomal dominant leukodystrophy (ADLD), adult-onset, 66 Avastin. See bevacizumab

bag of worms appearance, 504 Balo concentric sclerosis (BCS), 245–7 basal ganglia aspergillosis, 184, 186 chronic hepatic encephalopathy, 291 cystic degeneration, 7–8 hemolytic uremic syndrome, 141–2, 144 Huntington disease, 55–7 hybrid phakomatosis, 577, 580 hyperglycemic hemiballismus and hemichorea, 281–2 multiple system atrophy, 21–2 osmotic demyelination syndrome, 371–2 phospholipase-associated neurodegeneration, 13–14 Whipple disease, 18 Wilson disease, 340, 342–3 basal ganglia calcification aceruloplasminemia vs., 16 chronic hepatic encephalopathy, 292 familial idiopathic (FIBGC), 16, 614 mineralizing microangiopathy, 613–14 bat-wing pattern, osmotic demyelination syndrome, 372 Behçet disease, 228 neurologic (NBD), 227–9, 242 rhombencephalitis, 160 Bell palsy, 152 beta-propeller protein-associated neurodegeneration (BPAN), 2, 4 bevacizumab distal recurrence of glioblastoma after, 443–4 for hippocampal radiation necrosis, 430 persistent diffusion restriction after, 494 pseudoresponse of glioblastoma to, 439–42

Bickerstaff brainstem encephalitis (BBE), 237–40, 242 biphasic myelinopathy of Grinker, 288 blastomycosis, 181–2 blepharitis, lipoid proteinosis, 268 Bloch-Siemens syndrome. See incontinentia pigmenti BPAN. See beta-propeller protein-associated neurodegeneration brain atrophy corticobasal syndrome, 26 frontotemporal dementia with FUS mutation, 59 Machado-Joseph disease, 71–2 Menkes disease, 298 MPAN, 3–4 neuroferritinopathy, 8 phospholipase-associated neurodegeneration, 14 progressive supranuclear palsy, 27–8 brainstem. See also medulla oblongata; midbrain; pons adult-onset Alexander disease, 373, 376 blastomycosis, 181–2 encephalitis. See rhombencephalitis glioma, 182, 272 Machado-Joseph disease, 69, 72 metronidazole-induced encephalopathy, 322–4 neuromyelitis optica spectrum disorder, 258 progressive ataxia and palatal tremor, 43, 46 branch retinal artery occlusion (BRAO), 105 bright claustral sign, Wilson disease, 343 C19orf12 gene, 4 CADASIL, 107–11

Index calcification astroblastoma, 389, 392 basal ganglia. See basal ganglia calcification brain tumors with, 90, 268 cerebral amyloidoma, 87, 90 deep gray nuclei, 16 dermoid cysts, 484 intracranial chondroma, 458 lipoid proteinosis, 267–8 meningioangiomatosis, 383, 386 mineralizing microangiopathy, 613–14 rosette-forming glioneuronal tumor, 382 T1-weighted hyperintensity, 84 third ventricle craniopharyngioma, 461, 464 calcineurin inhibitor-mediated bilateral limbic injury, 599–602 capillary telangiectasias (CT), 128 carbon monoxide poisoning, 10, 342 delayed encephalopathy, 285–8 cardiac arrest, with global hypoxia, 84 caudate nuclei chronic infarcts, 56 frontotemporal dementia with FUS mutation, 59 Huntington disease, 55–7 Wilson disease, 342 cavernous malformations cavernous sinus, 530 developmental venous anomaly and, 128 meningioangiomatosis vs., 386 cavernous sinus hemangioma, 529–30 cephalocele, 148 cerebellar ataxia Erdheim-Chester disease, 623 idiopathic late-onset (ILOCA), 20 cerebellar atrophy, 14 cerebrotendinous xanthomatosis, 49 MPAN, 3–4 multiple system atrophy, 19–20 phospholipase-associated neurodegeneration, 13–14 progressive supranuclear palsy, 27–8 cerebellar liponeurocytoma, 421–4 cerebellar peduncles. See also middle cerebellar peduncles LBSL, 64, 66

Machado-Joseph disease, 69, 72 primary lateral sclerosis, 38, 40 cerebellitis, acute, 153–6 cerebellum cerebrotendinous xanthomatosis, 49, 52 rosette-forming glioneuronal tumor, 381–2 cerebral amyloid angiopathy (CAA) related inflammation (CAA-RI), 94–5 without inflammation, 94 cerebral autosomal-dominant arteriopathy with subcortical infarcts and leukoencephalopathy (CADASIL), 107–11 cerebral proliferative angiopathy (CPA), 78–9 cerebral venous thrombosis bilateral thrombosis of internal cerebral vein, 135–9 differential diagnosis, 128, 252, 278, 610 cerebrotendinous xanthomatosis (CTX), 49–52, 66, 72 ceruloplasmin gene mutations, 16 low serum, 16, 339, 342 cervical compressive myelopathy, 40 cervical spinal cord adult-onset Alexander disease, 376 cerebrotendinous xanthomatosis, 49, 52 Chagas disease, 199–203 child abuse, 298 childhood ataxia with central nervous system hypomyelination. See vanishing white matter (VWM) disease chondroma dural convexity, 455–8 meningioma vs., 452, 458 chondrosarcoma, 458, 530 chordoid glioma, 464, 548 chordoma, 530 choroid plexus carcinoma, 560 choroid plexus papilloma, temporal horn, 560 CLIPPERS (chronic lymphocytic inflammation with pontine perivascular enhancement responsive to steroids), 241–3 colloid cysts, 464 colonic adenocarcinoma, mucinous, 480 congenital diseases manifesting in adults, 566 copper deficiency, 366

deposition in brain, 342 metabolic/transport defects, 298, 342 corpus callosum (CC) hereditary spastic paraplegia with hypoplastic, 566 isolated infarction, 308 Marchiafava-Bignami disease, 307–9 neuromyelitis optica spectrum disorder, 258–9 partial agenesis, 308 phospholipase-associated neurodegeneration, 14 splenium. See splenium of corpus callosum Susac syndrome, 103–5 cortical dysplasia, pericallosal and left frontal surface lipoma with, 512–15 cortical laminar necrosis, 304 corticobasal degeneration (CBD), 26, 60 corticobasal syndrome (CBS), 25–6, 28, 60 corticospinal tract HTLV-1-related tropical spastic paraparesis, 212 multiple system atrophy, 21 primary lateral sclerosis, 38, 40 craniectomy, sinking skin flap syndrome after, 603–6 craniopharyngioma, third ventricle, 461–5 Creutzfeldt-Jakob disease (CJD), 26, 304 variant, 360 cutis verticis gyrata, 643–6 cyclosporine A, 602 cysticercosis. See neurocysticercosis cysts/cystic lesions astroblastoma, 389, 392 brain tumors, 392 dermoid cysts. See dermoid cysts epidermoid cysts. See epidermoid cysts extraventricular neurocytoma, 415, 418 glioblastoma with, 542 microcystic meningioma, 449, 452–3 neurocysticercosis, 213–14 neuroferritinopathy, 7–8 pilomyxoid astrocytoma, 545, 548 protoplasmic astrocytoma, 398 radiation-induced tumefactive cyst, 131–4 supratentorial ependymoma, 517, 520 third ventricle craniopharyngioma, 461, 464

vanishing white matter disease, 263–4 cytomegalovirus encephalitis, 360 dark dentate disease, 46 DARS2 gene, 66 delayed ischemic hyperintensity (DIH), 84–5 demyelination Balo concentric sclerosis, 246 delayed carbon monoxide poisoning, 288 Marchiafava-Bignami disease, 309 osmotic demyelination syndrome (ODS), 272, 369–72 progressive multifocal leukoencephalopathy, 166 subacute combined degeneration, 366 tumefactive, 272 dentate nuclei cerebrotendinous xanthomatosis, 49, 52 Erdheim-Chester disease, 620, 622 Machado-Joseph disease, 69, 72 dentatorubral-pallidoluysian atrophy (DRPLA), 72, 584 dentato-rubro-olivary pathway. See Guillain-Mollaret triangle dermoid cysts, 414, 484 intra-axial, 481–4 lipoma vs., 514 malignant transformation, 411–14 ruptured, 484, 536 white epidermoid cysts vs., 476 desmoplastic medulloblastoma, multifocal, 407–10 developmental venous anomalies (DVA), complications, 125–9 diabetes mellitus aceruloplasminemia, 15–16 hyperglycemic hemiballismus and hemichorea, 84, 281–3 DIDMOAD (diabetes insipidus, diabetes mellitus, optic atrophy, and deafness), 351–4 diffuse axonal injury (DAI), 122 drop metastases pinealoblastoma with, 498–9 renal cell carcinoma with, 503–4 dural convexity chondroma, 455–8 dural tail sign hemangiopericytoma, 510 meningioma, 452

659

Index dural venous sinuses arachnoid granulations, 610 idiopathic intracranial hypertension, 595–6 dysembryoplastic neuroepithelial tumor (DNET), 524 differential diagnosis, 268, 398 multifocal, 524 ear of the lynx sign, 564, 566 edema, cerebral. See also vasogenic edema hemolytic uremic syndrome, 144 internal cerebral vein thrombosis, 136, 138 meningioangiomatosis, 383, 386 metronidazole-induced encephalopathy, 324 microcystic meningioma, 453 empty sella, 594, 596 en coup de sabre appearance, 639–40 encephalitis brainstem. See rhombencephalitis paraneoplastic. See paraneoplastic encephalitis syndromes Susac syndrome vs., 104 viral. See viral encephalitis encephalocraniocutaneous lipomatosis (ECCL), 514 eosinophilic granuloma, calvarial, 148 ependymomas, 520 supratentorial, 392, 520 epidermoid cysts giant arachnoid granulation vs., 610 intraparenchymal, 467–70 white, 473–6 Epstein-Barr virus (EBV) encephalitis, 160 Erdheim-Chester disease, 619–23 extracellular matrix protein 1 (ECM1) gene, 268 eye-of-the-tiger sign absence, 4 differential diagnosis, 10 neuroferritinopathy, 8 PKAN, 10 Wilson disease, 342 face of the giant panda and her cub sign, 343 facial appearance lipoid proteinosis, 267–8 Parry-Romberg syndrome, 637, 640 trigeminal trophic syndrome, 647, 649–50 familial idiopathic basal ganglia calcification (FIBGC), 16, 614

660

fat deposits cerebellar liponeurocytoma, 421, 424 dermoid cyst, 412, 414, 484 intratumoral, differentiation, 414, 424 lipoma, 512, 514 ruptured dermoid cyst, 484, 533, 536 fat embolism, cerebral, 119–23 ferritin light chain gene mutations, 8 serum, 7, 16, 345, 348 fibrillary astrocytoma, 398 fibrinoid leukodystrophy, 376 filling-in enhancement, cavernous sinus hemangioma, 528, 530 flame appearance. See ear of the lynx sign FMR1 gene, 584 fourth ventricle, rosette-forming glioneuronal tumor, 381–2 fragile X-associated tremor/ ataxia syndrome (FXTAS), 72, 581–5 Friedrich ataxia, 354 frontal lobe corticobasal syndrome, 26 frontotemporal dementia with FUS mutation, 59 Whipple disease, 18 frontotemporal dementia (FTD), 28, 60 behavioral variant (bv-FTD), 60 hippocampal sclerosis dementia vs., 36 with FUS gene mutation (FTD-FUS), 59–61 with motor neuron disease (FTD-MND), 60 frontotemporal lobar degeneration (FTLD), 60 corticobasal syndrome, 26 right temporal variant, 32 frontotemporal lobar degeneration (FTLD)-FUS, 59–61 frontotemporal lobar degeneration (FTLD)-tau, 60 frontotemporal lobar degeneration (FTLD)-TDP43, 60 frontotemporal lobar degeneration (FTLD)-U, 60 fungal abscess, 542, 554 FUS gene mutation, frontotemporal dementia associated with (FTD-FUS), 59–61 fused in sarcoma (FUS) protein, 60

ganglioglioma, 268, 418, 524 germ cell tumors (GCT) hypothalamic, 464 pineal region, 498 testicular, paraneoplastic encephalitis, 445–7 glial fibrillary acidic protein (GFAP) gene, 376 glioblastoma (GBM), 542 cystic, 542 differential diagnosis, 134, 398, 554 distal recurrence after antiangiogenic therapy, 443–4 persistent diffusion restriction after bevacizumab, 494 pseudoprogression after chemoradiation, 433–7 pseudoresponse after antiangiogenic therapy, 439–42 glioma angiocentric, 524 brainstem, 182, 272 chordoid, 464, 548 gliomatosis cerebri, 488 globus pallidus BPAN, 2 MPAN, 4 neuroferritinopathy, 7–8 PKAN, 9–10 glutaric aciduria type 1, 298 granulomas Aspergillus, 187 neurosarcoidosis, 234 tuberculosis, 193, 196 Guillain-Mollaret triangle, 627–8 hypertrophic olivary degeneration, 628 progressive ataxia and palatal tremor, 46 gumma, syphilitic cerebral, 175–9 hair-curled pattern, epidermoid cyst, 470 headache idiopathic intracranial hypertension, 593, 596 postural, 589–90 recurrent acute thunderclap, 97, 100 hearing impairment, Susac syndrome, 105 hemangioblastoma (HGBL), 504 hemangiomas. See also venous malformations cavernous. See cavernous malformations cranial osseous, 148 hemangiopericytoma, intraventricular, 510 hemiballismus and hemichorea, 282

hyperglycemic (HHH), 84, 281–3 hemifacial atrophy, Parry-Romberg syndrome, 637, 640 hemifacial microsomia, congenital, 640 hemochromatosis, hereditary, 345–8 hemolytic uremic syndrome, 141–4 hemorrhage, intracranial. See also microhemorrhages; subarachnoid hemorrhage cerebral malaria, 208 chronic hepatic leukoencephalopathy, 288 developmental venous anomalies, 128 hypertensive, 278 posterior reversible encephalopathy syndrome, 275–9 hemorrhagic infarction internal cerebral vein thrombosis, 138 T1-weighted hyperintensity, 84 hepatic encephalopathy, 292 chronic, 282, 292, 342 hepatic leukoencephalopathy, chronic, 288 hepatolenticular degeneration. See Wilson disease hereditary spastic paraplegia (HSP), 566 differential diagnosis, 40, 566 with hypoplastic corpus callosum (HSP-HCC), 566 herpes simplex virus (HSV) acute cerebellitis, 153–6 encephalitis, 104, 160 herpes virus type 6 (HHV-6) encephalopathy, 602 herpes zoster oticus. See Ramsay Hunt syndrome Herpesviridae, 156 hippocampal sclerosis other associated neuropathologies, 36 pure, with dementia, 33–6 hippocampus anti-NMDA receptor encephalitis, 223, 225 calcineurin inhibitor-mediated injury, 599–602 radiation-induced necrosis, 427–31 transient global amnesia, 113, 116 HIV infection/AIDS cerebrovascular disease, 196 middle cerebellar peduncle infarction vs., 656

Index neurosyphilis, 169, 172 primary lateral sclerosis vs., 40 hoarseness, lipoid proteinosis, 268 hot-cross-bun sign Machado-Joseph disease, 69, 72 multiple system atrophy, 19–20 HTLV-1 infection, 40, 212 HTLV-1-related neurologic complex, 209–12 humming bird sign, 27–8 Hunter syndrome (mucopolysaccharidosis type II), 377–80 huntingtin protein, 56 Huntington disease (HD), 56–7, 60 hyalinosis cutis et mucosae. See lipoid proteinosis hybrid phakomatosis, 580 hydrocephalus choroid plexus papilloma, 560 developmental venous anomalies, 128 mucopolysaccharidosis, 380 27-hydroxylase, mitochondrial, 52 hyperalimentation, 84, 282 hyperammonemic encephalopathy, acute, 301–5 hypercoagulable state, multiple early infarcts, 110 hyperglycemic hemiballismus and hemichorea (HHH), 84, 281–3 hyperintense putaminal slit sign, 21–2 hypertensive emergency, 271–2, 275 hypertensive intracranial hemorrhage (HIC), 278 hypertrophic olivary degeneration, 625–9 hypoglycemia, acute, 84, 656 hypogonadotropic hypogonadism, hereditary hemochromatosis, 345, 348 hypomelanosis of Ito, 311–14 hyponatremia, rapid correction, 372 hypothalamic hamartoma isolated, 572 Pallister-Hall syndrome, 570, 572 syndromic associations, 572 hypothalamus anti-Ma2-associated paraneoplastic encephalitis, 445 Whipple disease, 18 Wolfram syndrome, 354 hypoxic ischemic injury (HII), 56

idiopathic late-onset cerebellar ataxia (ILOCA), 20 IKBKG gene, 576 immune reconstitution inflammatory syndrome, progressive multifocal leukoencephalopathyassociated, 163–6 immunosuppressed patients. See also HIV infection/AIDS aspergillosis, 186 calcineurin inhibitor-mediated bilateral limbic injury, 599– 602 Chagas disease, 202–3 PRES with hemorrhage, 279 incontinentia pigmenti, 314, 576 infantile neuroaxonal dystrophy (INAD). See phospholipaseassociated neurodegeneration infarction. See also stroke developmental venous anomalies, 128 hemorrhagic, 84, 138 incomplete, 84–5 isolated bilateral middle cerebellar peduncle, 653–7 isolated corpus callosum, 308 mammillothalamic tract degeneration after thalamic, 587–8 secondary to hypercoagulable state, 110 subcortical lacunar, 110 tuberculous vasculitis, 197 inferior olivary nuclei (ION) hypertrophic olivary degeneration, 628–9 metronidazole-induced encephalopathy, 322–4 progressive ataxia and palatal tremor, 43, 46 internal cerebral vein (ICV) thrombosis, bilateral, 135–9 intracranial hypertension, idiopathic, 593–7 intracranial hypotension spontaneous (SIH), 590 with midbrain swelling, 589–91 intracranial pressure (ICP), 596 inverted V appearance, subacute combined degeneration, 366 iron deposition, 4 aceruloplasminemia, 15–16 BPAN, 1–2 hereditary hemochromatosis, 348 MPAN, 4 multiple system atrophy, 21–2 neuroferritinopathy, 8 phospholipase-associated neurodegeneration, 14 PKAN, 10

ischemia, cerebral. See also infarction; small vessel ischemic disease; stroke bevacizumab-treated glioblastoma, 494 ruptured dermoid cyst with, 536 spectacular shrinking deficit, 84 transient global amnesia, 116 tuberculous vasculitis, 195–7 Japanese B encephalitis, 216 with neurocysticercosis, 215–16 JC virus, 166 Kayser-Fleischer (KF) rings, 342 kinky hair syndrome. See Menkes disease Korsakoff syndrome, 116, 360 LA2G6 gene mutations, 14 lactate LBSL, 65–6 MELAS, 336 lamellar pattern. See onionskin configuration lateral ventricles enlarged, Huntington disease, 55–6 intraventricular hemangiopericytoma, 510 ring-shaped nodules, 615–18 temporal horn choroid plexus papilloma, 560 LBSL. See leukoencephalopathy with brainstem and spinal cord involvement and lactate elevation Leber hereditary optic atrophy, 354 Leigh disease, 8, 252, 342 leptomeningeal carcinomatosis, 152, 220 leptomeningeal involvement acute cerebellitis, 155–6 neurocysticercosis, 213–14 neurosarcoidosis, 234 primary angiitis of CNS, 218 leptomeningeal melanoma, primary, 634 leukoencephalopathy with brainstem and spinal cord involvement and lactate elevation (LBSL), 63–6, 264 leukoencephalopathy with vanishing white matter, 261–5 Lhermitte-Duclos disease, 404 limbic encephalitis other causes, 446 paraneoplastic, 18, 446 limbic injury, calcineurin inhibitor-mediated bilateral, 599–602

lingual gyrus, MELAS, 333, 336 lipoblastic meningioma, 414 lipoid proteinosis, 267–9 lipoma, 514 interhemispheric, 514 pericallosal and left frontal surface, with cortical dysplasia, 512–15 liponeurocytoma, cerebellar, 421–4 liposarcoma, 414 Listeria rhombencephalitis, 157–61, 182 liver disease, chronic, 292 liver, hereditary hemochromatosis, 347–8 lymphoma, central nervous system (CNS) differential diagnosis, 90, 182, 554 perivascular spread, 242 lymphomatoid granulomatosis, 242 lymphomatosis cerebri, 488 Ma2-associated paraneoplastic encephalitis, 445–7 Machado-Joseph disease (MJD), 72 malaria, cerebral, 205–8 malignant transformation intracranial dermoid cyst, 411–14 neurocutaneous melanosis, 634–5 mammillary bodies, Wernicke encephalopathy, 357, 360–1 mammillothalamic tract degeneration, thalamic infarct with, 587–8 manganese (Mn) deposition, chronic liver disease, 292 Marchiafava-Bignami disease, 307–9 medulla oblongata. See also brainstem adult-onset Alexander disease, 376 hypertrophic olivary degeneration, 626, 628 medulloblastoma, 404, 410 cerebellar liponeurocytoma vs., 424 molecular subgroups, 404 multifocal desmoplastic, 407–10 with extensive nodularity, 401–5 megalencephalic leukoencephalopathy with subcortical cysts (MLSC), 264 melanoblastosis cutis. See incontinentia pigmenti melanocytic nevi, congenital (CMN), 633–4

661

Index melanoma neurocutaneous melanosis transforming to, 634–5 primary leptomeningeal, 634 MELAS (mitochondrial encephalomyopathy with lactic acidosis and strokelike episodes), 333–6 differential diagnosis, 110, 252 meningeal neurosyphilis, 172 meningioangiomatosis, 383–7 meningioma, 386, 452 angiomatous, 452 cavernous sinus, 530 chondroma vs., 452, 458 hemangiopericytoma vs., 510 lipoblastic, 414 microcystic, 452–3 pineal region, 498 sinking skin flap syndrome after, 603–6 trigeminal trophic syndrome after, 647–50 meningitis chemical, ruptured dermoid cyst, 536 tuberculous, 193, 196–7, 202 meningocele, 148 meningovascular syphilis (MVS), 196 Menkes disease, 295–8 mesial temporal sclerosis, 36 metabolic diseases, 261–5 metachromatic leukodystrophy, 40, 264 metastases, central nervous system, 480 differential diagnosis, 182, 186 mucinous adenocarcinoma, 477–80 pineal region, 498 renal cell carcinoma in posterior fossa, 501–5 methanol toxicity, 282, 342 methotrexate encephalopathy, 317–19 methotrexate-induced leukoencephalopathy, 318 metronidazole-induced encephalopathy, 46, 321–4 MGMT (O6-methyl-guanine methyltransferase) gene promoter, methylation status, 433, 436 Mickey Mouse appearance, 27–8 microhemorrhages amyloid β-related angiitis, 93–4 CADASIL, 109–10 cerebral fat embolism, 121–2 midbrain atrophy, progressive supranuclear palsy, 27–8 swelling, intracranial hypotension with, 589–91

662

Wilson disease, 341–3 middle cerebellar peduncles (MCPs). See also cerebellar peduncles fragile X-associated tremor/ ataxia syndrome, 581, 584–5 isolated bilateral infarcts, 653–7 progressive multifocal leukoencephalopathy, 163, 166 Wilson disease, 340, 342 Wolfram syndrome, 353–4 middle cerebral artery (MCA) focal stenosis, 83–4, 219–20 temporary occlusion, 84–5 Miller-Fisher syndrome, 240 mineralizing microangiopathy, 613–14 mitochondrial diseases, 66 mitochondrial encephalomyopathy with lactic acidosis and strokelike episodes. See MELAS mitochondrial membrane protein-associated neurodegeneration (MPAN), 2–5 morning glory sign, 27–8 motor cortex, primary lateral sclerosis, 37, 40 motor neuron disease, 40–1, 566, See also amyotrophic lateral sclerosis; primary lateral sclerosis motor neuropathy, MPAN, 4 motor trephine syndrome. See sinking skin flap syndrome MPAN. See mitochondrial membrane proteinassociated neurodegeneration mucinous metastasis, 182, 477– 80 mucopolysaccharidosis (MPS), 377–80 multiple sclerosis (MS) Balo concentric sclerosis, 246 Marchiafava-Bignami disease vs., 308 neuromyelitis optica vs., 258–9 primary lateral sclerosis vs., 40 Susac syndrome vs., 104 multiple system atrophy (MSA), 20 cerebellar type (MSA-C), 19–20 parkinsonism type (MSA-P), 20–2, 56 myoclonus epilepsy and ragged red fibers (MERRF), 252 nasal ulceration, trigeminal trophic syndrome, 648, 650 nasopharyngeal cancer, treated, 427, 430

NEMO gene, 576 neuroacanthocytosis, 60 neuro-Behçet disease, 227–9, 242, See also Behçet disease neurocutaneous melanosis, 631–5 neurocysticercosis Japanese B encephalitis with, 215–16 racemose, 213–14 neurocytoma, extraventricular, 415–18 neurodegeneration with brain iron accumulation (NBIA), 4 aceruloplasminemia, 15–16 BPAN, 2 MPAN (type 4), 3–5 neuroferritinopathy, 7–8 phospholipase-associated neurodegeneration, 13–14 PKAN (type 1), 9–11 neurodegenerative diseases, 1–2 neuroenteric cysts, 476 neuroferritinopathy (NFT), 7–8 aceruloplasminemia vs., 8, 16 neurofibromatosis type 1, with concomitant tuberous sclerosis, 580 neuroinfectious diseases, 149–52 neuroinflammatory diseases, 217–21 neuromyelitis optica spectrum disorder (NOSD), 255–9 neurosarcoidosis, 234 differential diagnosis, 182, 202, 234, 242 parenchymal, 231–4 neurosyphilis cerebral gumma, 175–9 differential diagnosis, 196, 446 manifestations, 172, 178 with temporal lobe involvement, 169–72 neurovascular diseases, 78–9 nitrous oxide (N2O) intoxication, 366 non-accidental trauma (child abuse), 298 NOTCH3 gene mutations, 110 obesity, idiopathic intracranial hypertension, 596 occipital lobe MELAS, 334–5 microcystic meningioma, 449, 452 oculofacial(-skeletal) myorhythmia, 18 oligodendroglioma, 90, 268, 392, 418, 524 olivary degeneration, hypertrophic, 625–9 olivopontocerebellar atrophy, sporadic, 72

onionskin configuration, Balo concentric sclerosis, 245–6 optic atrophy MPAN, 3–4 phospholipase-associated neurodegeneration, 13–14 Wolfram syndrome, 351, 354 optic nerve idiopathic intracranial hypertension, 593, 596 neuromyelitis optica spectrum disorder, 258 optic tracts, subacute combined degeneration involving, 363–6 osmotic demyelination syndrome (ODS), 272, 369–72 Pacchionian depressions. See arachnoid granulations, giant palatal tremor hypertrophic olivary degeneration, 625, 628 progressive ataxia and palatal tremor, 43, 46 Pallister-Hall syndrome, 572 panda sign, 343 PANK2 gene mutations, 10 pantothenate kinase 2 (PANK2), 10 pantothenate kinase-associated neurodegeneration (PKAN), 9–11 atypical, 10 differential diagnosis, 2, 4, 8 typical, 10 paradoxical herniation, postcraniectomy, 606 paraneoplastic encephalitis syndromes anti-Ma2-associated paraneoplastic encephalitis, 445–7 anti-NMDA receptor encephalitis, 223–6 limbic encephalitis, 18, 446 rhombencephalitis, 160 parental nutrition, long-term, 282 parietal lobe atrophy, corticobasal syndrome, 26 Parkinson-plus syndrome (PPS), 28 Parkinson disease (PD), 2, 4, 21–2, 28 Parry-Romberg syndrome, 637–41 penguin-silhouette sign, 27–8 peppered appearance, CLIPPERS, 242 Percheron artery occlusion, 138, 252 periaqueductal gray, Wernicke encephalopathy, 358, 360

Index perimesencephalic subarachnoid hemorrhage (PMSAH), 117–18 perivascular spaces, dilated hypomelanosis of Ito, 311, 314 mucopolysaccharidosis, 377, 380 phakomatosis, hybrid, 580 phospholipase-associated neurodegeneration (PLAN), 13–14 atypical, 14 differential diagnosis, 2, 4 Pick disease, 60 pilocytic astrocytoma (PA), 548 pilomyxoid astrocytoma (PMA), 18, 546–9 pineal parenchymal tumors, 498 pinealoblastoma, 498 with drop metastasis, 498–9 pituitary gland, hereditary hemochromatosis, 346, 348 PKAN. See pantothenate kinase-associated neurodegeneration PLAN. See phospholipaseassociated neurodegeneration pleomorphic xanthoastrocytoma, 524 polysyndactyly, Pallister-Hall syndrome, 569 pons central variant of PRES, 271–2 CLIPPERS, 241–2 hypertrophic olivary degeneration, 625, 628 Machado-Joseph disease, 69, 72 multiple system atrophy, 19–20 osmotic demyelination syndrome, 369, 372 Wilson disease, 341–3 Wolfram syndrome, 353–4 posterior cortical atrophy, 26 corticobasal syndrome, 26 posterior fossa, renal cell carcinoma metastasis, 501–5 posterior reversible encephalopathy syndrome (PRES), 272 atypical or severe forms, 144 calcineurin inhibitor-induced limbic injury vs., 602 central variant (CV), 271–3 hemorrhagic, 275–9 methotrexate-induced, 318 premotor cortex, primary lateral sclerosis, 39–40 PRES. See posterior reversible encephalopathy syndrome primary angiitis of central nervous system (PACNS), 217–21 differential diagnosis, 94, 100, 110

primary lateral sclerosis (PLS), 40–1 progressive ataxia and palatal tremor (PAPT) familial, 46 idiopathic, 43–7 progressive multifocal leukoencephalopathy (PML) classic lesions, 166 immune reconstitution inflammatory syndrome, 163–6 middle cerebellar peduncle infarction vs., 656 progressive nonfluent aphasia (PNFA), 60 progressive supranuclear palsy (PSP), 28–9 classic (Richardson) variant (PSP-R), 28 corticobasal syndrome, 26 frontotemporal lobar degeneration, 60 variants, 28 prosopagnosia, progressive (PP), 32 protoplasmic astrocytoma, 395–9 pseudobulbar affect, 40 pseudotumor cerebri. See intracranial hypertension, idiopathic punica granatum seed sign, chondroma, 458 putamen Huntington disease, 55–7 multiple system atrophyParkinsonian type, 21–2 Wilson disease, 342 pyogenic brain abscess, 186, 542, 554 pyramidal tracts LBSL, 64, 66 multiple system atrophy, 22 Quattrone’s magnetic resonance (MR) Parkinsonism index, 28 rabbit ears appearance, subacute combined degeneration, 366 radiation necrosis, delayed hippocampus, 427–31 tumefactive cyst formation vs., 134 radiation-induced tumefactive cyst, 131–4 radiation-induced tumors, 134 Ramsay Hunt syndrome, involving trigeminal spinal nucleus and tract, 149–52 Rasmussen encephalitis, 640 Refsum disease, 52 renal cell carcinoma, metastasis to posterior fossa, 501–5

retinal degeneration, 14, 16 reversible cerebral vasoconstriction syndrome (RCVS), 97–101, 278 reversible splenial lesion syndrome (RESLES), 327–30 Reye syndrome, 252 rhombencephalitis (RE). See also Bickerstaff brainstem encephalitis central variant of PRES vs., 272 etiology, 160 Listeria, 157–61, 182 right temporal lobe atrophy (RTLA), 31–2 right temporal variant of frontotemporal lobar degeneration, 32 ring-shaped lateral ventricular nodules (RSLVNs), 615–18 rosette-forming glioneuronal tumor of fourth ventricle, 381–2 sandwich sign, 308 sarcoidosis, nervous system. See neurosarcoidosis satellite lesions, rosette-forming glioneuronal tumor, 382 scalp, cutis verticis gyrata, 643–6 schistosomiasis, 189–92 schwannoma, cavernous sinus, 530 scleroderma, localized, 640 Seitelberger disease. See phospholipase-associated neurodegeneration semantic dementia (SD), 32, 60 seminoma, anti-Ma2-associated paraneoplastic encephalitis, 445–7 sinking skin flap syndrome, 603–6 sinus pericranii, 145–8 skeletal manifestations Erdheim-Chester disease, 621–2 Menkes disease, 298 mucopolysaccharidosis, 379–80 Pallister-Hall syndrome, 569 skin manifestations hypomelanosis of Ito, 313–14 incontinentia pigmenti, 573, 575–6 lipoid proteinosis, 267–8 neurocutaneous melanosis, 633–4 small vessel ischemic disease (SVID), 318, 488 snowball-like lesions, Susac syndrome, 103–5 spastic paraplegia, hereditary. See hereditary spastic paraplegia spectacular shrinking deficit (SSD), 84–5

SPG gene mutations, 566 spinal cord. See also cervical spinal cord HTLV-1-related tropical spastic paraparesis, 212 LBSL, 65–6 neuromyelitis optica spectrum disorder, 257–9 subacute combined degeneration, 363–6 spine drop metastases, 496, 498, 503–4 mucopolysaccharidosis, 379–80 spinocerebellar ataxia (SCA) type-3. See Machado-Joseph disease type-6, 72 splenium of corpus callosum metronidazole-induced encephalopathy, 321, 323–4 reversible/transient lesion, 327–30 sporadic subcortical arteriosclerotic encephalopathy (SSAE), 110 squamous cell cancer, dermoid cyst transformation to, 414 star field pattern, cerebral fat embolism, 122 static encephalopathy (of childhood) with neurodegeneration in adulthood (SENDA). See beta-propeller proteinassociated neurodegeneration striatum atrophy, Huntington disease, 56–7 hyperglycemic hemiballismus and hemichorea, 281–2 string-of-beads appearance, reversible cerebral vasoconstriction syndrome, 101 string-of-pearls appearance, Susac syndrome, 103–5 stroke. See also infarction; ischemia, cerebral CADASIL, 110 MELAS, 336 racemose neurocysticercosis, 213–14 reversible cerebral vasoconstriction syndrome, 100–1 ruptured dermoid cyst, 535–6 tuberculous vasculitis, 197 subacute combined degeneration, with optic tract involvement, 363–6 subarachnoid hemorrhage (SAH) aneurysmal, 100

663

Index subarachnoid hemorrhage (SAH) (cont.) perimesencephalic (PMSAH), 117–18 PRES with, 277–8 reversible cerebral vasoconstriction syndrome, 97, 100 subarachnoid spaces enhancing lesions, 202 enlarged, 55–6 subcortical arteriosclerotic encephalopathy, sporadic (SSAE), 110 subdural collections Menkes disease, 295, 298 spontaneous intracranial hypotension, 590 subependymal heterotopia, 618 subependymal nodules, 618 subependymoma, 90, 560, 618 substantia nigra aceruloplasminemia, 15–16 iron deposition, 1–2 neuroferritinopathy, 7 PKAN, 10 superior cerebellar artery (SCA), 656 Susac syndrome (SS), 103–5, 110, 308 Swiss cheese pattern, radiation necrosis, 429–30 syphilis, 172, 178, 196, See also neurosyphilis cerebral gumma, 175–9 T2-dark through effect, white epidermoid cyst, 476 tabes dorsalis, 172 tacrolimus-mediated bilateral limbic injury, 599–602 target sign, tuberculoma, 554 tauopathies, 26, 28

664

temporal horn choroid plexus papilloma, 560 temporal lobe atrophy, right-sided (RTLA), 31–2 neurosyphilis, 169–72 radiation-induced necrosis, 427–31 Whipple disease, 18 testicular tumor, anti-Ma2associated paraneoplastic encephalitis, 445–7 thalamus aceruloplasminemia, 15–16 anti-Ma2-associated paraneoplastic encephalitis, 445 infarct, with mammillothalamic tract degeneration, 587–8 internal cerebral vein thrombosis, 136, 138 Wernicke encephalopathy, 358, 360 Whipple disease, 18 thiamine deficiency, 360 third ventricle craniopharyngioma, 461–5 TORCH group of infections, 314 toxoplasmosis, 202 transient global amnesia (TGA), 113–16 transient ischemic attacks (TIA), multiple early, 107, 110 transient splenial lesion, 327–30 trichothiodystrophy. See Menkes disease trigeminal nerve (CN V) pathways LBSL, 66 Ramsay Hunt syndrome involving, 149–52

trigeminal trophic syndrome (TTS), 647–50 tropical spastic paraparesis, 209–12 tuberculoma, giant, 554–5 tuberculosis, 196–7 cerebral vasculitis, 193–7 meningitis, 193, 196–7, 202 tuberous sclerosis hypomelanosis of Ito vs., 314 subependymal nodules, 618 with concomitant neurofibromatosis type 1, 580 tumefactive cyst, radiationinduced, 131–4 tumefactive demyelination, 272 tumors, central nervous system, 381–2 Balo concentric sclerosis vs., 246 calcification, 90, 268 fat-containing, 414, 424 radiation-induced, 134 syphilitic gumma resembling, 178–9 ulceration en arc, 650 Urbach Wiethe disease. See lipoid proteinosis vanishing white matter (VWM) disease, 261–5 varicella-zoster virus (VZV), 152, 156 vascular dementia (VD), 36 vasculitis, cerebral. See also primary angiitis of central nervous system neurocysticercosis, 214 primary, 228 tuberculosis, 193–7 vasogenic edema. See also edema, cerebral

cerebral amyloidoma, 87–90 CNS aspergillosis, 183, 186 PRES with hemorrhage, 275, 277–8 venous malformations. See also sinus pericranii complications, 125–9 hypertrophic olivary degeneration, 625, 628 meningioangiomatosis vs., 386 venous sinuses, dural. See dural venous sinuses venous thrombosis, cerebral. See cerebral venous thrombosis viral encephalitis. See also Japanese B encephalitis differential diagnosis, 104, 160, 488 hemorrhagic, 208 visual disturbances. See also optic atrophy idiopathic intracranial hypertension, 593, 596 MELAS, 333 subacute combined degeneration, 366 Susac syndrome, 105 vitamin B12 deficiency, 40, 66, 366 Wernicke encephalopathy, 324, 357–61 WFS1 (wolframin) gene, 354 Whipple disease (WD), 17–18 whirlpool-like pattern, epidermoid cyst, 470 white epidermoid cyst, 473–6 Wilson disease (WD), 8, 339–43, 584 wine glass appearance, 40 Wolfram syndrome, 351–4 xanthoastrocytoma, pleomorphic, 524